Vous êtes sur la page 1sur 260

MP23

Calcul différentiel
Licence Sciences et Techniques

Recueil d’exercices corrigés et aide-mémoire.

Gloria Faccanoni
i http://faccanoni.univ-tln.fr/enseignements.html

Année 2013 – 2014


L1 PC

Table des matières


1 Approximations : polynômes de Taylor & développements limités 3

2 Fonctions de plusieurs variables 39


2.1 Introduction . . . . . . . . . . . . . . . . . . . . . . . . . . . . . . . . . . . . 39
2.2 Représentations de fonctions de deux variables . . . . . . . . . . . . . . . 40

3 Limites et continuité 51
3.1 Normes . . . . . . . . . . . . . . . . . . . . . . . . . . . . . . . . . . . . . . . 51
3.2 Limites . . . . . . . . . . . . . . . . . . . . . . . . . . . . . . . . . . . . . . . 52
3.3 Continuité . . . . . . . . . . . . . . . . . . . . . . . . . . . . . . . . . . . . . 54

4 Dérivabilité et différentiabilité, fonctions implicites 61


4.1 Dérivées partielles du premier ordre et gradient . . . . . . . . . . . . . . . 61
4.2 Différentiabilité . . . . . . . . . . . . . . . . . . . . . . . . . . . . . . . . . . 66
4.3 Dérivées partielles de deuxième ordre et matrice hessienne . . . . . . . . 72
4.4 Fonctions implicites . . . . . . . . . . . . . . . . . . . . . . . . . . . . . . . . 76

5 Extrema 117
5.1 Extrema libres . . . . . . . . . . . . . . . . . . . . . . . . . . . . . . . . . . . 118
5.2 Extrema liés . . . . . . . . . . . . . . . . . . . . . . . . . . . . . . . . . . . . . 127

6 Intégrales multiples 193


6.1 Intégrale double d’une fonction continue . . . . . . . . . . . . . . . . . . . 193
6.2 Intégrale triple . . . . . . . . . . . . . . . . . . . . . . . . . . . . . . . . . . . 196
6.3 Applications . . . . . . . . . . . . . . . . . . . . . . . . . . . . . . . . . . . . 197
6.4 Quelques intégrales remarquables . . . . . . . . . . . . . . . . . . . . . . . 200

7 Fonctions vectorielles d’une variable réelle : courbes paramétrées 233


7.1 Courbes planes en coordonnées polaires . . . . . . . . . . . . . . . . . . . 235

8 Champs de vecteurs, formes différentielles 237


8.1 Formes différentielles . . . . . . . . . . . . . . . . . . . . . . . . . . . . . . . 238
8.2 Champs de vecteurs . . . . . . . . . . . . . . . . . . . . . . . . . . . . . . . . 238

Dernière mise-à-jour
Jeudi 5 juin 2014

1
Dernière mise à jour : Jeudi 5 juin 2014

Ce cours s’adresse à des étudiants de la première année d’une Licence Physique-Chimie. Il a pour objectif de donner les bases
en calcul différentiel pour des fonctions de plusieurs variables indispensables à toute formation en physique et en chimie.
Les notions supposées connues correspondent au programme du premier semestre.

L’objet de ce aide-mémoire est de proposer une explication succincte des concepts vu en cours. De nombreux livres, parfois
très fournis, existent. Ici on a cherché, compte tenu des contraintes de volume horaire, des acquis des étudiants au premier
semestre et des exigences pour la suite du cursus, à dégager les points clés permettant de structurer le travail personnel de
l’étudiant voire de faciliter la lecture d’autres ouvrages. Ce polycopiée ne dispense pas des séances de cours et de TD ni de
prendre des notes complémentaires. Il est d’ailleurs important de comprendre et apprendre le cours au fur et à mesure car
on a très peu de temps et beaucoup de concepts nouveaux. Ce polycopié est là pour éviter un travail de copie qui empêche
parfois de se concentrer sur les explications données oralement mais ce n’est pas un livre auto-suffisant (il est loin d’être
exhaustif ) ! De plus, ne vous étonnez pas si vous découvrez des erreurs. Malgré de très nombreuses relectures, il restera
toujours des fautes, ce polycopié est donc fournit sans garanties ! N’hésitez pas à me signaler les erreurs que vous remarquez.

On a inclus dans ce texte nombreux exercices corrigés. Ceux-ci, de difficulté variée, répondent à une double nécessitée. Il est
important de jongler avec les différents concepts introduits en cours et même de faire certaines erreurs une fois pour bien
identifier les pièges. Les exercices permettent d’orienter les raisonnements vers d’autres domaines (physique, économie,
etc.), cela afin d’exhiber l’intérêt et l’omniprésence du calcul différentiel. Cependant, veuillez noter que vous n’obtiendrez
pas grande chose si vous vous limitez à choisir un exercice, y réfléchir une minute et aller vite voir le début de la correction
en passant tout le temps à essayer de comprendre la correction qui va paraitre incompréhensible. Pour que la méthode
d’étude soit vraiment efficace, il faut d’abord vraiment essayer de chercher la solution. En particulier, il faut avoir un papier
brouillon à coté de soi et un crayon. La première étape consiste alors à traduire l’énoncé (pas le recopier), en particulier
s’il est constitué de beaucoup de jargon mathématique. Ensuite il faut essayer de rapprocher les hypothèses de la conclusion
souhaitée, et pour cela faire quelques calculs ou transformer les hypothèses pour appliquer un théorème dont on aura vérifier
que les hypothèses sont bien satisfaite. C’est ici que l’intuition joue un grand rôle et il ne faut pas hésiter à remplir des
pages pour s’apercevoir que l’idée qu’on a eu n’est pas la bonne. Elle pourra toujours resservir dans une autre situation.
Quand finalement on pense tenir le bon bout, il faut rédiger soigneusement en s’interrogeant à chaque pas sur la validité
(logique, mathématique) de ce qu’on a écrit. Si l’étape précédente ne donne rien, il faut chercher de l’aide (voir le début de
la correction, en parler à un autre étudiant, interroger les tuteurs).

Gloria FACCANONI

IMATH Bâtiment U-318 T 0033 (0)4 94 14 23 81


Université du Sud Toulon-Var
Avenue de l’université B gloria.faccanoni@univ-tln.fr
83957 LA GARDE - FRANCE i http://faccanoni.univ-tln.fr

2 © G. Faccanoni
1 Approximations : polynômes de Taylor &
développements limités
Il est souvent plus avantageux de remplacer des fonctions compliquées par des fonctions plus simples qui les approchent.

Définition Linéarisation
Si on approche une fonction f au voisinage d’un point x 0 au moyen d’une fonction affine L(x) = q + mx, il est naturel de
choisir la fonction L dont le graphe est tangent au graphe de la fonction f en x 0 :

L(x) = f (x 0 ) + f 0 (x 0 )(x − x 0 ).

C’est ce qu’on appelle la linéarisation de f en x 0 . Dans certains cas on mentionne explicitement le point auquel la linéa-
risation est obtenue et on note la linéarisation de f en x 0 par L x0 .

Attention y L0
La linéarisation d’une fonction dépend du point auquel
on linéarise la fonction.
p Par exemple, la linéarisation de f
la fonction p f (x) = 1 + x en 0 donne L 0 (x) = 1 + x/2 (car
f 0 (x) = 1/(2 1 + x) et L 0 (x) = f (0) + f 0 (0)x = 1 + x/2) tan-
dis que la linéarisation en 3 donne L 3 (x) = (5 + x)/4. La
linéarisation L 0 fournit une meilleure approximation de L3
f tant que x < 1, la linéarisation L 3 devient meilleure
lorsque x > 1. En x = 1, les deux linéarisations fournissent
−2 −1 0 1 2 3 4 x
la même valeur.

Définition Notation
Lorsqu’elle est évaluée en x 0 , la linéarisation de la fonction f en x 0 coïncide avec la fonction f . Lorsque x reste proche
de x 0 , la linéarisation de L x0 fournit une approximation de f . On note

f (x) ' L(x) lorsque x ' x 0 .

Le signe ' signifie “est approximativement égal à” sans que l’on attache ici de sens plus précis à cette notation.

Exemple
La dérivée de f (x) = (1+x)n est égale à f 0 (x) = n(1+x)n−1 . La linéarisation de f à l’origine est donc égale à L 0 (x) = f (0)+ f 0 (0)x = 1+nx
est on a
(1 + x)n ' 1 + nx lorsque x ' 0.
Cette formule particulièrement simple à retenir est valable pour des valeurs quelconques de n. Elle permet de calculer rapidement des
approximations de racines et de puissances de nombres proches de l’unité. Ainsi, par exemple
p 0.2
1.2 = (1 + 0.2)0.3 ' 1 +
3
= 1.06.
3

La même formule permet d’évaluer immédiatement 1.002100 à 1.2 (la valeur exacte est 1.221 . . . ).

3
1 Approximations : polynômes de Taylor & développements limités Dernière mise à jour : Jeudi 5 juin 2014

Exemple
La linéarisation de la fonction sin à l’origine est L 0 (x) = x, donc

y L0

sin(x) ' x lorsque x ' 0. f

C’est la linéarisation que l’on effectue pour résoudre l’équation


du pendule en physique. x0 x
La qualité de cette approximation peut être également appréciée
au moyen de quelques valeurs :

x −0.2 −0.1 −0.05 0 0.05 0.1 0.2


f (x) = sin(x) −0.1986 −0.0998 −0.0499 0 0.0499 0.0998 0.1986
L 0 (x) = x −0.2 −0.1 −0.05 0 0.05 0.1 0.2

La linéarisation d’une fonction f en un point x 0 est un polynôme de degré inférieur ou égal à 1 tel que
(
L(x 0 ) = f (x 0 ),
L 0 (x 0 ) = f 0 (x 0 ).

Les polynômes de TAYLOR généralisent cette construction pour des polynômes de degrés quelconques.

Définition Polynôme de TAYLOR


Le polynôme de TAYLOR d’ordre n généré par f au point x 0 est le (seul) polynôme P n de degré inférieur ou égal à n qui
satisfait les conditions
P n(k) (x 0 ) = f (k) (x 0 ) pour k = 0, 1, . . . , n,
i.e. le polynôme
n f (k) (x )
0
(x − x 0 )k .
X
P n (x) =
k=0 k!
Le graphe du polynôme de TAYLOR d’une fonction f en x 0 est une courbe polynomiale tangente au graphe de f en x 0 .

Remarque
Soit la fonction f (x) = x 2 + 1. On a f 0 (x) = 2x, f 00 (x) = 2 et f (k) (x) = 0 lorsque k ≥ 3. Les polynômes de TAYLOR d’ordre 0,
1 et 2 générés par f à l’origine s’obtiennent par P 0 (x) = P 1 (x) = 1 et P 2 (x) = P k (x) = f (x) lorsque k ≥ 3 : les polynômes de
TAYLOR d’ordre ≥ 2 sont tous égaux à 1 + x 2 . C’est la raison pour laquelle on définit un polynôme de TAYLOR d’ordre n et
non pas de degré n : le degré d’un polynôme de TAYLOR peut être inférieur à son ordre.

Proposition
Le polynôme de TAYLOR d’ordre n généré au point x 0 par un polynôme f de degré m ≤ n coïncide avec f quel que soit x 0 .

Exemple y P2
f P1
Le polynôme de TAYLOR d’ordre n généré au point 0 par la fonc-
tion f (x) = e x est

x2 x3 xn
P n (x) = 1 + x + + +... .
2 6 n! P0
x0

4 © G. Faccanoni
Dernière mise à jour : Jeudi 5 juin 2014 1 Approximations : polynômes de Taylor & développements limités

Exemple y f
Le polynôme de TAYLOR d’ordre 4 généré au point 0 par le fonc- P3
tion f (x) = sin(x) est
x0 x
x3
P n (x) = x −
6
La qualité de l’approximation des fonctions circulaires par leurs
polynômes de TAYLOR est excellente.

Exemple
Le polynôme de TAYLOR d’ordre n généré au point 0 par le fonction f (x) = (1 − x)m , m > n est

n f (k) (0)
xk .
X
P n (x) =
k=0 k!

Comme à !
m−k m! m−k m
f (k)
(x) = m(m − 1) · · · (m − (k − 1))(1 − x) = (1 − x) = k!(1 − x)m−k
(m − k)! k
on trouve à !
n m
xk .
X
P n (x) =
k=0 k

Propriété
Soit f et g deux fonctions n fois dérivables en un point x 0 et P f et P g leurs polynômes de TAYLOR d’ordre n en x 0 . Alors
1. le polynôme de TAYLOR d’ordre n généré par la somme f + g au point x 0 est le polynôme P f + P g ;
2. le polynôme de TAYLOR d’ordre n généré par le produit f g au point x 0 est la somme des termes de degré inférieur
ou égaux à n du polynôme P f P g ;
3. le polynôme de TAYLOR d’ordre nk généré par f (x k ) au point x 0 est le polynôme P f (x k ) ;
4. le polynôme de TAYLOR d’ordre n − 1 généré par f 0 au point x 0 est le polynôme P 0f .

Exemple Somme
Sachant que le polynôme de TAYLOR d’ordre 3 généré par sin(x) à l’origine est égal à x − x 3 /6, on peut conclure que le polynôme de
TAYLOR d’ordre 3 généré par 4x + 3x 2 − sin(x) à l’origine est égal à 4x + 3x 2 − (x − x 3 /6), i.e. P 3 (x) = 3x + 3x 2 + x 3 /6.

Exemple Produit
Sachant que le polynôme de TAYLOR d’ordre 3 généré par e x à l’origine est égal à 1+ x + x 2 /2+ x 3 /6, on peut conclure que le polynôme
de TAYLOR d’ordre 3 généré par (x + 2x 2 − x 3 )e x à l’origine est donné par le polynôme obtenu par la somme des termes de degré
inférieur ou égaux à 3 du produit (x + 2x 2 − x 3 )(1 + x + x 2 /2 + x 3 /6), i.e. P 3 (x) = x + 3x 2 + 3x 3 /2.

Exemple Composition
Sachant que le polynôme de TAYLOR d’ordre n généré par 1/(1− x) = (1− x)−1 à l’origine est égal à 1+ x + x 2 +· · ·+ x n , on peut conclure
que le polynôme de TAYLOR d’ordre n généré par 1/(1 − (−x)) = (1 − (−x))−1 à l’origine est égal à 1 + (−x) + (−x)2 + · · · + (−x)n , i.e.
1−x+x 2 +· · ·+(−1)n x n et que le polynôme de TAYLOR d’ordre n généré par 1/(1+x 2 ) à l’origine est égal à 1+(−x 2 )+(−x 2 )2 +· · ·+(−x 2 )n ,
i.e. 1 − x 2 + x 4 + · · · + (−1)n x 2n .

Exemple
On veut calculer les polynômes de TAYLOR d’ordre 4 à l’origine générés par les fonctions f de R dans R suivantes :

1. f (x) = e x 2. f (x) = sin(x) 3. f (x) = e x + sin(x) 4. f (x) = e x sin(x) 5. f (x) = e sin(x)

© G. Faccanoni 5
1 Approximations : polynômes de Taylor & développements limités Dernière mise à jour : Jeudi 5 juin 2014

Par définition de polynôme de TAYLOR on a

f 00 (0) 2 f 000 (0) 3 f (i v) (0) 4


f (x) ' f (0) + f 0 (0)x + x + x + x lorsque x ' 0.
2 6 24
1. f (x) = e x

f (x) = e x f (0) = 1
f 0 (x) = e x f 0 (0) = 1
f 00 (x) = e x f 00 (0) = 1
000 x
f (x) = e f 000 (0) = 1
f (i v) (x) = e x f (i v) (0) = 1

donc
x2 x3 x4
ex ' 1 + x + + + lorsque x ' 0.
2 6 24
2. g (x) = sin(x)

f (x) = sin(x) f (0) = 0


f 0 (x) = cos(x) f 0 (0) = 1
f 00 (x) = − sin(x) f 00 (0) = 0
000
f (x) = − cos(x) f 000 (0) = −1
f (i v) (x) = sin(x) f (i v) (0) = 0

donc
x3
sin(x) ' x − lorsque x ' 0.
6
3. s(x) = e x + sin(x)
En utilisant la définition :
f (x) = e x + sin(x) f (0) = 1
0 x
f (x) = e + cos(x) f 0 (0) = 2
00 x
f (x) = e − sin(x) f 00 (0) = 1
f 000 (x) = e x − cos(x) f 000 (0) = 0
f (i v) (x) = e x + sin(x) f (i v) (0) = 1

donc
x2 x4
e x + sin(x) ' 1 + 2x + + lorsque x ' 0.
2 24
En utilisant la propriété de la somme :
à ! à !
x2 x3 x4 x3 x2 x4
e x + sin(x) ' 1 + x + + + + x− = 1 + 2x + + lorsque x ' 0.
2 6 24 6 2 24

4. p(x) = e x sin(x)
En utilisant la définition :
f (x) = e x sin(x) f (0) = 0
0 x
f (x) = e (sin(x) + cos(x)) f 0 (0) = 1
00 x
f (x) = 2e cos(x) f 00 (0) = 2
f 000 (x) = 2e x (cos(x) − sin(x)) f 000 (0) = 2
f (i v) (x) = −4e x sin(x) f (i v) (0) = 0

donc
x3
e x sin(x) ' x + x 2 + lorsque x ' 0.
3
En utilisant la propriété du produit :
!Ã ! Ã
x x2 x3 x4 x3
e sin(x) ' termes de degré inférieur ou égal à 4 du produit 1 + x + + + x−
2 6 24 6
à !
x3 x5 x6 x7
= termes de degré inférieur ou égal à 4 de x + x 2 + − − −
3 24 36 144
x3
= x + x2 + lorsque x ' 0.
3

6 © G. Faccanoni
Dernière mise à jour : Jeudi 5 juin 2014 1 Approximations : polynômes de Taylor & développements limités

5. c(x) = e sin(x)
En utilisant la définition :

f (x) = e sin(x) f (0) = 1


0 sin(x)
f (x) = cos(x)e f 0 (0) = 1
³ ´
f 00 (x) = cos2 (x) − sin(x) e sin(x) f 00 (0) = 1
³ ´
f 000 (x) = cos2 (x) − 3 sin(x) − 1 cos(x)e sin(x) f 000 (0) = 0
³ ´
f (i v) (x) = sin(x) − 4 cos2 (x) + 3 sin2 (x) − 6 sin(x) cos2 (x) + cos4 (x) e sin(x) f (i v) (0) = −3

donc
x2 x4
e sin(x) ' 1 + x + − lorsque x ' 0.
2 8
En utilisant la propriété de la composition :
3 2 3 3 3 4
 ³ ´ ³ ´ ³ ´ 
x − x6 x − x6 x − x6
à !
x 3
e sin(x) ' termes de degré inférieur ou égal à 4 de 1 + x − + + +
 
6 2 6 24

à !
x2 x4 x5 x6 x7 x8 x9 x 10 x 12
= termes de degré inférieur ou égal à 4 de 1 + x + − − − + + − − +
2 8 12 72 72 144 1296 1296 31104
x2 x4
= 1+x + − lorsque x ' 0.
2 8

Exemple
On veut déterminer la limite de visibilité depuis un point d’altitude H au-dessus du niveau de la mer. Du point A on voit jusqu’au point
B.

A L’on a
OB R 1
H cos(ϑ) = = = .
OA R + H 1+ H
C B R
Pour une altitude «raisonnable», HR est petit et ϑ également (R ≈
6371 km). On utilise les deux équivalences
R

ϑ
ϑ2 H −1 H
µ ¶
1
cos(ϑ) ≈ 1 − , = 1+ ≈ 1− ,
O 2 1+ H
R
R R
q
qui conduisent à ϑ ≈ 2 H R . On conclut que la distance C B à
la surface de la
p Terre, c’est-à-dire la longueur de l’arc de cercle,
est ` = Rϑ = 2R H . Par exemple, du sommet de la tour Eiffel
(H = 324 m) on peut voir à une distance de 66 kilomètres.

L’utilisation d’un polynôme de TAYLOR d’ordre n d’une fonction f en x 0 pour approcher f à proximité de x 0 induit une erreur
qui est d’autant plus élevée que le graphe de la fonction s’écarte du graphe de f . L’erreur commise n’est nulle partout que
lorsque la fonction f est un polynôme de degré inférieur ou égal à n. Lorsqu’on dispose d’une borne sur la dérivée (n + 1)-
ème de f , il est possible de borner l’erreur. De plus, lorsque l’on utilise un polynôme de TAYLOR d’ordre de plus en plus grand
pour approcher f , on s’attend à ce que l’approximation s’améliore.

Théorème Erreur d’approximation


Soit f : [a; b] → R une fonction n + 1 fois dérivable et soit P n le polynôme de TAYLOR d’ordre n généré par f en x 0 . Si
| f (n+1) (x)| ≤ M pour tous les x dans [a; b], alors

(b − a)n+1
| f (x) − P n (x)| ≤ M
(n + 1)!

pour tous les x dans [a; b].

© G. Faccanoni 7
1 Approximations : polynômes de Taylor & développements limités Dernière mise à jour : Jeudi 5 juin 2014

Exemple
La linéarisation de f (x) = sin(x) en x = 0 donne sin(x) ' x. Quelle est la précision de cette approximation lorsque |x| ≤ 0.5 ? Comme
max | f 00 (x)| = max |− sin(x)| = sin(0.5), une application directe du théorème permet d’écrire
|x|≤0.5 |x|≤0.5

(0.5)2
|sin(x) − x| ≤ sin(0.5) ≤ 0.06
2
pour tous les x dans [−0.5; 0.5].

Exemple
Le polynôme d’ordre 2 généré par la fonction f (x) = 1/(1 − x) à l’origine est donné par

P 2 (x) = 1 + x + x 2 .

Considérons ces fonctions sur l’intervalle [0; 0.1]. La fonction f 000 est dérivable sur cet intervalle et maxx∈[0;0.1] | f 000 (x)| =
maxx∈[0;0.1] |6/(1 − x)4 | = 6/(1 − 0.1)4 = M . Par conséquent

(0.1 − 0)2+1
| f (x) − P 2 (x)| ≤ 6/(1 − 0.1)4 ≤ 0.00152
(2 + 1)!

pour tous les x dans [0; 0.1].

Définition Développement limité.


f admet un développement limité à l’ordre n en x 0 , et on le note DL n (x 0 ), si D f ∩]x 0 − δ, x 0 + δ[\{x 0 } 6= ; pour tout δ > 0
et s’il existe un polynôme P n de degré ≤ n tel que

f (x) = P n (x − x 0 ) + (x − x 0 )n ε(x − x 0 ),

où ε est une fonction de D f ∩] − δ, +δ[\{0} à valeur dans R telle que lim ε(x) = 0. Au lieu de (x − x 0 )n ε(x − x 0 ) on écrit
x→0
souvent o((x − x 0 )n ).
Lorsqu’il existe, le polynôme P n est unique.

Théorème de TAYLOR-Y OUNG


Si f et toutes ses dérivées jusqu’à l’ordre n sont définies et continues en tout point d’un intervalle ouvert I contenant x 0
alors f admet un développement limité à l’ordre n en x 0 donné par la formule

n f (k) (x )
0
(x − x 0 )k + (x − x 0 )n ε(x − x 0 ).
X
f (x) =
k=0 k!

Attention
On dit qu’une fonction f admet un développement limité à l’ordre n (entier naturel) au point x 0 s’il existe un polynôme
P de degré au plus n tel que

f (x) = P (x − x 0 ) + o((x − x 0 )n ) = a 0 + a 1 (x − x 0 ) + a 2 (x − x 0 )2 + · · · + a n (x − x 0 )n + o((x − x 0 )n )

au voisinage de x 0 .
Si f est n fois continûment dérivable sur un intervalle I avec n entier positif ou nul, alors pour tout point x 0 ∈ I , f a un
développement limité au point x 0 donné par la formule de Taylor-Young
n f (k) (x )
0
(x − x 0 )k + o((x − x 0 )n ).
X
f (x) =
k=0 k!

Il est tentant de penser que la réciproque est vraie, c’est-à-dire qu’une fonction qui admet un développement limité à
l’ordre n en un point x 0 est dérivable n fois en ce point. C’est effectivement le cas si n = 0 ou n = 1. Dans le premier
cas, le développement limité s’écrit juste f (x) = a 0 + o(1) et donc a 0 = f (a) par définition d’un o(1). Si n = 1 et f (x) =
a 0 + a 1 (x − x 0 ) + o(x − x 0 ), alors a 0 = f (a) pour la même raison, et par suite :

f (x) − f (x 0 )
= a 1 + o(1)
x −a
8 © G. Faccanoni
Dernière mise à jour : Jeudi 5 juin 2014 1 Approximations : polynômes de Taylor & développements limités

ce qui implique que l’accroissement fini a une limite égale à a 1 , ainsi f est bien dérivable en x 0 et a 1 = f 0 (x 0 ). Mais la
réciproque est fausse pour n ≥ 2, autrement dit une fonction peut admettre un développement limité à l’ordre n ≥ 2 bien
qu’elle ne soit pas n fois continûment dérivable.

Exemple
Soit f : R → R l’application définie par

f : R→R
³ ´
x 2 sin x1
(
pour x 6= 0,
x 7→
0 pour x = 0.

On veut calculer le développement limité à l’ordre 1 puis à l’ordre 2 en 0 de f et montrer que f n’admet pas de développement limité
à l’ordre 3 en 0.
Commençons par calculer les dérivées de f :
³ ´
x 2 sin x1
(
pour x 6= 0
µ ¶
1
f (x) = et lim x 2 sin = 0 = f (0);
0 pour x = 0 x→0 x
³ ´ ³ ´
2x sin x1 − cos x1

pour x 6= 0 µ µ ¶
1
µ ¶¶
1
f 0 (x) = f (x)− f (0) x 2 sin 1 −0
¡ ¢
et lim f 0 (x) = lim 2x sin − cos n’existe pas.
 lim = lim x
= 0 pour x = 0 x→0 x→0 x x
x→0 x−0 x→0 x−0

Par conséquent f est continue mais f 0 n’est pas continue en x = 0. Cela implique que f n’admet pas de dérivée seconde en 0.
On peut utiliser la formule de TAYLOR-Y OUNG pour calculer le développement limité de f en 0 à l’ordre 1 :

f (x) = f (0) + f 0 (0)x + o(x) = 0 + o(x).

En revanche, comme f n’admet pas de dérivée seconde en 0, on ne peut pas utiliser la formule de TAYLOR-Y OUNG pour calculer le
développement limité de f en 0 à l’ordre 2. Cependant, ce développement limité existe est pour le calculer il suffit de remarquer que
³ ´
x 3 sin x1
lim = 0,
x→0 x2

c’est-à-dire f (x) = o(x 2 ) : le développement limité de f en 0 à l’ordre 2 est donc f (x) = 0 + o(x 2 ).
On montre enfin qu’il n’existe pas de développement limité de f en 0 à l’ordre 3 car la limite
³ ´
x 3 sin x1
lim
x→0 x3
n’existe pas.

Attention Développements limités usuels

k=n xk x2 x3 x4 xn
(−1)k+1 + o xn = x − + · · · + (−1)n+1 + o xn
X ¡ ¢ ¡ ¢
ln(1 + x) = + −
k=1 k 2 3 4 n
k=n xk x2 x3 x4 xn
+ o x n = −x − + o xn
X ¡ ¢ ¡ ¢
ln(1 − x) = − − − +···−
k=1 k 2 3 4 n
k=n xk x2 x3 x4 xn
ex = + o xn = 1 + x + + o xn
X ¡ ¢ ¡ ¢
+ + +···+
k=0 k! 2 6 24 n!

k=n x 2k+1 x3 x5 x 2n+1


(−1)k + o x 2n+2 = x − + · · · + (−1)n + o x 2n+2
X ¡ ¢ ¡ ¢
sin(x) = +
k=0 (2k + 1)! 6 120 (2n + 1)!
k=n x 2k x2 x4 x 2n
(−1)k + o x 2n+1 = 1 − + · · · + (−1)n + o x 2n+1
X ¡ ¢ ¡ ¢
cos(x) = +
k=0 (2k)! 2 24 (2n)!
x 3 2x 5 17x 7
+ o x8
¡ ¢
tan(x) = x + + +
3 15 315

© G. Faccanoni 9
1 Approximations : polynômes de Taylor & développements limités Dernière mise à jour : Jeudi 5 juin 2014

¯Ã !¯
x 3 3x 5 ¯ − 1 ¯ x 2n+1
+···+¯ 2 ¯ + o x 2n+2
¡ ¢
arcsin(x) = x + +
¯ ¯
6 40 ¯ n 2n + 1
¯
π
arccos(x) = − arcsin(x)
2
k=n x 2k+1 x3 x5 x7 x 2n+1
(−1)k + o x 2n+1 = x − + · · · + (−1)n + o x 2n+2
X ¡ ¢ ¡ ¢
arctan(x) = + −
k=0 2k + 1 3 5 7 2n + 1

k=n x 2k+1 x3 x5 x 2n+1


+ o x 2n+2 = x + + o x 2n+2
X ¡ ¢ ¡ ¢
sinh(x) = + +···+
k=0 (2k + 1)! 6 120 (2n + 1)!
k=n x 2k x2 x4 x 2n
+ o x 2n+1 = 1 + + o x 2n+1
X ¡ ¢ ¡ ¢
cosh(x) = + +···+
k=0 (2k)! 2 24 (2n)!
x 3 2x 5 17x 7
+ o x8
¡ ¢
tanh(x) = x − + −
3 15 315
à ! à !
α
X α k
k=n ¡ n¢ α(α − 1) 2 α(α − 1)(α − 2) 3 α n
x + o x = 1 + αx + x + o xn
¡ ¢
(1 + x) = x + x +···+
k=0 k 2 6 n

avec les cas particuliers

p x x 2 x 3 5x 4
+ o x4
¡ ¢
1+x = 1+ − + −
2 8 16 128
1 x 3x 2 5x 3 5x 4
+ o x4
¡ ¢
p == 1 − + − +
1+x 2 8 16 128
p x x 2 x 3 5x 4
+ o x4
¡ ¢
1−x = 1− − − −
2 8 16 128
1 k=n
(−1)k x k + o x n = 1 − x + x 2 − x 3 + · · · + (−1)n x n + o x n
X ¡ ¢ ¡ ¢
=
1 + x k=0
1 k=n
X k
x + o xn = 1 + x + x2 + x3 + · · · + xn + o xn
¡ ¢ ¡ ¢
=
1 − x k=0

Définition Prépondérance
Soit f et g deux applications de D vers R et soit x 0 ∈ R ∪ { ±∞ }. On dit que f est négligeable devant g au voisinage de x 0
s’il existe un voisinage V de x 0 et une fonction ϕ : V → R vérifiant

∀x ∈ V ∩ D, f (x) = g (x)ϕ(x), et lim ϕ(x) = 0.


x→x 0

On note alors f = o(g ).


x0
Si g ne s’annule pas dans un voisinage de x 0 , cette définition est équivalente à

f (x)
lim = 0.
x→x 0 g (x)

Exemple
ln(x) = o(x α ), ln(x) = o(x −α ), x α = o(e x ), |x|α = o(e −x ).
+∞ 0+ +∞ −∞

10 © G. Faccanoni
Dernière mise à jour : Jeudi 5 juin 2014 1 Approximations : polynômes de Taylor & développements limités

Propriété
B f = o(1) ⇐⇒ limx→x0 f (x) = 0
B f = o( f 1 ) et g = o(g 1 ) =⇒ f g = o( f 1 g 1 )
B f = o(h) et g = o(h) =⇒ f + g = o(h)
B f = o( f 1 ) =⇒ f n = o( f 1n ), n ∈ N
B f = o( f 1 ) =⇒ f α = o( f 1α ), α ∈ R si f et f 1 sont strictement positives au voisinage de x 0
B f ∼ g ⇐⇒ f − g = o(g )

Attention
B f = o( f 1 ) et g = o(g 1 ) ; f + g = o( f 1 + g 1 )
B f = o( f 1 ) ; e f = o(e f 1 )
B f = o( f 1 ) ; ln( f ) = o(ln( f 1 ))

Définition Asymptotes
Soit f définie sur un intervalle ]A, +∞[ (ou ]−∞, A[). On dit que f admet un développement limité généralisé à l’ordre n en
a = +∞ (ou en a = −∞) s’il existe un polynôme P tel que f (x) = P (1/x) + o(1/x n ) au voisinage de ce point. L’expression
P (1/x) n’est pas un polynôme, mais a des propriétés similaires. Dans la pratique, on peut obtenir un développement
limité de f (x) en a = +∞ (ou en a = −∞) en posant x = 1/t (de sorte que t tend vers 0, avec un signe fixe) et en tentant
de développer f (1/t ) au voisinage de 0.
Lorsque x et f (x) tendent vers l’infini, on obtient une asymptote oblique (si elle existe) en effectuant un développement
limité au voisinage de l’infini :
f (x) b c
µ ¶
1
= a + + k +o k
x x x x
où xck est le premier terme non nul après bx . Dans ce cas, la droite d’équation y = ax + b est asymptote à la courbe
représentative de f . Et la position relative de la courbe et de l’asymptote résulte du signe de xck lorsque x tend vers
l’infini.

Définition Équivalence
Soit f et g deux applications de D vers R et soit x 0 ∈ R ∪ { ±∞ }. On dit que f est équivalente à g au voisinage de x 0 s’il
existe un voisinage V de x 0 et une fonction ϕ : V → R vérifiant

∀x ∈ V ∩ D, f (x) = g (x)[1 + ϕ(x)], et lim ϕ(x) = 0.


x→x 0

On note alors f ∼ g .
x0
Si g ne s’annule pas dans un voisinage de x 0 , cette définition est équivalente à

f (x)
lim = 1.
x→x 0 g (x)

Exemple
B Soit P (x) = a0 + a1 x + a2 x 2 + · · · + an x n , (an 6= 0) une fonction polynomiale, alors
P (x) ∼ a n x n , P (x) ∼ a n x n , P (x) ∼ a 0 si a 0 6= 0.
+∞ −∞ 0

a0 + a1 x + a2 x 2 + · · · + an x n
B Soit F (x) = 6 0 et b p 6= 0) une fonction rationnelle, alors
, (a n =
b0 + b1 x + b2 x 2 + · · · + b p x p
an x n an x n a0
F (x) ∼ , F (x) ∼ , F (x) ∼ si a 0 , b 0 6= 0.
+∞ b p x p −∞ b p x p 0 b0

B Fonctions trigonométriques :
x2
sin(x) ∼ x, tan(x) ∼ x, 1 − cos(x) ∼ .
0 0 0 2
B Fonctions logarithmes, exponentielles, puissance :
ln(1 + x) ∼ x, e x − 1 ∼ x, a x − 1 ∼ x ln(a) (a > 0), (1 + x)α − 1 ∼ αx (α ∈ R).
0 0 0 0

© G. Faccanoni 11
1 Approximations : polynômes de Taylor & développements limités Dernière mise à jour : Jeudi 5 juin 2014

Théorème
Soit f et g deux applications de D vers R et soit x 0 ∈ R ∪ { ±∞ }. Si lim f (x) = ` ∈ R ∪ { ±∞ } et si f ∼ g , alors lim g (x) = `.
x→x 0 x0 x→x 0

Ce résultat est fondamentale car il permet de remplacer une limite par une limite plus simple.
Attention
Si deux fonctions ont même limite, elles ne sont pas nécessairement équivalentes. Par exemple, limx→+∞ x = limx→+∞ x 2
mais x 7→ x et x 7→ x 2 ne sont pas équivalentes en +∞.

Propriété
B f ∼ f 1 et g ∼ g 1 =⇒ f g ∼ f 1 g 1
B f ∼ f 1 =⇒ 1f ∼ f11 si f et f 1 ne s’annulent pas au voisinage de x 0
f f
B f ∼ f 1 et g ∼ g 1 =⇒ g ∼ g11 si g et g 1 ne s’annulent pas au voisinage de x 0
B f ∼ f 1 =⇒ f n ∼ f 1n , n ∈ N
B f ∼ f 1 =⇒ f α ∼ f 1α , α ∈ R si f et f 1 sont strictement positives au voisinage de x 0

Attention
B f ∼ f 1 et g ∼ g 1 ; f + g ∼ f 1 + g 1
B f ∼ f 1 ; e f ∼ e f1
B f ∼ f 1 ; ln( f ) ∼ ln( f 1 )

12 © G. Faccanoni
Dernière mise à jour : Jeudi 5 juin 2014 1 Approximations : polynômes de Taylor & développements limités

............... Exercices ..............


Exercice 1.1 Somme et produit de développements limités
Calculer les développements limités suivants au point 0 :

1 x
1.1−x − e à l’ordre 3, 4. cos(x) ln(1 + x) à l’ordre 4,
p p p
2. 1 − x + 1 + x à l’ordre 4, 5. (x 3 + 1) 1 − x à l’ordre 3,
3. sin(x) cos(2x) à l’ordre 5, 6. ln2 (x + 1) à l’ordre 4.

C ORRECTION .
¢ ³ 2 3
´ 2
1
− e x = 1 + x + x 2 + x 3 + o(x 3 ) − 1 + x + x2 + x6 + o(x 3 ) = x2 + 65 x 3 + o(x 3 )
¡
1. 1−x
p p ³ 2 x3 4
´ ³
x2 x3 5x 4
´
x2 x3
2. 1 − x + 1 + x = 1 − x2 − x8 − 16 − 5x 4 x 4 4
128 + o(x ) + 1 + 2 − 8 + 16 − 128 + o(x ) + o(x ) = 2 + x − 4 + 8 − 64 x + o(x )
5 4 4

2 (2x)4
³ 3
´³ ´
x5
3. sin(x) cos(2x) = x − x6 + 120 + o(x 5 ) 1 − (2x) 2 + 24 + o(x 5
) = x − 13 3 121 5 5
6 x + 120 x + o(x )
³ 2
´ ³ ´
x4 2 3 4 2 3
4. cos(x) ln(1 + x) = 1 − x2 + 24 + o(x 4 ) x − x2 + x3 + x4 + o(x 4 ) = x − x2 − x6 + o(x 4 )
p ³ 2 x3
´ 2
5. (x 3 + 1) 1 − x = (x 3 + 1) 1 − x2 − x8 − 16 + o(x 3 ) = 1 − x2 − x8 + 15 3
16 x + o(x )
3

³ 2 3
´³ 2 3
´
6. ln2 (x + 1) = (ln(x + 1)) (ln(x + 1)) = x − x2 + x3 + o(x 3 ) x − x2 + x3 + o(x 3 ) = x 2 − x 3 + 12 11 4
x + o(x 4 )

Exercice 1.2 Composition de développements limités


Calculer les développements limités suivants au point 0 :
³ ´
1. ln sin(x)
x à l’ordre 4, 2. e sin(x) à l’ordre 4, 3. (cos(x))sin(x) à l’ordre 5.

C ORRECTION .
3 5
x
x− x6 + 120 +o(x 5 ) 2 4 2 4
³ ´ 2
sin(x) sin(x)
1. x = x = 1 − x6 + 120
x
+ o(x 4 ). On note u = − x6 + 120
x
+ o(x 4 ) −−−→ 0, alors ln x = ln(1 + u) = u − u2 +
x→0
2
´2
x4
³
− x6 + 120 +o(x 4 ) x4
+o(x 4 )
³ 2 ´ ³ 2 ´
x4 x4 2 4 4
o(u 2 ) = − x6 + 120 + o(x 4 ) − 2 + o(x 4 ) = − x6 + 120 + o(x 4 ) − 36
2 + o(x 4 ) = − x6 + 120
x x
− 72 + o(x 4 ) =
2 4
− x6 − 180
x
+ o(x 4 )
x3 x3 u2 u3 u4
2. sin(x) = x − 6 + o(x 4 ). On note u = x − 6 + o(x 4 ) −−−→ 0, alors e sin(x) = e u = 1 + u + 2 + 6 + 24 + o(u 4 ). Comme
x→0
4
³ ´
3 4
³ ´ x 2 − x3
x3
u = x − x6 +o(x 4 ), alors u 2 = x 2 − x3 +o(x 4 ), u 3 3 4 4 4
= x +o(x ) et u = x +o(x ) et on trouve e 4 sin(x)
= 1+ x − 6 + 2 +
(x 3 ) + (x 4 ) + o(u 4 ) = 1 + x + x 2 − x 4 + o(x 4 )
6 24 2 8
2 4 2 4
3. (cos(x))sin(x) = exp ((sin(x)) ln(cos(x))). Comme cos(x) = 1 − x2 + 24
x
+ o(x 5 ), on pose u = − x2 + 24
x
+ o(x 5 ) −−−→ 0, alors
x→0
2 3 4 5 2 4 4
ln(cos(x)) = ln(1+u) = u − u2 + u3 − u4 + u5 +o(u 5 ). Comme u = − x2 + 24 x
+o(x 5 ), alors u 2 = x4 +o(x 5 ) et u 3 = u 4 = u 5 =
³ 4 ´2
³ 2 ´ x +o(x 5 )
x4 2 x4 3 x5
o(x 5 ), donc ln(cos(x)) = − x2 + 24
4
+ o(x 5 ) − 2 +o(u 5 ) = − x2 − 12 +o(x 5 ). Sachant que sin(x) = x− x6 + 120 +o(x 5 )
3 3
on en déduit (sin(x)) ln(cos(x)) = − x2 + o(x 5 ). On pose alors v = − x2 + o(x 5 ) −−−→ 0, et l’on a exp ((sin(x)) ln(cos(x))) =
x→0
2 3
exp(v) = 1 + v + v2 + o(v 2 ) = 1 − x2 + o(x 5 ).

Exercice 1.3 Division de développements limités


Calculer les développements limités suivants au point 0 :

sinh2 (t ) 3. tan(x) à l’ordre 5, 5. ln(1+x)


à l’ordre 3.
1. x 7→ 1−cos(t ) à l’ordre 4, sin(x)
1 sin(x)−1
2. 1+x+x 2
à l’ordre 4, 4. cos(x)+1 à l’ordre 2,

© G. Faccanoni 13
1 Approximations : polynômes de Taylor & développements limités Dernière mise à jour : Jeudi 5 juin 2014

Attention
Ne pas confondre la division selon les puissances croissantes avec la division euclidienne vue au premier semestre et qui
s’énonce comme suit :
Soient f et g deux polynômes avec g non nul et do g ≤ do f . Alors il existe un unique couple (Q, R) de poly-
nômes vérifiant
∀x ∈ C f (x) = g (x)Q(x) + R(x) et do R < do g .
Q s’appelle le quotient et R le reste. On dit que f est divisible par g si R = 0.

Exemple
Soient f (x) = x 3 + 2 et g (x) = x + 1. On a
f (x) R(x)
B f (x) = Q(x)g (x) + R(x) avec Q(x) = x 2 − x + 1 et R(x) = 1 calculés par division euclidienne, i.e. g (x) = Q(x) + g (x) ;
f (x)
B f (x) = (Q(x) + o(x 3 ))g (x) avec Q(x) = 2 − 2x + 2x 2 − x 3 calculé par division selon les puissances croissantes, i.e. g (x) = Q(x) + o(x 3 ).

C ORRECTION .
3 5 t2 4 6
1. sinh(t ) = t + t6 + 120
t
+ o(t 6 ) et 1 − cos(t ) = 2
t
− 24 t
+ 720 + o(t 7 ) donc
³ 3 5
´2
2 t + t6 + 120
t
+ o(t 6 ) t 2 + t3 + 2t 74 6
2 + 2t3 + 4t 5 2 4
sinh (t ) 45 + o(t ) 45 + o(t )
= = 2³ = .
t2 4 6 2 4
´
1 − cos(t ) t t t t2 t4 t t
2 − 24 + 720 + o(t 7 ) 2 1 − 12 + 360 + o(t )
5 1 − 12 + 360 + o(t 5 )

Maintenant on peux utiliser deux méthodes :


1 t2 t 4
1.1. la première utilise le développement limité de 1−A avec A = 12 − 360 + o(t 5 ) :
2 4
sinh2 (t ) 2 + 2t3 + 4t 5
45 + o(t )
=
1 − cos(t ) 1− A
2t 2 4t 4
µ ¶
+ o(t 5 ) 1 + A + A 2 + o(A 2 )
¡ ¢
= 2+ +
3 45
2t 2 4t 4 t2 t4 t4
µ ¶µ ¶
= 2+ + + o(t 5 ) 1 + − + o(t 5 ) +
3 45 12 360 144
5 2 11 4 4
= 2 + t + t + o(t );
6 72

1.2. la deuxième méthode (qu’on utilisera dans la suite de cet exercice) applique la technique de la division suivant
les puissances croissantes : si f et g ont au point a un développement limité à l’ordre n et si g (a) 6= 0, le polynôme
d’approximation de f /g à l’ordre n s’obtient en divisant suivant les puissances croissantes à l’ordre n le polynôme
2 4 t2 t4
d’approximation de f par celui de g . Ici f (x) = 2 + 2t3 + 4t
45 et g (x) = 1 − 12 + 360 et on pose la division suivant les
puissances croissantes en ne gardant que les termes de degré inférieur ou égal à 4 :

2 + 23 t 2 4 4
+ 45 t 1 1 2
− 12 t 1 4
+ 360 t

−2 + 16 t 2 1 4
− 180 t 2 + 65 t 2 + 11
72 t
4

+ 56 t 2 1 4
+ 12 t

− 56 t 2 5 4
+ 72 t

11 4
+ 72 t

11 4
− 72 t

sinh2 (t )
On conclut que 1−cos(t ) = 2 + 56 t 2 + 11 4 4
72 t + o(t ).

14 © G. Faccanoni
Dernière mise à jour : Jeudi 5 juin 2014 1 Approximations : polynômes de Taylor & développements limités

2. Ici f (x) = 1 et g (x) = 1 + x + x 2 et on pose la division suivant les puissances croissantes en ne gardant que les termes de
degré inférieur ou égal à 4 :

1 1 + x + x2
2
−1 −x −x 1 − x + x3 − x4
−x −x 2
x +x 2 +x 3
x3
−x 3 −x 4 +o(x 4 )
x4 +o(x 4 )
−x 4 +o(x 4 )
o(x 4 )

1
Le développement limité de 1+x+x 2
au point 0 à l’ordre 4 est donc 1 − x + x 3 − x 4 + o(x 4 )
sin(x) x− 16 x 3 + 120
1
x 5 +o(x 5 )
3. tan(x) = = 1
. Ici f (x) = x − 16 x 3 + 120 x 5 +o(x 5 ) et g (x) = 1+ 12 x 2 + 24
1 4
x +o(x 5 ) et on pose la division
cos(x) 1+ 12 x 2 + 24
1 4
x +o(x 5 )
suivant les puissances croissantes en ne gardant que les termes de degré inférieur ou égal à 5 :

x − 16 x 3 1
+ 120 x5 +o(x 5 ) 1 + 12 x 2 + 24
1 4
x + o(x 5 )
−x + 12 x 3 1 5
− 24 x +o(x 5 ) 1 3
x + 3 x + 15 x2 5
1 3 1 5
3x − 30 x +o(x 5 )
− 13 x 3 1 5
+6x +o(x 5 )
2 5
15 x +o(x 5 )
2 5
− 15 x +o(x 5 )
o(x 5 )

Le développement limité de tan(x) au point 0 à l’ordre 5 est donc x + 13 x 3 + 15


2 5
x + o(x 5 )
sin(x)−1 −1+x+o(x 2 )
4. = . Ici f (x) = −1 + x + o(x 2 ) et g (x) = 2 + 12 x 2 + o(x 2 ) et on pose la division suivant les puissances
cos(x)+1 2+ 12 x 2 +o(x 2 )
croissantes en ne gardant que les termes de degré inférieur ou égal à 2 :

−1 +x +o(x 2 ) 2 − 12 x 2 + o(x 2 )
+1 − 41 x 2 +o(x 2 ) − 21 + 12 x 2
x − 41 x 2 +o(x 2 )
−x +o(x 2 )
− 41 x 2 +o(x 2 )
− 41 x 2 +o(x 2 )
o(x 2 )

sin(x)−1
Le développement limité de cos(x)+1 au point 0 à l’ordre 2 est donc − 12 + 21 x 2 + o(x 2 )
x −1− 12 x+ 13 x 2 +o(x 2 )
¡ ¢
ln(1+x)
5. sin(x) = ¡ 1
2 2
¢ . Ici f (x) = −1 − 21 x + 13 x 2 + o(x 2 ) et g (x) = 1 − 61 x 2 + o(x 2 ) et on pose la division suivant les
x 1− 6 x +o(x )
puissances croissantes en ne gardant que les termes de degré inférieur ou égal à 2 :

−1 − 12 x + 31 x 2 +o(x 2 ) 1 − 16 x 2 + o(x 2 )
−1 + 16 x 2 +o(x 2 ) 1 − 12 x + 12 x 2
− 12 x + 21 x 2 +o(x 2 )
1
2x +o(x 2 )
1 2
2x +o(x 2 )
− 12 x 2 +o(x 2 )
o(x 2 )

ln(1+x)
Le développement limité de sin(x) au point 0 à l’ordre 2 est donc 1 − 12 x + 12 x 2 + o(x 2 ).

© G. Faccanoni 15
1 Approximations : polynômes de Taylor & développements limités Dernière mise à jour : Jeudi 5 juin 2014

Exercice 1.4
Développer dans le voisinage de zéro les fonctions suivantes :
p p 1
1. x 7→ 1 + x − 1 − x à l’ordre 3, 6. x 7→ (1 + sin(x)) x à l’ordre 2,
2. x 7→ ln(1 + 3x) à l’ordre 3, 7. x 7→ ex
à l’ordre 3,
2 cos(x)
3. x 7→ (1 − cos(x)) ln(1 + x ) à l’ordre plus bas,
cos(x)
4. x 7→ (sin(x 2 ))(e −x − 1) à l’ordre plus bas, 8. x 7→ ex à l’ordre 3,
x2 ex
5. x 7→ e x − 1 + sin(x) − 2x − 2 à l’ordre 4, 9. x 7→ (sin(x))/x à l’ordre 3.

C ORRECTION . Comme

x2 x4
cos(x) = 1 − + + o(x 5 ),
2 24
x3
sin(x) = x − + o(x 4 ),
6
x2 x3
ln(1 + x) = x − + + o(x 3 ),
2 3
x2
ex = 1 + x + + o(x 2 ),
2
p x x2 x3
1+x = 1+ − + + o(x 3 ),
2 8 16
p x x2 x3
1−x = 1− − − + o(x 3 ),
2 8 16
donc
p p ³ 2 x3
´ ³ 2 x3
´ 3
1. 1 + x − 1 − x = 1 + x2 − x8 + 16 + o(x 3 ) − 1 − x2 − x8 − 16 + o(x 3 ) = x + x8 + o(x 3 ) ;
2 (3x)3 2
2. ln(1 + 3x) = (3x) − (3x)
2 3 + o(x 3 ) = 3x − 9x2 + 9x 3 + o(x 3 ) ;
³ 2 ´³ ´
x4 4 4
3. (1 − cos(x)) ln(1 + x 2 ) = x2 − 24 + o(x 5 ) x 2 − x2 + o(x 4 ) = x2 + o(x 4 ) ;
³ 6
´³ 2
´
4. (sin(x 2 ))(e −x − 1) = x 2 − x6 + o(x 8 ) −x + x2 + o(x 2 ) = −x 3 + o(x 3 ).

5. À partir des développements limités des fonctions e x et sin(x) en 0 à l’ordre 4

x2 x3 x4
ex = 1 + x + + + + o(x 4 ),
2 6 24
x3
sin(x) = x − + o(x 4 ),
6
2 3 4 3 2 x4
on conclut que f (x) = 1 + x + x2 + x6 + 24
x
− 1 + x − x6 − 2x − x2 + o(x 4 ) = 24 + o(x 4 ).
6. On réécrit d’abord la fonction g (x) sous forme exponentielle :
1 −1 ln(1+sin(x))
(1 + sin(x)) x = e x .

On commence par déterminer le développement limité en 0 à l’ordre 2 de x 7→ x −1 ln(1 + sin(x)). Comme on divise par
x, il faut déterminer le développement limité à l’ordre 3 de x 7→ ln(1 + sin(x)). Or,

x3
sin(x) = x − + o(x 4 ),
6
donc
x3
µ ¶
ln(1 + sin(x)) = ln 1 + x − + o(x 4 ) .
6
3
On pose u = x − x6 + o(x 4 ). Comme limx→0 u = 0 et

u2 u3
ln(1 + u) = u − + + o(u 3 ),
2 3

16 © G. Faccanoni
Dernière mise à jour : Jeudi 5 juin 2014 1 Approximations : polynômes de Taylor & développements limités

on en déduit que
¶2 ¶3
x3 x3 x3 x2 x3
µ ¶ µ µ
1 1
ln(1 + sin(x)) = x − − x− + x− + o(x 3 ) = x − + + o(x 3 )
6 2 6 3 6 2 6

d’où
x x2
x −1 ln(1 + sin(x)) = 1 − + + o(x 2 ).
2 6
On a donc
x x2
−1 ln(1+sin(x)) +o(x 2 )
ex = ee − 2 + 6 .
x2
On pose v = − x2 + 6 + o(x 2 ). Comme limx→0 v = 0 et

v2
ev = 1 + v + + o(v 2 ),
2
on en déduit que
" ¶2 #
x x2 x x2 e
µ ¶ µ
1
x −1 ln(1+sin(x)) 1 7e
(1 + sin(x)) = ex = e 1+ − + + − + 2
+ o(x ) = e − x + x 2 + o(x 2 ).
2 6 2 2 6 2 24

7. À partir du DL donné dans le tableau cos(x) = 1 − 12 x 2 + o(x 3 ), on trouve le DL en zéro de 1


cos(x) (en utilisant le DL en
1
zéro de 1−x ) :
u= 12 x 2 +o(x 3 )
1 1 1
1 + u + o(u 2 ) = 1 + x 2 + o(x 3 ).
| {z }
= =
cos(x) 1 − 21 x 2 + o(x 3 ) 2

En multipliant ce résultat par le DL donné dans le tableau e x = 1 + x + 21 x 2 + 16 x 3 + o(x 3 ) on obtient finalement

2
1 + x + x 2 + x 3 + o(x 3 ).
3

8. À partir du DL donné dans le tableau e x = 1 + x + 21 x 2 + 61 x 3 + o(x 3 ), on trouve le DL en zéro de 1


ex (en utilisant le DL en
1
zéro de 1+x ):

u=x+ 21 x 2 + 61 x 3 +o(x 3 )
1 1 1
1 − u + u 2 + o(u 3 ) = 1 − x + x 2 + o(x 3 ).
| {z }
= =
e x 1 + x + 12 x 2 + 61 x 3 + o(x 3 ) 2

En multipliant ce résultat par le DL donné dans le tableau cos(x) = 1 − 12 x 2 + o(x 3 ) on obtient finalement

1
1 − x + x 3 + o(x 3 ).
3

9. À partir du DL donné dans le tableau cos(x) = 1 − 12 x 2 + o(x 3 ), on trouve le DL en zéro de 1


cos(x) (en utilisant le DL en
1
zéro de 1−x ) :
u= 12 x 2 +o(x 3 )
1 1 1
1 + u + o(u 2 ) = 1 + x 2 + o(x 3 ).
| {z }
= 1
=
cos(x) 1 − 2 x 2 + o(x 3 ) 2

En multipliant ce résultat par le DL donné dans le tableau e x = 1 + x + 21 x 2 + 16 x 3 + o(x 3 ) on obtient finalement

2
1 + x + x 2 + x 3 + o(x 3 ).
3

Exercice 1.5
Calculer le développement limité à l’ordre 3 au point 0 de
³ x ´
f (x) = arctan
x +2
de trois façons :
1. par la formule de Taylor-Young

© G. Faccanoni 17
1 Approximations : polynômes de Taylor & développements limités Dernière mise à jour : Jeudi 5 juin 2014

2. par composition de développements limités


3. en commençant par calculer le développement limité de f 0 .

C ORRECTION .
1. Pour utiliser la formule de Taylor-Young on doit d’abord calculer les dérivées de f jusqu’à l’ordre 3 en 0 :
³ x ´
f (x) = arctan f (0) = 0
x +2
1 2 1 1
f 0 (x) = = f 0 (0) =
x 2 (x + 2)2 x 2 + 2x + 2 2
¡ ¢
1+ x+2
x +1 −1
f 00 (x) = −2 f 00 (0) =
(x 2 + 2x + 2)2 2
3x 2 + 6x + 2 1
f 000 (x) = 2 f 000 (0) =
(x 2 + 2x + 2)3 2

d’où le développement limité

f 00 (0) 2 f 000 (0) 3 x x2 x3


f (x) = f (0) + f 0 (0)x + x + x + o(x 3 ) = − + + o(x 3 )
2 6 2 4 12
x
2. Comme u(x) ≡ −−→ 0, on a
x+2 −
x→0

(u(x))3
f (x) = arctan(u(x)) = u(x) − + o(u 3 )
3
x x3
= − + o(x 3 )
x + 2 3(x + 2)3
x 2 + 6x + 6
= 2x + o(x 3 )
3(x 3 + 6x 2 + 12x + 8)

En effectuant la division selon les puissances croissantes et en s’arrêtant à o(x 2 ) car le résultat sera multiplié par x on a

6 +6x +x 2 o(x 2 ) 8 + 12x + 6x 2 + x 3


−6 −9x − 29 x 2 o(x 2 ) 3 3 1 2
4 − 8x + 8x
−3x − 27 x 2 o(x 2 )
+3x + 29 x 2 o(x 2 )
x2 o(x 2 )
−x 2 o(x 2 )
o(x 2 )

ainsi

x ¡ x x2 x2
µ ¶
2x 3 3 1
− x + x 2 + o(x 3 ) = 6 − 3x + x 2 + o(x 3 ) = − + o(x 3 ).
¢
f (x) = +
3 4 8 8 12 2 4 12

1
3. f 0 (x) = x 2 +2x+2 et le le développement limité de f 0 au point 0 à l’ordre 2 peut être calculé par la division selon les
puissances croissantes :
1 o(x 2 ) 2 + 2x + x 2
1 2
−1 −x − 2 x o(x 2 ) 12 − 12 x + 14 x 2
1 2
−x − 2 x o(x 2 )
2
x +x o(x 2 )
1 2
2x o(x 2 )
1 2
−2x o(x 2 )
o(x 2 )
ainsi
x x x3
Z µ ¶
1 1 1
Z
f (x) = f 0 (x) dx = − x + x 2 dx + o(x 3 ) = − + + o(x 3 ).
2 2 4 2 4 12

18 © G. Faccanoni
Dernière mise à jour : Jeudi 5 juin 2014 1 Approximations : polynômes de Taylor & développements limités

Exercice 1.6
ln(1+sin2 (x))
1. Calculer le développement limité en x = 0 à l’ordre 3 de ln(1 + sin2 (x)). En déduire la limite lim 2x 2
.
x→0
ln(1+x cos2 (x))
2. Calculer le développement limité en x = 0 à l’ordre 3 de ln(1 + x cos2 (x)). En déduire la limite lim 4x .
x→0
ln(1−sin2 (x))
3. Calculer le développement limité en x = 0 à l’ordre 3 de ln(1 − sin2 (x)). En déduire la limite lim 2x 2
.
x→0
ln(1−x cos2 (x))
4. Calculer le développement limité en x = 0 à l’ordre 3 de ln(1 − x cos2 (x)). En déduire la limite lim 3x .
x→0
ln(1−sin2 (x))
5. Calculer le développement limité en x = 0 à l’ordre 3 de ln(1 − sin2 (x)). En déduire la limite lim 7x 2
.
x→0

C ORRECTION .
1. Étant donné que
x3
+ o x4
¡ ¢
sin(x) = x −
6
alors
sin2 (x) = x 2 + o x 4 .
¡ ¢

Comme x 2 + o x 4 −−−→ 0 et comme le développement limité de ln(1 + u) en u = 0 est


¡ ¢
x→0

u2 u3 u4 un
+ · · · + (−1)n+1 + o un
¡ ¢
ln(1 + u) = u − + −
2 3 4 n

on conclut que
ln(1 + sin2 (x)) = x 2 + o x 4 .
¡ ¢

Donc
ln(1 + sin2 (x)) x2 1
lim 2
= lim 2
= .
x→0 2x x→0 2x 2
2. Étant donné que
x2
+ o x4
¡ ¢
cos(x) = 1 −
2
alors
cos2 (x) = 1 − x 2 + o x 4 .
¡ ¢

Comme x cos2 (x) = x − x 3 + o x 4 −−−→ 0 et comme le développement limité de ln(1 + u) en u = 0 est


¡ ¢
x→0

u2 u3 u4 un
+ · · · + (−1)n+1 + o un
¡ ¢
ln(1 + u) = u − + −
2 3 4 n

on conclut que
x2 2 3
ln(1 + x cos2 (x)) = x − + x + o x4 .
¡ ¢
2 3
Donc
ln(1 + x cos2 (x)) x 1
lim = lim = .
x→0 4x x→0 4x 4
3. Étant donné que
x3
+ o x4
¡ ¢
sin(x) = x −
6
alors
sin2 (x) = x 2 + o x 4 .
¡ ¢

Comme sin2 (x) = x 2 + o x 4 −−−→ 0 et comme le développement limité de ln(1 − u) en u = 0 est


¡ ¢
x→0

u2 u3 u4 un
+ o un
¡ ¢
ln(1 − u) = −u − − − +···−
2 3 4 n

© G. Faccanoni 19
1 Approximations : polynômes de Taylor & développements limités Dernière mise à jour : Jeudi 5 juin 2014

on conclut que
ln(1 − sin2 (x)) = −x 2 + o x 4 .
¡ ¢

Donc
ln(1 − sin2 (x)) −x 2 1
lim = lim =− .
x→0 2x 2 x→0 2x 2 2
4. Étant donné que
x2
+ o x4
¡ ¢
cos(x) = 1 −
2
alors
cos2 (x) = 1 − x 2 + o x 4 .
¡ ¢

Comme x cos2 (x) = x − x 3 + o x 4 −−−→ 0 et comme le développement limité de ln(1 − u) en u = 0 est


¡ ¢
x→0

u2 u3 u4 un
+ o un
¡ ¢
ln(1 − u) = −u − − − +···−
2 3 4 n
on conclut que
x2 2 3
ln(1 − x cos2 (x)) = −x − + x + o x4 .
¡ ¢
2 3
Donc
ln(1 − x cos2 (x)) −x 1
lim = lim =− .
x→0 4x x→0 4x 4
5. Étant donné que
x3
+ o x4
¡ ¢
sin(x) = x −
6
alors
sin2 (x) = x 2 + o x 4 .
¡ ¢

Comme sin2 (x) = x 2 + o x 4 −−−→ 0 et comme le développement limité de ln(1 − u) en u = 0 est


¡ ¢
x→0

u2 u3 u4 un
+ o un
¡ ¢
ln(1 − u) = −u − − − +···−
2 3 4 n
on conclut que
ln(1 − sin2 (x)) = −x 2 + o x 4 .
¡ ¢

Donc
ln(1 − sin2 (x)) −x 2 1
lim 2
= lim =− .
x→0 7x x→0 7x 2 7

Exercice 1.7
1. Écrire le DL de cos(x) et de e 2x en 0 à l’ordre 2. En déduire le DL de cos(x)
e 2x
en 0 à l’ordre 2. Écrire le DL de ln(1 + u)
³ ´
cos(x)
en 0 à l’ordre 2. En déduire le DL de ln e 2x en 0 à l’ordre 2. Calculer

³ ´
cos(x)
ln e 2x
+ 2x
lim .
x→0 x2

2. Écrire le DL de cos(2x) et de e x en 0 à l’ordre 2. En déduire le DL de cos(2x)


ex en 0 à l’ordre 2. Écrire le DL de ln(1 + u)
³ ´
cos(2x)
en 0 à l’ordre 2. En déduire le DL de ln ex en 0 à l’ordre 2. Calculer

³ ´
cos(2x)
ln ex +x
lim .
x→0 x2

sin(2x)
3. Écrire le DL de sin(2x) et de e 2x en 0 à l’ordre 2. En déduire le DL de e 2x
en 0 à l’ordre 2. Écrire le DL de ln(1 + u)

20 © G. Faccanoni
Dernière mise à jour : Jeudi 5 juin 2014 1 Approximations : polynômes de Taylor & développements limités

³ ´
sin(2x)
en 0 à l’ordre 1. En déduire le DL de ln 2xe 2x
en 0 à l’ordre 1. Calculer
³ ´
sin(2x)
ln 2xe 2x
+ 2x
lim .
x→0 x

C ORRECTION .
2
1. cos(x) = 1 − x2 + o(x 2 ) et e 2x = 1 + 2x + 2x 2 + o(x 2 ) lorsque x ' 0. On pose la division suivant les puissances croissantes
en ne gardant que les termes de degré inférieur ou égal à 2 :

1 − 12 x 2 +o(x 2 ) 1 + 2x + 2x 2 + o(x 2 )
−1 −2x −2x 2 +o(x 2 ) 1 − 2x + 32 x 2
−2x − 52 x 2 +o(x 2 )
2x +4x 2 +o(x 2 )
3 2
2x +o(x 2 )
− 32 x 2 +o(x 2 )
o(x 2 )

2
cos(x) 1 − x2 + o(x 2 )
Donc = = 1 − 2x + 23 x 2 + o(x 2 ) lorsque x ' 0.
e 2x 1 + 2x + 2x 2 + o(x 2 ) ³ ´
2 cos(x)
ln(1 + u) = u − u2 + o(u 2 ) lorsque u ' 0 donc ln e 2x
= −2x − 12 x 2 + o(x 2 ) lorsque x ' 0. Notons que dans ce cas il
³ ´
cos(x) cos(x)
n’est pas nécessaire de calculer d’abord le DL de e 2x
mais il suffit le DL de cos(x) car ln e 2x
= ln(cos(x))−ln(e 2x ) =
³ 2
´ 2
ln(cos(x)) − 2x = ln 1 − x2 + o(x 2 ) − 2x = − x2 + o(x ) − 2x lorsque x ' 0.
2

Le DL précédent permet de calculer facilement la limite donnée :


³ ´
ln cos(x)
e 2x
+ 2x −2x − 12 x 2 + o(x 2 ) + 2x 1
lim = lim =− .
x→0 x2 x→0 x2 2

2. cos(2x) = 1 − 2x 2 + o(x 2 ) et e x = 1 + x + 21 x 2 + o(x 2 ) lorsque x ' 0. On pose la division suivant les puissances croissantes
en ne gardant que les termes de degré inférieur ou égal à 2 :

1 −2x 2 +o(x 2 ) 1 + x + 12 x 2 + o(x 2 )


−1 −x − 21 x 2 +o(x 2 ) 1 − x − 32 x 2
−x − 52 x 2 +o(x 2 )
x +x 2 +o(x 2 )
− 32 x 2 +o(x 2 )
3 2
2x +o(x 2 )
o(x 2 )

cos(2x) 1 − 2x 2 + o(x 2 )
Donc = = 1 − x − 23 x 2 + o(x 2 ) lorsque x ' 0.
ex 1 + x + 12 x 2 + o(x 2 )
2
³ ´
ln(1 + u) = u − u2 + o(u 2 ) lorsque u ' 0 donc ln cos(2x) e x = −x − 2x 2 + o(x 2 ) lorsque x ' 0. Notons que dans ce cas il n’est
³ ´
pas nécessaire de calculer d’abord le DL de cos(2x) ex mais il suffit le DL de cos(2x) car ln cos(2x)
ex = ln(cos(2x)) − ln(e x ) =
ln(cos(2x)) − x = ln(1 − 2x 2 + o(x 2 )) − x = −2x 2 + o(x 2 ) − x lorsque x ' 0.
Le DL précédent permet de calculer facilement la limite donnée :
³ ´
ln cos(x)
e 2x
+x −x − 2x 2 + o(x 2 ) + x
lim = lim = −2.
x→0 x2 x→0 x2

© G. Faccanoni 21
1 Approximations : polynômes de Taylor & développements limités Dernière mise à jour : Jeudi 5 juin 2014

3. sin(2x) = 2x + o(x 2 ) et e 2x = 1 + 2x + 2x 2 + o(x 2 ) lorsque x ' 0. On pose la division suivant les puissances croissantes en
ne gardant que les termes de degré inférieur ou égal à 2 :

2x +o(x 2 ) 1 + 2x + 2x 2 + o(x 2 )
−2x −4x 2 +o(x 2 ) 2x − 4x 2
−4x 2 +o(x 2 )
+4x 2 +o(x 2 )
o(x 2 )

sin(2x) 2x + o(x 2 )
Donc = = 2x − 4x 2 + o(x 2 ) lorsque x ' 0.
e 2x 1 + 2x + 2x 2 + o(x 2 )
2x−4x 2 +o(x 2 )
³ ´ ³ ´
sin(2x)
ln(1 + u) = u + o(u) lorsque u ' 0 donc ln 2xe 2x
= ln 2x = ln(1 − 2x + o(x)) = −2x + o(x) lorsque x ' 0.
Le DL précédent permet de calculer facilement la limite donnée :
³ ´
ln sin(2x)
2xe 2x + 2x −2x + o(x) + 2x
lim = lim = 0.
x→0 x x→0 x

Exercice 1.8 Développements limités au point x0 6= 0


Calculer les développements limités suivants :
p
1. x au point 2 à l’ordre 3, 7. x 3 au point x = −2 à l’ordre 4,
2. sin(x) au point π3 à l’ordre 3, 8. x 3 au point x = 2 à l’ordre 4,
3. cos(x) au point π6 à l’ordre 3,
ln(x) 9. x 3 au point x = −3 à l’ordre 4,
4. (1+x)2
en 1 à l’ordre 2.
3
5. x au point x = 1 à l’ordre 4, 10. x 4 − 1 au point −1 à l’ordre 3,

6. x 3 au point x = −1 à l’ordre 4, 11. x 4 + 1 au point 1 à l’ordre 3.

C ORRECTION . Soit f une fonction de R dans R et x 0 ∈ R. Si f et toutes ses dérivées jusqu’à l’ordre n sont définies et continues
en tout point d’un intervalle ouvert contenant x 0 alors f admet un développement limité à l’ordre n en x 0 donné par la
formule
n f (k) (x )
0
(x − x 0 )k + (x − x 0 )n ε(x − x 0 ).
X
f (x) =
k=0 k!
p p p q
1. On pose h = x − 2, alors x = h + 2 et on a x = 2 + h = 2 1 + h2 . On se rappelle que, pour u → 0, on peut écrire
p 2 3 p p p q p p p ³ 2 3
´
1 + u = 1 + u2 − u8 + u16 + o(u 3 ) ; ici u = h2 donc x = 2 + h = 2 1 + h2 = 2 1 + u = 2 1 + u2 − u8 + u16 + o(u 3 ) =
p ³ 2 h3
´ p ³
(x−2)2 (x−2)3
´
2 1 + h4 − h32 + 128 + o(h 3 ) = 2 1 + (x−2) 4 − 32 + 128 + o((x − 2) )
3

π
2. Pour calculer le développement limité de sin(x) en 3 à l’ordre 3 on utilise la formule de Taylor :
k=n f (k) (a)
(x − a)k + o((x − a)k ).
X
f (x) =
a
k=0 k!
π
Ici n = 3, a = 3 et f (x) = sin(x) d’où
p
3
f (x) = sin(x), f (a) = ;
2
1
f 0 (x) = cos(x), f 0 (a) = ;
2
p
3
f 00 (x) = − sin(x), f 00 (a) = − ;
2
1
f 000 (x) = − cos(x), f 000 (a) = − ;
2
donc p p ³
π´ π ´2 1 ³ π ´3 π ´3
µ³ ¶
3 1³ 3
sin(x) = + x− − x− − x− +o x − .
π/3 2 2 3 4 3 12 3 3

22 © G. Faccanoni
Dernière mise à jour : Jeudi 5 juin 2014 1 Approximations : polynômes de Taylor & développements limités

π
3. Pour calculer le développement limité de cos(x) en 6 à l’ordre 3 on utilise la formule de Taylor-Young :

k=n f (k) (a)


(x − a)k + o((x − a)k ).
X
f (x) =
a
k=0 k!

π
Ici n = 3, a = 6 et f (x) = cos(x) d’où
p
3
f (x) = cos(x), f (a) = ;
2
1
f 0 (x) = − sin(x), f 0 (a) = − ;
2
p
3
f 00 (x) = − cos(x), f 00 (a) = − ;
2
1
f 000 (x) = sin(x), f 000 (a) = ;
2
donc p p ³
π´ π ´2 1 ³ π ´3 π ´3
µ³ ¶
3 1³ 3
cos(x) = − x− − x− + x− +o x − .
π/6 2 2 6 4 6 12 6 6
ln(x) ln(1+y) ln(1+y)
4. On pose y = x − 1. Alors (1+x)2
= (2+y)2
. Comme limx→1 y = 0, on calcul le développement limité de (2+y)2
en zéro. Or,

1
ln(1 + y) = y − y 2 + o(y 2 ),
2
y ´−2 1 y (−2)(−2 − 1) y 2
µ ¶ µ ¶
1 ³
2 1 3 2 2
(2 + y)−2 = 1 + = 1−2 + + o(y ) = 1 − y + y + o(y ) ,
4 2 4 2 2 22 4 2

d’où µ ¶
1 3
1 − (x − 1) + (x − 1)2 + o((x − 1)2 ) .
4 2

5. n = 4, f (x) = x 3 et x 0 = 1 donc

(x − 1)0 (x − 1)1 (x − 1)2 (x − 1)3 (x − 1)4


f (x) = f (1) + f 0 (1) + f 00 (1) + f (3) (1) + f (4) (1)
0! 1! 2! 3! 4!
= 1 + 3(x − 1) + 3(x − 1)2 + (x − 1)3 .

6. n = 4, f (x) = x 3 et x 0 = −1 donc

(x + 1)0 (x + 1)1 (x + 1)2 (x + 1)3 (x + 1)4


f (x) = f (−1) + f 0 (−1) + f 00 (−1) + f (3) (−1) + f (4) (−1)
0! 1! 2! 3! 4!
= −1 + 3(x + 1) − 3(x + 1)2 + (x + 1)3 .

7. n = 4, f (x) = x 3 et x 0 = −2 donc

(x + 2)0 (x + 2)1 (x + 2)2 (x + 2)3 (x + 2)4


f (x) = f (−2) + f 0 (−2) + f 00 (−2) + f (3) (−2) + f (4) (−2)
0! 1! 2! 3! 4!
= −8 + 12(x + 2) − 6(x + 2)2 + (x + 2)3 .

8. n = 4, f (x) = x 3 et x 0 = 2 donc

(x − 2)0 (x − 2)1 (x − 2)2 (x − 2)3 (x − 2)4


f (x) = f (2) + f 0 (2) + f 00 (2) + f (3) (2) + f (4) (2)
0! 1! 2! 3! 4!
= 8 + 12(x − 2) + 6(x − 2)2 + (x − 2)3 .

9. n = 4, f (x) = x 3 et x 0 = −3 donc

(x + 3)0 (x + 3)1 (x + 3)2 (x + 3)3 (x + 3)4


f (x) = f (−3) + f 0 (−3) + f 00 (−3) + f (3) (−3) + f (4) (−3)
0! 1! 2! 3! 4!
2 3
= −27 + 27(x + 3) − 9(x + 3) + (x + 3) .

© G. Faccanoni 23
1 Approximations : polynômes de Taylor & développements limités Dernière mise à jour : Jeudi 5 juin 2014

10. Selon la formule de Taylor, le développement limité à l’ordre n au point x 0 de f peut être calculé selon la formule
k
f (x 0 ) (x−x 0)
Pk=n (k)
DL n (x 0 ) = k=0 k! + o((x − x 0 )n ). Ici f (x) = x 4 − 1, x 0 = −1 et

f (0) (x) = x 4 − 1 f (0) (−1) = 0,


f (i ) (x) = 4x 3 f (i ) (−1) = −4,
f (i i ) (x) = 12x 2 f (i i ) (−1) = 12,
f (i i i ) (x) = 24x f (i i i ) (−1) = −24,

donc

(x + 1)0 (x + 1)1 (x + 1)2 (x + 1)3


x 4 − 1 = f (0) (−1) + f (i ) (−1) + f (i i ) (−1) + f (i i i ) (−1) + o((x + 1)3 )
0! 1! 2! 3!
1 (x + 1) (x + 1)2 (x + 1)3
= 0 + (−4) + 12 + (−24) + o((x + 1)3 )
1 1 2 6
= −4(x + 1) + 6(x + 1)2 − 4(x + 1)3 + o((x + 1)3 ).

11. Selon la formule de Taylor, le développement limité à l’ordre n au point x 0 de f peut être calculé selon la formule
k
f (x 0 ) (x−x 0)
Pk=n (k)
DL n (x 0 ) = k=0 k! + o((x − x 0 )n ). Ici f (x) = x 4 + 1, x 0 = 1 et

f (0) (x) = x 4 + 1 f (0) (1) = 2,


f (i ) (x) = 4x 3 f (i ) (1) = 4,
f (i i ) (x) = 12x 2 f (i i ) (1) = 12,
f (i i i ) (x) = 24x f (i i i ) (1) = 24,

donc

(x − 1)0 (x − 1)1 (x − 1)2 (x − 1)3


x 4 + 1 = f (0) (1) + f (i ) (1) + f (i i ) (1) + f (i i i ) (1) + o((x − 1)3 )
0! 1! 2! 3!
1 (x − 1) (x − 1)2 (x − 1)3
= 2 +4 + 12 + 24 + o((x − 1)3 )
1 1 2 6
= 2 + 4(x − 1) + 6(x − 1)2 + 4(x − 1)3 + o((x − 1)3 ).

Exercice 1.9 Développements limités généralisés


Calculer les développements limités suivants :
p
6. f (x) = e − /x en −∞ à l’ordre 3,
1
x+2
1. f (x) = p
x
en +∞ à l’ordre 3,
x
p
7. f (x) = e /x en +∞ à l’ordre 3,
4
2. f (x) = 2
x−1 x + 1 en −∞ et en +∞ à l’ordre 1,
2
/x
3. f (x) = e en +∞ à l’ordre 3, x 2 −x
8. f (x) = 1+x en +∞ à l’ordre 2,
/x
3
4. f (x) = e en +∞ à l’ordre 3, p p
3+x+x 2 − 2−x+x 2
5. f (x) = e − /x en −∞ à l’ordre 3,
2
9. f (x) = x en +∞ à l’ordre 2.

C ORRECTION . Soit f définie sur un intervalle ]A, +∞[ (ou ]−∞, A[). On dit que f admet un développement limité généralisé
à l’ordre n en a = +∞ (ou en a = −∞) s’il existe un polynôme P tel que f (x) = P (1/x)+o(1/x n ) au voisinage de ce point. L’ex-
pression P (1/x) n’est pas un polynôme, mais a des propriétés similaires. Dans la pratique, on peut obtenir un développement
limité de f (x) en a = +∞ (ou en a = −∞) en posant x = 1/t (de sorte que t tend vers 0, avec un signe fixe) et en tentant de
développer f (1/t ) au voisinage de 0.
p q q p
x+2 x+2 (2u)2 (2u)3
1. On pose u = x1 , ainsi u −−−−−→ 0. Alors p
x
= x = 1 + x2 = 1 + 2u = 1 + 2u 3 1 1
2 − 8 + 16 + o(u ) = 1 + x − 2x 2 +
x→+∞
1
2x 3
+ o(x −3 )
1
2. B Développement limité asymptotique en +∞ : on pose y = x et on remarque que limx→+∞ y = 0+ . Alors
s
1 y 1 + y 2 (y>0) 1
q
+
` (y) = = 1 + y2
y 1− y y2 y(1 − y)

24 © G. Faccanoni
Dernière mise à jour : Jeudi 5 juin 2014 1 Approximations : polynômes de Taylor & développements limités

y2
µ ¶
1¡ 2 2 1 3
+ o(y ) = + 1 + y + o(y 2 )
2
¢
= 1 + y + y + o(y ) 1 +
y 2 y 2

d’où
31
+ o(x −1 ). `(x) = x + 1 +
2x
On en déduit que le graphe de la fonction ` admet la droite d’équation y = x + 1 comme asymptote en +∞.
B Développement limité asymptotique en −∞ : on pose encore y = x1 et on remarque que limx→−∞ y = 0− . Alors
s
1 y 1 + y 2 (y<0) 1
q

` (y) = = − 1 + y2
y 1− y y2 y(1 − y)
y2
µ ¶
1¡ 1 3
= − 1 + y + y 2 + o(y 2 ) 1 + + o(y 2 ) = + 1 + y + o(y 2 )
¢
y 2 y 2

d’où
31
+ o(x −1 ).
`(x) = −x − 1 −
2x
On en déduit que le graphe de la fonction ` admet la droite d’équation y = −x − 1 comme asymptote en −∞.
3. f (x) = e /x , n = 3 et a = +∞ donc
2

y2 y3 4t 2 8t 3
µ ¶
2 2 4 1
e /x = e 2t = e y = 1 + y + + o y 3 = 1 + 2t + +o t3 = 1+ + 2 + 3 +o 3 .
2 ¡ ¢ ¡ ¢
+ +
2 6 2 6 x x 3x x

4. f (x) = e /x , n = 3 et a = +∞ donc
3

y2 y3 9t 2 27t 3
µ ¶
3 9 9 1
e /x = e 3t = e y = 1 + y + + o y 3 = 1 + 3t + +o t3 = 1+ + 2 + 3 +o 3 .
3 ¡ ¢ ¡ ¢
+ +
2 6 2 6 x 2x 2x x

5. f (x) = e − /x , n = 3 et a = −∞ donc
2

y2 y3 t2 t3
µ ¶
−2/x −t y
¡ 3¢ ¡ 3¢ 2 2 4 1
e =e = e = 1+ y + + +o y = 1−t + − +o t = 1− + 2 − 3 +o 3 .
2 6 2 6 x 2x 3x x

6. f (x) = e − /x , n = 3 et a = −∞ donc
1

y2 y3 t2 t3
µ ¶
1 1 1 1
e − /x = e −t = e y = 1 + y + + o y3 = 1 − t + − + o t3 = 1 − + 2 − 3 + o 3 .
1 ¡ ¢ ¡ ¢
+
2 6 2 6 x 2x 6x x

7. f (x) = e /x , n = 3 et a = ∞ donc
4

y2 y3 t2 t3
µ ¶
4 8 32 1
e /x = e t = e y = 1 + y + + o y3 = 1 + t + + + o t3 = 1 + + 2 + 3 + o 3 .
4 ¡ ¢ ¡ ¢
+
2 6 2 6 x x 3x x
1−y
8. On pose y = x1 . Il s’agit de calculer le développement limité en zéro à l’ordre 3 de la fonction r (y) = y(1+y) .
1−y 1
Or, 1+y = (1− y)(1− y + y 2 − y 3 +o(y 3 )) donc r (y) = y −2y +2y 2 −2y 3 +o(y 3 ) et finalement f (x) = x −2+ x2 − x22 +o(x −2 ).
9.
Ãr r !
1 1 1 1
f (1/t ) = t 3+ + 2 − 2− + 2
t t t t
p p
= 3t 2 + t + 1 − 2t 2 − t + 1
= (1 − A) /2 − (1 − B ) /2 avec A = −t − 3t 2 −−−→ 0, B = t − 2t 2 −−−→ 0
1 1

t →0 t →0
µ ¶ µ ¶
1 1 2 1 1
= 1 − A + A + o(A 2 ) − 1 − B + B 2 + o(A 2 )
2 8 2 8
µ ¶ µ ¶
1 3 2 1 2 2 1 2 1 2 2
= 1 + t + t + t + o(t ) − 1 − t + t + t + o(t )
2 2 8 2 8
1 2
= t + t + o(t 2 )
2
1 1
= + 2 + o(1/x 2 ).
x 2x

© G. Faccanoni 25
1 Approximations : polynômes de Taylor & développements limités Dernière mise à jour : Jeudi 5 juin 2014

Exercice 1.10 Calculs de limites


Calculer les limites suivantes
³ ´
1−cos(x) sin(x)(tanh x−x) x(cosh(x)+cos(x)−2)
1. lim 9. lim ln(1+x) 18. lim
x→0 sin(x) x→0 x→0 sinh(x)+sin(x)−2x
³ ´ 1
arctan x−sin(x) (1+x) x −e sinh(x)+sin(x)−2x
2. lim 10. lim x 19. lim
x→0 tan x−arcsin x x→0 x→0 x(cosh(x)+cos(x)−2)
3
p p
³ ´x 2 11. lim x3 + x + 1 − x2 + x 20. lim x(cosh(x)+cos(x)−2)
3. lim cosh x12 x→+∞
x→0 tanh(x)+tan(x)−2x
x→+∞ x−sin(x)
12. lim 2 x ln(x)
cos(x)−e x x→0 e x −1−x− x2
³ ´
4. lim arcsin 21. lim 2
x−x 2 3 x→1 x −1
x→0
13. lim 2(tan x−sin(x))−x
x5
p p p
x→0
lim 3x+1+x 2 +x−2
2−x− 2x+7
³ ´
5. 1
lim x−sin(x) + x−sinh 1 22.
a
x→0 x 14. lim sin(x)−sin
x−a
x→1
³ ´ x→a p p p
x+2− 7−2x
6. lim x12 ln sin(x) 15. lim x−x cos(x) 23. lim 1−3x+x 2 −x−2
x x→−1
x→0 x→0 x−sin(x)
¢x π p
3
p
lim 1 + x7 16. limπ (cos(x)) 2 −x
¡
7. 24. lim x3 + x + 1 − x2 + x
x→+∞ x→ 2 x→+∞
³ ´ x −e −x −2x 1−cos(2x)
1
8. lim x1 − ln(1+x) 17. lim e x−sin(x) 25. lim x
x→0 x→0 x→0 e −1−x

C ORRECTION .
2
1−cos(x) 1−1+ x2 +o(x 2 ) x
2 +o(x)
1. lim = lim 2 = lim =0
x→0 sin(x) x→0 x+o(x ) x→0 1+o(x)
3 3
³ ´ ³ ´
3
x−sin(x) x− x3 +o(x 3 ) − x− x6 +o(x 3 ) − x6 +o(x 3 )
2. lim arctan = lim 3 3 = lim x3
= −1
x→0 tan x−arcsin x
³ ´ ³ ´
x→0 x+ x3 +o(x 3 ) − x+ x6 +o(x 3 ) x→0 6 +o(x 3 )
´´x 2 µ u +e −u ¶
ln( e )
³ ³ ³ ´
1 1 1/u ln(cosh(u)) 2
3. On pose u = x2
, alors limx→+∞
cosh = lim (cosh(u)) = lim expx2 u = lim exp u
u→0 u→0 u→0
µ ¡ u2 2
¢¶ µ u 2 2

ln 1+ 2 +o(u ) +o(u )
= lim exp u2 + o(u) = 1
¡ ¢
= lim exp u = lim exp 2 u
u→0 u→0 u→0
2 2
³ ´ ³ ´
cos(x)−e x 1− x2 +o(x 2 ) − 1+x+ x2 +o(x 2 )
4. lim 2 = lim = lim −1−x+o(x)
(x+o(x))−x 2
= −1
x→0 arcsin x−x x→0 x→0 1+x+o(x)
³ ´ ³ ´
1 1 3 3 6 1
5. lim x−sin(x) + x−sinh x = lim x 3 +o(x 3 ) + x 3 +o(x 3 ) = lim x 3 +o(x 3 ) n’existe pas car −−−−→ ±∞
x 3 x→0±
x→0 x→0 x→0
3
x− x6 +o(x 3 )
³ 2
´
6. lim 1
2 ln sin(x)
x = lim = lim x12 ln 1 − x6 + o(x 2 ) = lim ln(1+u+o(u))
1
2 ln x −6u = lim u+o(u) = − 16
x→0 x x→0 x x→0 u→0 u→0 −6u
¢x 2 2 ))
³ ´ ³ ´ ³ ´
lim 1 + x7 = lim (1 + 7u) /u = lim exp ln(1+7u) = lim exp 7 ln(1+v) = lim exp 7(v−v /2+o(v
¡ 1
7. u v v
x→+∞ u→0 u→0 v→0 v→0
= lim exp (7(1 − v/2 + o(v))) = e 7
v→0
2
x− x2 +o(x 2 )−x
³ ´
1 1 ln(1+x)−x 1 1
8. Comme x − ln(1+x) = x ln(1+x) = x 2 +o(x 2 )
d’où lim − ln(1+x) = − 12 .
x→0 x
9. En déterminant les développements limité en 0 à l’ordre 3 des fonctions x 7→ sin(x), x 7→ ln(1 + x) et x 7→ tanh(x) on
obtient
3 3
sin(x)(tanh(x) − x) (x + o(x))(− x3 + o(x 3 )) (1 + o(1))(− x3 + o(x 3 ))
lim = lim = lim = 0.
x→0 ln(1 + x) x→0 x + o(x) x→0 1 + o(1)
2
1 ln(1+x) 1 x− x2 +o(x 2 ) x
10. Comme (1 + x) x = e x et en utilisant le développement limité de ln(1 + x) en 0 on a (1 + x) x = e x = e 1− 2 +o(x)
1
(1+x) x −e
d’où lim x = − 2e .
x→0
p
3
p
11. On note f (x) = x 3 + x + 1 − x 2 + x. On réécrit d’abord la fonction comme
"µ ¶1 µ ¶1 #
1 1 3 1 2
f (x) = x 1 + 2 + 3 − 1 + .
x x x

On pose y = x1 . Alors
1 1
p
3
p (1 + y 2 + y 3 ) 3 − (1 + y) 2
lim x3 + x + 1 − x2 + x = lim .
x→+∞ y→0 y

26 © G. Faccanoni
Dernière mise à jour : Jeudi 5 juin 2014 1 Approximations : polynômes de Taylor & développements limités

En utilisant le développement limité de (1 + y)α en 0 on a


1 1 y
(1 + y 2 + y 3 ) 3 − (1 + y) 2 1 + o(y) − 1 − 2 + o(y) 1
lim = lim =− .
y→0 y y→0 y 2

12. En utilisant les développements limités on a :


3
x − sin(x) x − x + x6 + o(x 3 )
lim 2
= lim 2 3 2
=
x→0 e x − 1 − x − x2 x→0 1 + x + x2 + x6 − 1 − x − x2 + o(x 3 )
x3
+ o(x 3 )
= lim 63 = 1.
x→0 x + o(x 3 )
6

13. En utilisant les développements limités on a :


³ 3 5
´
x3 x5
2(tan(x) − sin(x)) − x 3 2 x + x3 + 2x
15 − x + 6 − 120 + o(x 5
) − x3
lim = lim =
x→0 x5 x→0 x5
x5
4 + o(x 5 ) 1
= lim = .
x→0 x5 4
sin(x)−sin(a)
14. lim x−a = cos(a). (On s’aperçoit que c’est la définition de la dérivée de sin(x) en x = a.)
x→a
15. En utilisant les développements limités on a :
³ 2
´
x − x cos(x) x − x 1 − x2 + o(x 2 )
lim = lim =
x→0 x − sin(x) x→0 x − x + x 3 + o(x 3 )
6
x3
+ o(x 3 )
= lim 23 = 3.
x→0 x + o(x 3 )
6

π
16. On pose y = 2 − x, alors
π
lim (cos(x) 2 −x = lim (cos( π2 − y)) y =
x→ π2 y→0

= lim (sin(y) y = lim e y ln(sin(y)) =


y→0 y→0
µ ¶ µ ¶
y3 y2
y ln y− 6 +o(y 3 ) y ln y(1− 6 +o(y 2 ))
= lim e = lim e =
y→0 y→0
µ ¶ µ ¶
y2 2 )) y2
y ln y y ln 1− 6 +o(y y ln y y − 6 +o(y 2 )
= lim e e = lim e e =
y→0 y→0
y3
+o(y 3 )
= lim e y ln y e − 6 = 1.
y→0

17. Pour calculer la limite donnée on peut utiliser les développements limités :
x2 3 2 3
e x − e −x − 2x 1+x + 2 + x6 − 1 + x − x2 + x6 − 2x + o(x 3 )
lim = lim 3
=
x→0 x − sin(x) x→0 x − x + x6 + o(x 3 )
3
2 x6 + o(x 3 )
= lim = 2.
x→0 x 3 + o(x 3 )
6

Sinon on peut utiliser la règle de L’Hôpital car

n(x) = e x − e −x − 2x, lim n(x) = 0;


x→0
d (x) = x − sin(x), lim d (x) = 0.
x→0

On a

n 0 (x) = e x + e −x − 2, lim n 0 (x) = 0;


x→0

© G. Faccanoni 27
1 Approximations : polynômes de Taylor & développements limités Dernière mise à jour : Jeudi 5 juin 2014

d 0 (x) = 1 − cos(x), lim d 0 (x) = 0;


x→0
n 00 (x) = e x − e −x , lim n 00 (x) = 0;
x→0
d 00 (x) = sin(x), lim d 00 (x) = 0;
x→0
n 000 (x) = e x + e −x , lim n 00 (x) = 2;
x→0
d 000 (x) = cos(x), lim d 00 (x) = 1;
x→0

donc
e x − e −x − 2x n 000 (x)
lim = lim 000 = 2.
x→0 x − sin(x) x→0 d (x)

3 5 3 5
18. sinh(x) + sin(x) − 2x = x + x6 + 120
x
+ x − x6 + 120
x
− 2x + o(x 5 )
2 4 2 4
x
cosh(x) + cos(x) − 2 = 1 + x3 + 24 + 1 − x2 + 24
x
− 2 + o(x 4 )
1
sinh(x)+sin(x)−2x 60 +o(1)
Donc lim = lim 1 = 15 .
x→0 x(cosh(x)+cos(x)−2) x→0 12 +o(1)
2 4 x2 4
x
19. cosh(x) + cos(x) − 2 = 1 + x3 + 24 +1− 2
x
+ 24 − 2 + o(x 4 )
3 5 3 5
sinh(x) + sin(x) − 2x = x + x6 + 120
x
+ x − x6 + 120
x
− 2x + o(x 5 )
1
x(cosh(x)+cos(x)−2) 12 +o(1)
Donc lim = lim 1 = 5.
x→0 sinh(x)+sin(x)−2x x→0 60 +o(1)
2 4 x2 4 x4
x
20. cosh(x) + cos(x) − 2 = 1 + x2 + 24 +1− 2
x
+ 24 − 2 + o(x 4 ) = 12 + o(x 4 ),
x3 2x 5 x3 2x 5 5 4x 5
tanh(x) + tan(x) − 2x = x − 3 + 15 +x + 3 + 15 − 2x + o(x ) = 15 + o(x 5 ),
1
x(cosh(x)+cos(x)−2) 12 +o(1) 5
Donc lim = lim 4 = 16 .
x→0 tanh(x)+tan(x)−2x x→0 15 +o(1)
µ ¶
y2 y3
(1+y) y− 2 + 3 +o(y 3 ) y y2
(1+y) ln(1+y) 1+ 2 − +o(y 2 )
21. On pose y = x − 1 Alors lim x ln(x)
2 = lim y(2+y) = lim y(2+y) = lim 6
y = 21 .
x→1 x −1 y→0 y→0 y→0 2(1+ 2 )
p p p
22. Soit f (x) = 3x + 1 + 2 − x − 2x + 7 et g (x) = x 2 + x − 2. f et g sont continues sur [0; 2], dérivables sur ]0; 2[ et f (1) =
g (1) = 0. On a

3 1 1
f 0 (x) = p − p −p , g 0 (x) = 2x + 1
2 3x + 1 2 2 − x 2x + 7
f 0 (x) 1 1 f (x)
d’où lim g 0 = − 36 . On en déduit que lim g (x) = − 36 .
x→1 (x) x→1
p p p
23. Soit f (x) = 1 − 3x + x + 2 − 7 − 2x et g (x) = x 2 − x − 2. f et g sont continues sur [−2; 0], dérivables sur ] − 2; 0[ et
f (−1) = g (−1) = 0. On a

3 1 1
f 0 (x) = − p + p +p , g 0 (x) = 2x − 1
2 1 − 3x 2 x + 2 7 − 2x
f 0 (x) 1 f (x) 1
d’où lim 0 = − 36 . On en déduit que lim = − 36 .
x→1 g (x) x→1 g (x)
24. On commence par factoriser le terme dominant (comme on s’intéresse à la limite lorsque x tend vers +∞, on peut
supposer x > 0) :
s µ ¶ s µ ¶ "r r #
p
3
p
3 1 1 1 3 1 1 1
f (x) = x3 + x + 1 − x2 + x = x3 2
1+ 2 + 3 − x 1+ =x 1+ 2 + 3 − 1+ .
x x x x x x

On peut alors écrire

t t
· ¸ · µ ¶¸ · ¸
1 ¡ ¢1 1 1 1 1
g (t ) = f (1/t ) = 1 + t 2 + t 3 3 − (1 + t ) 2 = (1 + o(t )) − 1 + + o(t ) = − + o(t ) = − + o(1).
t t 2 t 2 2

Donc lim f (x) = − 12 .


x→+∞

28 © G. Faccanoni
Dernière mise à jour : Jeudi 5 juin 2014 1 Approximations : polynômes de Taylor & développements limités

25. Pour calculer la limite donnée on peut utiliser les développements limités :
2
1 − cos(2x) 1 − 1 + (2x)
2 + o(x )
2
lim x = lim =
x→0 e − 1 − x x→0 1 + x + x 2 − 1 − x + o(x 2 )
2
2x 2 + o(x 2 )
= lim = 4.
x→0 x 2 + o(x 2 )
2

Sinon on peut utiliser la règle de l’H ÔPITAL car

n(x) = 1 − cos(2x), lim n(x) = 0;


x→0
d (x) = e x − 1 − x, lim d (x) = 0.
x→0

On a

n 0 (x) = 2 sin(2x), lim n 0 (x) = 0;


x→0
d 0 (x) = e x − 1, lim d 0 (x) = 0;
x→0
n 00 (x) = 4 cos(2x), lim n 00 (x) = 4;
x→0
d 00 (x) = e x , lim d 00 (x) = 1;
x→0

donc
1 − cos(2x) n 00 (x)
lim x
= lim 00 = 4.
x→0 e − 1 − x x→0 d (x)

Exercice 1.11
Écrire
B le développement limité de sin(x) en 0 à l’ordre 3 ;
B le développement limité de e 2x en 0 à l’ordre 3.
Puis calculer
B le développement limité de f (x) = sin(x) × e 2x en 0 à l’ordre 3.
B le développement limité de g (x) = sin(x)
e 2x
en 0 à l’ordre 3.

C ORRECTION . On commence par écrire


B le développement limité de sin(x) en 0 à l’ordre 3 :

x3
sin(x) = x − + o(x 3 );
0 6
B le développement limité de e 2x en 0 à l’ordre 3 :

(2x)2 (2x)3 4
e 2x = 1 + (2x) + + + o(x 3 ) = 1 + 2x + 2x 2 + x 3 + o(x 3 );
0 2 6 3
On peut alors facilement calculer
B le développement limité de f (x) = e 2x sin(x) en 0 à l’ordre 3 :

x3
· ¸ · ¸
3 2 4 3 3
f (x) = x − + o(x ) × 1 + 2x + 2x + x + o(x ) =
0 6 3
x3
=x− + 2x 2 + 2x 3 + o(x 3 ) =
6
11
= x + 2x 2 + x 3 + o(x 3 );
6

B le développement limité de g (x) = sin(x)


e 2x
en 0 à l’ordre 3 :

3 u=2x+2x 2 + 34 x 3 +o(x 3 )
x − x6 + o(x 3 ) | {z }
g (x) = =
0 1 + 2x + 2x 2 + 34 x 3 + o(x 3 )

© G. Faccanoni 29
1 Approximations : polynômes de Taylor & développements limités Dernière mise à jour : Jeudi 5 juin 2014

3
x − x6 + o(x 3 )
= =
1+u
x3
· ¸
+ o(x 3 ) 1 − u + u 2 − u 3 + o(u 3 ) =
£ ¤
= x−
6
x3
· ¸· ¸
3 2 4 3 2 4 3 2 2 4 3 3 3
= x− + o(x ) 1 − (2x + 2x + x ) + (2x + 2x + x ) − (2x + 2x + x ) + o(x ) =
6 3 3 3
3
x
· ¸· ¸
4
= x− + o(x 3 ) 1 − 2x − 2x 2 − x 3 + 4x 2 + x 3 − 8x 3 + o(x 3 ) =
6 3
3
x
· ¸
+ o(x 3 ) 1 − 2x + 2x 2 − 8x 3 + o(x 3 ) =
£ ¤
= x−
6
11
= x − 2x 2 + x 3 + o(x 3 ).
6

Exercice 1.12
p
On considère la fonction f (x) = x + rx 2 + x.
³ ´
B Écrire le développement limité de 1 + x1 en ±∞ à l’ordre 3.
B En déduire le développement asymptotique de f à l’ordre 2 en −∞ et en +∞.
B Écrire les équations des asymptotes pour f en −∞ et en +∞.

p
C ORRECTION . On considère la fonction f (x) = x + x 2 + x. On remarque que
 ³ q ´
x 1 + 1 + 1 , si x ≥ 0,
x
f (x) = ³ q ´
x 1 − 1 + 1 , si x < 0.
x

q
B Pour écrire le développement limité de 1 + x1 en ±∞ à l’ordre 3 on commence par se ramener à un développement limité
p
en 0 en posant y = 1/x : il s’agit alors de calculer le développement limité de 1 + y en 0 à l’ordre 3

1 1
1 + y = 1 + y − y 2 + o(y 2 )
p
0 2 8
d’où
r
1 1 1
1+ = 1+ − + o(x −2 )
x ∞ 2x 8x 2
B On en déduit le développement asymptotique de f à l’ordre 2
(i) en −∞ : µ ¶
1 1 −2 1 1
f (x) = x − + + o(x ) = − + + o(x −1 )
−∞ 2x 8x 2 2 8x
(ii) en +∞ : µ ¶
1 1 −2 1
f (x) = x 2 + − + o(x ) = 2x + 1 − + o(x −1 )
+∞ 2x 8x 2 4x
B On en déduit les équations des asymptotes pour f en
(i) en −∞ :
1
y =−
2
(ii) en +∞ :
y = 2x + 1.
En effet, le graphe de la fonction est le suivant :

30 © G. Faccanoni
Dernière mise à jour : Jeudi 5 juin 2014 1 Approximations : polynômes de Taylor & développements limités

−1 0
x

−1

Exercice 1.13
¡1¢
1. Écrire le développement limité généralisé de cos 2x en +∞ à l’ordre 3.
q
2. Écrire le développement limité généralisé de 2 + x1 en +∞ à l’ordre 3.
¡1¢
cos 2x
3. En déduire le développement limité généralisé de f (x) = q en +∞ à l’ordre 3.
2 + x1
4. Écrire l’équation de l’asymptote de f en +∞.

C ORRECTION . Soit f définie sur un intervalle ]A, +∞[. On dit que f admet un développement limité généralisé à l’ordre n
en a = +∞ s’il existe un polynôme P tel que f (x) = P (1/x)+o(1/x n ) au voisinage de ce point. L’expression P (1/x) n’est pas un
polynôme, mais a des propriétés similaires. Dans la pratique, on peut obtenir un développement limité de f (x) en a = +∞
en posant x = 1/t (de sorte que t tend vers 0, avec un signe fixe) et en tentant de développer f (1/t ) au voisinage de 0.
1
1. Soit t = −−−−→ 0. Alors
2x −
x→+∞

t2
µ ¶
1 1
= cos(t ) = 1 − + o t 3 = 1 − 2 + o x −3
¡ ¢ ¡ ¢
cos
2x 2 8x
1
2. Soit t = −−−−→ 0. Alors
2x −
x→+∞

t t2 t3
r r
1 p p p p p
µ ¶ µ ¶
1 1 1 1
+o t3 = 2 1+
¡ ¢ ¡ −3 ¢
2+ = 2 1+ = 2 1+t = 2 1+ − + − + + o x
x 2x 2 8 16 4x 32x 2 128x 3
1
3. Soit t = −−−−→ 0. Alors
2x −
x→+∞
¡1¢ 2
1 − t2 + o t 3
¡ ¢
cos 2x
p q =p ³ 2 ¡ ¢´
t3
1
2 1 + 2x 2 1 + 2t − t8 + 16 +o t3

Comme
1 − 12 t 2 o(t 3 ) 1 + 12 t − 18 t 2 + 16
1 3
t + o(t 3 )

−1 − 12 t + 18 t 2 1 3
− 16 t o(t 3 ) 1 − 12 t − 18 t 2 − 16
1 3
t + o(t 3 )

− 12 t − 38 t 2 1 3
− 16 t o(t 3 )

1
2t + 14 t 2 1 3
+ 32 t o(t 3 )

− 18 t 2 1 3
− 32 t o(t 3 )

1 2 1 3
8t + 16 t o(t 3 )

1 3
16 t o(t 3 )

1 3
− 16 t o(t 3 )

o(t 3 )

© G. Faccanoni 31
1 Approximations : polynômes de Taylor & développements limités Dernière mise à jour : Jeudi 5 juin 2014

alors
2
1 − t2 + o t 3
¡ ¢ µ ¶ µ ¶
1 1 1 2 1 3 3 1 1 1 1 −3
p ³ = p 1 − t − t − t + o(t ) = p 1 − − − + o(x )
4x 32x 2 128x 3
2
´
t3 2 8 16
2 1 + 2t − t8 + 16 2 2
¡ ¢
+o t3

4. L’équation de l’asymptote de f en +∞ est y = p1 .


2

Exercice 1.14
¡1¢
1. Écrire le développement limité généralisé de sin 2x en +∞ à l’ordre 3.
q
1
2. Écrire le développement limité généralisé de 1 + 2x en +∞ à l’ordre 3.
¡1¢
x sin 2x
3. En déduire le développement limité généralisé de f (x) = q en +∞ à l’ordre 3.
1
1 + 2x
4. Écrire l’équation de l’asymptote de f en +∞.

C ORRECTION . Soit f définie sur un intervalle ]A, +∞[. On dit que f admet un développement limité généralisé à l’ordre n
en a = +∞ s’il existe un polynôme P tel que f (x) = P (1/x)+o(1/x n ) au voisinage de ce point. L’expression P (1/x) n’est pas un
polynôme, mais a des propriétés similaires. Dans la pratique, on peut obtenir un développement limité de f (x) en a = +∞
en posant x = 1/t (de sorte que t tend vers 0, avec un signe fixe) et en tentant de développer f (1/t ) au voisinage de 0.
1
1. Soit t = −−−−→ 0. Alors
2x −
x→+∞

t3
µ ¶
1 1 1
= sin(t ) = t − + o t 4 = + o x −4
¡ ¢ ¡ ¢
sin − 3
2x 6 2x 48x
1
2. Soit t = −−−−→ 0. Alors
2x −
x→+∞

t t2 t3
r
1 p 1 1 1
+o t3 = 1+ + o x −3
¡ ¢ ¡ ¢
1+ = 1+t = 1+ − + − 2
+ 3
2x 2 8 16 4x 32x 128x
1
3. Soit t = −−−−→ 0. Alors
2x −
x→+∞
3 2
t − t6 + o t 4 1 − t6 + o t 3
¡1¢ ¡ ¢ ¡ ¢
x sin 2x sin (t )
= p = ³ ¡ ¢´ = 2 3
2t 1 + t 2t 1 + t − t 2 + t 3 + o t 3 2 + t − t4 + t8 + o t 3
q ¡ ¢
1
1 + 2x 2 8 16

Comme
1 − 16 t 2 o(t 3 ) 2 + t − 41 t 2 + 18 t 3 + o(t 3 )

−1 − 12 t − 18 t 2 1 3
− 16 t o(t 3 ) 1
2 − 14 t + 48
5 2
t − 11 3 3
96 t + o(t )

− 12 t 5 2
+ 24 t 1 3
− 16 t o(t 3 )

1
2t + 14 t 2 1 3
− 16 t o(t 3 )

5 2
24 t − 18 t 3 o(t 3 )

5 2 5 3
− 24 t − 48 t o(t 3 )

11 3
− 48 t o(t 3 )

11 3
48 t o(t 3 )

o(t 3 )

alors
2
1 − t6 + o t 3
¡ ¢
1 1 5 2 11 3 3 1 1 5 11
¡ ¢ = − t + t − t + o(t ) = − + 2
− 3
+ o(x −3 )
t2 t3 3 2 4 48 96 2 8x 192x 768x
2+t − 4 + 8 +o t

32 © G. Faccanoni
Dernière mise à jour : Jeudi 5 juin 2014 1 Approximations : polynômes de Taylor & développements limités

4. L’équation de l’asymptote de f en +∞ est y = 12 .

Exercice 1.15
a) Écrire le développement limité de sin(2x) et de cos(2x) en 0 à l’ordre 3 ; en déduire le développement limité de
sin(2x) 1
tan(2x) = cos(2x) en 0 à l’ordre 3 soit en utilisant le développement limité en u = 0 de la fonction u 7→ 1+u soit en
appliquant la technique de la division suivant les puissances croissantes.
Suggestion : comme on connait le développement limité de tan(x) en 0 à l’ordre 3, il est facile de calculer le développe-
ment limité de tan(2x) et de vérifier qu’on a bien fait les calculs.
b) En utilisant le développement limité de ln(1 + z) en 0 à l’ordre¡3 et le développement
¢ limité de tan(2x) en 0 à l’ordre 3
calculé au point a), déterminer le développement limité de ln 1 + tan(2x) en 0 à l’ordre 3.
¡ ¢
c) En utilisant le développement limité de arcsin(x) en 0 à l’ordre 3 et le développement limité de ln 1 + tan(2x) en 0 à
l’ordre 3 calculé au point b), calculer ¡ ¢
ln 1 + tan(2x) + 2x(x − 1)
lim .
x→0 arcsin(x) − x

C ORRECTION .
3 2
a) Comme sin(x) = x − x6 + o(x 3 ) et cos(x) = 1 − x2 + o(x 3 ), les développements limités de sin(2x) et cos(2x) en 0 à l’ordre 3
sont

(2x)3 4x 3 (2x)2
sin(2x) = (2x) − + o((2x)3 ) = 2x − + o(x 3 ), cos(2x) = 1 − + o((2x)3 ) = 1 − 2x 2 + o(x 3 );
6 3 2
sin(2x)
on peux alors utiliser deux méthodes pour calculer le développement limité de tan(2x) = cos(2x) en 0 à l’ordre 3 :
1
B la première méthode utilise le développement limité de 1+u :

3 u=−2x 2 +o(x 3 )
2x − 4x3 + o(x 3 ) | {z }
g (x) = =
1 − 2x 2 + o(x 3 )
3
2x − 4x3 + o(x 3 )
= =
1+u
4x 3
· ¸
+ o(x ) 1 − u + u 2 − u 3 + o(u 3 ) =
3
£ ¤
= 2x −
3
4x 3
· ¸
+ o(x ) 1 − (−2x 2 ) + (−2x 2 )2 − (−2x 2 )3 + o(x 3 ) =
3
£ ¤
= 2x −
3
4x 3
· ¸
+ o(x ) 1 + 2x 2 + o(x 3 ) =
3
£ ¤
= 2x −
3
8
= 2x + x 3 + o(x 3 ).
3
B la deuxième méthode applique la technique de la division suivant les puissances croissantes : si f et g ont au point a
un développement limité à l’ordre n et si g (a) 6= 0, le polynôme d’approximation de f /g à l’ordre n s’obtient en divisant
3
suivant les puissances croissantes à l’ordre n le polynôme d’approximation de f par celui de g . Ici f (x) = 2x − 4x3 et
2
g (x) = 1 − 2x et on pose la division suivant les puissances croissantes en ne gardant que les termes de degré inférieur
ou égal à 3 :

2x − 34 x 3 1 −2x 2

8 3
−2x +4x 3 2x 3x

8 3
3x

− 83 x 3

Le développement limité de tan(2x) à l’ordre 3 en x = 0 est donc 2x + 83 x 3 + o(x 3 ).

© G. Faccanoni 33
1 Approximations : polynômes de Taylor & développements limités Dernière mise à jour : Jeudi 5 juin 2014

3
B Pour vérifier nos calculs on va utiliser le développement limité de tan(z) : comme tan(z) = z + z3 + o(x 3 ), le développe-
8x 3
ment limité de tan(2x) en 0 à l’ordre 3 est bien tan(2x) = 2x + 3 + o(x 3 ).
b) Étant donné que les développements limités en u = 0 à l’ordre 3 de ln(1 + u) et tan(2u) sont
u2 u3 8u 3
ln(1 + u) = u − + + o(u 3 ), tan(2u) = 2u + + o(u 3 ),
2 3 3
on en déduit le développement limité en x = 0 à l’ordre 3 de ln(1 + tan(2x)) :
 

8x 3
 
3 
 
ln(1 + tan(2x)) = ln 1 + 2x +
 + o(x )
 | 3 {z }
=z −−−→0
x→0

= ln(1 + z)
z2 z3
=z− + + o(z 3 )
2 3
3
si on remplace z = 2x + 8x3 + o(x 3 ) on a

3 2 3 3
³ ´ ³ ´
8x 3 2x + 8x3 2x + 8x3
= 2x + − + + o(x 3 )
3 2 3
8x 3 4x 2 8x 3
= 2x + − + + o(x 3 )
3 2 3
16
= 2x − 2x 2 + x 3 + o(x 3 ).
3
c) Étant donné que les développements limités en u = 0 à l’ordre 3 de arcsin(u) et ln(1 + tan(2u)) sont
u3 16 3
arcsin(u) = u + + o(u 3 ) ln(1 + tan(2u)) = 2u − 2u 2 + u + o(u 3 )
6 3
on a
ln(1 + tan(2x)) + 2x(x − 1) 2x − 2x 2 + 16 3 3 2
3 x + o(x ) + 2x − 2x
16 3
3 x + o(x )
3
lim = lim 3
= lim 3
= 32.
x→0 arcsin(x) − x x→0 x + x6 + o(x 3 ) − x x→0 x 3
6 + o(x )

Exercice 1.16
¡ 1−x ¢ln(x)
1. Soit f (x) := 1+x . Quel est son domaine de définition ? Est-elle prolongeable par continuité en 0 ?
¡ 1+x ¢ln(x)
2. Soit f (x) := 1−x . Quel est son domaine de définition ? Est-elle prolongeable par continuité en 1 ?

C ORRECTION .
1. Pour que f soit bien définie il faut que, pour tout x ∈ R, x vérifie le système suivant

1 + x 6= 0,

1−x
 1+x > 0,
x > 0,

donc D f =]0; 1[. De plus


¶ln(x)
1−x
µ ¡ 1−x ¢ ¡ 1−x ¢
lim f (x) = lim = lim e ln(x) ln 1+x = e limx→0+ ln(x) ln 1+x
x→0+ x→0+ 1+x x→0+

et
¡ 1−x ¢ ¡ 1+x ¢ −(1+x)−(1−x)
ln 1+x 1−x (1−x)2 −2x ln2 (x)(1 + x)
lim = lim 2
= lim
x→0+ 1/ ln(x) x→0+ 1/(x ln (x)) x→0+ (1 − x)3
−2(1 + x)
µ ¶µ ¶
2
= lim lim x ln (x) = −2 lim x ln2 (x) = 0
x→0+ (1 − x)3 x→0+ x→0+

donc
lim f (x) = e 0 = 1.
x→0+

34 © G. Faccanoni
Dernière mise à jour : Jeudi 5 juin 2014 1 Approximations : polynômes de Taylor & développements limités

2. Pour que f soit bien définie il faut que, pour tout x ∈ R, x vérifie le système suivant

1 + x 6= 0,

1+x
 1−x > 0,
x > 0,

donc D f =]0; 1[. De plus


¶ln(x)
1+x
µ ¡ 1+x ¢ ¡ 1+x ¢
= lim− e ln(x) ln = e limx→1 ln(x) ln 1−x

lim− f (x) = lim− 1−x
x→1 x→1 1−x x→1

et
¡ 1+x ¢ ¡ 1−x ¢ (1−x)+(1+x)
ln 1−x 1+x (1−x)2 2x ln2 (x)
lim− = lim− 2
= lim−
x→1 1/ ln(x) x→1 1/(x ln (x)) x→1 (1 − x 2 )
ln2 (x)
µ ¶µ ¶
−2x 2 ln(x)
= lim− lim− = −1 lim− =0
x→1 1 + x x→1 1 − x x→1 −x

donc
lim f (x) = e 0 = 1.
x→1−

Attention
On sait que, pour qu’une fonction admette un développement limité d’ordre n au voisinage de 0, il est suffisante qu’elle
soit n fois dérivable en 0. Les exercices suivants montrent que ce n’est pas nécessaire.

Exercice 1.17
Soit f la fonction définie par (
2 + 3x + 4x 2 + x 3 sin x1
¡ ¢
si x 6= 0,
f (x) =
2 si x = 0.

Montrez que f admet, au voisinage de 0, un développement limité d’ordre 2 et que, pourtant, f 00 (0) n’existe pas.

¡1¢
C ORRECTION . Puisque limx→0 x sin x = 0, on peut écrire

f (x) = 2 + 3x + 4x 2 + o(x 2 )

ce qui, par définition, montre que f admet un développement limité d’ordre 2 au voisinage de 0.
D’autre part,
3 + 8x + 3x 2 sin x1 − x cos x1
( ¡ ¢ ¡ ¢
0
si x 6= 0,
f (x) = f (x)− f (0) ¡1¢
lim x−0 = lim 3 + 4x + x 2 sin x = 3 si x = 0.
x→0 x→0

Mais f 00 (0) n’existe pas car le quotient


f 0 (x) − f 0 (0)
µ ¶ µ ¶
1 1
= 8 + 3x sin − cos
x −0 x x
n’a pas de limite quand x tend vers 0.

Exercice 1.18
Soit f : R → R une application définie par
 3 ¡1¢
x sin x , si x > 0,

f (x) = 0, si x = 0,

 3
x ln|x|, si x < 0.
1. Montrer que f est continue en 0.

© G. Faccanoni 35
1 Approximations : polynômes de Taylor & développements limités Dernière mise à jour : Jeudi 5 juin 2014

2. Calculer f 0 (x) pour x 6= 0, puis f 0 (0).


3. Étudier la continuité de f 0 en 0.
4. Est-ce que f ∈ C 1 (R) ?
5. Calculer la dérivée seconde f 00 (0) si elle existe.
6. Peut-on utiliser la formule de TAYLOR-Y OUNG pour écrire le développement limité de f en 0 à l’ordre 2 ? Pourquoi ?
7. Écrire le développement limité de f en 0 à l’ordre 2 s’il existe.

C ORRECTION . L’application f : R → R se réécrit


 3 ¡1¢
x sin x , si x > 0,

f (x) = 0, si x = 0,

 3
x ln(−x), si x < 0.

1. f est continue en 0 car µ ¶


31
lim f (x) = lim x sin = 0 = f (0)
x→0+ x→0+ x
et
lim f (x) = lim− x 3 ln(−x) = 0 = f (0).
x→0− x→0

2. Comme
x 3 sin x1 − 0
¡ ¢
f (x) − f (0)
lim = lim =0
x→0+ x −0 x→0+ x −0
et
f (x) − f (0) x 3 ln(−x) − 0
lim− = lim− =0
x→0 x −0 x→0 x −0
la dérivée de f est l’application f : R → R définie par
0

 2 ¡1¢ ¡1¢
3x sin x − x cos x , si x > 0,

f 0 (x) = 0, si x = 0,

 2 2
3x ln(−x) + x , si x < 0.

3. f 0 est continue en x = 0 car µ ¶ µ ¶


1 1
lim f 0 (x) = lim 3x 2 sin − x cos = 0 = f 0 (0)
x→0+ x→0+ x x
et
lim f 0 (x) = lim 3x 2 ln(−x) + x 2 = 0 = f 0 (0),
x→0− x→0+

donc f (x) = f (0) + x f 0 (0) + o(x) = o(x).


4. f ∈ C 1 (R) car elle est continue, dérivable et sa dérivée est continue.
5. La dérivée seconde de f en x = 0 n’existe pas car
¡1¢ ¡1¢
f 0 (x) − f 0 (0) 3x 2 sin − x cos −0
µ µ ¶ µ ¶¶
x x 1 1
lim = lim = lim 3x sin − cos
x→0+ x −0 x→0+ x −0 x→0 + x x

n’existe pas.
6. On ne peut pas utiliser la formule de TAYLOR-Y OUNG pour écrire le développement limité de f en 0 à l’ordre 2 car f 00 (0)
n’existe pas.
7. Puisque limx→0+ x sin x1 = limx→0− x ln |x| = 0, on peut écrire
¡ ¢

 ¡1¢
x sin x
 si x > 0,
f (x) = x 2 ε(x) = o(x 2 ), avec ε(x) = 0 si x = 0,

x ln |x| si x < 0

ce qui, par définition, montre que f admet un développement limité d’ordre 2 au voisinage de 0.

36 © G. Faccanoni
Dernière mise à jour : Jeudi 5 juin 2014 1 Approximations : polynômes de Taylor & développements limités

Exercice 1.19 Gravitation et approximation


À l’altitude z, la norme du champ de gravitation de la Terre est la fonction

g : R+ → R+
GM
z 7→
(R + z)2

où G est la constante de gravitation, M la masse de la Terre et R le rayon de la Terre. Si z ¿ R, donner en fonction de z et


R une expression approchée de g (z) en utilisant un développement limité au second ordre en z/R.

C ORRECTION . Posons u = z/R ' 0 si z ¿ R. On a g̃ (u) = (1 + u)−2G M /R 2 . Le développement limité de g̃ à l’ordre 2 au


voisinage de 0 est
GM ¡
g̃ (u) = 2 1 − 2u + 3u 2 + o(u 2 ) .
¢
R
Une expression approchée de g pour z ¿ R est alors

GM ¡ 2
R − 2zR + 3z 2 + o(z 2 ) .
¢
g (z) =
R4

Exercice 1.20
TAYLOR en 0 à l’ordre n de k x . En déduire le polynôme de TAYLOR
Soit k une constante positive. Calculer le polynôme de p
k x −1 m
en 0 à l’ordre n − 1 de x . En déduire ensuite que m( k − 1) ' ln(k) lorsque m est grand.

C ORRECTION .
dn(k x ) dn(e cx )
B k x = e c x avec c = ln(k) donc dx n = dx n = c n e c x = c n k x . On a alors

n ln j (k)
kx ' xj
X
si x ' 0.
j =0 j !

kx − 1 P n
ln j (k) j −1
Par conséquence
x
' j! x si x ' 0 (attention aux indices de la somme).
j =1
B En posant m = x1 on trouve
³p ´ X n ln j (k)
m
m k −1 ' j −1
si m → +∞.
j =1 j !m
p
m
Pour n = 1 on a alors m( k − 1) ' ln(k).

Exercice 1.21 `
x

Considérons un secteur circulaire de rayon r et angle 2α,


avec α petit.
B Exprimer x en fonction de cos(α). Utiliser un dévelop-
pement limité en 0 à l’ordre 3 de cos(α) pour appro-
cher x par un polynôme en α et r .
r −x

B En approchant la Terre par une sphère de rayon r =


r

6360 km, utiliser le résultat ainsi obtenu pour estimer α


de combien un arc de ` = 100 km s’écarte de sa corde
(i.e. estimer x).

C ORRECTION .
r − x = r cos(α)
et
cos(0) − sin(0) − cos(0) sin(0) 1
cos(α) ' (α − 0)0 + (α − 0)1 + (α − 0)2 + (α − 0)3 = 1 − α2 lorsque α ' 0
0! 1! 2! 3! 2

© G. Faccanoni 37
1 Approximations : polynômes de Taylor & développements limités Dernière mise à jour : Jeudi 5 juin 2014

donc
r α2
x' lorsque α ' 0.
2
` 100 5 6360×1002 125
Si ` = 100 km alors α = r = 6360 = 318 ' 0 donc x ' 2×63602
= 159 ' 0.79 km.

38 © G. Faccanoni
2 Fonctions de plusieurs variables
Qu’il s’agisse de traiter des questions relatives à la biologie, la chimie, la physique, la production, la consommation ou encore
l’environnement, etc. une modélisation adéquate s’exprime le plus souvent à l’aide de fonctions de plusieurs variables. Ce
chapitre introduit les fonctions de plusieurs variables réelles en élargissant les définitions énoncées dans le module M11
pour les fonctions d’une variable réelle. Nous prenons le parti de privilégier les thèmes qui s’écartent des notions vues pour
les fonctions d’une seule variable. À l’opposé, les définitions et les propriétés qui apparaissent comme des généralisations
évidentes sont évoquées ou présentées brièvement.

2.1 Introduction
Les fonctions de plusieurs variables sont naturelles, par exemple
B la température dépend de la latitude, de la longitude et du temps :

T : R3 → R
(x, y, t ) 7→ T (x, y, t )

B le coût d’une brochure publicitaire dépend de son format (A4, A5), du nombre de pages, du nombre de couleurs utilisées. . .

Définition Fonction de plusieurs variables


B Une fonction f , définie sur une partie D de Rn et à valeurs réelles, fait correspondre à tout point x ≡ (x 1 , x 2 , . . . , x n ) de
D un réel unique f (x).
B Le domaine de définition de f est l’ensemble©D ⊂ R¯n .
B L’image par f de D est l’ensemble Im¯f (D) =ª r ∈ R ¯ r = f (x), x ∈ Rn ⊂ R.
ª

B L’ensemble des points S = (x, f (x)) ¯ x ∈ D de Rn+1 est la surface représentative de f ; c’est l’analogue de la courbe
©

représentative d’une fonction d’une variable.


x ∈ Rn se note aussi ~
x ou x. Si n = 2, on utilise souvent la notation (x, y), si n = 3 la notation (x, y, z).

Exemple
p
Le domaine de la fonction f (x, y) = x + y est donné par D = (x, y) ∈ R2 ¯ x + y = 0 . Il se représente donc naturellement comme une
© ¯ ª
2
portion du plan R . En outre, les valeurs prises par la fonction parcourent tout l’ensemble des réels positifs ou nuls : Im f (D) = R+ .
y
D

Exemple d’applications à la gestion


1. La production P d’une entreprise est souvent exprimée en fonction de deux facteurs synthétiques, le capital, noté K , et le travail,
noté W : P = f (K ,W ).
2. L’utilité U d’un consommateur dépend de ses quantités consommées. En présence de n biens, la fonction d’utilité s’exprime
sous la forme U = f (x 1 , x 2 , . . . , x n ), où x i désigne la quantité consommée du i -ème bien disponible (i = 1, . . . , n).

Évidemment, la représentation géométrique devient plus lourde que pour les fonctions d’une seule variable : une fonction
de n variables se visualise à priori dans un espace à n + 1 dimensions (n pour les variables, 1 pour le résultat de la fonction),
alors que les pages d’un livre sont, par nature, bidimensionnelles. Pour contourner cette impossibilité technique, nous nous
limiterons aux représentations des fonctions de deux variables, soit sous forme de dessins en perspective, soit sous forme
de coupes par des plans horizontaux ou verticaux qui donnent des informations souvent utiles, quoique parcellaires. Ce
problème de visualisation introduit une rupture nette par rapport aux fonctions d’une variable étudiées antérieurement.

39
2 Fonctions de plusieurs variables Dernière mise à jour : Jeudi 5 juin 2014

2.2 Représentations de fonctions de deux variables


Lorsque n = 2, le graphe
G f ≡ (x, y, z = f (x, y)) ¯ (x, y) ∈ D
© ¯ ª

est tridimensionnel. On peut considérer le graphe d’une fonction de deux variables comme étant le relief d’une région (par
exemple, l’altitude en fonction de la longitude et de la latitude).
Les axes relatifs aux variables, x et y, sont conventionnellement situés dans un plan horizontal (le domaine D apparaît alors
comme un sous-ensemble de ce plan), tandis que la dimension verticale est réservée aux valeurs de z. Ainsi, à tout (x, y) ∈ D,
dont l’image est f (x, y) ∈ R, correspond le point suivant du graphe : (x, y, f (x, y)) ∈ R3 . Une mise en perspective permet
la visualisation des surfaces à trois dimensions. Dans ce cas, l’axe z est toujours placé verticalement. Toutefois, pour des
raisons de lisibilité, les axes x et y ne sont pas toujours présentés selon la même orientation.

B Équation du plane x y : z = 0 ;
B Équation du plane xz : y = 0 ;
B Équation du plane y z : x = 0.

2.2.1 Représentation surfacique


On visualise le graphe d’une fonction

f : R2 → R
(x, y) 7→ f (x, y)

par l’altitude z = f (x, y).

Exemple
1.

2. Le graphe de la fonction f : R2 → R définie par f (x, y) = x 2 − y 2 est une surface de R3 qui a la forme d’une selle de cheval, comme
l’indique la représentation en perspective de la figure ci-dessous.

40 © G. Faccanoni
Dernière mise à jour : Jeudi 5 juin 2014 2 Fonctions de plusieurs variables

2.2.2 Fonctions partielles

Les fonctions partielles associées à f : R2 → R sont des fonctions de R dans R données par l’intersection de la surface repré-
sentative de f avec des plans «verticaux parallèles aux axes».

Définition Fonctions partielles


Soit f une fonction de R2 dans R et (a, b) ∈ D le domaine de définition de f . Les fonctions

fb : R → R fa : R → R
x 7→ f (x, b) y 7→ f (a, y)

définies sur un intervalle ouvert contenant respectivement a et b, sont appelées les fonctions partielles associées à f au
point (a, b).

Exemple
Grâce aux fonctions partielles on peut deviner la représentation surfacique des fonctions f : R2 → R suivantes :
2,0

1,5

1,0

0,5
-2,6

-1,6 2,4
x
( -0,6 1,4

f b (x) = 1 pour tout b ∈ R


0,0
0,4 0,4
-0,6
1,4
-1,6
2,4

1. si f (x, y) = 1 alors donc la représentation surfacique de f est -2,6


y

f a (y) = 1 pour tout a ∈ R

(
f b (x) = x 2 pour tout b ∈ R
1
-2,6 -2,6
-1,6 -1,6

2. si f (x, y) = x 2 alors
0
-0,6 -0,6

donc la représentation surfacique de f est y


1,4
0,4
0,4
1,4
x

f a (y) = a 2 pour tout a ∈ R

12,5

10,0

7,5

5,0
-2,6

-2,6
y x -1,6
2,5
-1,6
(
f b (x) = x 2 + b 2 pour tout b ∈ R
-0,6
-0,6
0,0 0,4
0,4

3. si f (x, y) = x 2 + y 2 alors
1,4

donc la représentation surfacique de f est 1,4 2,4

f a (y) = y 2 + a 2 pour tout a ∈ R


2,4

© G. Faccanoni 41
2 Fonctions de plusieurs variables Dernière mise à jour : Jeudi 5 juin 2014

2.2.3 Représentation planaire

Les nuances de gris d’une photo noir et blanc sont la représentation d’une fonction définie sur un rectangle à valeurs dans
l’intervalle [0; 1] : 0 noir, 1 blanc. On parle de représentation planaire.

Exemple
Représentation planaire et surfacique de la fonction

f : R2 → R
(x, y) 7→ x 2 + y 2

5,0

2,5

-5,0
y 0,0
-2,5
y

0,0

50 2,5
-2,5
40 5,0

-5,0
30
-2,5
20 -5,0
0,0
-5,0 -2,5 0,0 2,5 5,0
10 x
2,5
x
0 5,0

Exemple Cartes météorologiques


Sur une carte météorologique, les nuances de
couleur sont la représentation de la tempéra-
ture : ici on va du vert (≈ 7 ◦C) au rouge (≈
25 ◦C).

25 ◦C

7 ◦C

2.2.4 Représentation par lignes de niveau

Pour obtenir les fonctions partielles, on a considéré des coupes verticales du graphe d’une fonction de deux variables. De
la même manière, on peut considérer des coupes horizontales et on obtient, de façon générale, des courbes planes, dites
courbes ou lignes de niveau.

Définition Lignes de niveau


Soit k ∈ R et f une fonction de R2 dans R ; la courbe de niveau k de la fonction f est la projection sur
© le plan d’équation z ª=
0 de l’intersection de la surface représentative de f avec le plan horizontal z = k, i.e. l’ensemble (x, y) ∈ D ¯ f (x, y) = k .
¯

En pratique, on représente simultanément différentes courbes de niveau pour visualiser la progression du graphe. Cette
représentation s’apparente aux cartes géographiques où le niveau correspond à l’altitude. Les courbes de niveau d’une fonc-
tion f (x, y) fournissent une représentation géométrique de f sur le plan, alors que son graphe en donne une dans l’es-
pace.

42 © G. Faccanoni
Dernière mise à jour : Jeudi 5 juin 2014 2 Fonctions de plusieurs variables

Exemple
Représentation par lignes de niveau et surfacique de la fonction

f : R2 → R
(x, y) 7→ x 2 + y 2

5,0

2,5

-5,0
y 0,0
-2,5
y

0,0

50 2,5
-2,5
40 5,0

-5,0
30
-2,5
-5,0 20
0,0
-5,0 -2,5 0,0 2,5 5,0
10 x
2,5
x
0 5,0

Exemple
L’image ci-contre montre les courbes de niveaux d’une fonc-
tion f . On peut alors se faire une idée de l’allure de la fonction.
Par exemple f (1; 3) ≈ 72, f (4; 5) ≈ 56, soit 40 < f (3; 3) < 50 soit
50 < f (3; 3) < 60, etc.

Exemple
Soit α > 0 et considérons la fonction f (µ, σ) = µ − ασ2 . Cette fonction est souvent utilisée en gestion de portefeuille : µ désigne la
rentabilité attendue du portefeuille et σ sa volatilité (écart-type de la rentabilité). La fonction f représente l’utilité attendue d’un
investisseur qui présente de l’aversion vis-à-vis du risque (σ2 est affecté d’un coefficient négatif). Les courbes de niveau pour α fixé
sont ici appelées courbes d’indifférence ou courbes d’iso-utilité puisqu’elles caractérisent les rentabilités attendues et les volatilités
des portefeuilles qui atteignent, pour l’investisseur considéré, un niveau fixé d’utilité attendue.

© G. Faccanoni 43
2 Fonctions de plusieurs variables Dernière mise à jour : Jeudi 5 juin 2014

Exemple Cartes topographiques


On peut considérer le relief d’une région comme étant le graphe d’une fonction de deux variables (par exemple, l’altitude en fonction
de la longitude et de la latitude). Une courbe de niveau nous indique les points de même altitude (ici, l’altitude du point A est 940+d =
940+cb/a). En dessinant les courbes de niveau avec leur altitude correspondante, on obtient la carte topographique du relief. La lecture
d’une carte topographique permet non seulement d’obtenir des mesures quantitatives du relief, mais aussi de faire rapidement des
observations qualitatives sur sa nature. Par exemple, localiser les points de plus haute et de plus basse altitude ; les crêtes, les fonds, les
vallées, les cols, etc. ; les endroits du relief où les pentes sont plus escarpées ou plus douces, puisqu’ils correspondent respectivement
aux courbes de niveau très rapprochées ou très distantes.

Exemple Cartes météorologique

Sur une carte météorologique, les courbes de


niveau sont les isothermes (lignes reliant les
points d’égale température) ou les isobares
(lignes reliant les points d’égale pression).

Exemple
Alice se rend chez son épicier pour y acheter des grappes de raisin et du fromage à la coupe. Le prix du raisin est fixé à 5 euros le Kg,
celui du fromage considéré est de 18 euros le Kg. Si Alice décide d’acheter x Kg de raisin et y Kg de fromage, sa dépense en euros est de

D(x, y) = 5x + 18y.

Cette dépense constitue donc une fonction linéaire D : R2+ → R+ des variables x et y. Le graphe obtenu est une portion de plan.
Si Alice décide de payer en liquide et dispose de L euros seulement, elle ne peut envisager que des budgets (x, y) tels que D(x, y) ≤ L.
Graphiquement, ceci revient essentiellement à envisager une intersection de la surface S avec des plans horizontaux d’équation z =
k ∈ [0; L].

Exemple
Le nombre de voitures produites annuellement par une usine dépend du nombre x d’heures de travail et du capital y à disposition de
la manière suivante :
N (x, y) = 2x 2/3 y 1/3 .
On suppose par ailleurs que le coût de production de q voitures s’élève à

C (q) = 1 + 3q + q 2

et l’on définit aussi une fonction bénéfice B : R2+ → R par

B (q, p) = q p −C (q).

où p est le prix. N et B sont des fonctions définies sur R2+ , à valeurs réelles, non linéaires : les surfaces ne sont plus des portions de
plan ! On peut envisager d’étudier le comportement de la fonction B à prix fixé, c’est-à-dire lorsque p est fixé à une certaine valeur p 0 .
Graphiquement cela correspond à considérer l’intersection de la surface avec le plan vertical d’équation p = p 0 . On peut aussi s’inté-
resser à l’ensemble des couples (q, p) permettant de réaliser le bénéfice B 0 . Graphiquement cela correspond à considérer l’intersection
de la surface avec le plan horizontal d’équation z = B 0 . On peut de plus envisager la fonction Be : R3+ → R donnant le bénéfice annuel
réalisé en fonction du nombre d’heures de travail, du capital et du prix unitaire fixé : Be(x, y, p) = B (N (x, y), p). Be est une fonction de 3
variables, son graphe se situe donc dans R4 . Cependant les surfaces de niveaux peuvent être visualisées, ainsi que les variation de Be en
fonction de x lorsque les variables y et p sont fixées.

44 © G. Faccanoni
Dernière mise à jour : Jeudi 5 juin 2014 2 Fonctions de plusieurs variables

............... Exercices ..............


Exercice 2.1
Dans chaque cas, déterminez et représentez le domaine de définition des fonctions données.
p
−y+x 2 ln(y) ln(x 2 +1)
1. f (x, y) = p 2. f (x, y) = p
x−y
3. f (x, y) = ln(x + y) 4. f (x, y, z) = yz
y

C ORRECTION .

1. D = (x, y) ∈ R2 ¯ y ≤ x 2 et y > 0 3. D = (x, y) ∈ R2 ¯ y > −x


© ¯ ª © ¯ ª
y
y
x
2
2. D = (x, y) ∈ R ¯ y > 0 et x > y
© ¯ ª
x
y

4. D = (x, y, z) ∈ R3 ¯ y 6= 0 et z 6= 0
© ¯ ª
x

Exercice 2.2
Déterminer les courbes de niveau des fonctions suivantes :
2
f (x, y) = x, f (x, y) = y + 1, f (x, y) = x + y − 1, f (x, y) = e y−x , f (x, y) = y − cos(x).

Esquissez ensuite leurs graphes (le graphe peut être vu comme un empilement de courbes de niveau qui forment une
surface dans R3 ).

C ORRECTION .
B f (x, y) = x :
f (x, y) = κ ssi x = κ, les courbes de niveau sont donc des droites verticales et la surface représentative de f est un plan.
2

1
y 0

-1
-1
2

1
-2
-2 2 0
1 x
-2 -1 0 1 2 0 -1
y -1
x -2
-2

B f (x, y) = y + 1 :
f (x, y) = κ ssi y = κ − 1, les courbes de niveau sont donc des droites horizontales et la surface représentative de f est un
plan.
2

2
y 0

-1
0
2

1
-1
-2 2 0
1 x
-2 -1 0 1 2 0 -1
y -1
x -2
-2

B f (x, y) = x + y − 1 :
f (x, y) = κ ssi y = −x + (κ + 1), les courbes de niveau sont donc des droites de pente −1 et la surface représentative de f
est un plan.

© G. Faccanoni 45
2 Fonctions de plusieurs variables Dernière mise à jour : Jeudi 5 juin 2014

3,2
y 0

1,2

-1
-0,8

-2,8 1

-2 2 0
1 x
-2 -1 0 1 2 0 -1
y -1
x -2
-2

2
B f (x, y) = e y−x :
f (x, y) = κ ssi y = x 2 + ln(κ), les courbes de niveau sont donc des paraboles. On observe notamment la croissance expo-
nentielle marquée lorsque les valeurs prises par y sont grandes et celles prises par |x| sont petites.
2

3 000

1
2 500

2 000

1 500
0

-2 -1 0 1 2 -2 8,0
1 000
y
x 5,5

-1 500 3,0
x
0,5
y -1 0
-2,0 0

2
-2

B f (x, y) = y − cos(x) :
f (x, y) = κ ssi y = cos(x) + κ
10

9
4 -10 -10
y x
-5 4 -5
2

0 0
0
-1
-10 -8 -6 -4 -2 0 2 4 6 8 10
5 5
-2
x
-6
10 10
-4

y -11
-6

-8

-10

Exercice 2.3 Domaine de définition, courbes de niveau


1. Déterminer et représenter le domaine de définition de la fonction f : R2 → R définie par f (x, y) = ln(x − y 2 ).
2. Déterminer et représenter ses courbes de niveau.

C ORRECTION .

D f = (x, y) ∈ R2 ¯ x − y 2 > 0 = (x, y) ∈ R2 ¯ y 2 < x x − y2 = ek x = y2 + ek


© ¯ ª © ¯ ª
f (x, y) = k ⇐⇒ ⇐⇒

y k → −∞
y k = −2
k =0
1
1 k =1
0
1 2 3 x 0
1 2 3 x
−1
−1

−2
−2

46 © G. Faccanoni
Dernière mise à jour : Jeudi 5 juin 2014 2 Fonctions de plusieurs variables

Exercice 2.4

Dans la figure ci-contre on a tracé les isobares de l’Amé-


rique du Nord au 12 août 2008. La pression indiquée est
mesurée en millibars (mbar).
1. Donner une estimation de la pression
B à Nashville (point N),
B à Chicago (point C),
B à San Francisco (point S)
B et à Vancouver (point V).
2. Dans quelle ville le vent est le plus fort ?

C ORRECTION .
1. B Au point N la pression est de 1012 mbar environ,
B au point C la pression est de 1013 mbar environ,
B au point S la pression est de 1010 mbar environ,
B au point V la pression est comprise entre 1016 mbar et 1020 mbar ou entre 1012 mbar et 1016 mbar.
2. Le vent est plus fort à San Francisco car les lignes de pression sont le plus rapprochées.

Exercice 2.5
Associer chaque fonction (1-6) à sa surface (I-VI).

1. f (x, y) = |x| + |y|

2. f (x, y) = |x y|

1
3. f (x, y) =
1 + x2 + y 2

4. f (x, y) = (x 2 − y 2 )2

5. f (x, y) = (x − y)2

6. f (x, y) = sin(|x| + |y|)

C ORRECTION . 1-VI, 2-V, 3-I, 4-IV, 5-II, 6-III.

Exercice 2.6
Associer chaque fonction (1-6) à sa surface (A-F) et à ses courbes de niveau (I-VI) :
1. f (x, y) = sin(x y)

© G. Faccanoni 47
2 Fonctions de plusieurs variables Dernière mise à jour : Jeudi 5 juin 2014

2. f (x, y) = sin(x − y)
3. f (x, y) = (1 − x 2 )(1 − y 2 )
x−y
4. f (x, y) = 1+x 2 +y 2
x
5. f (x, y) = e cos(y)
6. f (x, y) = sin(x) − sin(y)

C ORRECTION .
1. C-II : la fonction est périodique en x et en y ; f ne change pas quand on échange x et y, i.e. le graphe est symétrique
par rapport au plan d’équation y = x ; f (0, y) = f (x, 0) = 0.
2. F-I : la fonction est périodique en x et en y ; f est constante si y = x + κ.

48 © G. Faccanoni
Dernière mise à jour : Jeudi 5 juin 2014 2 Fonctions de plusieurs variables

3. B-VI : f (±1, y) = f (x, ±1) = 0 ; la trace dans le plan xz est z = 1 − x 2 et dans le plan y z est z = 1 − y 2 .
4. D-V : f (x, x) = 0 ; f (x, y) > 0 si x > y ; f (x, y) < 0 si x < y.
5. A-IV : la fonction est périodique en y ;
6. E-III : la fonction est périodique en x et en y.

Exercice 2.7 Cartes topographiques du relief

Sur une carte topographique, les courbes de niveau dési-


gnent les points de même altitude. On observe sur l’ex-
trait de carte ci-contre de l’institut Géographique National
(IGN), des courbes qui donnent une idée du relief (Massif
du Sancy). Elles représentent des coupes horizontales suc-
cessives du terrain à des altitudes qui varient de 10 mètres
en 10 mètres. Tous les points de même altitude sont situés
sur la même courbe de niveau.

1. Compléter le tableau
Point A B C D E F G
Altitude 1370
2. Lorsque les courbes de niveau se resserrent, que peut-on dire du relief ?
3. La rivière coule-t-elle d’est en ouest ou vice-versa ?

C ORRECTION .
1. On a
Point A B C D E F G
Altitude 1470 1370 1380 1470 1400 1460 1520
2. Les endroits du relief où les pentes sont plus escarpées ou plus douces correspondent respectivement aux courbes de
niveau très rapprochées ou très distantes.
3. La rivière coule de l’est à l’ouest.

Exercice 2.8
1. Deviner l’expression d’une fonction dont l’iso-0 (c’est-à-dire la ligne de niveau 0) représente la berge d’un fleuve
rectiligne :

2. Modifier la fonction précédente pour que l’eau coule dans la direction des x positifs :

© G. Faccanoni 49
2 Fonctions de plusieurs variables Dernière mise à jour : Jeudi 5 juin 2014

3. Deviner l’expression d’une fonction dont l’iso-0 représente la berge d’un fleuve rectiligne qui s’est séparé en deux
affluents.

4. Modifier ensuite la fonction pour que l’eau coule dans la direction des x positifs :

C ORRECTION .

f 1 : R2 → R f 2 : R2 → R f 3 : R2 → R f 4 : R2 → R
x x
(x, y) 7→ f 1 (x, y) = y 2 (x, y) 7→ f 2 (x, y) = y 2 − (x, y) 7→ f 3 (x, y) = y 4 − y 2 (x, y) 7→ f 4 (x, y) = y 4 − y 2 −
10 10

Exercice 2.9 Fonctions radiales


p
On pose r = x 2 + y 2 (distance à l’origine). On s’intéresse au cas particulier f (x, y) = g (r ).
1. Donner les équations des fonctions polynomiales en double puits dont les graphes sont les suivants :
y
y
4
4 x4

3
3

2
2

1
1

−2 −1 1 2 x
−2 −1 1 2 x
−1
−1

−2
−2
−x 2

2. En déduire l’expression des fonction qui présentent un «canyon» circulaire dont les surfaces sont les suivantes :

C ORRECTION .
1.

f : R→R f : R→R
2 2
x 7→ (x − 1)(x − 4) x 7→ x 4 − x 2

2.

g: R→R g: R→R
2 2
r 7→ (r − 1)(r − 4) r 7→ r 4 − r 2

50 © G. Faccanoni
3 Limites et continuité des fonctions de Rn dans R
La notion de limite pour une fonction de plusieurs variables généralise naturellement la notion correspondante dans le cas
des fonctions d’une seule variable. Toutefois, un nouvel élément entre en jeu : les limites unilatérales (i.e. de la gauche et
de la droite) perdent leur sens et sont remplacées par les nombreuses limites directionnelles possibles. En effet, dès que le
domaine se situe dans un espace à deux dimensions au moins, les chemins qui mènent à un point donné peuvent suivre
divers axes. Ainsi, l’ensemble des points en lesquels une limite peut être considérée, doit être défini en tenant compte de
toutes les possibilités d’accès (voir par exemple la figure 3.1). Une façon commode de procéder s’appuie sur la notion de
boule ouverte dans Rn qui généralise celle d’intervalle ouvert dans R. Pour cela il faut d’abord introduire la notion de norme
dans Rn qui généralise la notion de distance dans R.

F IGURE 3.1: Différents façons de s’approcher du point x0 .

3.1 Normes
Définition Norme
On appelle norme sur Rn toute application N : Rn → [0; +∞[ possédant les propriétés suivantes :

N (x) = 0 ⇐⇒ x = 0 pour tout x ∈ Rn (séparation),


n
N (λx) = |λ|N (x) pour tout x ∈ R et pour tout λ ∈ R (homogénéité),
n
N (x + y) ≤ N (x) + N (y) pour tout x, y ∈ R (inégalité triangulaire).

On emploie généralement la notation kxk pour N (x), qui rappelle l’analogie avec la valeur absolue dans R ou le module
dans C.

Normes classiques
Soit p ∈ N∗ . Dans Rn , on utilise les normes classiques suivantes définies pour x = (x 1 , x 2 , . . . , x n ) par :
s
n
X
|x i |p ,
p
© ª
kxkp = kxk∞ = sup |x 1 |, |x 2 |, . . . , |x n | .
i =1

Si p = 2, on parle de norme euclidienne.

Propriété
Pour tout x, y ∈ Rn muni d’une norme k·k on a
¯ ¯
¯kxk − kyk¯ ≤ kx − yk ≤ kxk + kyk.
¯ ¯

51
3 Limites et continuité Dernière mise à jour : Jeudi 5 juin 2014

Définition Boules
Dans Rn muni d’une norme k·k, on appelle
B boule ouverte de centre A ∈ Rn et de rayon r > 0 l’ensemble B(A, r ) = {x ∈ Rn tel que kx − Ak < r } ;
B boule fermée de centre A ∈ Rn et de rayon r > 0 l’ensemble B(A, r ) = {x ∈ Rn tel que kx − Ak ≤ r }.

Exemple
On veut dessiner les boules fermées de centre (0, 0) et de rayon r > 0 relatives aux trois normes classiques de R2 : k·k1 , k·k2 et k·k∞ .
La boule fermée de centre (0, 0) et rayon r > 0 est l’ensemble de points

B((0, 0), r ) = {(x, y) ∈ R2 | k(x, y) − (0, 0)k ≤ r } = {(x, y) ∈ R2 | k(x, y)k ≤ r }.

B Pour la norme k·k∞ on a


k(x, y)k∞ < r ⇐⇒ sup{|x|, |y|} ≤ r.
B Pour la norme k·k1 on a
k(x, y)k1 < r ⇐⇒ |x| + |y| ≤ r.
B Pour la norme k·k2 on a
k(x, y)k2 < r ⇐⇒ |x|2 + |y|2 ≤ r 2 .
y y y
r r r

−r r x −r r x −r r x

−r −r −r

max{|x|, |y|} ≤ r |x| + |y| ≤ r x2 + y 2 ≤ r

La mot «boule» a donc ici un sens plus générale que dans la vie courante !

3.2 Limites
Définition Limite en un point
Soit D un ouvert de Rn , A ∈ D et f une fonction définie sur D, éventuellement non définie en A, à valeurs réelles.
B On dit que f a pour limite ` ∈ R au point A, ce que l’on écrit lim f (x) = `, si pour tout ε > 0 il existe δ > 0 tel que
x→A

x ∈ D \ { A } et kx − Ak < δ =⇒ | f (x) − `| < ε,

autrement dit
x ∈ D ∩ B(A, δ) =⇒ | f (x) − `| < ε.
Voir la figure 3.2 pour un exemple avec une fonction de deux variables et la norme euclidienne.
B On dit que f tend vers +∞ quand x tend vers A, ce que l’on écrit lim f (x) = +∞, si pour tout M > 0 il existe δ > 0 tel
x→A
que
x ∈ D \ { A } et kx − Ak < δ =⇒ f (x) > M ,
autrement dit
x ∈ D ∩ B(A, δ) =⇒ | f (x) − `| > M .
B On dit que f tend vers −∞ quand x tend vers A, ce que l’on écrit lim f (x) = −∞, si pour tout M < 0 il existe δ > 0 tel
x→A
que
x ∈ D \ { A } et kx − Ak < δ =⇒ f (x) < M ,
autrement dit
x ∈ D ∩ B(A, δ) =⇒ | f (x) − `| < M .

Propriété
L’existence et la valeur éventuelle de la limite sont indépendantes de la norme choisie dans Rn . Lorsqu’elle existe, la limite
est unique.

52 © G. Faccanoni
Dernière mise à jour : Jeudi 5 juin 2014 3 Limites et continuité

F IGURE 3.2: lim f (x, y) = L


(x,y)→(a,b)

Les théorèmes sur les opérations pour les fonctions de n variables admettant des limites en A sont les mêmes que pour les
focntions d’une variable réelle à valeur réelle.
Propriété
Si f a pour limite ` en A, la restriction de f à toute courbe continue (non seulement les droites !) passant par A admet la
même limite `.

Attention
Pour prouver qu’une fonction de plusieurs variables
• n’admet pas de limite en A, il suffit d’expliciter une restriction à une courbe continue dans D ∪ { A } passant par A
qui n’admet pas de limite, ou deux restrictions qui conduisent à des limites différentes ;
• admet ` comme limite en A, il faut considérer le cas général : si on a juste que la restriction à toute droite passant
par A admet la même limite, on ne peut pas conclure que la limite existe !

Exemple n = 2
Soit A = (x 0 , y 0 ) ∈ R2 . Des courbes continues passant par A sont par exemple
B les droites :
B la droite verticale qui a équation x = x 0 ,
B les droites qui ont équation y = m(x − x 0 ) + y 0 pour tout m ∈ R ;
B quelques paraboles, par exemple :
B les paraboles d’équation y = m(x − x 0 )2 + y 0 pour tout m ∈ R
B les paraboles d’équation x = m(y − y 0 )2 + x 0 pour tout m ∈ R

Exemple
x2 − y 2
On veut calculer lim .
(x,y)→(0,0) x 2 + y 2
x 2 −y 2
B La fonction f (x, y) = 2 2 est définie sur R2 \ { (0, 0 }.
x +y
B On cherche des courbes continues définies sur R2 , qui passent par (0, 0), et on calcule la restriction de f à ces courbes :
2
B la droite d’équation y = 0 donne la restriction g (x) = f (x, 0) = x 2 = 1 pour tout x ∈ R,
x
−y 2
B la droite d’équation x = 0 donne la restriction g (y) = f (0, y) = 2 = −1 pour tout y ∈ R,
y
2 4 2
B la parabole d’équation y = x 2 donne la restriction g (x) = f (x, x 2 ) = x 2 −x 4 = 1−x 2 pour tout x ∈ R,
x +x 1+x
B etc.
B On calcule les limites (d’une seul variable !) de ces restrictions et si on trouve deux limites différentes on s’arrête :
B la restriction de f à la droite d’équation y = 0 donne g (x) = f (x, 0) = 1 pour tout x ∈ R et limx→0 g (x) = 1 ;
B la restriction de f à la droite d’équation x = 0 donne g (y) = f (0, y) = −1 pour tout y ∈ R et lim y→0 g (y) = −1.
Comme les restrictions de f à deux courbes continues passant par (0, 0) donnent deux limites différents, on conclut que la
lim f (x, y) n’existe pas.
(x,y)→(0,0)

3.2.1 R2 : coordonnées polaires pour le calcul de limites

Lorsque n = 2, il est souvent utile de passer aux coordonnées polaires pour ramener le calcul de la limite d’une fonction
de deux variables à celui de la limite d’une fonction d’une seule variable. En effet, tout point (x, y) de R2 \ { (a, b) } peut être
représenté par ses coordonnées polaires centrées autour d’un point (a, b) grâce aux relations : x = a +r cos(ϑ), y = b +r sin(ϑ)
avec r > 0 et ϑ ∈ [0; 2π[.

© G. Faccanoni 53
3 Limites et continuité Dernière mise à jour : Jeudi 5 juin 2014

R
y
r B((a, b), R) = (x, y) ∈ R2 ¯ k(x − a, y − b)k < R
© ¯ ª
b ϑ

a x

Dans cette écriture, r représente la distance entre (a, b) et (x, y) de sorte que

lim f (x, y) = lim f (a + r cos(ϑ), b + r sin(ϑ))


(x,y)→(a,b) r →0
∀ϑ

On peut alors utiliser la condition suffisante suivante :


Proposition
B S’il existe ` ∈ R et une fonction r 7→ s = s(r ) telle que au voisinage de (a, b) on a
∀ϑ
| f (a + r cos(ϑ), b + r sin(ϑ)) − `| ≤ s(r ) −−−→ 0,
r →0

alors
lim f (x, y) = `.
(x,y)→(a,b)

B S’il existe une fonction r 7→ M = M (r ) telle que au voisinage de (a, b) on a


∀ϑ
| f (a + r cos(ϑ), b + r sin(ϑ))| ≥ M (r ) −−−→ +∞,
r →0

alors la limite lim(x,y)→(a,b) f (x, y) n’existe pas.

Exemple
x 2 −y 2
Montrons d’une autre manière que lim f (x, y) avec f (x, y) = n’existe pas en utilisant les coordonnées polaires. En posant
(x,y)→(0,0) x 2 +y 2
x = r cos(ϑ) et y = r sin(ϑ), on a

r 2 (cos2 (ϑ) − sin2 (ϑ))


lim f (x, y) = lim f (r cos(ϑ), r sin(ϑ)) = lim = lim (cos2 (ϑ) − sin2 (ϑ)) = cos(2ϑ).
(x,y)→(0,0) r →0 r →0 r 2 (cos2 (ϑ) + sin2 (ϑ)) r →0
∀ϑ ∀ϑ ∀ϑ

x 2 −y 2
Le résultat varie selon la direction ϑ, donc lim 2 2 n’existe pas.
(x,y)→(0,0) x +y

3.3 Continuité
Définition Continuité
Soit f une fonction de D ⊂ Rn dans R. On dit que f est continue en A ∈ D si f possède en A une limite égale à f (A). Si f
est continue en tout point de D, on dit que f est continue sur D.

54 © G. Faccanoni
Dernière mise à jour : Jeudi 5 juin 2014 3 Limites et continuité

Définition Prolongement par continuité


Soit f une fonction de D ⊂ Rn \ { A } dans R. Si lim f (x) = `, la fonction fe définie sur D ∪ { A } par fe(A) = ` et fe(x) = f (x)
x→A
pour x ∈ D est la seule fonction continue en A dont la restriction à D soit f . On l’appelle le prolongement par continuité
de f à A.

Propriété Opérations algébriques


Les fonctions continues de plusieurs variables jouissent des mêmes propriétés que les fonctions continues d’une seule
variable. Les fonctions élémentaires telles que les polynômes, les fonctions exponentielles, logarithmiques et trigonomé-
triques sont continues dans leurs domaines de définition respectifs. La continuité des autres fonctions s’établit, le cas
échéant, en tant que somme, produit, composée, le quotient (lorsque le dénominateur ne s’annule pas) etc., de fonctions
continues.

Exemple
1. f (x, y) = x 2 + y 2 − x y + y est continue dans R2 (polynôme du second degré à deux variables).
2. f (x, y, z) = e y + x y 2 − z est continue dans R3 (somme d’une exponentielle et d’un polynôme).
3. f (x, y) = ln(x + y 2 ) − 3 est continue dans D = (x, y) ∈ R2 ¯ x + y 2 > 0 comme somme du logarithme d’un polynôme (fonction
© ¯ ª

composée) et d’une constante.

Exemple
B Soit f la fonction définie sur R2 \ {(0, 0)} par
xy
f (x, y) = 2 .
x + y2
Comme f (0, t ) = 0 et f (t , t ) = 12 alors f ne peut pas être prolongée par continuité en (0, 0).

B Soit f la fonction définie sur R2 \ {(0, 0)} par


xy
f (x, y) = q .
x2 + y 2
Pour trouver la valeur de la limite, si elle existe, il suffit de calculer la limite d’une restriction à une courbe continue passant par
(0, 0) : comme f (0, t ) = 0 pour tout t , si la limite existe elle est 0. Pour vérifier que c’est bien la limite on passe en coordonnées
polaires : on pose x = r cos(ϑ) et y = r sin(ϑ) pour r > 0 et ϑ ∈ [0, 2π] ; on obtient

r 2 cos(ϑ) sin(ϑ) r
f (x(r, ϑ), y(r, ϑ)) = p = r cos(ϑ) sin(ϑ) = sin(2ϑ).
r 2 (cos2 (ϑ) + sin2 (ϑ)) 2

Comme |sin(2ϑ)| ≤ 1, alors | f (x, y)| ≤ r2 −−−→ 0 indépendamment de ϑ, donc


r →0

lim f (x, y) = 0.
(x,y)→(0,0)

f peut donc être prolongée par continuité en (0, 0) par la valeur 0.

© G. Faccanoni 55
3 Limites et continuité Dernière mise à jour : Jeudi 5 juin 2014

............... Exercices ..............


Exercice 3.1
Ces limites existent-elles dans R ?

1 y3
1. lim ; 2. lim ;
(x,y)→(1,1) x − y (x,y)→(1,0) (x − 1)2 + y 2

C ORRECTION .
1 1 1
1. La limite lim(x,y)→(1,1) x−y n’existe pas car f (1, y) = −−−→ −∞ et
1−y − f (1, y) = 1−y −
−−−→ +∞.
y→1+ y→1−

2. En polaire x = 1 + r cos(ϑ), y = r sin(ϑ) : f (r, ϑ) = r sin3 ϑ et |r sin3 ϑ| ≤ r ainsi lim f (r, ϑ) = 0 et finalement
r →0

y3
lim = 0.
(x,y)→(1,0) (x − 1)2 + y 2

On peut également utiliser le théorème du pincement sans passer par les coordonnées polaires, grâce aux inégalités
suivantes :
¯ y 3 ¯ |y|3
¯ ¯
2 2 2
x + y ≥ y =⇒ ¯ 2
¯ ¯≤ = |y| ∀y 6= 0.
x + y2 ¯ y2

Exercice 3.2
Soit f la fonction définie sur R2 par

x2 y


 si (x, y) 6= (0, 0)
f (x, y) = x 4 − 2x 2 y + 3y 2

0 sinon.

Montrer que la restriction de f à toute droite passant par l’origine est continue mais que f n’est pas continue à l’origine.

C ORRECTION . Remarquons tout d’abord que la fonction est bien définie dans R2 puisque

x 4 − 2x 2 y + 3y 2 = (x 2 − y)2 + 2y 2

ne s’annule qu’en (0, 0).


La restriction de f aux droites x = 0 et y = 0 est la fonction nulle. De plus, la restriction de f à la droite y = mx, avec m 6= 0,
donne
mx
f (x, mx) = 2
x − 2mx + 3m 2
et tend vers 0 quand x tend vers 0. Comme f (0, 0) = 0, la restriction de f à toute droite passant par l’origine est donc continue.
Considérons la restriction de f à la parabole y = x 2 . On a

x4 1
f (x, x 2 ) = 4
= .
2x 2

Par conséquent, f (x, x 2 ) ne tend pas vers 0 quand x tend vers 0.

Exercice 3.3
Soit f la fonction définie sur R2 \ {(0, 0)} par
x ln(1 + x 3 )
f (x, y) = .
y(x 2 + y 2 )
Calculer, si elle existe, la limite de f pour (x, y) qui tend vers (0, 0).

56 © G. Faccanoni
Dernière mise à jour : Jeudi 5 juin 2014 3 Limites et continuité

ln(1 + x 3 ) 2 ln(1 + x 3 ) 1
C ORRECTION . Pour m 6= 0 on a f (x, mx) = −
− −
→ 0 mais f (x, x ) = −−−→ 1. La limite n’existe
m(1 + m 2 )x 2 x→0 x3 1 + x 2 x→0
donc pas.

Exercice 3.4
Soit f : R2 → R la fonction ainsi définie
3 3

x +y

si (x, y) 6= (0, 0),
f (x, y) = x 2 + y 2

0 si (x, y) = (0, 0).

Est-elle continue sur R2 ?

C ORRECTION . On utilise les coordonnées polaires : x = r cos(ϑ), y = r sin(ϑ) ; alors f (r, ϑ) = r (cos3 ϑ + sin3 ϑ) et comme
| f (r, ϑ)| ≤ |2r | −−−→ 0, on en déduit que limr →0 f (r, ϑ) = 0 indépendamment de ϑ, ce qui prouve que lim(x,y)→(0,0) f (x, y) = 0,
r →0
i.e. que f est continue en (0, 0). De plus elle est continue sur R2 \ { (0, 0) } comme quotient de fonctions continues dont le
dénominateur ne s’annule pas. Nous pouvons donc conclure que f est continue sur R2 .

Exercice 3.5
Montrer que la fonction f : R2 \ { (0, 0) } → R définie par

x y2
f (x, y) =
x2 + y 4

n’est pas prolongeable par continuité en (0, 0).

C ORRECTION . Pour que f soit prolongeable par continuité en (0, 0) il faut que lim(x,y)→(0,0) f (x, y) = ` ∈ R. Pour prouver que
cette limite n’existe pas il suffit d’expliciter deux restrictions à deux courbes continues passant par (0, 0) qui conduisent à des
3
limites différentes. La restriction de f à la courbe continue y = x qui passe par le point (0, 0) est la fonction f (x, x) = x 2x+y 4 =
x y4
1+x 2
. La restriction de f à la courbe continue x = y 2 qui passe par le point (0, 0) est la fonction f (y 2 , y) = y 4 +y 4
= 12 . Comme
f (x, x) −−−→ 0 mais f (y 2 , y) = 21 , la fonction n’est pas prolongeable par continuité en (0, 0).
x→0

Exercice 3.6
Montrer que la fonction f : R2 \ { (0, 0) } → R définie par

sin(x 2 ) − sin(y 2 )
f (x, y) =
x2 + y 2

n’est pas prolongeable par continuité en (0, 0).

C ORRECTION . Pour que f soit prolongeable par continuité en (0, 0) il faut que lim(x,y)→(0,0) f (x, y) = ` ∈ R. Pour prouver que
cette limite n’existe pas il suffit d’expliciter deux restrictions à deux courbes continues passant par (0, 0) qui conduisent à des
2)
limites différentes. La restriction de f à la courbe continue y = 0 qui passe par le point (0, 0) est la fonction f (x, 0) = sin(x
x2
. La
sin(y 2 )
restriction de f à la courbe continue x = 0 qui passe par le point (0, 0) est la fonction f (0, y) = − y2
. Comme f (x, 0) −−−→ 1
x→0
mais f (0, y) = −1, la fonction n’est pas prolongeable par continuité en (0, 0).

© G. Faccanoni 57
3 Limites et continuité Dernière mise à jour : Jeudi 5 juin 2014

Exercice 3.7
6x 2 y
Soit f : R2 \ { (0, 0) } → R la fonction définie par f (x, y) = x 2 +y 2
. Montrer que lim f (x, y) = 0 de trois façons :
(x,y)→(0,0)
1. d’après la définition (utiliser la norme euclidienne),
2. d’après le théorème de pincement,
3. en utilisant les coordonnées polaires.

C ORRECTION .
1. Soit ε > 0. Il faut trouver r > tel que
¯q ¯ ¯ 2 ¯
¯ x 2 + y 2 − 0¯ < r =⇒ ¯ 6x y − 0¯ < ε.
¯ ¯ ¯ ¯
¯ ¯ ¯ x2 + y 2 ¯

Comme on a
¯ 6x 2 y ¯ 6x 2 |y|
¯ ¯ q
x 2 + y 2 ≥ x 2 =⇒ ¯¯ 2 ¯≤ = 6|y| ≤ x2 + y 2 ∀(x, y) 6= (0, 0),
x + y2 ¯ x2
il suffit de choisir r = ε/6.
2. Pour tout (x, y) 6= (0, 0) on a
¯ 6x 2 y ¯
¯ ¯
0 ≤ ¯¯ 2 ¯ ≤ 6|y| −−−−−−−−→ 0
x + y2 ¯ (x,y)→(0,0)

3. Posons x = r cos(ϑ) et y = r sin(ϑ). On obtient

6x 2 y
lim = lim (6r cos2 (ϑ) sin(ϑ)).
(x,y)→(0,0) x 2 + y 2 r →0
∀ϑ

∀ϑ
Or, 0 ≤ |6r cos2 (ϑ) sin(ϑ)| ≤ 6r −−−→ 0, donc lim(x,y)→(0,0) f (x, y) = 0.
r →0

Exercice 3.8
Calculer la limite si elle existe ou montrer qu’elle n’existe pas :

x 4 +y 4 2x 2 +3x y+y 2 3x 2 y
1. lim x 2 2 ; 12. lim x 2 +5y 2
; 22. lim x 2 2 ;
(x,y)→(0,0) +y (x,y)→(0,0) (x,y)→(0,0) +y
2 2
e −x −y −1 (x+y)2 x2 y
2. lim 2 2 ; 13. lim ; 23. lim ;
(x,y)→(0,0) x +y 2
(x,y)→(0,0) x +y
2 4
(x,y)→(0,0) x +y
2

x 3 +y 3
3. lim ; x 2 sin2 (y) x y+y z 2 +xz 2
2
(x,y)→(0,0) x +y
2 14. lim 2 2 ; 24. lim ;
(x,y)→(0,0) x +3y
2 2 4
(x,y,z)→(0,0,0) x +y +z
x 4 −4y 2
4. lim x 2 +2y 2 ; arctan((x+y)2 ) 2
(x,y)→(0,0) 15. lim x2
; 25. lim p x2 ;
(x,y)→(0,0) (x,y)→(0,0) x +y 2
x 2 ye y
5. lim x 4 +4y 2 ; x 2 −y 2
(x,y)→(0,0) x 2 −x 3 +y 2 +y 3
16. lim 2 2 ; 26. lim ;
(x,y)→(0,0) x +y (x,y)→(0,0) x 2 +y 2
x 2 +y 2
6. lim p ;
(x,y)→(0,0) x 2 +y 2 +1−1 x2 y 3 x−y
17. lim ; 27. lim ;
|x|+|y| (x,y)→(0,0) 2x 2 +y 2
(x,y)→(0,0) x+y
7. lim 2 2 ;
(x,y)→(0,0) x +y
18. lim
sin(x 2 +y 2 )
; 28. lim (x 2 + y 2 ) ln(x 2 + y 2 ) + 5 ;
x3 x 2 +y 2 (x,y)→(0,0)
8. lim 2 2 ; (x,y)→(0,0)
(x,y)→(0,0) x +y
y 2 sin2 (x)
x y−2y x 3 +y 2 29. lim ;
9. lim ; 19. lim x 2 2 ; 4
(x,y)→(0,0) x +y
4
2 2
(x,y)→(2,0) x +y −4x+4 (x,y)→(0,0) +y
2
10. lim
y sin(x+1)
; 20. lim xe x/y ; 30. lim e y tan(xz) ;
2 (x,y,z)→(π,0,1/3)
(x,y)→(1,2) x −2x+1 (x,y)→(0,0)
x 2 +y 2 x ln y x y+y z 2 +xz 2
11. lim p ; 21. lim p ; 31. lim 2 2 .
(x,y)→(0,0) x 2 +y 2 +1−1 (x,y)→(0,1) x 2 +(y−1)2 (x,y,z)→(0,0,0) x +y +z4

58 © G. Faccanoni
Dernière mise à jour : Jeudi 5 juin 2014 3 Limites et continuité

C ORRECTION .
(x 2 +y 2 )2 ∀ϑ x 4 +y 4
1. Comme | f (x, y)| ≤ x 2 +y 2
= x 2 + y 2 et en polaire x 2 + y 2 = r 2 −−−→ 0 alors lim 2 2 = 0.
r →0 (x,y)→(0,0) x +y
2
e −r −1 ∀ϑ
2. On utilise les coordonnées polaires : x = r cos(ϑ), y = r sin(ϑ) ; alors f (r, ϑ) = r2
−−−→ −1, on en déduit que lim f (r, ϑ) =
r →0 r →0
∀ϑ
2 2
e −x −y −1
−1 indépendamment de ϑ, ce qui prouve que lim 2 2 = −1.
(x,y)→(0,0) x +y

3. On utilise les coordonnées polaires : x = r cos(ϑ), y = r sin(ϑ) ; alors f (r, ϑ) = r (cos3 (ϑ) + sin3 (ϑ)) et comme | f (r, ϑ)| ≤
|2r | −−−→ 0, on en déduit que lim f (r, ϑ) = 0 indépendamment de ϑ, ce qui prouve que lim f (x, y) = 0.
r →0 r →0 (x,y)→(0,0)
∀ϑ
x 4 −4y 2
4. Comme f (x, 0) = x 2 −−−→ 0 mais f (0, y) = −2 −−−→ −2 la limite lim 2 2 n’existe pas.
x→0 y→0 (x,y)→(0,0) x +2y
2
ex 1 x 2 ye y
5. Comme f (x, 0) = 0 −−−→ 0 mais f (x, x 2 ) = 5 −−−→ la limite lim 4 2 n’existe pas.
x→0 y→0 5 (x,y)→(0,0) x +4y
x 2 +y 2 p
6. lim p = lim x 2 + y 2 + 1 + 1 = 2.
(x,y)→(0,0) x 2 +y 2 +1−1 (x,y)→(0,0)
|x| |x|+|y|
7. Comme f (x, x) = −−−→ +∞ alors la limite lim
x 2 x→0 2 2 n’existe pas.
(x,y)→(0,0) x +y
x3
8. En polaire : f (r, ϑ) = r cos3 ϑ et |r cos3 ϑ| ≤ r . Comme lim f (r, ϑ) = 0 alors lim x 2 +y 2 = 0.
r →0 (x,y)→(0,0)
∀ϑ
1 x y−2y
9. Comme f (x, 0) = 0 et f (x, x − 2) = 2 la limite (x,y)→(2,0)
lim x 2 +y 2 −4x+4 n’existe pas.

2 sin(x+1) y sin(x+1)
10. Comme f (x, 2) = x−1 x+1 − −−→ ∞ alors lim 2 n’existe pas.
x→1 (x,y)→(1,2) x −2x+1
2 x 2 +y 2
11. En polaires : f (r, ϑ) = p r qui ne dépend que de r . Comme lim f (r, ϑ) = 2 alors lim p = 2.
r 2 +1−1 r →0 (x,y)→(0,0) x 2 +y 2 +1−1
∀ϑ
1 2x 2 +3x y+y 2
12. Comme f (x, 0) = 2 et f (0, y) = 5 alors lim x 2 +5y 2
n’existe pas.
(x,y)→(0,0)
(x+y)2
13. Comme f (x, x) = 2 et f (x, −x) = 0 alors lim 2 2 n’existe pas.
(x,y)→(0,0) x +y

cos2 (ϑ) sin2 (r sin(ϑ)) x 2 sin2 (y)


14. En polaires : f (r, ϑ) = 1+2 sin2 (ϑ)
et | f (r, ϑ)| ≤ sin2 (r sin(ϑ)). Comme lim f (r, ϑ) = 0 alors lim 2 2 = 0.
r →0 (x,y)→(0,0) x +3y
∀ϑ
arctan((2x)2 ) arctan(x 2 ) arctan((x+y)2 )
15. Comme f (x, x) = x2
−−−→ 4 et f (x, 0) = x2
−−−→ 1 alors lim x2
n’existe pas.
x→0 y→0 (x,y)→(0,0)
x 2 −y 2
16. Comme f (x, 0) = 1 et f (0, y) = −1 alors lim 2 2 n’existe pas.
(x,y)→(0,0) x +y
(ϑ) sin ϑ 2 3
r3
17. En polaires : f (r, ϑ) = r 3 cos
1+cos2 (ϑ)
et | f (r, ϑ)| ≤ 1+cos2 (ϑ)
car |cos2 (ϑ) sin3 ϑ| ≤ 1.
x2 y 3
Comme lim f (r, ϑ) = 0 alors lim 2 2 = 0.
r →0 (x,y)→(0,0) 2x +y
∀ϑ
sin(r 2 ) sin(x 2 +y 2 )
18. En polaires : f (r, ϑ) = r2
. Comme lim f (r, ϑ) = 1 alors lim x 2 +y 2
= 1.
r →0 (x,y)→(0,0)
x 3 +y 2
19. Comme f (x, 0) = x −−−→ 0 mais f (0, y) = 1 −−−→ 1 alors lim 2 2 = 0 n’existe pas.
x→0 y→0 (x,y)→(0,0) x +y

20. Comme f (x, x) = xe −−−→ 0 mais f (x, x 2 ) = xe 1/x −−−−→ +∞ alors lim xe x/y n’existe pas.
x→0 x→0+ (x,y)→(0,0)
21. En polaire : f (r, ϑ) = cos(ϑ) ln(1 + r sin(ϑ)) et |cos(ϑ) ln(1 + r sin(ϑ))| ≤ |ln(1 + r sin(ϑ))|. Comme on calcule la limite
pour r → 0, on peut supposer r < 21 ; dans ce cas |ln(1 + r sin(ϑ))| < |ln(1 + r )| −−−→ 0 et comme lim f (r, ϑ) = 0 alors
r →0 r →0
∀ϑ
x ln y
lim p = 0.
(x,y)→(0,1) x 2 +(y−1)2
3x 2 y
22. En polaire : f (r, ϑ) = 3r cos2 (ϑ) sin(ϑ) et |3r (cos2 (ϑ) sin(ϑ))| ≤ 3r . Comme lim f (r, ϑ) = 0 alors lim x 2 2 = 0.
r →0 (x,y)→(0,0) +y

x3 x x4 1 x2 y
23. Comme f (x, x) = x 4 +x 2
= −−−→ 0 mais
x 2 +1 x→0
f (x, x 2 ) = x 4 +x 4
= 2 alors lim x 4 +y 2 n’existe pas.
(x,y)→(0,0)

x 2 +2x 3 1+2x x3 x x y+y z 2 +xz 2


24. Comme f (x, x, x) = 2x 2 +x 4
= −−−→ 1
2+x 2 x→0 2
mais f (x, 0, x) = x 2 +x 4
= −−−→ 0 alors
1+x 2 x→0
lim 2 2 4 n’existe pas.
(x,y,z)→(0,0,0) x +y +z

© G. Faccanoni 59
3 Limites et continuité Dernière mise à jour : Jeudi 5 juin 2014

2
25. En polaire : f (r, ϑ) = r cos2 (ϑ) et |r cos2 (ϑ)| ≤ r . Comme lim f (r, ϑ) = 0 alors lim p x2 = 0.
r →0 (x,y)→(0,0) x +y 2

x 2 −x 3 +y 2 +y 3 y 3 −x 3 y 3 −x 3
26. x 2 +y 2
= 1+ x 2 +y 2
. Si on définit g (x, y) = x 2 +y 2
, en polaire g (r, ϑ) = r (sin3 ϑ − cos3 ϑ) et |r (sin3 ϑ − cos3 ϑ)| ≤ 2r .
x 2 −x 3 +y 2 +y 3
Comme lim f (r, ϑ) = 0 alors lim x 2 +y 2
= 1.
r →0 (x,y)→(0,0)
∀ϑ
x−y
27. Comme f (x, 0) = 1 et f (0, y) = −1 alors lim n’existe pas.
(x,y)→(0,0) x+y

28. En polaire : f (r, ϑ) = r 2 ln(r 2 ) + 5 −−−→ 5 alors lim (x 2 + y 2 ) ln(x 2 + y 2 ) + 5 = 5.


r →0 (x,y)→(0,0)
∀ϑ
29. La restriction de f à la courbe continue y = 0 qui passe par le point (0, 0) est la fonction f (x, 0) = 0. La restriction de f
³ ´2
à la courbe continue y = x qui passe par le point (0, 0) est la fonction f (x, x) = 12 sin(x)
x . Comme f (x, x) −−−→ 12 mais
x→0
y 2 sin2 (x)
f (x, 0) = 0, la limite lim 4 4 n’existe pas.
(x,y)→(0,0) x +y
2 2 p 2 π
30. lim e y tan(xz) = e 0 tan(π/3) = 3 car f (x, y, z) = e y tan(xz) est continue si xz 6= 2 + 2kπ ;
(x,y,z)→(π,0,1/3)
x y+y z 2 +xz 2
31. La restriction de f (x, y, z) = x 2 +y 2 +z4 à la courbe continue d’équation y = 0 et z = 0 qui passe par le point (0, 0, 0) est
la fonction f (x, 0, 0) = 0. La restriction de f à la courbe continue d’équation y = x et z = 0 qui passe par le point (0, 0, 0)
x y+y z 2 +xz 2
est la fonction f (x, x, 0) = 21 . Par conséquent la limite lim 2 2 n’existe pas.
(x,y,z)→(0,0,0) x +y +z4

60 © G. Faccanoni
4 Dérivabilité et différentiabilité, fonctions
implicites
4.1 Dérivées partielles du premier ordre et gradient
L’unique dérivée d’une fonction d’une variable réelle, lorsqu’elle existe, est liée aux variations de la fonction tandis que la
variable parcourt l’axe des abscisses. Pour une fonction f : R2 → R, dont le graphe est une surface de R3 , la situation est très
différente. En effet, l’axe réel n’offre que deux types de mouvements possibles : de gauche à droite et de droite à gauche tan-
dis que le plan R2 possède une infinité de directions. Il peut s’avérer intéressant d’étudier comment une fonction f : R2 → R
évolue lorsque la variable suit l’une ou l’autre direction du plan. À cet égard, considérons d’abord la direction horizontale.
Prenons le point (x 0 , y 0 ) du domaine de f . Son image est f (x 0 , y 0 ) ∈ R et le graphe de la fonction, qui est la surface d’équa-
tion z = f (x, y) de R3 , comporte le point (x 0 , y 0 , f (x 0 , y 0 )). L’intersection du graphe de f avec le plan vertical y = y 0 est la
courbe d’équation z = f (x, y 0 ) de R3 . Le point (x 0 , y 0 ) étant fixé, on peut alors interpréter cette courbe comme le graphe
de la fonction f y 0 d’une seule variable définie par f y 0 (x) = f (x, y 0 ). Si f y 0 est dérivable en x 0 , alors sa dérivée nous ren-
seigne sur la variation de la fonction f lorsque (x, y) se déplace le long de la droite horizontale de R2 passant par le point
(x 0 , y 0 ).
Lorsqu’on pose toutes les variables d’une fonction égales à une constante, sauf une, on obtient alors une fonction d’une seule
variable qui peut être dérivée suivant les règles habituelles.

Définition Dérivées partielles premières


Soit f une fonction à valeurs réelles définie sur une partie ouverte D de R2 . Soit (x 0 , y 0 ) ∈ D. Les dérivées partielles de f
en (x 0 , y 0 ) sont les dérivées des fonctions partielles f y 0 et f x0 évaluées en (x 0 , y 0 ), i.e. les fonctions

∂f f (x 0 + h, y 0 ) − f (x 0 , y 0 )
(x 0 , y 0 ) = f y00 (x) = lim , dérivée partielle de f par rapport à x au point (x 0 , y 0 ),
∂x h→0 h
∂f f (x 0 , y 0 + h) − f (x 0 , y 0 )
(x 0 , y 0 ) = f x00 (y) = lim , dérivée partielle de f par rapport à y au point (x 0 , y 0 ).
∂y h→0 h

Il s’agit de limites d’une fonction réelle de variable réelle !


Si f admet toutes les dérivées partielles premières, on dit que f est dérivable.

Remarque Notation
∂f
se note aussi ∂x f ou f ,x .
∂x
(Attention à ne pas confondre f ,x la dérivée de f par rapport à x avec f x0 la fonction partielle associée à f .)

Définition Vecteur gradient


Le gradient de f en (x 0 , y 0 ), noté ∇ f (x 0 , y 0 ) ou encore grad f (x 0 , y 0 ), est le vecteur dont les composantes sont les dérivées
partielles premières. Il est orthogonal à la courbe de niveau de f passant par (x 0 , y 0 ).

Astuce
En pratique, pour calculer la dérivée partielle ∂x f (resp. ∂ y f ), on dérive f comme si elle était une fonction de la seule
variable x (resp. y) et que l’autre variable, y (resp. x), était une constante.

Ces définitions se généralisent de manière naturelle à toute fonction de n > 2 variables.

61
4 Dérivabilité et différentiabilité, fonctions implicites Dernière mise à jour : Jeudi 5 juin 2014

Exemple
Soit f (x, y) = 4− x 2 −2y 2 . On a ∂x f (x, y) = −2x y et ∂ y f (x, y) = −4y. Le graphe de f est le paraboloïde z = 4− x 2 −2y 2 et le plan vertical
y = 1 l’intersecte dans la parabole z = 2 − x 2 , y = 1 (et on appelle cette courbe C 1 comme dans la figure à gauche). La pente de la droite
tangente à cette parabole au point (1, 1, 1) est ∂x f (1, 1) = −2. De la même façon, le plan vertical x = 1 intersecte le paraboloïde dans la
parabole z = 2−2y 2 , x = 1 (et on appelle cette courbe C 2 comme dans la figure à droite). La pente de la droite tangente à cette parabole
au point (1, 1, 1) est ∂ y f (1, 1) = −4.

Les dérivées partielles jouissent des mêmes propriétés que les dérivées de fonctions d’une seule variable. En particulier,
les fonctions élémentaires telles que les polynômes, les fonctions exponentielles, logarithmiques et trigonométriques sont
dérivables dans leur domaine respectif. La dérivabilité (partielle) des autres fonctions s’établit, le cas échéant, en tant que
somme, produit, composée, etc., de fonctions dérivables. Les règles de dérivation sont également similaires, à l’exception
de celle relative à la dérivation des fonctions composées. En effet, lorsque n > 1, il est impossible de réaliser un produit de
composition entre deux fonctions de Rn → R.

Exemple
1. Soit la fonction f (x, y) = 3x 2 + x y − 2y 2 . Alors D ≡ R2 , f est continue, ∂x f (x, y) = 6x + y (car y est considérée constante) et
∂ y f (x, y) = x − 4y (car x est considérée constante).
2. Soit la fonction f (x, y, z) = 5xz ln(1+7y). Alors D ≡ {(x, y, z) | y > −1/7}, f est continue et ∂x f (x, y, z) = 5z ln(1+7y), ∂ y f (x, y, z) =
35xz
1+7y et ∂z f (x, y, z) = 5x ln(1 + 7y).
3. Considérons ¢ l’entropie d’un gaz parfait en fonction de l’énergie interne spécifique ε et du volume spécifique τ : s(τ, ε) =
c v ln ετγ−1 = c v ln(ε) + c v (γ − 1) ln(τ) (avec c v et γ > 1 deux constantes). Comme la température et la pression sont définies
¡

c v (γ−1)τγ−2
respectivement par T = 1/s ε et P = T s τ , on obtient T = s1 = cε et P = T s τ = cε = (γ − 1) τε . On retrouve ainsi la
ε v v ετγ−1
relation bien connue P τ = RT avec R = c v (γ − 1).
4. La résistance totale R d’un conducteur produite par trois conducteurs de résistances R 1 , R 2 R 3 , connectés en parallèle, est
donnée par la formule
1 1 1 1
= + + .
R R1 R2 R3
On a alors ∂Ri R(R 1 , R 2 , R 3 ) = R 2 /R i2 .

Fonction de production de C OBB -D OUGLAS


C OBB et D OUGLAS ont cherché une fonction, définie et dérivable dans R2+ , qui caractérise la production en fonction du
travail w et du capital k :
f : R2+ → R∗+
(w, k) 7→ f (w, k)
La dérivée partielle ∂w f est la vitesse à laquelle la production change en fonction des variations du travail. On appelle
cela production marginale par rapport au travail ou productivité marginale du travail. De la même manière, la dérivée
partielle ∂k f est la vitesse à laquelle la production change en fonction des variations du capital. On appelle cela produc-
tion marginale par rapport au capital ou productivité marginale du capital. C OBB et D OUGLAS ont fait les hypothèses
suivantes :
B si le travail ou le capital s’annulent, la production aussi s’annule : f (w, 0) = f (0, k) = 0 ;
B la productivité marginale du travail est proportionnelle à la production par unité de travail :
α
∃α ∈ R tel que ∂w f (w, k) = f (w, k);
w
α
pour chaque k fixé, cela correspond à l’EDO f k0 (w) = w f k (w) dont la solution est f k (w) = g (k)w α , i.e. f (w, k) =
α
g (k)w où g est une fonction qui ne dépend que de k ;
B la productivité marginale du capital est proportionnelle à la production par unité de capital :
β
∃β ∈ R tel que ∂k f (w, k) = f (w, k).
k
β
pour chaque w fixé, cela correspond à l’EDO f w0 (k) = k f w (k) dont la solution est f w (k) = h(w)k β , i.e. f (w, k) =
h(w)k β où h est une fonction qui ne dépend que de w.

62 © G. Faccanoni
Dernière mise à jour : Jeudi 5 juin 2014 4 Dérivabilité et différentiabilité, fonctions implicites

On obtient alors la fonction f : R2+ → R définie par f (w, k) = c w α k β , où c est une constante, appelée fonction de produc-
tion de C OBB -D OUGLAS.
α
Comme ∂w f (w, k) = w f (w, k), alors c = 1. De plus, on a fait l’hypothèse que si le travail ou le capital s’annulent alors la
production aussi s’annule, par conséquent α, β > 0 et l’on a bien

α β
∂w f (w, k) = αw α−1 k β = f (w, k), ∂k f (w, k) = βw α k β−1 = f (w, k).
w k
Remarquons que si le travail et le capital augment en même temps d’un facteur m alors

f (mw, mk) = c(mw)α (mk)β = m α+β f (w, k).

Pour que la production augmente elle aussi d’un facteur m il faut imposer α + β = 1.
Les courbes de niveau d’une telle fonction sont nommées isocantes ou courbes d’isoproduction. Pour un niveau fixé
K > 0 de production, l’équation w α k β = K détermine les points du plan (w, k) donnant toutes les combinaisons des
quantités de facteurs qui permettent de produire ce niveau K .

Définition Matrice jacobienne


Dans certaines applications, on est amené à considérer des fonctions de D ⊂ Rn → Rm qui peuvent être vues comme des
empilements de fonctions de D ⊂ Rn → R :

f : D ⊂ Rn → Rm
 
f 1 (x)
 f (x) 
 2 
 ..  .
x 7→  
 . 
f m (x)

De telles fonctions s’étudient comme s’il s’agissait de m fonctions de D ⊂ Rn → R. On obtiendra ainsi un vecteur gradient
pour chaque fonction f j , j = 1, . . . , m. La matrice de n lignes et m colonnes dont la colonne j contient le vecteur ∇ f j est
appelée matrice jacobienne de f .

Définition Dérivées des fonctions composées : règle de dérivation en chaîne (chain rule)
Cas R → R2 → R Soit f une fonction de deux variables admettant des dérivées partielles premières et x et y deux fonc-
tions dérivables de R dans R :

f : R2 → R x: R→R y: R→R
(x, y) 7→ f (x, y) t 7→ x(t ) t 7→ y(t )

Alors la fonction g

g:R R2 R
t (x = x(t ), y = y(t )) g (t ) = f (x(t ), y(t ))

est dérivable et
∂f ∂f
g 0 (t ) = (x(t ), y(t )) × x 0 (t ) + (x(t ), y(t )) × y 0 (t ).
∂x ∂y

Cas R2 → R2 → R Soit f une fonction de deux variables x et y elles-même fonctions des deux variables u et v.

f : R2 → R2 x : R2 → R y : R2 → R

© G. Faccanoni 63
4 Dérivabilité et différentiabilité, fonctions implicites Dernière mise à jour : Jeudi 5 juin 2014

(x, y) 7→ f (x, y) (u, v) 7→ x(u, v) (u, v) 7→ y(u, v)

On peut définir la fonction composée g (u, v) = f (x(u, v), y(u, v)) :

g : R2 R2 R
(u, v) (x = x(u, v), y = y(u, v)) g (u, v) = f (x(u, v), y(u, v))

Lorsque les dérivées partielles premières qui interviennent sont définies, on a

∂g ∂f ∂x ∂f ∂y
(u, v) = (x(u, v), y(u, v)) × (u, v) + (x(u, v), y(u, v)) × (u, v),
∂u ∂x ∂u ∂y ∂u
∂g ∂f ∂x ∂f ∂y
(u, v) = (x(u, v), y(u, v)) × (u, v) + (x(u, v), y(u, v)) × (u, v).
∂v ∂x ∂v ∂y ∂v

Exemple Gaz parfaits


La pression P (en kilopascals), le volume V (en litres) et la température T (en kelvins) d’une mole d’un gaz parfait sont liés par l’équa-
tion PV = RT avec R = 8.31. On veut déterminer la vitesse à laquelle la pression change quand la température est de 300 K et est en
train d’augmenter à raison de 0.1 K s−1 et quand le volume est de 100 L et est en train de croître à raison de 0.2 L s−1 .
Soit P une fonction de deux variables admettant des dérivées partielles premières et T et V sont deux fonctions dérivables de R dans
R:

P : R2 → R T : R→R V : R→R
(T,V ) 7→ P (T,V ) t 7→ T (t ) t 7→ V (y)

Alors la fonction P
P :R R2 R
t (T (t ),V (t )) P (T (t ),V (t )) = R VT (t )
(t )

est dérivable et
∂P ∂P
P 0 (t ) = (T (t ),V (t ))T 0 (t ) + (T (t ),V (t ))V 0 (t ).
∂T ∂V
À l’instant considéré, T = 300 et d T /d t = 0.1, V = 100 et dV /d t = 0.2, donc

dP ∂P d T ∂P dV R d T RT dV 3.81 3.81 × 300


= + = − 2 = 0.1 − 0.2 = −0.04155.
dt ∂T d t ∂V d t V dt V dt 100 1002

La pression est donc en train de diminuer d’environ 0.042 kPa s−1 .

Exemple
p température en un point (x, y) est notée T (x, y) et mesurée en degrés Celsius. Un insecte en train de voler suit la trajectoire (x =
La
1 + t , y = 2+ t /3) où t est mesuré en secondes et x et y en centimètres. Si la fonction température vérifie ∂x T (2, 3) = 4 et ∂ y T (2, 3) = 3,
à quelle vitesse croît la température sur la trajectoire de l’insecte après 3 secondes ?
Comme x et y sont chacune fonction du temps t , la fonction T (x, y) est aussi fonction du temps
T :R R2 R
p
t (x(t ), y(t )) T (x(t ), y(t )) = T ( 1 + t , 2 + t /3)

et l’on a
dT ∂T d x ∂T d y ∂T 1 ∂T 1
= + = p + .
dt ∂x d t ∂y d t ∂x 2 1 + t ∂y 3

Après 3 secondes, x = 2 et y = 3 et
dT 1 1
(2, 3) = 4 + 3 = 2.
dt 2×2 3
Ainsi la température croît de 2 ◦C par second.

Exemple
Le consommateur dont la fonction d’utilité est donnée par U (x, y) est soumis à la contrainte de budget P x + Q y = R, où P > 0 et Q > 0
sont les prix des deux biens et R > 0 le montant dévolu à l’achat de ces deux biens. La quantité consommée du second bien devient
ainsi fonction de celle du premier : y = f (x) avec f (x) = R−P x
Q , de sorte que, sous la contrainte de budget, la fonction d’utilité peut être
exprimée sous la forme d’une fonction d’une seule variable : V (x) = U (x, f (x)), qui est la composée des fonctions

y : R → R2 U : R2 → R

64 © G. Faccanoni
Dernière mise à jour : Jeudi 5 juin 2014 4 Dérivabilité et différentiabilité, fonctions implicites

x 7→ f (x) (x, y) 7→ U (x, y)

Grâce à la chain rule, on a


dV P
V 0 (x) = = ∂x U − ∂ y U .
dx Q

Attention
Une fonction f peut admettre des dérivées partielles en
tout point d’un ouvert de R2 et ne pas être continue en
tout point de cet ouvert.

Exemple
La fonction
( xy
si (x, y) 6= (0, 0)
f (x, y) = x 2 +y 2
0 sinon

n’est pas continue en (0, 0) car f (x, x) = 1/2 6= f (0, 0), cepen-
dant elle admet des dérivées partielles en (0, 0) car ∂x f (0, 0) =
f (h,0)− f (0,0) f (0,h)− f (0,0)
limh→0 h
= 0 et ∂ y f (0, 0) = h
= 0.

Définition Fonction de classe C 1


Si f et ses fonctions dérivées partielles ∂x f et ∂ y f sont continues sur D, on dit que f est de classe C 1 sur D et on note
f ∈ C 1 (D).

Les directions correspondant aux dérivées partielles sont celles des axes. D’autres dérivées directionnelles sont aussi envisa-
geables :

Définition Dérivée directionnelle


Les dérivées partielles d’une fonction de deux variables décrivent le taux de variation de cette fonction lorsqu’une de ses
variable varie, l’autre restant constante. De manière plus générale, on peut s’intéresser au taux de variation de f lorsque
ses arguments (x, y) varient dans une direction fixée. La dérivée de f en (x 0 , y 0 ) selon la direction v = (a, b) est définie par

∂f f (x 0 + ha, y 0 + hb) − f (x 0 , y 0 )
(x 0 , y 0 ) = lim .
∂v h→0 h
∂f
Lorsque f est différentiable (cf. prochaine section), cette limite vaut ∂v (x 0 , y 0 ) = ∇ f (x 0 , y 0 )·v = ∂x f (x 0 , y 0 )a+∂ y f (x 0 , y 0 )b.

Exemple
Calculons la dérivée de f = x 2 − y 2 en (1, 2) selon la direction v = (3, 5) :
∂f 2 2 )−(12 −22 ) 2
B en appliquant la définition on a ∂v
(1, 2) = limh→0 ((1+3h) −(2+5h) h
= limh→0 −16hh−14h = −14 ;
∂f
B comme f est différentiable, on a ∂x f (1, 2) = 2 et ∂ y f (1, 2) = −4 donc ∂v (1, 2) = ∇ f (1, 2) · v = 3∂x f (1, 2) + 5∂ y f (1, 2) = 6 − 20 = −14.

Élasticité
Soit f une fonction à valeurs réelles définie sur une partie ouverte D de R2 . Soit (x 0 , y 0 ) ∈ D et supposons f dérivable par
rapport à x (resp. y) en (x 0 , y 0 ) avec x 0 6= 0 (resp. y 0 6= 0) et f (x 0 , y 0 ) 6= 0. L’élasticité (partielle) de f par rapport à x (resp. y)
en (x 0 , y 0 ) est le changement proportionnel de la fonction f relativement à la variable x (resp. y) en considérant l’autre

© G. Faccanoni 65
4 Dérivabilité et différentiabilité, fonctions implicites Dernière mise à jour : Jeudi 5 juin 2014

variable constante et est donc définie par les relations :


f (x 0 +h,y 0 )− f (x 0 ,y 0 ) f (x 0 ,y 0 +h)− f (x 0 ,y 0 )
f (x 0 ,y 0 ) y f (x 0 ,y 0 )
E xf (x 0 , y 0 ) = lim h
, E f (x 0 , y 0 ) = lim h
.
h→0 h→0
x0 y0

Par conséquent
x0 y y0
E xf (x 0 , y 0 ) = ∂x f (x 0 , y 0 ), E f (x 0 , y 0 ) = ∂ y f (x 0 , y 0 ).
f (x 0 , y 0 ) f (x 0 , y 0 )

Si la valeur absolue de l’élasticité est > 1, on dit que la fonction est élastique, si elle est < 1 on dit qu’elle est rigide et si elle
est = 1 on dit qu’elle est unitaire. Par exemple, une élasticité E xf de −2 signifie qu’une augmentation des x (par exemple
le prix) de 1% diminue de 2% la quantité f (par exemple la quantité vendue). Le concept de l’élasticité peut être employé
toutes les fois qu’il y a un rapport de cause et d’effet. Elle est souvent exprimée pour une variation de 1% de x (resp. y). Si
y
E xf (x 0 , y 0 ) = 0 (resp. E f (x 0 , y 0 ) = 0), alors f est parfaitement inélastique relativement à la variable x (resp. y).

Exemple
La fonction de production de C OBB -D OUGLAS f : R2+ → R s’écrit f (x, y) = x α y β (avec α, β > 0). Elle est dérivable dans (R∗ 2
+ ) et on
obtient :
x x
E xf (x, y) = ∂x f (x, y) = αx α−1 y β = α,
f (x, y) x yβ
α
y y
βx α y β−1 = β.
y
E f (x, y) = ∂ y f (x, y) =
f (x, y) xα y β

En conséquence, toutes les fonctions de C OBB -D OUGLAS bénéficient d’élasticités constantes, ce qui contribue à expliquer l’attrait
qu’elles représentent pour le modélisateur.

Exemple
p
La fonction de demande D d’un bien dépend du prix p et des revenues r du consommateur. Si D(p, r ) = 100 − p + r , on a
p
p p −p r r r
E D (p, r ) = ∂p D(p, r ) = p , ED (p, r ) = ∂r D(p, r ) = p .
f (p, r ) 100 − p + r D(p, r ) 2(100 − p + r )
p r (200, 12000) = 5.74 >
Si p = 200 et r = 12000, on a E D (200, 12000) = −20.95 < 0 (la demande est une fonction décroissante du prix) et E D
0 (la demande est une fonction croissante des revenues). Dans les deux cas, la demande est élastique car > 1 en valeur absolue.

4.2 Différentiabilité
La différentiabilité d’une fonction f en un point x0 correspond à l’existence d’une approximation linéaire de la fonction au
voisinage du point (x0 , f (x0 )) du graphe de la fonction.
Pour les fonctions d’une seule variable, cette approximation linéaire est fournie par la droite tangente d’équation y = f (x 0 ) +
f 0 (x)(x − x 0 ), impliquant directement l’équivalence entre la dérivabilité et la différentiabilité. Il n’était donc pas nécessaire
d’ajouter une définition.
Dans le cas des fonctions de deux variables et plus, l’équivalence disparaît entre l’existence des dérivées partielles, d’une
part, et celle d’un plan tangent, d’autre part. Cela provient du fait que la dérivabilité repose seulement sur des limites le long
de directions particulières. La dérivabilité apparaît donc comme un concept trop faible pour garantir l’existence d’un plan
tangent et la notion de différentiabilité va combler ce déficit.
Pour les fonctions de deux variables, l’intuition géométrique peut encore servir de guide. Ainsi, si la fonction f est déri-
vable en (x 0 , y 0 ), on peut affirmer l’existence de deux droites tangentes, chacune par rapport à la trace verticale du graphe
dans les plans d’équation x = x 0 et y = y 0 . Dans le meilleur des cas, ces deux droites, nécessairement concourantes en
(x 0 , y 0 , f (x 0 , y 0 )), forment un plan qui est tangent au graphe. Toutefois, certaines irrégularités peuvent surgir (par exemple, la
présence d’une discontinuité en (x 0 , y 0 )) qui excluent l’existence d’un plan tangent. Dans pareil cas, les deux droites existent

66 © G. Faccanoni
Dernière mise à jour : Jeudi 5 juin 2014 4 Dérivabilité et différentiabilité, fonctions implicites

et définissent un plan qui n’est pas un plan tangent, parce qu’un tel plan n’existe pas. Ces deux droites déterminent donc un
«candidat plan tangent», dont l’existence doit encore être vérifiée.
Plus généralement, la différentiabilité d’une fonction de n variables, dérivable au point x0 , s’étudie en deux étapes. La pre-
mière consiste à introduire la «candidate différentielle». La seconde teste si cette candidate constitue effectivement une ap-
proximation locale de l’accroissement de la fonction. Les définitions suivantes précisent ces notions.

Définition Fonction différentiable


Soit f une fonction à valeurs réelles définie sur une partie ouverte D de R2 . Soit (x 0 , y 0 ) ∈ D. On dit que f est différentiable
en (x 0 , y 0 ) s’il existe deux constantes réelles A et B telles que

f (x 0 + h, y 0 + k) − f (x 0 , y 0 ) = h A + kB + k(h, k)kε(h, k)

avec lim(h,k)→(0,0) ε(h, k) = 0 et k·k une norme quelconque de R2 .

La différentiabilité est une notion plus forte que la continuité et que la dérivabilité :
Théorème
Soit f une fonction à valeurs réelles définie sur une partie ouverte D de R2 . Soit (x 0 , y 0 ) ∈ D. Si f est différentiable en
(x 0 , y 0 ), alors f est continue et dérivable en (x 0 , y 0 ). Dans ce cas on a A = ∂x f (x 0 , y 0 ) et B = ∂ y f (x 0 , y 0 ) et on peut définir
la fonction

df (x0 ,y 0 ) : B((0, 0), δ > 0) ⊂ R2 → R


(h, k) 7→ h∂x f (x 0 , y 0 ) + k∂ y f (x 0 , y 0 ),

appelée différentielle de f au point (x 0 , y 0 ).

Si f est différentiable en (x 0 , y 0 ), en générale on note h = dx, k = dy et on écrit

df (x 0 , y 0 ) = ∂x f (x 0 , y 0 ) dx + ∂ y f (x 0 , y 0 ) dy.

Par conséquent on a la définition/notation suivante :

Définition Différentielle
En tout point (x, y) où f est différentiable, on définit la différentielle de f comme

df (x, y) = ∂x f (x, y) dx + ∂ y f (x, y) dy.

Exemple
Soit f : R2 → R la fonction définie par f (x, y) = 1 + x 2 − x y. La différentielle de f au point (x 0 , y 0 ) est la fonction

df (x0 ,y 0 ) : B((x 0 , y 0 ), δ > 0) ⊂ R2 → R


(h, k) 7→ h(2x 0 − y 0 ) + k(−x 0 ).

Si on note h = dx et k = dy, elle se réécrit

df (x0 ,y 0 ) : B((x 0 , y 0 ), δ > 0) ⊂ R2 → R


(d x, d y) 7→ (2x 0 − y 0 ) dx − x 0 dy.

et on écrit
d f (x, y) = (2x − y) dx − x dy.

Théorème
Soit f une fonction à valeurs réelles définie sur une partie ouverte D de R2 . Soit (x 0 , y 0 ) ∈ D. Si f est de classe C 1 au
voisinage de (x 0 , y 0 ), alors f est différentiable en (x 0 , y 0 ). La réciproque est fausse.

© G. Faccanoni 67
4 Dérivabilité et différentiabilité, fonctions implicites Dernière mise à jour : Jeudi 5 juin 2014

Exemple
1. La fonction f : R2 → R définie par
 x y2

si (x, y) 6= (0, 0)
f (x, y) = x 2 +y 4
0 si (x, y) = (0, 0)

n’est pas continue en (0, 0) puisque lim y→0 f (y 2 , y) = 21 =


6 f (0, 0). Elle n’est donc pas différentiable en (0, 0).
2
2. La fonction f : R → R définie par f (x, y) = |x| + |y| n’est pas dérivable en (x, 0) pour tout x ∈ R et n’est pas dérivable en (0, y)
pour tout y ∈ R, donc n’est pas différentiable en tous ces points.

Notons que les fonctions élémentaires telles que les polynômes, les fonctions exponentielles, logarithmiques et trigonomé-
triques sont automatiquement différentiables dans leur domaine respectif et que les propriétés de différentiabilité relatives
aux sommes, produits, etc., existent.

Théorème
Soit f une fonction à valeurs réelles définie sur une partie ouverte D de R2 . Soit (x 0 , y 0 ) ∈ D un point en lequel f est
dérivable. Elle est différentiable en (x 0 , y 0 ) ssi

f (x 0 + h, y 0 + k) − f (x 0 , y 0 ) − h∂x f (x 0 , y 0 ) − k∂ y f (x 0 , y 0 )
lim p = 0.
(h,k)→(0,0) h2 + k 2

Exemple
1. La fonction f (x, y) = x y − 2x + 3y est différentiable en (0, 0) car

f (h, k) − f (0, 0) − h∂x f (0, 0) − k∂ y f (0, 0) hk − 2h + 3k − h(−2)) − k(3) hk


lim p = lim p = lim p =0
(h,k)→(0,0) 2
h +k 2 (h,k)→(0,0) 2
h +k 2 (h,k)→(0,0) h2 + k 2

puisque p hk = r cos(ϑ) sin(ϑ) et |r cos(ϑ) sin(ϑ)| ≤ r −−−→ 0.


2 h +k 2 r →0
2. Soit f : R2 → R la fonction définie par f (x, y) = x 2 + y 2 . Si f est différentiable, alors sa différentielle est la fonction

df (x0 ,y 0 ) : Rn → R
(h, k) 7→ 2hx 0 + 2k y 0 .

Afin de voir s’il s’agit effectivement d’une différentielle, on calcule

(x 0 + h)2 + (y 0 + k)2 − x 02 − y 02 − 2hx 02 − 2k y 02 h2 + k 2 p


lim p = lim p = lim h 2 + k 2 = 0.
(h,k)→(0,0) h2 + k 2 (h,k)→(0,0) h 2 + k 2 (h,k)→(0,0)

Par conséquent, la fonction f est différentiable dans R2 et pour tout (x, y) ∈ R2 on a df = 2x dx + 2y dy.

Astuce
En résumé, pour répondre à la question “la fonction f : Rn → R est-elle différentiable en x0 ∈ Rn ?” il est utile de se rappeler
le schéma suivant :
f continue

⇐ si n = 1
: si n > 1

:
1 ⇒
f ∈C
;

: f différentiable

:s
in>
⇐ 1
si n
=1 f dérivable

Contrexemples
Soit f : R2 → R définie par

(
g (x, y) si (x, y) 6= (0, 0)
f (x, y) =
0 si (x, y) = (0, 0)

68 © G. Faccanoni
Dernière mise à jour : Jeudi 5 juin 2014 4 Dérivabilité et différentiabilité, fonctions implicites

x2 y 2
B Si g (x, y) = , f est de classe C 1 (R2 )
x2 + y 2
P REUVE : il suffit de remarquer que
 2 2
 x y si (x, y) 6= (0, 0)
| f (r cos(ϑ), r sin(ϑ))| ≤ r 2 −−−→ 0
2 2
f (x, y) = x +y
0 si (x, y) = (0, 0) r →0

4

 2x y si (x, y) 6= (0, 0)
2 2 2
∂x f (x, y) = (x +y ) |∂x f (r cos(ϑ), r sin(ϑ))| ≤ 2r −−−→ 0
f (h,0)− f (0,0) r →0
limh→0 =0 si (x, y) = (0, 0)

h
4y

2x

2 2 2 si (x, y) 6= (0, 0)
∂ y f (x, y) = (x +y ) |∂ y f (r cos(ϑ), r sin(ϑ))| ≤ 2r −−−→ 0
f (0,k)− f (0,0) r →0
limk→0 =0 si (x, y) = (0, 0)

k

donc f , ∂x f et ∂ y f sont continues sur R2 .


à !
2 2 1
B Si g (x, y) = (x + y ) sin p alors f n’est pas de classe C 1 (R2 ) mais elle est différentiable sur R2
x2 + y 2
P REUVE : on a
 Ã !
(x 2 + y 2 ) sin q 1

si (x, y) 6= (0, 0)
µ ¶
1
f (x, y) = x 2 +y 2 f (r cos(ϑ), r sin(ϑ)) = r 2 sin −−−→ 0
 r r →0
si (x, y) = (0, 0)

0
 Ã ! Ã !
2x sin q 1 −q x cos q 1

si (x, y) 6= (0, 0)

x
µ ¶ µ ¶
1 1
∂x f (x, y) = x 2 +y 2 x 2 +y 2 x 2 +y 2 ∂x f (x, 0) = 2x sin − cos −−−→0


 f (h,0)− f (0,0) |x| |x| |x|  
x→0
lim
h→0 h
=0 si (x, y) = (0, 0)
 Ã ! Ã !
y
2y sin q 1 cos q 1

−q si (x, y) 6= (0, 0)

y
µ ¶ µ ¶
 1 1
∂ y f (x, y) = x 2 +y 2 x 2 +y 2 x 2 +y 2 ∂ y f (0, y) = 2y sin − cos −−−→0

 f (0,k)− f (0,0) |y| |y| |y|  
y→0

lim
k→0 k
=0 si (x, y) = (0, 0)

donc f est continue sur R2 , les dérivées partielle ∂x f et ∂ y f existent sur R2 mais elles ne sont pas continues en (0, 0). Bien que f ne
soit pas de classe C 1 (R2 ), elle est différentiable sur R2 car
³ ´
f (h, k) − f (0, 0) − h∂x f (0, 0) − k∂ y f (0, 0) (h 2 + k 2 ) sin p 21 2 µ ¶
1
h +k
lim p = lim p = lim r sin = 0.
(h,k)→(0,0) 2
h +k 2 (h,k)→(0,0) 2
h +k 2 r →0 r

B Si g (x, y) = (x y)1/3 , f n’est pas différentiable mais elle est continue et dérivable
f (h,0)− f (0,0)
P REUVE : f est clairement continue en (0, 0). Elle admet des dérivées partielles en (0, 0) car ∂x f (0, 0) = limh→0 h
= 0 et
f (0,h)− f (0,0)
∂ y f (0, 0) = h
= 0. Cependant elle n’est pas différentiable en (0, 0) car

f (h, k) − f (0, 0) − h∂x f (0, 0) − k∂ y f (0, 0) (hk)1/3


lim p = lim p 6= 0
(h,k)→(0,0) h2 + k 2 (h,k)→(0,0) h 2 + k 2

−1/3
puisque la restriction à la courbe continue d’équation k = h qui passe par (0, 0) donne hp −−−→ +∞.
2 h→0
xy
B Si g (x, y) = 2 , f n’est pas continue mais elle est dérivable
x + y2
P REUVE : f n’est pas continue en (0, 0) car f (x, x) = 1/2 6= f (0, 0), cependant elle admet des dérivées partielles en (0, 0) car ∂x f (0, 0) =
f (h,0)− f (0,0) f (0,h)− f (0,0)
limh→0 h
= 0 et ∂ y f (0, 0) = limh→0 h
= 0.
B Si g (x, y) = |x| + |y|, f n’est pas dérivable mais elle est continue
P REUVE : f est clairement continue en (0, 0), cependant elle n’admet pas de dérivées partielles en (0, 0) car les deux limites
f (h,0)− f (0,0) f (0,h)− f (0,0)
limh→0 h
et limh→0 h
n’existent pas.
y
B Si g (x, y) = , f n’est ni dérivable ni continue
x2 + y 2
P REUVE : f n’est pas continue en (0, 0) car f (y, y) = 1/y −−−→ ∞. Elle n’admet pas de dérivée partielle ∂ y f (0, 0) car
y→0
f (0,h)− f (0,0)
limh→0 h
= 12 −−−→ ∞.
h y→0

4.2.1 Développement limité à l’ordre 1 et plan tangent

À l’approche analytique de la notion de différentiabilité correspond la vision géométrique d’un plan tangent. En effet, lorsque
f est différentiable en (x 0 , y 0 ), on peut, dans un voisinage de (x 0 , y 0 ), approcher la différence E (h, k) ≡ f (x 0 + h, y 0 + k) −

© G. Faccanoni 69
4 Dérivabilité et différentiabilité, fonctions implicites Dernière mise à jour : Jeudi 5 juin 2014

F IGURE 4.1: La fonction f (x, y) = 2x 2 + y semble coïncider avec son plan tangent au point (1, 1, f (1, 1)) lorsqu’on zoom vers
ce point. Si on regarde les courbes de niveau, lorsque on zoom vers ce point les courbes tendent vers des droites
parallèles toutes à la même distance les unes des autres.

f (x 0 , y 0 ) par la différentielle df (x0 ,y 0 ) (h, k), ce qui revient à approcher f (x 0 + h, y 0 + k) par le plan d’équation f (x 0 , y 0 ) +
df (x0 ,y 0 ) (h, k) (voir la figure 4.1).

Définition Plan tangent et linéarisation


Soit D une partie ouverte de R2 et soit f : D → R une fonction différentiable en (x 0 , y 0 ). L’équation du plan tangent au
graphe de la fonction f (x, y) en (x 0 , y 0 ) est

L(x, y) = f (x 0 , y 0 ) + (x − x 0 )∂x f (x 0 , y 0 ) + (y − y 0 )∂ y f (x 0 , y 0 ).

Si on approche une fonction f au voisinage d’un point (x 0 , y 0 ) au moyen d’une fonction affine, il est naturel de choisir
une fonction dont le graphe est tangent au graphe de la fonction f en (x 0 , y 0 ). C’est ce qu’on appelle la linéarisation de f
en (x 0 , y 0 ).

Graphe de la fonction (x, y) 7→ f (x, y) et sa linéarisation


(x, y) 7→ L(x, y) au voisinage du point (a, b). La fonction
(h, k) 7→ E (h, k) ≡ f (a +h, b +k)−L(a +h, b +k) mesure l’er-
reur qu’on fait au point (a + h, b + k) lorsqu’on approche la
valeur de f par la valeur du plan tangent L. Si f est diffé-
rentiable au point (a, b) alors

E (h, k)
lim p = 0.
(h,k)→(0,0) h2 + k 2

Cette notion se généralise naturellement pour n > 2 : il s’agit en fait d’un plan tangent pour n = 2 et d’un hyperplan tangent
pour n > 2. Dans un espace de dimension n, un hyperplan est une variété linéaire de dimension n − 1.

Exemple
On veut montrer que la fonction f : R2 → R définie par f (x, y) = xe x y est différentiable en (1, 0) et utiliser sa linéarisation pour appro-
cher f (1.1, −0.1). On a

∂x f (x, y) = e x y + x ye x y ∂x f (1, 0) = 1
∂ y f (x, y) = x 2 e x y ∂ y f (1, 0) = 1

Les deux fonctions ∂x f et ∂ y f sont continues, donc f est différentiable. Sa linéarisation donne

f (x, y) ' f (1, 0) + (x − 1)∂x f (1, 0) + (y − 0)∂ y f (1, 0) = x + y,

autrement dit xe x y ' x + y lorsque (x, y) ' (1, 0), ainsi f (1.1, −0.1) ' 1.1 − 0.1 = 1. En effet, f (1.1, −0.1) = 1.1e −0.11 ≈ 0.98542

70 © G. Faccanoni
Dernière mise à jour : Jeudi 5 juin 2014 4 Dérivabilité et différentiabilité, fonctions implicites

Exemple
q
Considérons la fonction f : R2 → R définie par f (x, y) = x 2 + y 2 . Elle n’est pas différentiable en (0, 0) car elle n’est pas linéaire au
voisinage de (0, 0), autrement dit, faire un zoom vers (0, 0) ne donne pas le graphe d’un plan. Pour prouver cela analytiquement, il suffit
de montrer qu’il n’existe aucune approximation linéaire de f au voisinage de (0, 0). Chaque plan qui passe par le point (0, 0, f (0, 0))
aura équation L(x, y) = ax + b y pour certaines constantes a et b.
Si on pose E (h, k) ≡ f (0 + h, 0 + k) − L(h, k), alors
p
E (h, k) h 2 + k 2 − ah − bk
lim p = lim p .
(h,k)→(0,0) h 2 + k 2 (h,k)→(0,0) h2 + k 2
h
Si on prend k = 0 on a limh→0 1 − a |h| qui ne vaut pas 0. En effet,
∂x f (0, 0) n’existe pas car

f (h, 0) − f (0, 0) |h|


∂x f (0, 0) = lim = lim
h→0 h h→0 h

et cette limite n’existe pas.

4.2.2 Estimation d’erreurs

On rencontre couramment en physique le problème suivant : on a une quantité Q, fonction connue des quantités x ≡
(x 1 , x 2 , . . . ). Ayant fait des mesures des quantités x 1 , x 2 , . . . avec une certaine incertitude, on se demande avec quelle incer-
titude est connue Q. Mathématiquement, on dispose des objets suivants :
B un rectangle R = x ∈ Rn | |x − x i | < r i , i = 1, . . . , n , les nombres r i représentent les erreurs maximales de mesure,
© ª

B une fonction Q : R → R,
B un point xM (c’est le point qu’on mesure),
B la valeur Q(xM ).
On ne connait pas Q(x) pour tout x ∈ R mais on peut estimer sa valeur à partir de la différentielle de Q évaluée au point xM
en appliquant le théorème suivant qui est une conséquence de la formule de TAYLOR à une variable appliquée à la fonction
−−→
g (z) = f (xM + z xM x).
Proposition Estimation d’erreurs
Considérons n constantes positives r i et R un rectangle de Rn centré en xM défini par

R = x ∈ Rn ¯ |x − x i | < r i , i = 1, . . . , n .
© ¯ ª

Soit Q : R → R une fonction de classe C 1 (R). Alors, pour tout x ∈ R, on a la majoration suivante :
n
X n
X
|Q(x) −Q(xM )| ≤ sup |(x i − (x M )i )∂xi Q(x)| ≤ Ci ri
x∈R i =1 i =1

avec C i un majorant de |∂xi Q(x)| sur R.

Estimation d’erreurs dans R2


Soit r 1 et r 2 deux constantes positives et R un rectangle de R2 centré en (x 0 , y 0 ) défini par

R = (x, y) ∈ R2 ¯ |x − x 0 | < r 1 , ¯ y − y 0 ¯ < r 2 .


© ¯ ¯ ¯ ª

© G. Faccanoni 71
4 Dérivabilité et différentiabilité, fonctions implicites Dernière mise à jour : Jeudi 5 juin 2014

Soit f : R → R une fonction de classe C 1 (R). Alors, pour tout (x 1 , y 1 ) ∈ R, on a la majoration suivante :
¯ ¡ ¢ ¡ ¢¯
¯ f x 1 , y 1 − f x 0 , y 0 ¯ ≤ sup |(x 1 − x 0 )∂x f (x, y) + (y 1 − y 0 )∂ y f (x, y)| ≤ Ar 1 + Br 2
(x,y)∈R

avec A (resp. B ) un majorant de |∂x f (x, y)| (resp. |∂ y f (x, y)|) sur R.

4.2.3 Fonctions homogènes

Définition Fonction homogène


Soit la fonction f : Rn → R. Elle est dite homogène de degré k si

f (λx) = λk f (x), ∀(x, λ) ∈ Rn × R∗+ .

Les fonctions homogènes bénéficient de la propriété suivante qui découle de la chain rule.
Théorème d’E ULER
Si la fonction f : Rn → R est homogène de degré k et différentiable dans Rn , alors
n
x i ∂xi f (x) = k f (x), ∀∈Rn .
X
i =1

Exemple
La fonction de production de C OBB -D OUGLAS à rendements constants s’écrit f (x, y) = x α y 1−α , où α ∈ R∗
+ , est homogène de degré 1
puisque f (λx, λy) = (λx)α (λy)1−α = λ(x α y 1−α ) = λ f (x, y). Dans ce cas, la formule d’Euler s’écrit

x∂x f (x, y) + y∂ y f (x, y) = f (x, y).

Le résultat f (x, y) de la production obtenue avec les quantités respectives x et y de facteurs permet donc de rémunérer ces facteurs au
niveau de leur productivité marginale.

4.3 Dérivées partielles de deuxième ordre et matrice hessienne


Si les fonctions dérivées partielles admettent elles-mêmes des dérivées partielles en (x 0 , y 0 ), ces dérivées sont appelées déri-
vées partielles secondes, ou dérivées partielles d’ordre 2, de f en (x 0 , y 0 ). On peut, de la même façon, introduire les dérivées
partielles d’ordres supérieurs. Les définitions suivantes s’énoncent dans des ensembles ouverts pour éviter les problèmes
liés au calcul de limites au bord du domaine.

Définition Dérivées partielles d’ordre 2 pour une fonction de deux variables


Soit la fonction f : D ⊂ R2 → R où D est un ouvert de R2 . On a 2 dérivées partielles d’ordre 1 et donc 4 dérivées partielles
d’ordre 2 ainsi notées :

∂2 f ∂ ∂f
µ ¶
(x 0 , y 0 ) = (x 0 , y 0 ) (notée aussi ∂xx f (x 0 , y 0 )),
∂x 2 ∂x ∂x
∂2 f ∂ ∂f
µ ¶
(x 0 , y 0 ) = (x 0 , y 0 ) (notée aussi ∂x y f (x 0 , y 0 )),
∂x ∂y ∂x ∂y
∂2 f ∂ ∂f
µ ¶
(x 0 , y 0 ) = (x 0 , y 0 ) (notée aussi ∂ y x f (x 0 , y 0 )),
∂y ∂x ∂y ∂x
∂2 f ∂ ∂f
µ ¶
(x 0 , y 0 ) = (x 0 , y 0 ) (notée aussi ∂ y y f (x 0 , y 0 )).
∂y 2 ∂y ∂y

Les dérivées partielles d’ordre supérieur à 2 se définissent par récurrence de façon analogue.

72 © G. Faccanoni
Dernière mise à jour : Jeudi 5 juin 2014 4 Dérivabilité et différentiabilité, fonctions implicites

Théorème Théorème de S CHWARZ (ou de C LAIRAUT)


Si les dérivées partielles mixtes ∂x y f et ∂ y x f sont continues en (x 0 , y 0 ) alors ∂x y f (x 0 , y 0 ) = ∂ y x f (x 0 , y 0 ).

Relations de M AXWELL et Carré thermodynamique


En thermodynamique, on appelle relations de M AXWELL des équations obtenues grâce aux définitions des potentiels
thermodynamiques et à l’égalité de S CHWARZ. Pour un système entièrement décrit par les variables pression P , tempéra-
ture T , entropie S et volume V , on retient généralement un ensemble de quatre relations relatives aux quatre potentiels
thermodynamiques énergie interne U , enthalpie H , énergie libre F et enthalpie libre (ou potentiel de G IBBS) G :

∂F ¯¯ ∂F ¯¯
¯ ¯
dF (V, T ) = −P (V, T )dV − S(V, T )dT, =⇒ P (V, T ) = − , S(V, T ) = − ;
∂V ¯T ∂T ¯V
∂G ¯¯ ∂G ¯¯
¯ ¯
dG(P, T ) = V (P, T )dP − S(P, T )dT, =⇒ V (P, T ) = , S(P, T ) = − ;
∂P ¯T ∂T ¯P
∂U ¯¯ ∂U ¯¯
¯ ¯
dU (S,V ) = T (S,V )dS − P (S,V )dV, =⇒ T (S,V ) = − , P (S,V ) = − ;
∂S ¯V ∂V ¯S
∂H ¯¯ ∂H ¯¯
¯ ¯
dH (P, S) = V (P, S)dP + T (P, S)dS, =⇒ V (P, S) = , T (P, S) = ;
∂P S¯ ∂S ¯P

si bien qu’en utilisant l’égalité de S CHWARZ l’on a les relations de M AXWELL

∂P ¯¯ ∂S ¯¯ ∂V ¯¯ ∂S ¯¯ ∂T ¯¯ ∂P ¯¯ ∂T ¯¯ ∂V ¯¯
¯ ¯ ¯ ¯ ¯ ¯ ¯ ¯
= , =− , =− , = .
∂T ¯V ∂V ¯T ∂T ¯P ∂P ¯T ∂V ¯S ∂S ¯V ∂P ¯S ∂S ¯P

Le «carré thermodynamique» ci-dessous a été conçu pour mémoriser simplement l’ensemble de ces relations (différen-
tielles des potentiels thermodynamiques et relations de M AXWELL).
f
V T

ε g

S P
h
Le long des quatre cotés on écrit les quatre potentiels thermodynamiques f , g , h et ε, disposés en ordre alphabétique et
en sens horaire, à partir de f sur le coté haut. Aux deux coins de gauche on pose les variables extensives V et S tandis
qu’aux coins de droite on pose les variables intensives P et T .
Chacun des quatre potentiels thermodynamiques se trouve entre les variables indépendantes des quelles il est fonction
naturelle. On a donc que ² est fonction naturelle de V et S ; f de V et T etc.
Ce carré permet de lire facilement les dérivées partielles qui lient les potentiels aux variables naturelles respectives : plus
précisément, la dérivée partielle d’un potentiel par rapport à une variable (coin) est donnée simplement par la variable
qui se trouve au coin opposé et les flèches diagonales en déterminent le signe (positif dans la direction de la flèche, négatif
∂f ¯ ∂g ¯
¯ ¯
sinon). Par exemple, on a (V, T ) 7→ f et ∂V ¯ = −P mais (P, T ) 7→ g et ∂P ¯ = V .
T T
Même les relations de M AXWELL peuvent se lire sur le schéma : la dérivée de la variable (coin) par rapport à la variable sur
la même arrête (en considérant constante la variable dans la diagonale opposée) est égale à la dérivée le¯long de¯ l’autre
diagonale. De nouveau, les signes doivent être choisis selon la direction des diagonales. Par exemple ∂V ∂T ¯
∂S ¯P = ∂P ¯S mais
¯
¯ ¯
∂T ¯ ∂P ¯
∂V ¯ = − ∂S ¯ .
S V

© G. Faccanoni 73
4 Dérivabilité et différentiabilité, fonctions implicites Dernière mise à jour : Jeudi 5 juin 2014

4.3.1 Développements limités à l’ordre 2

La linéarisation d’une fonction f en un point (x 0 , y 0 ) est un polynôme de degré inférieur ou égal à 1 tel que

L(x 0 , y 0 ) = f (x 0 , y 0 ),

∂x L(x 0 , y 0 ) = ∂x f (x 0 , y 0 ),

∂ y L(x 0 , y 0 ) = ∂ y f (x 0 , y 0 ).

Les polynômes de TAYLOR généralisent cette construction pour des polynômes de degrés quelconques. Ici on va se limiter
aux polynômes de degré au plus 2.

Définition Développement limité à l’ordre 2 dans R2


Soit f : D ⊂ R2 → R une fonction de classe C 2 (D). Alors au voisinage de (x 0 , y 0 ) ∈ D on a

f (x, y) = f (x 0 , y 0 )
+ (x − x 0 )∂x f (x 0 , y 0 ) + (y − y 0 )∂ y f (x 0 , y 0 )

+ (x − x 0 )2 ∂xx f (x 0 , y 0 ) + 2(x − x 0 )(y − y 0 )∂x y f (x 0 , y 0 ) + (y − y 0 )2 ∂ y y f (x 0 , y 0 )
¤
2
+ o((x − x 0 )2 + (y − y 0 )2 ).

Définition Matrice hessienne


Soit la fonction f : D ⊂ R2 → R où D est un ouvert de R2 . La matrice hessienne de f en (x 0 , y 0 ) est la matrice de taille 2 × 2
dont les entrées sont les dérivées partielles secondes :

∂xx f (x 0 , y 0 ) ∂x y f (x 0 , y 0 )
µ ¶
H f (x 0 , y 0 ) = .
∂ y x f (x 0 , y 0 ) ∂ y y f (x 0 , y 0 )

Son déterminant est le réel dét(H f (x 0 , y 0 )) ≡ ∂xx f (x 0 , y 0 )∂ y y f (x 0 , y 0 ) − ∂x y f (x 0 , y 0 )∂ y x f (x 0 , y 0 ).

Cette notion se généralise naturellement pour n > 2.

Exemple
Les dérivées premières et secondes de la fonction f (x, y) = −2x 2 + 3x y 2 − y 3 sont

∂x f (x, y) = −4x + 3y 2 , ∂ y f (x, y) = 6x y − 3y 2 ,


∂xx f (x, y) = −4, ∂x y f (x, y) = 6y, ∂ y x f (x, y) = 6y, ∂ y y f (x, y) = 6x − 6y.

La matrice hessienne est µ ¶


−4 6y
H f (x, y) = .
6y 6x − 6y
Dans cet exemple, on remarque que la matrice hessienne de f est symétrique du fait que les dérivées secondes mixtes, ∂x y f et ∂ y x f ,
sont égales.

Remarque Développement limité à l’ordre 2 dans Rn


La notation matricielle permet d’écrire le développement limité à l’ordre 2 d’une manière plus compacte et généralisable
à un nombre quelconque de variables : soit f : D ⊂ Rn → R une fonction de classe C 2 (D). Alors pour tout x ∈ B(x0 , δ) ∩ D
on a
1
f (x) = f (x0 ) + (x − x0 )T · ∇ f (x0 ) + (x − x0 )T · H f (x0 ) · (x − x0 ) + kx − x0 k2 ε(x)
2
avec limx→x0 ε(x) = 0.

Définition Fonction de classe C k


Si les fonctions dérivées partielles d’ordre k sont continues sur D, on dit que f est de classe C k sur D. Si les dérivées
partielles de tout ordre existent, f est dite de classe C ∞ sur D.

74 © G. Faccanoni
Dernière mise à jour : Jeudi 5 juin 2014 4 Dérivabilité et différentiabilité, fonctions implicites

Comme la dérivée seconde pour les fonctions d’une seule variable, la matrice hessienne permet d’étudier la convexité des
fonctions de plusieurs variables et joue, dès lors, un rôle important dans leur optimisation.

Définition Convexité
Soit D un sous-ensemble convexe de R2 et f : D → R une fonction.
B f est concave dans D si

∀(x 0 , y 0 ), (x 1 , y 1 ) ∈ D et ∀t ∈ [0; 1];


¡ ¢
f (1 − t )(x 0 , y 0 ) + t (x 1 , y 1 ) ≥ (1 − t ) f (x 0 , y 0 ) + t f (x 1 , y 1 )

B f est strictement concave dans D si

∀(x 0 , y 0 ), (x 1 , y 1 ) ∈ D et ∀t ∈]0; 1[;


¡ ¢
f (1 − t )(x 0 , y 0 ) + t (x 1 , y 1 ) > (1 − t ) f (x 0 , y 0 ) + t f (x 1 , y 1 )

B f est convexe dans D si

∀(x 0 , y 0 ), (x 1 , y 1 ) ∈ D et ∀t ∈ [0; 1];


¡ ¢
f (1 − t )(x 0 , y 0 ) + t (x 1 , y 1 ) ≤ (1 − t ) f (x 0 , y 0 ) + t f (x 1 , y 1 )

B f est strictement convexe dans D si

∀(x 0 , y 0 ), (x 1 , y 1 ) ∈ D et ∀t ∈]0; 1[;


¡ ¢
f (1 − t )(x 0 , y 0 ) + t (x 1 , y 1 ) < (1 − t ) f (x 0 , y 0 ) + t f (x 1 , y 1 )

Comme pour les fonctions d’une variable, la concavité et la convexité des fonctions de n variables suffisamment régulières
peuvent être caractérisées à l’aide des dérivées d’ordres 1 ou 2.
Propriété
Soit D un sous-ensemble convexe de R2 et f : D → R une une fonction différentiable dans D. Alors
1. f est concave dans D ⇐⇒ ∀(x 0 , y 0 ), (x 1 , y 1 ) ∈ D, f (x 1 , y 1 ) ≤ f (x 0 , y 0 ) + (x 1 − x 0 )∂x f (x 0 , y 0 ) + (y 1 − y 0 )∂ y f (x 0 , y 0 ) ;
2. f est convexe dans D ⇐⇒ ∀(x 0 , y 0 ), (x 1 , y 1 ) ∈ D, f (x 1 , y 1 ) ≥ f (x 0 , y 0 ) + (x 1 − x 0 )∂x f (x 0 , y 0 ) + (y 1 − y 0 )∂ y f (x 0 , y 0 ).
Si, de plus, f ∈ C 2 (D), alors
1. f est concave dans D ⇐⇒ ∀(x, y) ∈ D, H f (x, y) est semi-définie négative (i.e. dét(H f (x, y)) ≥ 0 et ∂xx f (x, y) ≤ 0) ;
2. f est convexe dans D ⇐⇒ ∀(x, y) ∈ D, H f (x, y) est semi-définie positive (i.e. dét(H f (x, y)) ≥ 0 et ∂xx f (x, y) ≥ 0) ;
3. ∀(x, y) ∈ D, H f (x, y) est définie négative (i.e. dét(H f (x, y)) > 0 et ∂xx f (x, y) < 0) =⇒ f est strictement concave
dans D ;
4. ∀(x, y) ∈ D, H f (x, y) est définie positive (i.e. dét(H f (x, y)) > 0 et ∂xx f (x, y) < 0) =⇒ f est strictement convexe
dans D.

Les deux premiers énoncés expriment que tout plan tangent au graphe d’une fonction concave (resp. convexe) se trouve au-
dessus (resp. au-dessous) de ce graphe. Les deux derniers, relatifs à la matrice hessienne, rappellent celui qui fait référence au
signe de la dérivée seconde d’une fonction d’une seule variable. De la même manière, la condition stricte ne s’applique que
dans un seul sens. Dès lors, une fonction peut être strictement convexe sans que sa matrice hessienne soit définie positive
en tout point.

Exemple
1. Soit f (x, y) = x 2 + y 2 . On a
B f ∈ C 2 (R2 ),¡ ¢
B H f (x, y) = 20 02 pour tout (x, y) ∈ R2 ,
B dét(H f (x, y)) = 4 > 0 et ∂xx f (x, y) = 2 > 0 donc H f (x, y) est définie positive pour tout (x, y) ∈ R2 .
Il s’en suit que f est strictement convexe dans R2 .
2. Soit f (x, y) = x 4 + y 4 . On a
B f ∈ C 2 (R2 ),³
12x 2 0
´
B H f (x, y) = 0 12y 2
pour tout (x, y) ∈ R2 ,
B dét(H f (x, y)) = 144x 2 y 2 ≥ 0 et ∂xx f (x, y) = 12x 2 ≥ 0 donc H f (x, y) est définie positive pour tout (x, y) ∈ (R∗ )2 et semi-définie
positive en (0, 0).
On peut cependant montrer à l’aide de la définition que la fonction est strictement convexe dans R2 car

f (x, y) = x 4 + y 4 > f (0, 0) + x∂x f (0, 0) + y∂ y f (0, 0) = 0 ∀(x, y) 6= (0, 0).

Exemple Gaz parfaits


Considérons l’entropie d’un gaz parfait en fonction de l’énergie interne spécifique ε et du volume spécifique τ :
2
s : ( R∗
+) → R

© G. Faccanoni 75
4 Dérivabilité et différentiabilité, fonctions implicites Dernière mise à jour : Jeudi 5 juin 2014

(τ, ε) 7→ c v ln ετγ−1 = c v ln(ε) + c v (γ − 1) ln(τ)


³ ´

avec c v et γ > 1 deux constantes. L’entropie s est de classe ∈ C ∞ ((R∗ 2


+ ) ) et l’on a

c v (γ − 1) cv
∂τ s = >0 ∂ε s = >0
τ ε

donc l’entropie est strictement croissante en τ et en ε et

c v (γ − 1) cv
∂ττ s = − <0 ∂τε s = ∂ετ s = 0 ∂εε s = − 2
τ2 ε
c v2 (γ−1)
donc dét(H s (τ, ε)) = > 0 : l’entropie est strictement concave.
τ2 ε 2

4.4 Fonctions implicites


Si b 6= 0, l’équation ax + b y + c = 0 définit une fonction y = −(ax + c)/b. Nous allons généraliser ce fait aux équations du type
f (x, y) = 0 où f est une fonction différentiable : une fonction ϕ(x) est définie implicitement près de x = x 0 par l’équation
f (x, y) = 0 si toutes les solutions de cette équation dans un voisinage de (x 0 , ϕ(x 0 )) sont sur le graphe {(x, y) ∈ R2 | y = ϕ(x)}
de ϕ.

Exemple
Si f (x, y) = x 2 + y 2 − 1, l’équation f (x, y) = 0 est celle d’un cercle de rayon 1 centré en (0, 0). Ce cercle n’est pas globalement le
pgraphe
d’une fonction, cependant p l’équation f (x, y)
p = 0 peut être résolue explicitement pour y. On trouve les deux solutions y = ± 1 − x2.
Les fonctions ϕ1 (x) = 1 − x 2 et ϕ2 (x) = − 1 − x 2 sont définies implicitement par l’équation f (x, y) = 0 près de x = 1.

Théorème de la fonction implicite (cas d’une fonction à deux variables)


Soit f : D ⊂ R2 → R une fonction de classe C 1 au voisinage de (x 0 , y 0 ) ∈ D. Si f (x 0 , y 0 ) = k (k constante réelle) et
∂ y f (x 0 , y 0 ) 6= 0, alors il existe un intervalle ouvert I contenant x 0 et une unique fonction ϕ de I dans R de classe C 1
sur I telle que
1. ϕ(x 0 ) = y 0
2. f (x, ϕ(x)) = k pour tout x ∈ I
∂x f (x, ϕ(x))
3. ϕ0 (x) = − pour tout x ∈ I
∂ y f (x, ϕ(x))
4. la droite tangente à y = ϕ(x) en x = x 0 a pour équation y = ϕ0 (x 0 )(x − x 0 ) + y 0 .
Notons que les points 1 et 3 constituent un problème de C AUCHY dont la solution est bien la fonction cherchée y = ϕ(x) pour
tout x ∈ I .
Exemple
Soit f : R2 → R la fonction définie par f (x, y) = y 2 − y −3x et considérons le point (x 0 , y 0 ) = (2, −2). On a f (2, −2) = 0. Considérons alors
la courbe de niveau 0 : le théorème de la fonction implicite permet d’affirmer qu’au voisinage de x = 2 il existe un intervalle I et une
seule fonction y = ϕ(x) définie implicitement par f (x, y) = 0 telle que
B f (x, ϕ(x)) = 0 pour tout x ∈ I
B ϕ(2) = −2
3
B ϕ0 (x) = 2ϕ(x)−1 pour tout x ∈ I
B la droite tangente au graphe de la courbe d’équation y = ϕ(x) a équation

−∂x f (2, ϕ(2)) 3 3 4


y = (x − 2) + (−2) = (x − 2) −2 = − x − .
∂ y f (2, ϕ(2)) 2ϕ(2) − 1 5 5

En résolvant le problème de C AUCHY


3
ϕ0 (x) = 2ϕ(x)−1
(

ϕ(2) = −2
p
on obtient ϕ(x) = 1− 12x+1
2 .

76 © G. Faccanoni
Dernière mise à jour : Jeudi 5 juin 2014 4 Dérivabilité et différentiabilité, fonctions implicites

y
y 2 − y = 3x

³ ´
−1 1
12 , 2
2
x

−2 y = ϕ(x)

On aurait pu trouver l’expression analytique de ϕ en notant que


p
1 ± 1 + 12x
y 2 − y − 3x = 0 ⇐⇒ y=
2
p
et que le point (−2, 2) appartient à la branche d’équation y = 1− 1+12x
2 .

Formules de dérivation implicite et chain rule


d
B On calcule dx de f (x, ϕ(x)) = k. On obtient

∂x f (x, ϕ(x)) + ∂ y f (x, ϕ(x)) × ϕ0 (x) = 0

d’où
∂x f (x, ϕ(x))
ϕ0 (x) = − .
∂ y f (x, ϕ(x))
d
B On calcule dx de ∂x f (x, ϕ(x)) + ∂ y f (x, ϕ(x)) × ϕ0 (x) = 0. On obtient
³ ´
∂xx f (x, ϕ(x)) + ∂x y f (x, ϕ(x)) + ∂ y y f (x, ϕ(x)) × ϕ0 (x) × ϕ0 (x) + ∂ y f (x, ϕ(x)) × ϕ00 (x) = 0

d’où

∂xx f (x, ϕ(x)) + ∂x y f (x, ϕ(x)) + ∂ y y f (x, ϕ(x)) × ϕ0 (x) × ϕ0 (x)


¡ ¢
00
ϕ (x) = −
∂ y f (x, ϕ(x))
∂ f (x,ϕ(x)) ∂x f (x,ϕ(x))
³ ´
∂xx f (x, ϕ(x)) − ∂x y f (x, ϕ(x)) − ∂ y y f (x, ϕ(x)) ∂xy f (x,ϕ(x)) ∂ y f (x,ϕ(x))
=−
∂ y f (x, ϕ(x))
∂ f (x,ϕ(x)) (∂ f (x,ϕ(x)))2
∂xx f (x, ϕ(x)) − ∂x y f (x, ϕ(x)) ∂xy f (x,ϕ(x)) + ∂ y y f (x, ϕ(x)) (∂x f (x,ϕ(x)))2
y
=−
∂ y f (x, ϕ(x))
(∂xx f (x, ϕ(x)))×(∂ y f (x, ϕ(x)))2 − (∂x y f (x, ϕ(x)))×(∂x f (x, ϕ(x)))×(∂ y f (x, ϕ(x))) + (∂ y y f (x, ϕ(x)))×(∂x f (x, ϕ(x)))2
=−
(∂ y f (x, ϕ(x)))3

Théorème de la fonction implicite (cas d’une fonction à trois variables)


Soit f : D ⊂ R3 → R une fonction de classe C 1 au voisinage de (x 0 , y 0 , z 0 ) ∈ D. Si f (x 0 , y 0 , z 0 ) = k et ∂z f (x 0 , y 0 , z 0 ) 6= 0, alors
il existe un ouvert I ⊂ R2 contenant (x 0 , y 0 ) et une fonction ϕ de I dans R de classe C 1 sur I telle que ϕ(x 0 , y 0 ) = z 0 et pour
tout (x, y) ∈ I

∂x f (x, y, ϕ(x, y)) ∂ y f (x, y, ϕ(x, y))


f (x, y, ϕ(x, y)) = k, ∂x ϕ(x, y) = − , ∂ y ϕ(x, y) = − .
∂z f (x, y, ϕ(x, y)) ∂z f (x, y, ϕ(x, y))

© G. Faccanoni 77
4 Dérivabilité et différentiabilité, fonctions implicites Dernière mise à jour : Jeudi 5 juin 2014

............... Exercices ..............


Calcul de dérivées partielles

Exercice 4.1
Calculer toutes les dérivées partielles d’ordre 1 des fonctions données :

1. f (x, y) = y 5 − 3x y 6. f (x, y, z) = x cos(xz) + ln(2 − sin2 (y + z))


2. f (x, y) = x 2 + 3x y 2 − 6y 5 7. f (x, t ) = e −t cos(πx)
3. f (x, y) = x cos(e x y ) 8. z = (2x + 3y)10
ax+b y
4. f (x, y) = x/y 9. f (x, y) = c x+d y
Rx
5. f (x, y) = x y 10. F (x, y) = y cos(e t ) dt

C ORRECTION .
1. ∂x f (x, y) = −3y et ∂ y f (x, y) = 5y 4 − 3x
2. ∂x f (x, y) = 2x + 3y 2 et ∂ y f (x, y) = 6x y − 30y 4
3. ∂x f (x, y) = cos(e x y ) − x ye x y sin(e x y ) et ∂ y f (x, y) = −x 2 e x y sin(e x y )
4. ∂x f (x, y) = 1/y et ∂ y f (x, y) = −x/y 2
5. ∂x f (x, y) = y x y /x et ∂ y f (x, y) = ln(x)x y Bx y = e y ln(x) donc x > 0
−2 sin(y + z) cos(y + z) −2 sin(y + z) cos(y + z)
6. ∂x f (x, y, z) = cos(xz)−xz sin(xz), ∂ y f (x, y) = et ∂z f (x, y) = −x 2 sin(xz)+
2 − sin2 (y + z) 2 − sin2 (y + z)
7. ∂x f (x, t ) = −πe −t sin(πx) et ∂t f (x, t ) = −e −t cos(πx)
8. ∂x z(x, y) = 20(2x + 3y)9 et ∂ y z(x, y) = 30(2x + 3y)9
(ad −bc)y (bc−ad )x
9. ∂x f (x, y) = (c x+d y)2
et ∂ y f (x, y) = (cx+d y)2
x y
10. ∂x F (x, y) = cos(e ) et ∂ y F (x, y) = − cos(e )

Exercice 4.2
1 mole de gaz parfait qui occupe le volume V à la température T et à la pression P vérifie la relation PV = RT où R est
une constante. Montrer que
∂P ∂V ∂T ∂P ∂V
= −1, T = R.
∂V ∂T ∂P ∂T ∂T

C ORRECTION . On a
∂P T ∂V R ∂T V ∂P R
= −R 2 = = =
∂V V ∂T P ∂P R ∂T V
donc
∂P ∂V ∂T T RV ∂P ∂V RR
= −R 2 = −1, T =T = R.
∂V ∂T ∂P V P R ∂T ∂T V P

Exercice 4.3 Loi de VAN DER WAALS


La loi de VAN DER WAALS pour n moles d’un gaz s’écrit

n2 a
µ ¶
P + 2 (V − nb) = nRT
V

où P est la pression, V le volume, T la température du gaz, R la constant universelle des gaz et a et b deux constantes
positives. Calculer ∂T ∂P
∂P et ∂V .

78 © G. Faccanoni
Dernière mise à jour : Jeudi 5 juin 2014 4 Dérivabilité et différentiabilité, fonctions implicites

C ORRECTION .

∂T V − nb ∂P n(2naV 2 − 4n 2 abV − RT v 3 + 2n 3 ab 2 )
= , = .
∂P nR ∂V V 3 (nb − V )2

Exercice 4.4
Une étude des glaciers a montré que la température T à l’instant t (mesuré en jours) à la profondeur x (mesurée en pied)
peut être modélisée par la fonction
T (x, t ) = T0 + T1 e −λx sin(ωt − λx)
où ω = 2π/365 et λ sont deux constantes positives.
1. Calculer ∂x T et ∂t T .
2. Montrer que T satisfait l’équation de la chaleur ∂t T = k∂xx T pour une certaine constante k.

C ORRECTION .
1. ∂x T = −λT1 e −λx (sin(ωt − λx) + cos(ωt − λx)) et ∂t T = ωT1 e −λx cos(ωt − λx).
∂t T ωT1 e −λx cos(ωt −λx) ω
2. On a ∂xx T = 2λ2 T1 e −λx cos(ωt − λx) donc ∂xx T = 2λ2 T1 e −λx cos(ωt −λx)
= 2λ2
.

Exercice 4.5 Gaz parfait


Pour un gaz parfait, l’énergie interne ε s’écrit en fonction du volume spécifique τ et de l’entropie spécifique s comme

ε : R2 → R
(τ, s) 7→ τ1−γ e s/c v

où γ > 1 et c v > 0 sont deux constantes. Sachant que la pression p et la température T sont liées à l’énergie interne par les
relations
∂ε ∂ε
p =− , T= ,
∂τ ∂s
prouver la loi des gaz parfait, i.e. prouver que

= constante.
T

C ORRECTION .
∂ε γ−1 ∂ε 1 ε pτ
p =− = −(1 − γ)τ−γ e s/c v = ε, T= = τ1−γ e s/c v = , = (γ − 1)c v .
∂τ τ ∂s c v cv T

Exercice 4.6 Gaz raidis


Notons
B τ le volume, B T la température, B h l’enthalpie,
B ε l’énergie interne, B P la pression, B Γ le coefficient de G RÜNEISEN,
B s l’entropie, B g le potentiel de G IBBS (ou enthalpie libre), B c ∗ la vitesse du son.
On a les relations suivantes :
T = 1/∂ε s(τ, ε) P = T ∂τ s(τ, ε)
g = ε−Ts +Pτ h = ε+Pτ
Γ = −τT P ∂2εε s(τ, ε) − ∂2ετ s(τ, ε) (c ) = τ2 (P ∂ε P (τ, ε) − ∂τ P (τ, ε)) .
∗ 2
¡ ¢

La loi d’état des gaz raidis qui exprime l’entropie en fonction du volume et de l’énergie interne s’écrit
(τ, ε) 7→ s(τ, ε) = c v ln(ε − q − πτ) + c v (γ − 1) ln(τ)
où les paramètres c v > 0, γ > 1, π > 0 et q sont des constantes qui¯décrivent les propriétés
ª thermodynamique du fluide. Le
domaine de définition de l’entropie s est l’ensemble (τ, ε) ∈ R2 ¯ τ > 0, ε − q − πτ > 0 . La loi des gaz parfaits correspond
©

© G. Faccanoni 79
4 Dérivabilité et différentiabilité, fonctions implicites Dernière mise à jour : Jeudi 5 juin 2014

au cas π = q = 0.
Calculer T (τ, ε), P (τ, ε), h(τ, ε), g (τ, ε), Γ(τ, ε) et (c ∗ )2 (τ, ε) pour un gaz raidi.

C ORRECTION .
ε − q − πτ ε − q − πτ ε−q
T= > 0, P= (γ − 1) − π = (γ − 1) − γπ,
cv τ τ
h = q + (ε − q − πτ)γ > q, g = q + (ε − q − πτ) γ − ln (ε − q − πτ)τ(γ−1) ,
¡ ¡ ¢¢

Γ = γ − 1 > 0, (c ∗ )2 = γ(γ − 1)(ε − q − πτ) = γ(P + π)τ = γ(γ − 1)c v T = (γ − 1)(h − q) > 0.

Chain rule

Exercice 4.7
Calculer z 0 (t ) ou w 0 (t ) :
1. z(t ) = f (x(t ), y(t )), f (x, y) = x 2 + y 2 + x y, x(t ) = sin(t ), y(t ) = e t ;
2. z(t ) = f (x(t ), y(t )), f (x, y) = cos(x + 4y), x(t ) = 5t 4 , y(t ) = 1/t ;
y/z 2
3. w(t ) = f (x(t ), y(t ), z(t )), f (x, y, z) = xe , x(t ) = t , y(t ) = 1 − t , z(t ) = 1 + 2t ;
p
4. w(t ) = f (x(t ), y(t ), z(t )), 2 2
f (x, y, z) = ln( x + y + z ),2 x(t ) = sin(t ), y(t ) = cos(t ), z(t ) = tan(t ).

C ORRECTION .
1. z 0 (t ) = (2x(t ) + y(t ))x 0 (t ) + (2y(t ) + x(t ))y 0 (t ) = (2 sin(t ) + e t ) cos(t ) + (2e t + sin(t ))e t ;
³ ´
2. z 0 (t ) = − sin(x(t ) + 4y(t ))x 0 (t ) − 4 sin(x(t ) + 4y(t ))y 0 (t ) = t42 − 20t 3 sin 5t 4 + 4t ;
¡ ¢

t 2 (1−t )
³ ´
x(t )y(t ) t2 8t 2 +5t +2 (1−t )/(1+2t )
3. w 0 (t ) = e y(t )/z(t ) x 0 (t ) + x(t ) y(t )/z(t ) 0
z(t ) e y (t ) − z 2 (t ) e y(t )/z(t ) z 0 (t ) = 2t − 1+2t − 2 (1+2t )2
e (1−t )/(1+2t ) = (1+2t )2
te ;
2x(t ) 2y(t )
4. w 0 (t ) = p x 0 (t )+ p 2 y 0 (t )+ p 2 2z(t2 ) 2 z 0 (t ) = − p 2 2 sin(t ) cos(t )
+ p 22 cos(t2) sin(t ) 2 +
x 2 (t )+y 2 (t )+z 2 (t ) x (t )+y 2 (t )+z 2 (t ) x (t )+y (t )+z (t ) sin (t )+cos2 (t )+tan2 (t ) x (t )+y (t )+tan (t )
2 tan(t ) 1 2 sin(t )
p 2 = cos(t ) .
sin2 (t )+cos2 (t )+tan2 (t ) cos (t )

Exercice 4.8
Soient les fonctions f : R → R, g : R → R et z : R2 → R. Calculer ∂x z et ∂ y z :

1. z = f (x) + g (y) 3. z = f (x)/g (y) 5. z = f (x y)


2. z = f (x)g (y) 4. z = f (x + 2y) 6. z = f (x/y)

C ORRECTION .
1. ∂x z(x, y) = f 0 (x) et ∂ y z(x, y) = g 0 (y)
2. ∂x z(x, y) = g (y) f 0 (x) et ∂ y z(x, y) = f (x)g 0 (y)
f 0 (x) f (x)g 0 (y)
3. ∂x z(x, y) = g (y) et ∂ y z(x, y) = − g 2 (y)
4. Soit w(x, y) = x+2y, alors z = f (w(x, y)) donc ∂x z(x, y) = f 0 (w(x, y))∂x w(x, y) = f 0 (x+y) et ∂ y z(x, y) = f 0 (w(x, y))∂ y w(x, y) =
2 f 0 (x + y)
5. Soit w(x, y) = x y, alors z = f (w(x, y)) donc ∂x z(x, y) = f 0 (w(x, y))∂x w(x, y) = y f 0 (x y) et ∂ y z(x, y) = f 0 (w(x, y))∂ y w(x, y) =
x f 0 (x y)
f 0 (x/y)
6. Soit w(x, y) = x/y, alors z = f (w(x, y)) donc ∂x z(x, y) = f 0 (w(x, y))∂x w(x, y) = y et ∂ y z(x, y) = f 0 (w(x, y))∂ y w(x, y) =
f 0 (x/y)
− y2

Exercice 4.9
La production annuelle de blé B dépend de la température moyenne T et des précipitations R. Les scientifiques es-
timent que la température moyenne est en train de croître de 0.15 ◦C an−1 et que les précipitations diminuent à raison de

80 © G. Faccanoni
Dernière mise à jour : Jeudi 5 juin 2014 4 Dérivabilité et différentiabilité, fonctions implicites

0.1 cm an−1 . Ils pensent aussi qu’aux niveaux de production actuels ∂T B = −2 et ∂R B = 8. Que signifient les signes de ces
dérivées partielles ? Estimez le taux actuel de variation de la production de blé d B /d t .

C ORRECTION . Comme ∂T B est négative, une augmentation de la température moyenne (tout en gardant les précipitations
annuelles constant) entraîne une diminution de la production de blé aux niveaux de production actuels. Puisque ∂R B est
positive, une augmentation de la pluviosité annuelle (tout en gardant la température moyenne constante) provoque une
augmentation de la production de blé.
Puisque la température moyenne augmente à un taux de 0.15 ◦C an−1 , nous savons que d T /d t = 0.15. Comme les précipita-
tions diminuent à raison de 0.1 cm an−1 , nous savons que d R/d t = 0.1. Par conséquent,
d B ∂B d T ∂B d R
= + = (−2) × (0.15) + (8) × (−0.1) = −1.1.
dt ∂T d t ∂R d t
Ainsi, nous estimons que la production de blé diminuera à raison de 1.1 unités par an.

Exercice 4.10
La longueur `, la largeur w et la hauteur h d’une boite changent en fonction du temps t . À un instant donné les dimen-
sions sont ` = 1 m, w = h = 2 m, et ` et w augmentent au taux de 2 m s−1 tandis que h diminue au taux de 3 m s−1 . Calculer
les taux de variation du volume, de la surface et de la longueur de la diagonale.

C ORRECTION . On a

V (t ) = V (`(t ), w(t ), h(t )) = `(t ) × w(t ) × h(t ),


S(t ) = S(`(t ), w(t ), h(t )) = 2 w(t ) × h(t ) + w(t ) × `(t ) + h(t ) × `(t ) ,
¡ ¢
p
d (t ) = d (`(t ), w(t ), h(t )) = `2 (t ) + w 2 (t ) + h 2 (t ),

donc
∂V
= ∂` V × `0 + ∂w V × w 0 + ∂h V × h 0 = w(t ) × h(t ) × `0 (t ) + `(t ) × h(t ) × w 0 (t ) + w(t ) × `(t ) × h 0 (t ) = 6 m3 s−1 ,
∂t
∂S ³ ´
= ∂` S × `0 + ∂w S × w 0 + ∂h S × h 0 = 2 (w(t ) + h(t )) × `0 (t ) + (h(t ) + `(t )) × w 0 (t ) + (w(t ) + `(t )) × h 0 (t ) = 10 m2 s−1 ,
∂t
∂d ``0 (t ) + w w 0 (t ) + hh 0 (t )
= ∂` d × `0 + ∂w d × w 0 + ∂h d × h 0 = p = 0 m s−1 .
∂t ` (t ) + w (t ) + h (t )
2 2 2

Linéarisation
Exercice 4.11
Calculer les différentielles des fonctions suivantes :
v
q
2 −z 2
m = p5q3 u = x 2 + 3y 2 R = αβ2 cos(γ) T= L = xze −y
1 + uv w

C ORRECTION .
1. dm = 5p 4 q 3 dp + 3p 5 q 2 dq
x 3y
2. du = p dx + p dy
x 2 + 3y 2 x 2 + 3y 2
3. dR = β2 cos(γ) dα + 2αβ cos(γ) dβ − αβ2 sin(γ) dγ
v2w 1 v 2u
4. dT = − du + dv − dw
(1 + uv w)2 (1 + uv w)2 (1 + uv w)2
2 −z 2 2 −z 2 2 −z 2
5. dL = ze −y dx − 2x y ze −y dy − x(2z 2 − 1)e −y dz

© G. Faccanoni 81
4 Dérivabilité et différentiabilité, fonctions implicites Dernière mise à jour : Jeudi 5 juin 2014

Exercice 4.12
Sachant que la fonction f : R2 → R est différentiable et que

f (2, 5) = 6, ∂x f (2, 5) = 1, ∂ y f (2, 5) = −1,

donner une valeur approchée de f (2.2, 4.9).

C ORRECTION . On approche f (2.2, 4.9) par L (2,5) (2.2, 4.9) ou L (2,5) est l’équation du plan tangent à f au point (2, 5) :

L (2,5) (x, y) = f (2, 5) + (x − 2)∂x f (2, 5) + (y − 5)∂ y f (2, 5) = 6 + (x − 2) − (y − 5) = x − y + 9

donc f (2.2, 4.9) ≈ 6.3.

Exercice 4.13
p p 4
La fonction de production d’un entrepreneur est donné par K L, où K représente le capital utilisé et L le travail. Actuel-
lement, il utilise 9 unités de capital et 16 unités de travail. Déterminez par approximation linéaire la production obtenue
s’il augmente d’une unité le capital et de deux unités le facteur travail.

p p
4
C ORRECTION . La fonction f (K , L) = K L est différentiable dans (R∗+ )2 et
p
4
p
L K
∂K f (K , L) = p , ∂L f (K , L) = p
4
,
2 K 4 L3
par conséquent
p4
p
p p
4 16 9 2 3 313
f (9 + 1, 16 + 2) ≈ f (9, 16) + 1∂K f (9, 16) + 2∂L f (9, 16) = 9 16 + p + p4
×2 = 3×2+ + ×2 = .
2 9 4 163 2×3 4×8 48

Exercice 4.14
La tension T de la cordelette du yo-yo en figure est décrite
par la fonction
mg R
T= 2
2r + R 2
où m est la masse du yo-yo et g l’accélération due à la gra-
vité. Donner une estimation de la variation de T lorsque
R passe de 3 cm à 3.1 cm et r passe de 0.7 cm à 0.8 cm.

C ORRECTION .

mg (2r 2 − R 2 ) −4mg r R (−8.02 − 8.4) × 0.1


dT = ∂R T dR + ∂r T dr = dR + dr = mg ≈ −0.0165mg < 0.
(2r 2 + R 2 )2 (2r 2 + R 2 )2 99.6004

Exercice 4.15
Un sac contient 1.6 kg ± 40 g de bonbons. Pour estimer le nombre de bonbons présents dans le sac, on pèse un bonbon
au hasard et on obtient 19 g±2 g. On suppose que tous les bonbons sont identiques. Calculer le nombre total de bonbons
avec l’incertitude absolue et relative.

82 © G. Faccanoni
Dernière mise à jour : Jeudi 5 juin 2014 4 Dérivabilité et différentiabilité, fonctions implicites

C ORRECTION . Soit x le poids total et y le poids d’un seul bonbon (en grammes). Considérons le rectangle R centré en
(1600, 19) avec r 1 = 40 et r 2 = 2. Le nombre de bonbons en fonction de x et y est donné par l’application

N : R → R∗+
x
(x, y) 7→ N (x, y) ≡
y
N est une fonction de classe C 1 (R) donc pour tout (x, y) ∈ R on a la majoration ¯N (x, y) − N (1600, 19)¯ ≤ Ar 1 + Br 2 avec A
¯ ¯

(resp. B ) un majorant de |∂x N (x, y)| (resp. |∂ y N (x, y)|) sur R :


¯ ¯ ¯ ¯
¯1¯ 1 ¯ x ¯ 1600 + 40
|∂x N (x, y)| = ¯¯ ¯¯ ≤ |∂ y N (x, y)| = ¯¯− 2 ¯¯ ≤
y 19 − 2 y (19 − 2)2
d’où
¯N (x, y) − N (1600, 19)¯ ≤ 40 1 + 2 1640 ' 13.71.
¯ ¯
17 172
On conclut que le nombre de bonbons est égale à 84.21 à ±13.71 près et que l’erreur relative est de 13.71/84.21 qui est
inférieure à 17%.

Exercice 4.16
On mesure un rectangle et on obtient une largeur de 30 cm et une longueur de 24 cm, avec une erreur d’au plus 0.1 cm
pour chaque mesure. Estimer l’aire du rectangle.
C ORRECTION . L’aire du rectangle est donnée par la fonction f (x, y) = x y avec x la largeur et y la longueur (en cm). Lorsque
(h, k) ' (0, 0), on a
f (x + h, y + k) ' f (x, y) + h∂x f (x, y) + k∂ y f (x, y)
donc, pour h, k ∈ [−0.1, 0.1], on a

f (30 + 0.1, 24 + 0.1) ' 30 × 24 + 0.1 × 24 + 0.1 × 30 = 725.4,


f (30 − 0.1, 24 − 0.1) ' 30 × 24 − 0.1 × 24 − 0.1 × 30 = 714.6,

donc l’aire est comprise entre 714.6 cm2 et 725.4 cm2 .

Exercice 4.17
x+y
Soit f : R2 → R la fonction définie par f (x, y) = .
1 + x2 + y 2
1. Déterminer et représenter ses courbes de niveau.
2. Calculer ses dérivées partielles premières.
3. Écrire l’équation du plan tangent à f en (0, 0).

C ORRECTION .
1. Les courbes de niveau de f sont les courbes d’équation f (x, y) = k, i.e. la droite d’équation y = −x pour k q
= 0 et les
courbes d’équation x + y − k x − k y + 1 = 0 pour 0 < k < 1/2 qui sont des cercles de centre 2k , 2k et rayon 2k1 2 − 1.
1 1
2 2 2
¡1 1¢

y
2 0,55

0,3

0,05

K4 K2 0 2 4
x -0,2

K2
-0,45

5,0
-0,7 2,5
0,0
-5,0 y
-2,5 -2,5

K4
0,0 -5,0
2,5
x 5,0

2. Les deux dérivées premières partielles de f sont


1 − x 2 − 2x y + y 2 1 + x 2 − 2x y − y 2
∂x f (x, y) = , ∂ y f (x, y) = .
(1 + x 2 + y 2 )2 (1 + x 2 + y 2 )2
3. L’équation du plan tangent à f en (0, 0) est

z = f (0, 0) + x∂x (0, 0) + y∂ y (0, 0) = x + y.

© G. Faccanoni 83
4 Dérivabilité et différentiabilité, fonctions implicites Dernière mise à jour : Jeudi 5 juin 2014

Différentiabilité
Exercice 4.18 Différentiabilité
Vérifier, en utilisant la définition, que les fonctions f : R2 → R suivantes sont différentiables dans le point indiqué :
p
a) f (x, y) = x y − 3x 2 en (1; 2), c) f (x, y) = x y + 3y 2 en (2; 1), e) f (x, y) = y x en (4; 1),
b) f (x, y) = x y − 3y 2 en (2; 1), d) f (x, y) = x y − 2y 2 en (−2; 3), f) f (x, y) = |y| ln(1 + x) en (0; 0).

C ORRECTION . Une application f : R2 → R est différentiable en (x 0 , y 0 ) ∈ R2 ssi


f (x, y) − f (x 0 , y 0 ) − ∂x f (x 0 , y 0 )(x − x 0 ) − ∂ y f (x 0 , y 0 )(y − y 0 )
lim p = 0.
(x,y)→(x 0 ,y 0 ) (x − x 0 )2 + (y − y 0 )2
a) On a

f (x, y) = x y − 3x 2 f (1, 2) = −1,


∂x f (x, y) = y − 6x ∂x f (1, 2) = −4,
∂ y f (x, y) = x ∂ y f (1, 2) = 1,

donc
f (x, y) − f (x 0 , y 0 ) − ∂x f (x 0 , y 0 )(x − x 0 ) − ∂ y f (x 0 , y 0 )(y − y 0 ) x y − 3x 2 + 1 + 4(x − 1) − (y − 2)
p = p .
(x − x 0 )2 + (y − y 0 )2 (x − 1)2 + (y − 2)2
Avec le changement de variables x = 1 + r cos(θ) et y = 2 + r sin(θ) ce rapport se réécrit

x y − 3x 2 + 1 + 4(x − 1) − (y − 2) (1 + r cos(θ))(2 + r sin(θ)) − 3(1 + r cos(θ))2 + 1 + 4r cos(θ) − r sin(θ)


p = = r cos(θ)(sin(θ)−3 cos(θ))
(x − 1)2 + (y − 2)2 r

On a alors ¯ ¯
¯ x y − 3x 2 + 1 + 4(x − 1) − (y − 2) ¯
¯ ≤ 4r −−−→ 0
¯ ¯
¯ p
¯ 2
(x − 1) + (y − 2) 2 ¯ r →0

d’où
x y − 3x 2 + 1 + 4(x − 1) − (y − 2)
lim p = 0.
(x,y)→(1,2) (x − 1)2 + (y − 2)2
La fonction est donc différentiable en (1, 2).
Plus généralement, on peut prouver que la fonction est différentiable pour tout (x, y) ∈ R2 et la différentielle est d f (x, y) =
(y − 6x) dx + x dy.
b) On a

f (x, y) = x y − 3y 2 f (2, 1) = −1,


∂x f (x, y) = y ∂x f (2, 1) = 1,
∂ y f (x, y) = x − 6y ∂ y f (2, 1) = −4,

donc
f (x, y) − f (x 0 , y 0 ) − ∂x f (x 0 , y 0 )(x − x 0 ) − ∂ y f (x 0 , y 0 )(y − y 0 ) x y − 3y 2 + 1 − (x − 2) + 4(y − 1)
p = p .
(x − x 0 )2 + (y − y 0 )2 (x − 2)2 + (y − 1)2
Avec le changement de variables x = 2 + r cos(θ) et y = 1 + r sin(θ) ce rapport se réécrit

x y − 3y 2 + 1 − (x − 2) + 4(y − 1) (2 + r cos(θ))(1 + r sin(θ)) − 3(1 + r sin(θ))2 + 1 − r cos(θ) + 4r sin(θ)


p = = r sin(θ)(cos(θ)−3 sin(θ)).
(x − 2)2 + (y − 1)2 r

On a alors ¯ ¯
¯ x y − 3y 2 + 1 − (x − 2) + 4(y − 1) ¯
¯ ≤ 4r −−−→ 0
¯ ¯
¯ p
¯ (x − 2)2 + (y − 1)2 ¯ r →0

d’où
x y − 3y 2 + 1 − (x − 2) + 4(y − 1)
lim p = 0.
(x,y)→(2,1) (x − 2)2 + (y − 1)2
La fonction est donc différentiable en (2, 1).
Plus généralement, on peut prouver que la fonction est différentiable pour tout (x, y) ∈ R2 et la différentielle est d f (x, y) =
y dx + (x − 6y) dy.

84 © G. Faccanoni
Dernière mise à jour : Jeudi 5 juin 2014 4 Dérivabilité et différentiabilité, fonctions implicites

c) On a

f (x, y) = x y + 3y 2 f (2, 1) = 5,
∂x f (x, y) = y ∂x f (2, 1) = 1,
∂ y f (x, y) = x + 6y ∂ y f (2, 1) = 8,

donc
f (x, y) − f (x 0 , y 0 ) − ∂x f (x 0 , y 0 )(x − x 0 ) − ∂ y f (x 0 , y 0 )(y − y 0 ) x y + 3y 2 − 5 − (x − 2) − 8(y − 1)
p = p .
(x − x 0 )2 + (y − y 0 )2 (x − 2)2 + (y − 1)2
Avec le changement de variables x = 2 + r cos(θ) et y = 1 + r sin(θ) ce rapport se réécrit

x y + 3y 2 − 5 − (x − 2) − 8(y − 1) (2 + r cos(θ))(1 + r sin(θ)) + 3(1 + r sin(θ))2 − 5 − r cos(θ) − 8r sin(θ)


p = = r sin(θ)(3 sin(θ)+cos(θ)).
(x − 2)2 + (y − 1)2 r

On a alors ¯ ¯
¯ x y + 3y 2 − 5 − (x − 2) − 8(y − 1) ¯
¯ ≤ 4r −−−→ 0
¯ ¯
¯ p
¯ (x − 2)2 + (y − 1)2 ¯ r →0

d’où
x y + 3y 2 − 5 − (x − 2) − 8(y − 1)
lim p = 0.
(x,y)→(2,1) (x − 2)2 + (y − 1)2
La fonction est donc différentiable en (−2, 3).
Plus généralement, on peut prouver que la fonction est différentiable pour tout (x, y) ∈ R2 et la différentielle est d f (x, y) =
y dx + (x + 6y) dy.
d) On a

f (x, y) = x y − 2y 2 f (−2, 3) = −24,


∂x f (x, y) = y ∂x f (−2, 3) = 3,
∂ y f (x, y) = x − 4y ∂ y f (−2, 3) = −14,

donc
f (x, y) − f (x 0 , y 0 ) − ∂x f (x 0 , y 0 )(x − x 0 ) − ∂ y f (x 0 , y 0 )(y − y 0 ) x y − 2y 2 + 24 − 3(x + 2) + 14(y − 3)
p = p .
(x − x 0 )2 + (y − y 0 )2 (x + 2)2 + (y − 3)2
Avec le changement de variables x = −2 + r cos(θ) et y = 3 + r sin(θ) ce rapport se réécrit

x y − y 2 + 24 − 3(x + 2) + 14(y − 3) (−2 + r cos(θ))(3 + r sin(θ)) − 2(3 + r sin(θ))2 − 24 − 3r cos(θ) + 14r sin(θ)
p =
(x + 2)2 + (y − 3)2 r
= r sin(θ)(cos(θ) − 2 sin(θ)).

On a alors ¯ ¯
¯ x y − y 2 + 24 − 3(x + 2) + 14(y − 3) ¯
¯ ≤ 3r −−−→ 0
¯ ¯
¯ p
¯ 2
(x + 2) + (y − 3) 2 ¯ r →0

d’où
x y − y 2 + 24 − 3(x + 2) + 14(y − 3)
lim p = 0.
(x,y)→(−2,3) (x + 2)2 + (y − 3)2
La fonction est donc différentiable en (−2, 3).
Plus généralement, on peut prouver que la fonction est différentiable pour tout (x, y) ∈ R2 et la différentielle est d f (x, y) =
y dx + (x − 4y) dy.
e) On a
p
f (x, y) = y x f (4, 1) = 2,
y 1
∂x f (x, y) = p ∂x f (4, 1) = ,
2 x 4
p
∂ y f (x, y) = x ∂ y f (4, 1) = 2,

donc p
f (x, y) − f (x 0 , y 0 ) − ∂x f (x 0 , y 0 )(x − x 0 ) − ∂ y f (x 0 , y 0 )(y − y 0 ) y x − 2 − 14 (x − 4) − 2(y − 1)
p = p .
(x − x 0 )2 + (y − y 0 )2 (x − 4)2 + (y − 1)2

© G. Faccanoni 85
4 Dérivabilité et différentiabilité, fonctions implicites Dernière mise à jour : Jeudi 5 juin 2014

Avec le changement de variables x = 4 + r cos(θ) et y = 1 + r sin(θ) ce rapport se réécrit


p p
y x − 2 − 14 (x − 4) − 2(y − 1) (1 + r sin(θ)) 4 + r cos(θ) − 2 − r cos(θ)
4 − 2r sin(θ)
p =
2
(x − 4) + (y − 1) 2 r
q
2(1 + r sin(θ)) 1 + r cos(θ)
4 − 2 − r cos(θ)
4 − 2r sin(θ)
=
³ r ´
2(1 + r sin(θ)) 1 + 8 + o(r ) − 2 − r cos(θ)
r cos(θ)
4 − 2r sin(θ)
=
r
r o(r )
= cos(θ) sin(θ) + −−−→ 0
4 r r →0
p
où on a utilisé l’approximation 1 + t ' 1 + 2t + o(t ) lorsque x ' 0. Par conséquent

f (x, y) − f (x 0 , y 0 ) − ∂x f (x 0 , y 0 )(x − x 0 ) − ∂ y f (x 0 , y 0 )(y − y 0 )


lim p = 0.
(x,y)→(4,1) (x − x 0 )2 + (y − y 0 )2

La fonction est donc différentiable en (4, 1).


f) Comme f (x, 0) = f (0, y = 0) alors ∂x f (0, 0) = ∂ y f (0, 0) ≡ 0 donc

f (x, y) − f (x 0 , y 0 ) − ∂x f (x 0 , y 0 )(x − x 0 ) − ∂ y f (x 0 , y 0 )(y − y 0 ) |y| ln(1 + x)


p = p .
(x − x 0 )2 + (y − y 0 )2 x2 + y 2

On passe aux coordonnées polaires : x = r cos(θ) et y = r sin(θ) et on a


¯ ¯
¯ |y| ln(1 + x) ¯ ¯ ¯
¯ = sin(ϑ) ln(1 + r cos(ϑ))¯ ≤ 2r |sin(ϑ) cos(ϑ)| ≤ 2r −−−→ 0.
¯ ¯ ¯
¯ p
¯ 2
x +y 2 ¯ r →0

La fonction est donc différentiable en (0, 0).

86 © G. Faccanoni
Dernière mise à jour : Jeudi 5 juin 2014 4 Dérivabilité et différentiabilité, fonctions implicites

Continuité, dérivabilité, différentiabilité

Exercice 4.19
Soit f : R2 → R une application. Dire si les affirmations suivantes sont V raies ou F ausses.
1. Si f ∈ C 1 (R2 ) alors f est différentiable en tout point.
2. Si f ∈ C 1 (R2 ) alors il existe ∇ f en tout point.
3. Si f ∈ C 1 (R2 ) alors f est continue.
4. Si f est différentiable en tout point alors f ∈ C 1 (R2 ).
5. S’il existe ∇ f en tout point alors f ∈ C 1 (R2 ).
6. Si f est continue alors f ∈ C 1 (R2 ).
7. Si f est différentiable en (x 0 , y 0 ) alors il existe ∇ f (x 0 , y 0 ).
8. Si f est différentiable alors f est continue.
9. S’il existe ∇ f (x 0 , y 0 ) alors f est différentiable en (x 0 , y 0 ).
10. Si f est continue alors f est différentiable.
11. Si f est continue alors il existe ∇ f en tout point.
12. S’il existe ∇ f en tout point alors f est continue.

C ORRECTION .
1. V Si f ∈ C 1 (R2 ) alors f est différentiable en tout point.
2. V Si f ∈ C 1 (R2 ) alors il existe ∇ f en tout point.
3. V Si f ∈ C 1 (R2 ) alors f est continue.
4. F Si f est différentiable en tout point alors f ∈ C 1 (R2 ).
5. F S’il existe ∇ f en tout point alors f ∈ C 1 (R2 ).
6. F Si f est continue alors f ∈ C 1 (R2 ).
7. V Si f est différentiable en (x 0 , y 0 ) alors il existe ∇ f (x 0 , y 0 ).
8. V Si f est différentiable alors f est continue.
9. F S’il existe ∇ f (x 0 , y 0 ) alors f est différentiable en (x 0 , y 0 ).
10. F Si f est continue alors f est différentiable.
11. F Si f est continue alors il existe ∇ f en tout point.
12. F S’il existe ∇ f en tout point alors f est continue.

Exercice 4.20
Soit f une fonction définie sur R2 telle que lim(x,y)→(3,1) f (x, y) = 6.
1. Que peut-on dire de f (3, 1) ?
2. Si f est continue en (3, 1), que peut-on dire de f (3, 1) ?
3. Si f est dérivable en (3, 1), que peut-on dire de f (3, 1) ?
4. Si f est différentiable en (3, 1), que peut-on dire de f (3, 1) ?
5. Si f est de classe C 1 au voisinage de (3, 1), que peut-on dire de f (3, 1) ?

C ORRECTION .
1. On ne peut rien dire de la valeur f (3, 1).
2. f (3, 1) = 6.
3. On ne peut rien dire de la valeur f (3, 1).
4. f (3, 1) = 6 car la différentiabilité implique la continuité.

© G. Faccanoni 87
4 Dérivabilité et différentiabilité, fonctions implicites Dernière mise à jour : Jeudi 5 juin 2014

5. f (3, 1) = 6 car être de classe C 1 implique la continuité.

Exercice 4.21 Exemple de P EANO


Soit f la fonction définie sur R2 par
2 2

x y x − y

si (x, y) 6= (0, 0),
f (x, y) = x2 + y 2

0 si (x, y) = (0, 0).

1. Montrer que f est continue sur R2 .


2. Calculer ∇ f (x, y).
3. Montrer que ∇ f est continu sur R2 .
4. Montrez que f admet des dérivées partielles secondes en tout point.
5. Que pouvez-vous déduire du calcul de ∂x y f (0, 0) et de ∂ y x f (0, 0) ?

C ORRECTION .
1. La fonction est clairement continue dans R2 \ {(0, 0)} car quotient de fonctions continues dont le dénominateur ne
s’annule pas. Elle est aussi continue en (0, 0) car

¯ x y(x 2 − y 2 ) ¯
¯ ¯ ¯ ¯
lim | f (x, y)| = lim ¯ ¯ ¯ = lim ρ 2 ¯¯ cos(ϑ) sin(ϑ)(cos2 (θ) − sin2 (θ))¯¯ ≤ lim 2ρ 2 = 0 = f (0, 0).
(x,y)→(0,0) (x,y)→(0,0) x2 + y 2 ¯ ρ→0 ρ→0
∀θ

f est donc continue sur R2 .


2. On a
 4
x − 4x 2 y 2 − y 4
 4 2 2 4
 y x + 4x y − y

si (x, y) 6= (0, 0), x
 si (x, y) 6= (0, 0),
∂x f (x, y) = (x 2 + y 2 )2 ∂ y f (x, y) = (x 2 + y 2 )2
 lim f (x,0)− f (0,0) = 0 si (x, y) = (0, 0), f (0,y)− f (0,0)

x
 lim

y = 0 si (x, y) = (0, 0).
x→0 y→0

3. La fonction ∂x f est clairement continue dans R2 \ {(0, 0)} car quotient de fonctions continues dont le dénominateur ne
s’annule pas. Elle est aussi continue en (0, 0) car
¯ 4
¯ x + 4x 2 y 2 − y 4 ¯
¯ ¯ ¢¯
¯ = lim ρ ¯¯ sin(ϑ) cos4 (ϑ)+4 cos2 (θ) sin2 (θ)−sin4 (ϑ) ¯¯ ≤ lim 5ρ = 0 = ∂x f (0, 0).
¡
lim |∂x f (x, y)| = lim ¯y
(x,y)→(0,0) (x,y)→(0,0)
¯ (x 2 + y 2 )2 ¯ ρ→0 ρ→0
∀θ

∂x f est donc continue sur R2 .


La fonction ∂ y f est clairement continue dans R2 \ {(0, 0)} car quotient de fonctions continues dont le dénominateur ne
s’annule pas. Elle est aussi continue en (0, 0) car
¯ 4
¯ x − 4x 2 y 2 − y 4 ¯
¯ ¯ ¢¯
¯ = lim ρ ¯¯ cos(ϑ) cos4 (ϑ)−4 cos2 (θ) sin2 (θ)−sin4 (ϑ) ¯¯ ≤ lim 5ρ = 0 = ∂ y f (0, 0).
¡
lim |∂ y f (x, y)| = lim ¯x
(x,y)→(0,0) (x,y)→(0,0)
¯ (x 2 + y 2 )2 ¯ ρ→0 ρ→0
∀θ

∂ y f est donc continue sur R2 .


4. Passons aux dérivées secondes :
 4x y 3 (x 2 −3y 2 )  6 4 2 2 4 6
2 2 3 si (x, y) 6= (0, 0),  x +9x y2 −9x

2 3
y −y
si (x, y) 6= (0, 0),
(x +y )

(x +y )
∂xx f (x, y) = ∂x y f (x, y) = ∂ y f (x,0)−∂ y f (0,0)
 lim ∂x f (x,0)−∂x f (0,0) = 0 si (x, y) = (0, 0),  lim
 = 1 si (x, y) = (0, 0),
x→0 x x→0 x
 3
4x y(3x 2 −y 2 )
 6 4 2 2 4 6
x +9x y −9x y −y

 (x 2 +y 2 )3
si (x, y) 6= (0, 0), 
 (x 2 +y 2 )3 si (x, y) 6= (0, 0),
∂ y x f (x, y) = ∂ f (0,y)−∂x f (0,y) ∂ y y f (x, y) = ∂ f (0,y)−∂ y f (0,0)
 lim x

y = −1 si (x, y) = (0, 0),  lim y

y = 0 si (x, y) = (0, 0),
y→0 y→0

5. Comme ∂x y f (0, 0) 6= ∂ y x f (0, 0), le théorème de S CHWARZ permet de conclure que les dérivées secondes ∂x y f (x, y) et
∂ y x f (x, y) ne sont pas continue en (0, 0). On peut donc conclure que f est de classe C 1 (R2 ) mais pas de classe C 2 (R2 ).

88 © G. Faccanoni
Dernière mise à jour : Jeudi 5 juin 2014 4 Dérivabilité et différentiabilité, fonctions implicites

Exercice 4.22 Continuité, dérivabilité, différentiabilité


Soit f : R2 → R la fonction ainsi définie
2

 x y

si (x, y) 6= (0, 0),
f (x, y) = x 2 + y 2

0 si (x, y) = (0, 0).

1. Est-elle continue sur R2 ?


2. Calculer ∇ f (x, y).
3. La fonction f est-elle de classe C 1 (R2 ) ?
4. Que peut-on conclure sur la différentiabilité de la fonction f sur R2 ?

C ORRECTION .
1. f est continue sur R2 \ { (0, 0) } car quotient de fonctions continues dont le dénominateur ne s’annule pas. Pour que f
soit continue sur R2 il faut alors que lim(x,y)→(0,0) f (x, y) = f (0, 0) = 0. Passons on coordonnées polaires : x = 0+r cos(ϑ),
y = 0 + r sin(ϑ),
f˜(r, ϑ) = f (x(r, ϑ), y(r, ϑ)) = r cos2 ϑ sin(ϑ).
Comme | f˜(r, ϑ)| ≤ r −−−→ 0 alors lim(x,y)→(0,0) f (x, y) = 0 donc f est continue sur R2 .
r →0
∂x f (x, y)
µ ¶
2. ∇ f (x, y) = avec
∂ y f (x, y)
 4 2 2
2x y 3 x −x y

si (x, y) 6= (0, 0) 
 (x 2 +y 2 )2 si (x, y) 6= (0, 0)
(x 2 +y 2 )2

∂x f (x, y) = ∂ y f (x, y) = f (0,y)− f (0,0)
 lim f (x,0)− f (0,0) = 0 si (x, y) = (0, 0)  lim
 = 0 si (x, y) = (0, 0)
x−0 y→0 y−0
x→0

3. Comme f est continue et dérivable sur R2 et ses dérivées partielles sont continues sur R2 \ { (0, 0) } alors f est de classe
C 1 (R2 \ { (0, 0) }). Pour qu’elle soit de classe C 1 (R2 ) il faut que les dérivées partielles soient continues sur R2 , autrement
dit il faut que

lim ∂x f (x, y) = ∂x f (0, 0) lim ∂ y f (x, y) = ∂ y f (0, 0)


(x,y)→(0,0) (x,y)→(0,0)

1
Comme ∂x f (x, x) = 2 6= 0 = ∂x f (0, 0) on conclut que ∂x f n’est pas continue en (0, 0), donc f n’est pas de classe C 1 (R2 ).
4. Comme f est de classe C 1 (R2 \ { (0, 0) }) alors f est différentiable sur R2 \ { (0, 0) }. Pour qu’elle soit différentiable sur R2
il faut que
f (x, y) − f (0, 0) − ∂x f (0, 0)(x − 0) − ∂ y f (0, 0)(y − 0)
lim p = 0.
(x,y)→(0,0) (x − 0)2 + (y − 0)2
f (x,y)− f (0,0)−∂x f (0,0)(x−0)−∂ y f (0,0)(y−0) x2 y 1
Si on note h(x, y) = p = , comme h(x, x) = 23/2 6= 0, alors f n’est pas différen-
(x−0)2 +(y−0)2 (x 2 +y 2 )3/2
2
tiable sur R .

Exercice 4.23 Continuité, dérivabilité, différentiabilité


Soit f la fonction définie sur R2 par
3 3

x y −xy

si (x, y) 6= (0, 0),
f (x, y) = 2
x +y 2

0 sinon.
1. La fonction f est-elle continue en (0, 0) ?
2. Déterminer si les dérivées partielles ∂x f (0, 0) et ∂ y f (0, 0) existent et les calculer le cas échéant.
3. La fonction f est-elle de classe C 1 sur R2 ?
4. La fonction f est-elle différentiable en (0, 0) ?

© G. Faccanoni 89
4 Dérivabilité et différentiabilité, fonctions implicites Dernière mise à jour : Jeudi 5 juin 2014

C ORRECTION .
x 3 y−x y 3
1. On étudie la limite lim 2 2 . On a
(x,y)→(0,0) x +y

x3 y − x y 3 x2 − y 2 2
³
2 2
´
= x y = r cos(ϑ) sin(ϑ) cos (ϑ) − sin (ϑ) −−−→ 0
x2 + y 2 x2 + y 2 r →0
∀ϑ
¯ ³ ´¯¯
car ¯r cos(ϑ) sin(ϑ) cos (ϑ) − sin (ϑ) ¯¯ ≤ 2r 2 . On en déduit que la limite de f (x, y) quand (x, y) tend vers (0, 0) est
¯ 2 2 2
¯

0 = f (0, 0). La fonction est donc continue en (0, 0). De plus, f est continue sur (R∗ )2 car quotient de fonctions continues
dont le dénominateur ne s’annule pas, donc f est continue sur R2 .
2. Attention : il faut revenir à la définition de dérivée partielle en un point puisque la fonction au point (0, 0) n’est pas
définie de la même façon que sur le reste du domaine.
B Calcul de ∂x f (0, 0) :
2 2
f (0 + t , 0) − f (0, 0) (0 + t )0 (0+t ) −0
(0+t )2 +02
−0 0
∂x f (0, 0) = lim = lim = lim = lim 0 = 0.
t →0 t t →0 t t →0 t t →0

La dérivée partielle par rapport à x en (0, 0) existe et est égale à 0.


B Calcul de ∂ y f (0, 0) :
2 2
f (0, 0 + t ) − f (0, 0) 0(0 + t ) 002 −(0+t )
+(0+t )2
−0 0
∂ y f (0, 0) = lim = lim = lim = lim 0 = 0.
t →0 t t →0 t t →0 t t →0

La dérivée partielle par rapport à y en (0, 0) existe et est égale à 0.


3. Ici on doit calculer les dérivées partielles sur le reste du domaine et vérifier si elles sont continues ou non en (0, 0) qui
est le seul point qui peut poser des problèmes.
B Calcul de ∂x f (x, y) pour (x, y) 6= (0, 0) :

x3 y − x y 3 (3x 2 y − y 3 )(x 2 + y 2 ) − (x 3 y − x y 3 )(2x) x 4 − y 4 + 4x 2 y 2


µ ¶
∂x 2 2
= 2 2 2
=y .
x +y (x + y ) (x 2 + y 2 )2

On a
x 4 − y 4 + 4x 2 y 2 ³
4 4 2 2
´
y = r sin(ϑ) cos (ϑ) − sin (ϑ) + 4 cos (ϑ) sin (ϑ) −−−→ 0
(x 2 + y 2 )2 r →0
∀ϑ
¯ ³ ´¯¯
4 4 2 2
car ¯r sin(ϑ) cos (ϑ) − sin (ϑ) + 4 cos (ϑ) sin (ϑ) ¯¯ ≤ 6r . On en déduit que la limite de ∂x f (x, y) quand (x, y) tend
¯
¯

vers (0, 0) est 0 = ∂x f (0, 0). La fonction ∂x f est donc continue en (0, 0). De plus, elle est continue sur R2 \ { (0, 0) } car
quotient de fonctions continues dont le dénominateur ne s’annule pas.
B Calcul de ∂ y f (x, y) pour (x, y) 6= (0, 0) :

x3 y − x y 3 (x 3 − 3x y 2 )(x 2 + y 2 ) − (x 3 y − x y 3 )(2y) x 4 − y 4 − 4x 2 y 2
µ ¶
∂y 2 2
= 2 2 2
=x .
x +y (x + y ) (x 2 + y 2 )2

On a
x 4 − y 4 − 4x 2 y 2 ³
4 4 2 2
´
x = r cos(ϑ) cos (ϑ) − sin (ϑ) − 4 cos (ϑ) sin (ϑ) −−−→ 0
(x 2 + y 2 )2 r →0
∀ϑ
¯ ³ ´¯¯
4 4 2 2
car ¯r cos(ϑ) cos (ϑ) − sin (ϑ) − 4 cos (ϑ) sin (ϑ) ¯¯ ≤ 6r . On en déduit que la limite de ∂ y f (x, y) quand (x, y) tend
¯
¯

vers (0, 0) est 0 = ∂ y f (0, 0). La fonction ∂ y f est donc continue en (0, 0). De plus, elle est continue sur R2 \ { (0, 0) } car
quotient de fonctions continues dont le dénominateur ne s’annule pas.
La fonction f est continues sur R2 , elle admets des dérivées partielles sur R2 et ses dérivées partielles sont continues
sur R2 . On conclut que la fonction f est de classe C 1 sur R2 .
4. La fonction f est différentiable en (0, 0) car elle est de classe C 1 .

90 © G. Faccanoni
Dernière mise à jour : Jeudi 5 juin 2014 4 Dérivabilité et différentiabilité, fonctions implicites

Exercice 4.24 Continuité, dérivabilité, différentiabilité


Soit f : R2 → R la fonction ainsi définie
2

 xy

si (x, y) 6= (0, 0),
f (x, y) = x 2 + y 2

0 si (x, y) = (0, 0).

1. Est-elle continue sur R2 ?


2. Est-elle dérivable sur R2 ?
3. Est-elle de classe C 1 (R2 ) ?
4. Est-elle différentiable sur R2 ?

C ORRECTION .
1. É TUDE DE LA CONTINUITÉ SUR R2
Étude sur R2 \ { (0, 0) } : f est continue sur R2 \ { (0, 0) } car quotient de fonctions continues dont le dénominateur ne
s’annule pas.
Étude en (0, 0) : pour que f soit continue en (0, 0) il faut que lim(x,y)→(0,0) f (x, y) = f (0, 0) = 0. Passons on coordon-
nées polaires : x = 0 + r cos(ϑ), y = 0 + r sin(ϑ),

f˜(r, ϑ) = f (x(r, ϑ), y(r, ϑ)) = r cos(ϑ) sin2 (ϑ).

Comme | f˜(r, ϑ)| ≤ r −−−→ 0 alors lim(x,y)→(0,0) f (x, y) = 0.


r →0
Donc f est continue sur R2 .
2. É TUDE DE LA DÉRIVABILITÉ SUR R2
f est dérivable sur R2 si les deux dérivées partielles ∂x f et ∂ y f sont définie pour tout (x, y) ∈ R2 :

2x 3 y
 y 4 −x 2 y 2 
 2 22 si (x, y) 6= (0, 0) 
 (x 2 +y 2 )2 si (x, y) 6= (0, 0)
(x +y )
∂x f (x, y) = ∂ y f (x, y) = f (0,y)− f (0,0)
 lim f (x,0)− f (0,0) = 0 si (x, y) = (0, 0)  lim
 = 0 si (x, y) = (0, 0)
x−0 y→0 y−0
x→0

Donc f est dérivable sur R2 .


3. É TUDE DE LA RÉGULARITÉ SUR R2
f est continue et dérivable sur R2 .
Étude sur R2 \ { (0, 0) } : les dérivées partielles sont continues sur R2 \{ (0, 0) } car quotient de fonctions continues dont
le dénominateur ne s’annule pas. Alors f est de classe C 1 (R2 \ { (0, 0) })
Étude en (0, 0) : les dérivées partielles sont continues en (0, 0) ssi

lim ∂x f (x, y) = ∂x f (0, 0) lim ∂ y f (x, y) = ∂ y f (0, 0)


(x,y)→(0,0) (x,y)→(0,0)

Comme ∂ y f (x, x) = 21 6= 0 = ∂ y f (0, 0) on conclut que ∂ y f n’est pas continue en (0, 0)


f est de classe C 1 (R2 \ { (0, 0) }) mais n’est pas de classe C 1 (R2 ).
4. É TUDE DE LA DIFFÉRENTIABILITÉ SUR R2
Étude sur R2 \ { (0, 0) } : comme f est de classe C 1 (R2 \ { (0, 0) }) alors f est différentiable sur R2 \ { (0, 0) }.
Étude en (0, 0) : pour que f soit différentiable en (0, 0) il faut que

f (x, y) − f (0, 0) − ∂x f (0, 0)(x − 0) − ∂ y f (0, 0)(y − 0)


lim p = 0.
(x,y)→(0,0) (x − 0)2 + (y − 0)2

f (x,y)− f (0,0)−∂x f (0,0)(x−0)−∂ y f (0,0)(y−0) x y2 1


Si on note h(x, y) = p = p , comme h(x, x) = 6= 0, alors f n’est pas
(x−0)2 +(y−0)2 (x 2 +y 2 )3 23/2
différentiable en (0, 0).
f est différentiable sur R2 \ { (0, 0) } mais n’est pas différentiable en (0, 0).

© G. Faccanoni 91
4 Dérivabilité et différentiabilité, fonctions implicites Dernière mise à jour : Jeudi 5 juin 2014

Exercice 4.25
Soit f : R2 → R la fonction ainsi définie
2

 (x − 1)y

si (x, y) 6= (1, 0),
f (x, y) = (x − 1)2 + y 2

0 si (x, y) = (1, 0).

1. Étude de la fonction sur R2 \ { (1, 0) } :


1.1. montrer que f est continue sur R2 \ { (1, 0) } ;
1.2. calculer le gradient de f pour (x, y) ∈ R2 \ { (1, 0) } ;
1.3. montrer que f est de classe C 1 sur R2 \ { (1, 0) } ;
1.4. que peut-on conclure sur la différentiabilité de f sur R2 \ { (1, 0) } ?
2. Étude de la fonction en (1, 0) :
2.1. montrer que f est continue en (1, 0) ;
2.2. calculer le gradient de f en (1, 0) ;
2.3. montrer que f n’est pas différentiable en (1, 0) ;
2.4. f est-elle de classe C 1 en (1, 0) ?

C ORRECTION .
1. Étude de la fonction sur R2 \ { (1, 0) }.
1.1. f est continue sur R2 \ { (1, 0) } car quotient de fonctions continues dont le dénominateur ne s’annule pas.
1.2. Le gradient de f pour (x, y) ∈ R2 \ { (1, 0) } est le vecteur de composantes

(x − 1)2 − y 2 2(x − 1)3 y


∂x f (x, y) = −y 2 , ∂ y f (x, y) = .
((x − 1)2 + y 2 )2 ((x − 1)2 + y 2 )2

1.3. f est continue et dérivable sur R2 \ { (1, 0) } ; ses dérivées partielles sont continues sur R2 \ { (1, 0) } car quotients de
fonctions continues dont les dénominateurs ne s’annulent pas. Alors f est de classe C 1 (R2 \ { (1, 0) }).
1.4. Comme f est de classe C 1 (R2 \ { (1, 0) }) alors f est différentiable sur R2 \ { (1, 0) }.
2. Étude de la fonction en (1, 0).
2.1. Pour que f soit continue en (1, 0) il faut que lim(x,y)→(1,0) f (x, y) = f (1, 0). Passons en coordonnées polaires : x =
1 + r cos(ϑ), y = 0 + r sin(ϑ),
f˜(r, ϑ) = f (x(r, ϑ), y(r, ϑ)) = r cos(ϑ) sin2 ϑ.
Comme | f˜(r, ϑ)| ≤ r −−−→ 0 alors lim(x,y)→(1,0) f (x, y) = 0 donc f est continue en (1, 0).
r →0
2.2. Le gradient de f en (1, 0) est le vecteur de composantes

f (x, 0) − f (1, 0) f (0, y) − f (1, 0)


∂x f (1, 0) = lim = 0, ∂ y f (1, 0) = lim = 0.
x→1 x −1 y→0 y −0

2.3. Pour prouver que f n’est pas différentiable en (1, 0) il faut montrer que

f (x, y) − f (1, 0) − ∂x f (1, 0)(x − 1) − ∂ y f (1, 0)(y − 0)


lim p 6= 0.
(x,y)→(1,0) (x − 1)2 + (y − 0)2

f (x,y)− f (1,0)−∂x f (1,0)(x−1)−∂ y f (1,0)(y−0) (x−1)y 2 1


Notons h(x, y) = p = ; comme h(x, x −1) = 6= 0, alors f n’est pas
(x−1)2 +(y−0)2 ((x−1)2 +y 2 )3/2 23/2
différentiable en (1, 0).
2.4. Comme f n’est pas différentiable en (1, 0), elle n’est pas de classe C 1 en (1, 0). Si on a oublié ce théorème on peut
le prouver directement : pour qu’elle soit de classe C 1 en (1, 0) il faut que les dérivées partielles soient continues,
autrement dit il faut que

lim ∂x f (x, y) = ∂x f (1, 0) et lim ∂ y f (x, y) = ∂ y f (1, 0).


(x,y)→(1,0) (x,y)→(1,0)

1
Comme ∂ y f (x, x −1) = 2 6= ∂ y f (1, 0) = 0 on conclut que ∂x f n’est pas continue en (1, 0), donc f n’est pas de classe
C 1 en (1, 0).

92 © G. Faccanoni
Dernière mise à jour : Jeudi 5 juin 2014 4 Dérivabilité et différentiabilité, fonctions implicites

Exercice 4.26
Soit f : R2 → R la fonction ainsi définie
2

 x (y + 1)

si (x, y) 6= (0, −1),
f (x, y) = x 2 + (y + 1)2

0 si (x, y) = (0, −1).

1. Étude de la fonction sur R2 \ { (0, −1) } :


1.1. montrer que f est continue sur R2 \ { (0, −1) } ;
1.2. calculer le gradient de f pour (x, y) ∈ R2 \ { (0, −1) } ;
1.3. montrer que f est de classe C 1 sur R2 \ { (0, −1) } ;
1.4. que peut-on conclure sur la différentiabilité de f sur R2 \ { (0, −1) } ?
2. Étude de la fonction en (0, −1) :
2.1. montrer que f est continue en (0, −1) ;
2.2. calculer le gradient de f en (0, −1) ;
2.3. montrer que f n’est pas différentiable en (0, −1) ;
2.4. f est-elle de classe C 1 en (0, −1) ?

C ORRECTION .
1. Étude de la fonction sur R2 \ { (0, −1) }.
1.1. f est continue sur R2 \ { (0, −1) } car quotient de fonctions continues dont le dénominateur ne s’annule pas.
1.2. Le gradient de f pour (x, y) ∈ R2 \ { (0, −1) } est le vecteur de composantes

2x(y + 1)3 x 2 − (y + 1)2


∂x f (x, y) = , ∂ y f (x, y) = x 2 .
(x 2 + (y + 1)2 )2 (x 2 + (y + 1)2 )2

1.3. f est continue et dérivable sur R2 \ { (0, −1) } ; ses dérivées partielles sont continues sur R2 \ { (0, −1) } car quotients
de fonctions continues dont les dénominateurs ne s’annulent pas. Alors f est de classe C 1 (R2 \ { (0, −1) }).
1.4. Comme f est de classe C 1 (R2 \ { (0, −1) }) alors f est différentiable sur R2 \ { (0, −1) }.
2. Étude de la fonction en (0, −1).
2.1. Pour que f soit continue en (0, −1) il faut que lim(x,y)→(0,−1) f (x, y) = f (0, −1). Passons en coordonnées polaires :
x = 0 + r cos(ϑ), y = −1 + r sin(ϑ),

f˜(r, ϑ) = f (x(r, ϑ), y(r, ϑ)) = r cos2 ϑ sin(ϑ).

Comme | f˜(r, ϑ)| ≤ r −−−→ 0 alors lim(x,y)→(0,−1) f (x, y) = 0 donc f est continue en (0, −1).
r →0
2.2. Le gradient de f en (0, −1) est le vecteur de composantes

f (x, 0) − f (0, −1) f (0, y) − f (0, −1)


∂x f (0, −1) = lim = 0, ∂ y f (0, −1) = lim = 0.
x→0 x −0 y→−1 y +1

2.3. Pour prouver que f n’est pas différentiable en (0, −1) il faut montrer que

f (x, y) − f (0, −1) − ∂x f (0, −1)(x − 0) − ∂ y f (0, −1)(y + 1)


lim p 6= 0.
(x,y)→(0,−1) x 2 + (y + 1)2

f (x,y)− f (0,−1)−∂x f (0,−1)(x−0)−∂ y f (0,−1)(y+1) x 2 (y+1) 1


Notons h(x, y) = p = ; comme h(y + 1, y) = 6= 0, alors f n’est
(x−0)2 +(y+1)2 (x 2 +(y+1)2 )3/2 23/2
pas différentiable en (0, −1).
2.4. Comme f n’est pas différentiable en (0, −1), elle n’est pas de classe C 1 en (0, −1). Si on a oublié ce théorème on
peut le prouver directement : pour qu’elle soit de classe C 1 en (0, −1) il faut que les dérivées partielles soient
continues, autrement dit il faut que

lim ∂x f (x, y) = ∂x f (0, −1) et lim ∂ y f (x, y) = ∂ y f (0, −1).


(x,y)→(0,−1) (x,y)→(0,−1)

1
Comme ∂ y f (y + 1, y) = 2 6= ∂ y f (0, −1) = 0 on conclut que ∂x f n’est pas continue en (0, −1), donc f n’est pas de
classe C 1 en (0, −1).

© G. Faccanoni 93
4 Dérivabilité et différentiabilité, fonctions implicites Dernière mise à jour : Jeudi 5 juin 2014

Exercice 4.27 Continuité, dérivabilité, différentiabilité


Soit f : R2 → R la fonction ainsi définie
2 2

 x y

si (x, y) 6= (0, 0),
f (x, y) = x 2 + y 2

0 si (x, y) = (0, 0).

1. Est-elle continue sur R2 ?


2. Calculer ∇ f (x, y).
3. La fonction f est-elle de classe C 1 (R2 ) ?
4. Que peut-on conclure sur la différentiabilité de la fonction f sur R2 ?

C ORRECTION .
1. La fonction est clairement continue dans R2 \ {(0, 0)}. Elle est aussi continue en (0, 0) car

x2 y 2
lim f (x, y) = lim = lim ρ 2 cos2 (θ) sin2 (θ) ≤ lim ρ 2 = 0 = f (0, 0).
(x,y)→(0,0) (x,y)→(0,0) x 2 + y 2 ρ→0 ρ→0
∀θ

f est donc continue sur R2 .


2. Pour (x, y) 6= (0, 0) on a
2x y 4
 
∂x f (x, y)
µ ¶
(x 2 +y 2 )2
∇ f (x, y) = =  2x 4 y  .
∂ y f (x, y)
(x 2 +y 2 )2

Pour (x, y) = (0, 0) on a


¶ Ã f (x,0)− f (0,0)
!
∂x f (0, 0) limx→0
µ µ ¶
x−0 0
∇ f (0, 0) = = f (0,y)− f (0,0) = .
∂ y f (0, 0) lim y→0 y−0
0

3. On a déjà vérifié que la fonction est continue sur R2 . Vérifions si ses dérivées partielles sont continues sur R2 . On a

2x y 4
(
(x 2 +y 2 )2
si (x, y) 6= (0, 0),
∂x f (x, y) =
0 si (x, y) = (0, 0),
2ρ 5 (cos(θ) sin4 (θ))
lim ∂x f (x, y) = lim = lim 2ρ = 0 = ∂x f (0, 0)
(x,y)→(0,0) ρ→0 ρ4 ρ→0
∀θ
2x 4 y
(
(x 2 +y 2 )2
si (x, y) 6= (0, 0),
∂ y f (x, y) =
0 si (x, y) = (0, 0).
2ρ (cos4 θ sin(θ))
5
lim ∂ y f (x, y) = lim = lim 2ρ = 0 = ∂ y f (0, 0),
(x,y)→(0,0) ρ→0 ρ4 ρ→0
∀θ

la fonction est donc de classe C 1 (R2 ).


4. Puisque toute fonction de classe C 1 (Ω) est différentiable sur Ω, on conclut que f est différentiable sur R2 .

Exercice 4.28 Continuité, dérivabilité, différentiabilité


Soit f : R2 → R la fonction ainsi définie
3 2

 x y

si (x, y) 6= (0, 0),
f (x, y) = x 2 + y 2

0 si (x, y) = (0, 0).

1. Est-elle continue sur R2 ?


2. Calculer ∇ f (x, y).
3. La fonction f est-elle de classe C 1 (R2 ) ?
4. Que peut-on conclure sur la différentiabilité de la fonction f sur R2 ?

94 © G. Faccanoni
Dernière mise à jour : Jeudi 5 juin 2014 4 Dérivabilité et différentiabilité, fonctions implicites

C ORRECTION .
1. La fonction est clairement continue dans R2 \ {(0, 0)}. Elle est aussi continue en (0, 0) car
x3 y 2
lim f (x, y) = lim = lim ρ 3 cos3 (θ) sin2 (θ) = lim ρ 3 = 0 = f (0, 0).
(x,y)→(0,0) (x,y)→(0,0) x 2 + y 2 ρ→0 ρ→0
∀θ

f est donc continue sur R2 .


2. Pour (x, y) 6= (0, 0) on a  2 2 2 2 
x y (x +3y )
∂x f (x, y)
µ ¶
(x 2 +y 2 )2 
∇ f (x, y) = = 2x 5 y
.
∂ y f (x, y)

(x 2 +y 2 )2
Pour (x, y) = (0, 0) on a
¶ Ã f (x,0)− f (0,0)
!
∂x f (0, 0) limx→0
µ µ ¶
x−0 0
∇ f (0, 0) = = f (0,y)− f (0,0) = .
∂ y f (0, 0) lim y→0 y−0
0

3. On a déjà vérifié que la fonction est continue sur R2 . Vérifions si ses dérivées partielles sont continues sur R2 . On a
( x 2 y 2 (x 2 +3y 2 )
(x 2 +y 2 )2
si (x, y) 6= (0, 0),
∂x f (x, y) =
0 si (x, y) = (0, 0),
ρ 6 cos2 (θ) sin2 (θ)(cos2 (θ) + 3 sin2 (θ))
lim ∂x f (x, y) = lim = lim 4ρ 2 = 0 = ∂x f (0, 0)
(x,y)→(0,0) ρ→0 ρ4 ρ→0
∀θ
2x 5 y
(
(x 2 +y 2 )2
si (x, y) 6= (0, 0),
∂ y f (x, y) =
0 si (x, y) = (0, 0).
2ρ (cos5 (θ) sin(θ))
6
lim ∂ y f (x, y) = lim = lim 2ρ 2 = 0 = ∂ y f (0, 0),
(x,y)→(0,0) ρ→0 ρ4 ρ→0
∀θ

la fonction est donc de classe C (R ). 1 2

4. Puisque toute fonction de classe C 1 (Ω) est différentiable sur Ω, on conclut que f est différentiable sur R2 .

Exercice 4.29 Continuité, dérivabilité, différentiabilité


Soit f : R2 → R la fonction ainsi définie
2 3

 x y

si (x, y) 6= (0, 0),
f (x, y) = x 2 + y 2

0 si (x, y) = (0, 0).

1. Est-elle continue sur R2 ?


2. Calculer ∇ f (x, y).
3. La fonction f est-elle de classe C 1 (R2 ) ?
4. Que peut-on conclure sur la différentiabilité de la fonction f sur R2 ?

C ORRECTION .
1. La fonction est clairement continue dans R2 \ {(0, 0)}. Elle est aussi continue en (0, 0) car
x2 y 3
lim f (x, y) = lim = lim ρ 3 cos2 (θ) sin3 (θ) = lim ρ 3 = 0 = f (0, 0).
(x,y)→(0,0) (x,y)→(0,0) x 2 + y 2 ρ→0 ρ→0
∀θ
2
f est donc continue sur R .
2. Pour (x, y) 6= (0, 0) on a
2x y 5
 
∂x f (x, y)
µ ¶
2 2 2
∇ f (x, y) = =  x 2 (x +y )
y 2 (3x 2 +y 2 )
.
∂ y f (x, y)
(x 2 +y 2 )2
Pour (x, y) = (0, 0) on a
¶ Ã f (x,0)− f (0,0)
!
∂x f (0, 0) limx→0
µ µ ¶
x−0 0
∇ f (0, 0) = = f (0,y)− f (0,0) = .
∂ y f (0, 0) lim y→0 y−0
0

© G. Faccanoni 95
4 Dérivabilité et différentiabilité, fonctions implicites Dernière mise à jour : Jeudi 5 juin 2014

3. On a déjà vérifié que la fonction est continue sur R2 . Vérifions si ses dérivées partielles sont continues sur R2 . On a
( 2x y 5
2 2 2 si (x, y) 6= (0, 0),
∂x f (x, y) = (x +y )
0 si (x, y) = (0, 0),
2ρ 6 (cos(θ) sin5 (θ))
lim ∂x f (x, y) = lim = lim 2ρ 2 = 0 = ∂x f (0, 0)
(x,y)→(0,0) ρ→0 ρ4 ρ→0
∀θ
( x 2 y 2 (3x 2 +y 2 )
(x 2 +y 2 )2
si (x, y) 6= (0, 0),
∂ y f (x, y) =
0 si (x, y) = (0, 0).
ρ 6 (3 cos2 (θ) + sin2 (θ))
lim ∂ y f (x, y) = lim = lim 4ρ 2 = 0 = ∂ y f (0, 0),
(x,y)→(0,0) ρ→0 ρ4 ρ→0
∀θ

la fonction est donc de classe C (R ).1 2

4. Puisque toute fonction de classe C 1 (Ω) est différentiable sur Ω, on conclut que f est différentiable sur R2 .

Exercice 4.30 Continuité, dérivabilité, différentiabilité


Soit f : R2 → R la fonction ainsi définie
( ³ ´
1
(x 2 + y 2 )3 cos x 2 +y 2 si (x, y) 6= (0, 0),
f (x, y) =
0 si (x, y) = (0, 0).

1. Est-elle continue sur R2 ?


2. Calculer ∇ f (x, y).
3. La fonction f est-elle de classe C 1 (R2 ) ?
4. Sans faire de calculs, que peut-on conclure sur la différentiabilité de la fonction f sur R2 ?

C ORRECTION .
1. La fonction est clairement continue dans R2 \ {(0, 0)}. Elle est aussi continue en (0, 0) car
µ ¶
1
lim f (x, y) = lim ρ 6 cos 2 = 0 = f (0, 0).
(x,y)→(0,0) ρ→0 ρ
∀θ

f est donc continue sur R2 .


2. Pour (x, y) 6= (0, 0) on a
 ³ ´ ³ ´
1 1
∂x f (x, y) 6x(x 2 + y 2 )2 cos x 2 +y 2 2
2 + 2x(x + y ) sin x 2 +y 2
µ ¶
∇ f (x, y) = = ´ .
∂ y f (x, y)
³ ´ ³
1 1
6y(x 2 + y 2 )2 cos x 2 +y 2 2
2 + 2y(x + y ) sin x 2 +y 2

Pour (x, y) = (0, 0) on a


¶ Ã f (x,0)− f (0,0)
!
∂x f (0, 0) limx→0
µ µ ¶
x−0 0
∇ f (0, 0) = = f (0,y)− f (0,0) = .
∂ y f (0, 0) lim y→0 y−0
0

3. On a déjà vérifié que la fonction est continue sur R2 . Vérifions si ses dérivées partielles sont continues sur R2 . On a
( ³ ´ ³ ´
1 1
6x(x 2 + y 2 )2 cos x 2 +y 2 2
2 + 2x(x + y ) sin x 2 +y 2 si (x, y) 6= (0, 0),
∂x f (x, y) =
0 si (x, y) = (0, 0),
µ ¶ µ ¶
1 1
lim ∂x f (x, y) = lim 6ρ 5 cos(θ) cos 2 + 2ρ 3 cos(θ) sin 2 = 0 = ∂x f (0, 0)
(x,y)→(0,0) ρ→0 ρ ρ
∀θ
( ³ ´ ³ ´
1 1
6y(x 2 + y 2 )2 cos x 2 +y 2 + 2y(x 2
+ y 2
) sin x 2 +y 2
si (x, y) 6= (0, 0),
∂ y f (x, y) =
0 si (x, y) = (0, 0).
µ ¶ µ ¶
1 1
lim ∂ y f (x, y) = lim 6ρ 5 sin(θ) cos 2 + 2ρ 3 sin(θ) sin 2 = 0 = ∂ y f (0, 0),
(x,y)→(0,0) ρ→0 ρ ρ
∀θ

la fonction est donc de classe C 1 (R2 ).

96 © G. Faccanoni
Dernière mise à jour : Jeudi 5 juin 2014 4 Dérivabilité et différentiabilité, fonctions implicites

4. Puisque toute fonction de classe C 1 (Ω) est différentiable sur Ω, on conclut que f est différentiable sur R2 .

Exercice 4.31 Continuité, dérivabilité, différentiabilité


Soit f : R2 → R la fonction ainsi définie

y4


 si (x, y) 6= (0, 0),
f (x, y) = x2 + y 2

0 si (x, y) = (0, 0).

1. La fonction f est-elle continue sur R2 ?


2. Calculer ∇ f (x, y) pour (x, y) 6= (0, 0) ; calculer ensuite ∇ f (0, 0). La fonction f est-elle de classe C 1 (R2 ) ?
3. La fonction f est-elle différentiable sur R2 ?

C ORRECTION .
1. La fonction est clairement continue sur R2 \ {(0, 0)}. De plus
y4 (r sin(θ))4
= = r 2 sin4 (θ) ≤ r 2 −−−→ 0
x 2 + y 2 (r cos(θ))2 + (r sin(θ))2 r →0

d’où
lim f (x, y) = 0 = f (0, 0).
(x,y)→(0,0)

f est donc continue sur R2 .


2. ~
∇ f (x, y) est le vecteur de composantes ∂x f (x, y) et ∂ y f (x, y) et on a :

−2x y 4


 , si (x, y) 6= (0, 0),
(x 2 + y 2 )2

∂x f (x, y) =
 lim f (x, 0) − f (0, 0) = 0,

sinon;

x→0 x −0
2
y2

 3 2x +
2y
 , si (x, y) 6= (0, 0),
(x + y 2 )2
2

∂ y f (x, y) =
 f (0, y) − f (0, 0)
 lim = 0, sinon.


y→0 y −0
Étant donné que
¯ −2x y 4 ¯ 4
¯ ¯ ¯ ¯
¯ = 2 ¯ (r cos(θ))(r sin(θ))
¯ ¯
¯ ((r cos(θ))2 + (r sin(θ))2 )2 ¯ ≤ 2r −r−→0
−→ 0
¯ ¯
¯ (x 2 + y 2 )2 ¯

et
¯ 3
¯ 2y (2x 2 + y 2 ) ¯ 3 2 2 2 ¯¯
¯ ¯
¯ = 2 ¯ (r sin(θ)) (2(r cos(θ)) + (r sin(θ)) ) ¯ ≤ 18r 3 −−−→ 0
¯
¯
¯ (x 2 + y 2 )2 ¯ ¯ ((r cos(θ))2 + (r sin(θ))2 )2 ¯ r →0

on a
lim ∂x f (x, y) = 0 = ∂x f (0, 0),
(x,y)→(0,0)

lim ∂ y f (x, y) = 0 = ∂ y f (0, 0).


(x,y)→(0,0)

La fonction f est donc de classe C 1 (R2 ).


3. Étant de classe C 1 (R2 ), f est différentiable sur R2 . Si on ne se rappelle pas du théorème on peut le vérifier directement
par la définition : une application f : R2 → R est différentiable en (x 0 , y 0 ) ∈ R2 ssi
f (x, y) − f (x 0 , y 0 ) − ∂x f (x 0 , y 0 )(x − x 0 ) − ∂ y f (x 0 , y 0 )(y − y 0 )
lim p = 0.
(x,y)→(x 0 ,y 0 ) x2 + y 2
On a bien
f (x, y) − f (x 0 , y 0 ) − ∂x f (x 0 , y 0 )(x − x 0 ) − ∂ y f (x 0 , y 0 )(y − y 0 ) y4
lim = lim =0
(x,y)→(0,0) (x 2 + y 2 )3/2
p
(x,y)→(x 0 ,y 0 ) (x − x 0 )2 + (y − y 0 )2
car
y4 (r sin(θ))4
= ≤ r −−−→ 0.
(x 2 + y 2 )3/2 ((r cos(θ))2 + (r sin(θ))2 )3/2 r →0

© G. Faccanoni 97
4 Dérivabilité et différentiabilité, fonctions implicites Dernière mise à jour : Jeudi 5 juin 2014

Exercice 4.32 Continuité, dérivabilité, différentiabilité


Soit f : R2 → R la fonction ainsi définie
 y

2 + y2
si (x, y) 6= (0, 0),
f (x, y) = x
0 si (x, y) = (0, 0).

1. La fonction f est-elle continue sur R2 ?


2. Calculer ∇ f (x, y) pour (x, y) 6= (0, 0) ; calculer ensuite ∇ f (0, 0).
3. La fonction f est-elle différentiable sur R2 ?

C ORRECTION .
1. La fonction est continue sur R2 \ {(0, 0)} mais elle n’est pas continue sur R2 car si on considère la restriction de f à la
courbe y = x, qui passe par (0, 0), on a
1
lim f (x, x) = lim 6= f (0, 0).
x→0 x→0 2x

2. ~
∇ f (x, y) est le vecteur de composantes ∂x f (x, y) et ∂ y f (x, y) et on a :

−2x y

x 2 −y 2
(
(x 2 +y 2 )2
 , si (x, y) 6= (0, 0), , si (x, y) 6= (0, 0),
∂x f = ∂y f = (x 2 +y 2 )2
 lim f (x,0)− f (0,0) = 0, sinon; 6 ∃, sinon
x→0 x−0

f (0,y)− f (0,0)
car lim y−0 = ∞.
y→0

3. N’étant pas continue, f n’est pas différentiable sur R2 .

Exercice 4.33 Continuité, dérivabilité, différentiabilité


Soit f la fonction définie sur R2 par
2 2

x y +xy

si (x, y) 6= (0, 0),
f (x, y) = x2 + y 2

0 sinon.
1. La fonction f est-elle continue en (0, 0) ?
2. Déterminer si les dérivées partielles ∂x f (0, 0) et ∂ y f (0, 0) existent et les calculer le cas échéant.
3. La fonction f est-elle de classe C 1 sur R2 ?
4. La fonction f est-elle différentiable en (0, 0) ?

C ORRECTION .
1. On étudie la limite
x2 y + x y 2
lim .
(x,y)→(0,0) x 2 + y 2

On a
x2 y + x y 2 x+y ³ ´
= x y = r cos(ϑ) sin(ϑ) cos(ϑ) + sin(ϑ) −−−→ 0
x2 + y 2 x2 + y 2 r →0
∀ϑ
¯ ³ ´¯¯
¯
car ¯r cos(ϑ) sin(ϑ) cos(ϑ)+sin(ϑ) ¯¯ ≤ 2r . On en déduit que la limite de f (x, y) quand (x, y) tend vers (0, 0) est 0 = f (0, 0).
¯

La fonction est donc continue en (0, 0).


2. Attention : il faut revenir à la définition de dérivée partielle en un point puisque la fonction au point (0, 0) n’est pas
définie de la même façon que sur le reste du domaine.
(0+t )+0
f (0 + t , 0) − f (0, 0) (0 + t )0 (0+t )2 +02
−0 0
∂x f (0, 0) = lim = lim = lim = lim 0 = 0.
t →0 t t →0 t t →0 t t →0

La dérivée partielle par rapport à x en (0, 0) existe et est égale à 0.


La fonction est symétrique en x et y, donc la dérivée partielle par rapport à y en (0, 0) existe et est égale à 0.

98 © G. Faccanoni
Dernière mise à jour : Jeudi 5 juin 2014 4 Dérivabilité et différentiabilité, fonctions implicites

3. Ici on doit calculer la dérivée partielle sur le reste du domaine et vérifier si elle est continue ou non en (0, 0) qui est le
seul point qui peut poser des problèmes.

x2 y + x y 2 (2x y + y 2 )(x 2 + y 2 ) − (x 2 y + x y 2 )(2x) y 2 − x 2 + 2x y


µ ¶
∂x 2 2
= 2 2 2
= y2 .
x +y (x + y ) (x 2 + y 2 )2

On remarque que sur la droite y = x elle vaut 1/2 6= ∂x f (0, 0). On en déduit que la dérivée partielle par rapport à x n’est
pas continue en (0, 0) donc la fonction f n’est pas de classe C 1 sur R2 .
4. La fonction f est différentiable en (0, 0) ssi

f (x, y) − f (0, 0) − ∂x f (0, 0)(x − 0) − ∂ y f (0, 0)(y − 0)


lim p = 0.
(x,y)→(0,0) (x − 0)2 + (y − 0)2

On a
f (x, y) − f (0, 0) − ∂x f (0, 0)(x − 0) − ∂ y f (0, 0)(y − 0) x+y
lim = lim x y 2 .
(x + y 2 )3/2
p
(x,y)→(0,0) 2
(x − 0) + (y − 0) 2 (x,y)→(0,0)

On remarque que sur la droite y = x elle n’existe pas. La fonction f n’est pas différentiable en (0, 0).

Exercice 4.34 Continuité, dérivabilité, différentiabilité


Soit α ∈ R une constante et f : R2 → R la fonction ainsi définie
2 2

 ln(1 + x + y )

si (x, y) 6= (0, 0),
f (x, y) = 2
x +y 2
α

si (x, y) = (0, 0).

1. Étude de la fonction sur R2 \ { (0, 0) } :


1.1. calculer le gradient de f pour (x, y) ∈ R2 \ { (0, 0) } ;
1.2. montrer que f est de classe C 1 sur R2 \ { (0, 0) } ; que peut-on conclure sur la différentiabilité de f ?
2. Étude de la fonction en (0, 0) :
2.1. calculer le DL de ln(1 + r 2 ) à l’ordre 4 lorsque r ' 0 ;
2.2. calculer lim(x,y)→(0,0) f (x, y) et en déduire qu’il existe une valeur α0 de α (à préciser) pour laquelle f est conti-
nue en (0, 0) ;
2.3. montrer que f admet des dérivées partielles en (0, 0) si et seulement si α = α0 et calculer ∇ f (0, 0) ;
2.4. soit α = α0 , montrer que f est de classe C 1 en (0, 0) ; que peut-on déduire sur la différentiabilité en (0, 0) ?
2.5. soit α = α0 , utiliser la définition pour prouver qu’effectivement f est différentiable en (0, 0).

C ORRECTION .
1. Étude de la fonction sur R2 \ { (0, 0) }.
1.1. Le gradient de f pour (x, y) ∈ R2 \ { (0, 0) } est le vecteur de composantes

ln(1 + x 2 + y 2 ) ln(1 + x 2 + y 2 )
µ ¶ µ ¶
2x 1 2y 1
∂x f (x, y) = − , ∂ y f (x, y) = − .
(x 2 + y 2 ) 1 + x 2 + y 2 x2 + y 2 (x 2 + y 2 ) 1 + x 2 + y 2 x2 + y 2

1.2. f et ses dérivées partielles sont continues sur R2 \ { (0, 0) } car composition et quotients de fonctions continues
dont les dénominateurs ne s’annulent pas, f est donc de classe C 1 (R2 \ { (0, 0) }). Comme tout fonction de classe
C 1 est différentiable, on conclut que f est différentiable sur R2 \ { (0, 0) }.
2. Étude de la fonction en (0, 0).
2 4
2.1. ln(1 + x) ' x − x2 + o(x 2 ) lorsque x ' 0 donc ln(1 + r 2 ) ' r 2 − r2 + o(r 4 ) lorsque r ' 0.
2.2. Passons en coordonnées polaires :

f˜ : R∗+ × [0; 2π[ → R


ln(1 + r 2 )
(r, ϑ) 7→ f˜(r, ϑ) ≡ f (r cos(ϑ), r sin(ϑ)) = .
r2

Comme f˜(r, ϑ) −−−→ 1 pour tout ϑ, alors lim(x,y)→(0,0) f (x, y) = 1. Pour que f soit continue en (0, 0) il faut que
r →0
lim(x,y)→(0,0) f (x, y) = f (0, 0) donc α0 = 1.

© G. Faccanoni 99
4 Dérivabilité et différentiabilité, fonctions implicites Dernière mise à jour : Jeudi 5 juin 2014

2.3. Le gradient de f en (0, 0) est le vecteur de composantes

ln(1+x 2 ) Ã 2 x4
−α
!
f (x, 0) − f (0, 0) 1 ln(1 + x 2 ) 1 x − 2 1−α x
µ ¶
x2
∂x f (0, 0) = lim = lim = lim − α = lim − α , = lim − ,
x→0 x −0 x→0 x x→0 x x2 x→0 x x2 x→0 x 2
ln(1+y 2 )
f (0, y) − f (0, 0) y2
−α
∂ y f (0, 0) = lim = lim .
y→0 y −0 y→0 y

Ces deux limites existent et sont finies si et seulement si α = 1. Dans ce cas, ∇ f (0, 0) = (0, 0)T .
2.4. Pour que f soit de classe C 1 en (0, 0) il ne reste à vérifier que les dérivées partielles premières sont continues,
autrement dit il faut que

lim ∂x f (x, y) = ∂x f (0, 0) et lim ∂ y f (x, y) = ∂ y f (0, 0).


(x,y)→(0,0) (x,y)→(0,0)

Comme

ln(1 + x 2 + y 2 )
µ ¶
2x 1
lim ∂x f (x, y) = lim −
(x,y)→(0,0) (x,y)→(0,0) (x 2 + y 2 ) 1 + x 2 + y 2 x2 + y 2
ln(1 + r 2 ) r2 r (r 2 − 1)
µ ¶ µ ¶
2r cos(ϑ) 1 2 cos(ϑ) 1
= lim 2 2
− 2
= lim 2
− 1 + = lim 2 cos(ϑ) = 0,
r →0 r 1+r r r →0 r 1+r 2 r →0 (r + 1)
∀ϑ ∀ϑ ∀ϑ
ln(1 + x 2 + y 2 )
µ ¶
2y 1
lim ∂ y f (x, y) = lim −
(x,y)→(0,0) (x,y)→(0,0) (x 2 + y 2 ) 1 + x 2 + y 2 x2 + y 2
2 ¶
ln(1 + r ) r (r 2 − 1)
µ
2r sin(ϑ) 1
= lim − = lim sin(ϑ) = 0,
r →0 r2 1+r2 r2 r →0 (r 2 + 1)
∀ϑ ∀ϑ

on a bien ∂x f et ∂ y f sont continues en (0, 0). Comme f aussi est continue en (0, 0), on conclut que f est de classe
C 1 en (0, 0) ce qui implique que f est différentiable en (0, 0).
2.5. Pour prouver que f est différentiable en (0, 0) il faut montrer que

f (x, y) − f (0, 0) − ∂x f (0, 0)(x − 0) − ∂ y f (0, 0)(y − 0)


lim p = 0.
(x,y)→(0,0) (x − 0)2 + (y − 0)2

ln(1+x 2 +y 2 )
f (x,y)− f (0,0)−∂x f (0,0)(x−0)−∂ y f (0,0)(y−0) −1
x 2 +y 2
Notons h(x, y) = p = p . Passons en coordonnées polaires :
(x−0)2 +(y−0)2 x 2 +y 2

1 ln(1 + r 2 ) r
µ ¶
h̃(r, ϑ) = h(r cos(ϑ), r sin(ϑ)) = 2
− 1 ' − −−−→ 0.
r r 2 r →0

donc f est bien différentiable en (0, 0).

Exercice 4.35 Continuité, dérivabilité, différentiabilité


Soit α ∈ R une constante et f : R2 → R la fonction ainsi définie
2 2 2

 (1 + x + y ) − 1

si (x, y) 6= (0, 0),
f (x, y) = x2 + y 2
α

si (x, y) = (0, 0).

1. Étude de la fonction sur R2 \ { (0, 0) }.


1.1. Calculer le gradient de f α pour (x, y) ∈ R2 \ { (0, 0) }.
1.2. Montrer que f α est de classe C 1 sur R2 \ { (0, 0) } ; que peut-on conclure sur la différentiabilité de f α ?
2. Étude de la fonction en (0, 0).
2.1. Calculer lim(x,y)→(0,0) f α (x, y) et en déduire qu’il existe une valeur α0 de α (à préciser) pour laquelle f α est
continue en (0, 0).
2.2. Montrer que f α admet des dérivées partielles en (0, 0) si et seulement si α = α0 et calculer ∇ f α0 (0, 0).
2.3. En utilisant la définition, prouver que f α0 est différentiable en (0, 0).
2.4. Montrer que f α0 est de classe C 1 en (0, 0).

100 © G. Faccanoni
Dernière mise à jour : Jeudi 5 juin 2014 4 Dérivabilité et différentiabilité, fonctions implicites

C ORRECTION .
1. Étude de la fonction sur R2 \ { (0, 0) }.
1.1. Le gradient de f α pour (x, y) ∈ R2 \ { (0, 0) } est le vecteur de composantes

(1 + x 2 + y 2 )2 − 1 (1 + x 2 + y 2 )2 − 1
µ ¶ µ ¶
2x 2 2 2y 2 2
∂x f α (x, y) = 2 2(1 + x + y ) − , ∂ y f α (x, y) = 2 2(1 + x + y ) − .
x + y2 x2 + y 2 x + y2 x2 + y 2

1.2. f α et ses dérivées partielles sont continues sur R2 \ { (0, 0) } car composition et quotients de fonctions continues
dont les dénominateurs ne s’annulent pas, f α est donc de classe C 1 (R2 \ { (0, 0) }). Comme tout fonction de classe
C 1 est différentiable, on conclut que f α est différentiable sur R2 \ { (0, 0) }.
2. Étude de la fonction en (0, 0).
2.1. Passons en coordonnées polaires :

f˜α : R∗+ × [0; 2π[ → R


(1 + r 2 )2 − 1
(r, ϑ) 7→ f˜α (r, ϑ) ≡ f α (r cos(ϑ), r sin(ϑ)) = = r 2 + 2.
r2

Comme f˜α (r, ϑ) −−−→ 2 alors lim(x,y)→(0,0) f α (x, y) = 2.


r →0
∀ϑ
Pour que f α soit continue en (0, 0) il faut que lim(x,y)→(0,0) f α (x, y) = f α (0, 0) donc pour que f α soit continue en
(0, 0) il faut que α soit égal à α0 = 2.
2.2. Le gradient de f α en (0, 0) est le vecteur de composantes

(1+x 2 )2 −1
f α (x, 0) − f α (0, 0) −α 2−α
µ ¶
x2
∂x f α (0, 0) = lim = lim = lim x + ,
x→0 x −0 x→0 x x→0 x
(1+y 2 )2 −1
f α (0, y) − f α (0, 0) y2
−α µ
2−α

∂ y f α (0, 0) = lim = lim = lim y + .
y→0 y −0 y→0 y y→0 y

Ces deux limites existent et sont finies si et seulement si α = 2. Dans ce cas, ∇ f α (0, 0) = (0, 0)T .
2.3. Pour prouver que f α0 est différentiable en (0, 0) il faut montrer que

f α0 (x, y) − f α0 (0, 0) − ∂x f α0 (0, 0)(x − 0) − ∂ y f α0 (0, 0)(y − 0)


lim p = 0.
(x,y)→(0,0) (x − 0)2 + (y − 0)2

(1+x 2 +y 2 )2 −1
f α0 (x,y)− f α0 (0,0)−∂x f α0 (0,0)(x−0)−∂ y f α0 (0,0)(y−0) −2
x 2 +y 2
Notons h(x, y) = p = p . Passons en coordonnées polaires :
(x−0)2 +(y−0)2 x 2 +y 2

∀ϑ
h̃(r, ϑ) = h(r cos(ϑ), r sin(ϑ)) = r −−−→ 0
r →0

donc f α0 est bien différentiable en (0, 0).


2.4. Pour que f α0 soit de classe C 1 en (0, 0) il faut qu’elle et ses dérivées partielles premières soient continues, donc il
faut que

lim f α0 (x, y) = f α0 (0, 0), lim ∂x f α0 (x, y) = ∂x f α0 (0, 0) et lim ∂ y f α0 (x, y) = ∂ y f α0 (0, 0).
(x,y)→(0,0) (x,y)→(0,0) (x,y)→(0,0)

Comme

lim f α0 (x, y) = 2,
(x,y)→(0,0)
2x
∂x f α0 (x, y) = 2(1 + x 2 + y 2 ) − f α0 (x, y) = lim 4r cos(ϑ) = 0,
¡ ¢
lim lim
(x,y)→(0,0) (x,y)→(0,0) (x 2 + y 2 ) r →0
∀ϑ
2y
∂ y f α0 (x, y) = 2(1 + x 2 + y 2 ) − f α0 (x, y) = lim 4r sin(ϑ) = 0,
¡ ¢
lim lim
(x,y)→(0,0) (x,y)→(0,0) (x 2 + y 2 ) r →0
∀ϑ

par conséquent f α0 , ∂x f α0 et ∂ y f α0 sont continues en (0, 0) et on conclut que f α0 est de classe C 1 en (0, 0).

© G. Faccanoni 101
4 Dérivabilité et différentiabilité, fonctions implicites Dernière mise à jour : Jeudi 5 juin 2014

Exercice 4.36 Continuité, dérivabilité, différentiabilité


Soit α ∈ R une constante et f : R2 → R la fonction ainsi définie
 2 2
x +y
e
 −1
si (x, y) 6= (0, 0),
f α (x, y) = x + y2
2

α

si (x, y) = (0, 0).

1. Étude de la fonction sur R2 \ { (0, 0) }.


1.1. Calculer le gradient de f α pour (x, y) ∈ R2 \ { (0, 0) }.
1.2. Montrer que f α est de classe C 1 sur R2 \ { (0, 0) } ; que peut-on conclure sur la différentiabilité de f α ?
2. Étude de la fonction en (0, 0).
2.1. Calculer lim(x,y)→(0,0) f α (x, y) et en déduire qu’il existe une valeur α0 de α (à préciser) pour laquelle f α est
continue en (0, 0).
2.2. Montrer que f α admet des dérivées partielles en (0, 0) si et seulement si α = α0 et calculer ∇ f α0 (0, 0).
2.3. En utilisant la définition, prouver que f α0 est différentiable en (0, 0).
2.4. Montrer que f α0 est de classe C 1 en (0, 0).

C ORRECTION .
1. Étude de la fonction sur R2 \ { (0, 0) }.
1.1. Le gradient de f α pour (x, y) ∈ R2 \ { (0, 0) } est le vecteur de composantes

2x ³ x 2 +y 2 ´ 2y ³ x 2 +y 2 ´
∂x f α (x, y) = e − f α (x, y) , ∂ y f α (x, y) = e − f α (x, y) .
x2 + y 2 x2 + y 2

1.2. f α et ses dérivées partielles sont continues sur R2 \ { (0, 0) } car composition et quotients de fonctions continues
dont les dénominateurs ne s’annulent pas, f α est donc de classe C 1 (R2 \ { (0, 0) }). Comme tout fonction de classe
C 1 est différentiable, on conclut que f α est différentiable sur R2 \ { (0, 0) }.
2. Étude de la fonction en (0, 0).
2.1. Passons en coordonnées polaires :

f˜α : R∗+ × [0; 2π[ → R


2
e r −1
(r, ϑ) 7→ f˜α (r, ϑ) ≡ f (r cos(ϑ), r sin(ϑ)) = r2
.

2) 2 4
Lorsque r ' 0, le développement limité de e (r à l’ordre 4 s’écrit e (r ) ' 1 + r 2 + r2 + o(r 4 ), ainsi
2
e r −1 1+r 2 +o(r 3 )−1
f˜α (r, ϑ) = r2
' r2
= 1 + o(r )

donc
lim f α (x, y) = lim f˜α (r, ϑ) = 1.
(x,y)→(0,0) r →0
∀ϑ

Pour que f α soit continue en (0, 0) il faut que lim(x,y)→(0,0) f α (x, y) = f α (0, 0) donc pour que f α soit continue en
(0, 0) il faut que α soit égal à α0 = 1.
2.2. Le gradient de f α en (0, 0) est le vecteur de composantes
2
e x −1
f (x, 0) − f (0, 0) −α 1 1 + x 2 + o(x 3 ) − 1 1 + o(x) − α
µ ¶
x2
∂x f α (0, 0) = lim = lim = lim − α = lim ,
x→0 x −0 x→0 x x→0 x x2 x→0 x
f α (0, y) − f α (0, 0) 1 + o(y) − α
∂ y f α (0, 0) = lim = lim .
y→0 y −0 y→0 y

Ces deux limites existent et sont finies si et seulement si α = 1. Dans ce cas, ∇ f α (0, 0) = (0, 0)T .
2.3. Pour prouver que f α0 est différentiable en (0, 0) il faut montrer que
2 +y 2
ex −1
f α0 (x, y) − f α0 (0, 0) − ∂x f α0 (0, 0)(x − 0) − ∂ y f α0 (0, 0)(y − 0) x 2 +y 2
lim h(x, y) = 0, avec h(x, y) = p =p .
(x,y)→(0,0) (x − 0)2 + (y − 0)2 x2 + y 2

102 © G. Faccanoni
Dernière mise à jour : Jeudi 5 juin 2014 4 Dérivabilité et différentiabilité, fonctions implicites

Passons aux coordonnées polaires :

h̃ : R∗+ × [0; 2π[ → R


2
e (r ) −1 2
−1
r2 e (r ) −1−r 2
(r, ϑ) 7→ h̃(r, ϑ) ≡ h(r cos(ϑ), r sin(ϑ)) = r = r3

donc, lorsque r ' 0,


4
1 + r 2 + r2 + o(r 4 ) − 1 − r 2 r ∀ϑ
h̃(r, ϑ) ' = + o(r ) −−−→ 0
r3 2 r →0

et on conclut que f α0 est bien différentiable en (0, 0).


2.4. Pour que f α0 soit de classe C 1 en (0, 0) il faut qu’elle et ses dérivées partielles premières soient continues, donc il
faut que

lim f α0 (x, y) = f α0 (0, 0), lim ∂x f α0 (x, y) = ∂x f α0 (0, 0) et lim ∂ y f α0 (x, y) = ∂ y f α0 (0, 0).
(x,y)→(0,0) (x,y)→(0,0) (x,y)→(0,0)

Comme

lim f α0 (x, y) = 1,
(x,y)→(0,0)
2x ³ 2 2 ´
lim ∂x f α0 (x, y) = lim e x +y − f α0 (x, y) = lim (r + o(r )) cos(ϑ) = 0,
(x,y)→(0,0) (x,y)→(0,0) x 2 + y 2 r →0
∀ϑ
2y ³ x 2 +y 2 ´
lim ∂ y f α0 (x, y) = lim e − f α0 (x, y) = lim (r + o(r )) sin(ϑ) = 0,
(x,y)→(0,0) (x,y)→(0,0) x 2 + y 2 r →0
∀ϑ

par conséquent f α0 , ∂x f α0 et ∂ y f α0 sont continues en (0, 0) et on conclut que f α0 est de classe C 1 en (0, 0).

Exercice 4.37 Continuité, dérivabilité, différentiabilité


Soit α ∈ R une constante et f : R2 → R la fonction ainsi définie
2 2

 cos(x + y ) − 1

si (x, y) 6= (0, 0),
f α (x, y) = (x 2 + y 2 )2
α

si (x, y) = (0, 0).

1. Étude de la fonction sur R2 \ { (0, 0) }.


1.1. Calculer le gradient de f α pour (x, y) ∈ R2 \ { (0, 0) }.
1.2. Montrer que f α est de classe C 1 sur R2 \ { (0, 0) } ; que peut-on conclure sur la différentiabilité de f α ?
2. Étude de la fonction en (0, 0).
2.1. Calculer lim(x,y)→(0,0) f α (x, y) et en déduire qu’il existe une valeur α0 de α (à préciser) pour laquelle f α est
continue en (0, 0).
2.2. Montrer que f α admet des dérivées partielles en (0, 0) si et seulement si α = α0 et calculer ∇ f α0 (0, 0).
2.3. En utilisant la définition, prouver que f α0 est différentiable en (0, 0).
2.4. Montrer que f α0 est de classe C 1 en (0, 0).

C ORRECTION .
1. Étude de la fonction sur R2 \ { (0, 0) }.
1.1. Le gradient de f α pour (x, y) ∈ R2 \ { (0, 0) } est le vecteur de composantes

sin(x 2 + y 2 ) sin(x 2 + y 2 )
µ ¶ µ ¶
−2x −2y
∂x f α (x, y) = + 2 f α (x, y) , ∂ y f α (x, y) = + 2 f α (x, y) .
x2 + y 2 x2 + y 2 x2 + y 2 x2 + y 2

1.2. f α et ses dérivées partielles sont continues sur R2 \ { (0, 0) } car composition et quotients de fonctions continues
dont les dénominateurs ne s’annulent pas, f α est donc de classe C 1 (R2 \ { (0, 0) }). Comme tout fonction de classe
C 1 est différentiable, on conclut que f α est différentiable sur R2 \ { (0, 0) }.
2. Étude de la fonction en (0, 0).

© G. Faccanoni 103
4 Dérivabilité et différentiabilité, fonctions implicites Dernière mise à jour : Jeudi 5 juin 2014

2.1. Passons en coordonnées polaires :

f˜α : R∗+ × [0; 2π[ → R


cos(r 2 )−1
(r, ϑ) 7→ f˜α (r, ϑ) ≡ f (r cos(ϑ), r sin(ϑ)) = r4
.
4 6
r
Lorsque r ' 0, le développement limité de cos(r 2 ) à l’ordre 6 s’écrit cos(r 2 ) ' 1 − r2 + 24 + o(r 6 ), ainsi
4 6
cos(r 2 )−1 1− r2 + r24 +o(r 6 )−1 1
f˜α (r, ϑ) = r4
' r4
= − + o(r 2 )
2
donc
1
lim f α (x, y) = lim f˜α (r, ϑ) = − .
(x,y)→(0,0) r →0 2
∀ϑ

Pour que f α soit continue en (0, 0) il faut que lim(x,y)→(0,0) f α (x, y) = f α (0, 0) donc pour que f α soit continue en
(0, 0) il faut que α soit égal à α0 = − 12 .
2.2. Le gradient de f α en (0, 0) est le vecteur de composantes
4 6
cos(x 2 )−1 1− x2 + x24 +o(x 6 )−1
−α −α − 12 − α
à !
f (x, 0) − f (0, 0) x4 x4 x
∂x f α (0, 0) = lim = lim = lim = lim + + o(x)
x→0 x −0 x→0 x x→0 x x→0 x 24
− 12 −α
à !
f α (0, y) − f α (0, 0) y
∂ y f α (0, 0) = lim = lim + + o(y) .
y→0 y −0 y→0 y 24

Ces deux limites existent et sont finies si et seulement si α = − 21 . Dans ce cas, ∇ f α (0, 0) = (0, 0)T .
2.3. Pour prouver que f α0 est différentiable en (0, 0) il faut montrer que

cos(x 2 +y 2 )−1
f α0 (x, y) − f α0 (0, 0) − ∂x f α0 (0, 0)(x − 0) − ∂ y f α0 (0, 0)(y − 0) (x 2 +y 2 )2
+ 12
lim h(x, y) = 0, avec h(x, y) = p = p .
(x,y)→(0,0) (x − 0)2 + (y − 0)2 x2 + y 2

Passons aux coordonnées polaires :

h̃ : R∗+ × [0; 2π[ → R


cos(r 2 )−1 1 4
r4
+2 cos(r 2 )−1+ r2
(r, ϑ) 7→ h̃(r, ϑ) ≡ h(r cos(ϑ), r sin(ϑ)) = r = r5

donc, lorsque r ' 0,


4 6 4
r
1 − r2 + 24 + o(r 6 ) − 1 + r2 r ∀ϑ
h̃(r, ϑ) ' = + o(r ) −−−→ 0
r5 24 r →0

et on conclut que f α0 est bien différentiable en (0, 0).


2.4. Pour que f α0 soit de classe C 1 en (0, 0) il faut qu’elle et ses dérivées partielles premières soient continues, donc il
faut que

lim f α0 (x, y) = f α0 (0, 0), lim ∂x f α0 (x, y) = ∂x f α0 (0, 0) et lim ∂ y f α0 (x, y) = ∂ y f α0 (0, 0).
(x,y)→(0,0) (x,y)→(0,0) (x,y)→(0,0)

On a
1
lim f α0 (x, y) = − ,
(x,y)→(0,0) 2
sin(x 2 + y 2 ) sin(x 2 + y 2 )
µ ¶ µ ¶
−2x −2x
lim ∂x f α0 (x, y) = lim + 2 f α0 (x, y) = lim −1 ,
(x,y)→(0,0) (x,y)→(0,0) x 2 + y 2 x2 + y 2 (x,y)→(0,0) x 2 + y 2 x2 + y 2
sin(x 2 + y 2 ) sin(x 2 + y 2 )
µ ¶ µ ¶
−2y −2y
lim ∂ y f α0 (x, y) = lim + 2 f α0 (x, y) = lim −1 .
(x,y)→(0,0) (x,y)→(0,0) x 2 + y 2 x2 + y 2 (x,y)→(0,0) x 2 + y 2 x2 + y 2
6
Lorsque r ' 0, le développement limité de sin(r 2 ) à l’ordre 6 s’écrit sin(r 2 ) ' r 2 − r6 + o(r 6 ), ainsi

1
lim f α0 (x, y) = − ,
(x,y)→(0,0) 2
−2r cos(ϑ) sin(r 2 ) r4
µ ¶ µ ¶
−2 cos(ϑ) 4
lim ∂x f α0 (x, y) = lim − 1 = lim 1− + o(r ) − 1 = 0,
(x,y)→(0,0) r →0 r2 r2 r →0 r 6
∀ϑ ∀ϑ

104 © G. Faccanoni
Dernière mise à jour : Jeudi 5 juin 2014 4 Dérivabilité et différentiabilité, fonctions implicites

−2r sin(ϑ) sin(r 2 ) r4


µ ¶ µ ¶
−2 sin(ϑ) 4
lim ∂ y f α0 (x, y) = lim − 1 = lim 1− + o(r ) − 1 = 0;
(x,y)→(0,0) r →0 r2 r2 r →0 r 6
∀ϑ ∀ϑ

par conséquent f α0 , ∂x f α0 et ∂ y f α0 sont continues en (0, 0) et on conclut que f α0 est de classe C 1 en (0, 0).

Exercice 4.38 Continuité, dérivabilité, différentiabilité


Soit m ∈ N et f : R2 → R la fonction définie comme suit :
 m
 x y si (x, y) 6= (0, 0),
f (x, y) = x 2 + y 2
0 si (x, y) = (0, 0).

1. Étude de la fonction sur R2 \ { (0, 0) } :


1.1. montrer que f est continue sur R2 \ { (0, 0) } pour tout m ∈ N ;
1.2. calculer le gradient de f pour (x, y) ∈ R2 \ { (0, 0) } pour tout m ∈ N ;
1.3. montrer que f est de classe C 1 sur R2 \ { (0, 0) } pour tout m ∈ N ;
1.4. que peut-on conclure sur la différentiabilité de f sur R2 \ { (0, 0) } ?
2. Étude de la fonction en (0, 0) :
2.1. pour quelles valeurs de m ∈ N la fonction f est-elle continue en (0, 0) ?
2.2. calculer le gradient de f en (0, 0) pour tout m ∈ N ;
2.3. pour quelles valeurs de m ∈ N la fonction f est-elle différentiable en (0, 0) ?
2.4. pour quelles valeurs de m ∈ N la fonction f est-elle de classe C 1 en (0, 0) ?

C ORRECTION .
1. Étude de la fonction sur R2 \ { (0, 0) }.
1.1. f est continue sur R2 \ { (0, 0) } car quotient de fonctions continues dont le dénominateur ne s’annule pas.
1.2. Le gradient de f pour (x, y) ∈ R2 \ { (0, 0) } est le vecteur de composantes

x m−1 y(mx 2 + m y 2 − 2x 2 ) x m (x 2 − y 2 )
∂x f (x, y) = , ∂ y f (x, y) = .
(x 2 + y 2 )2 (x 2 + y 2 )2

1.3. f est continue et dérivable sur R2 \ { (0, 0) } ; ses dérivées partielles sont continues sur R2 \ { (0, 0) } car quotients de
fonctions continues dont les dénominateurs ne s’annulent pas. Alors f est de classe C 1 (R2 \ { (0, 0) }).
1.4. Comme f est de classe C 1 (R2 \ { (0, 0) }) alors f est différentiable sur R2 \ { (0, 0) }.
2. Étude de la fonction en (0, 0).
2.1. Pour que f soit continue en (0, 0) il faut que lim(x,y)→(0,0) f (x, y) = f (0, 0).
B Si m = 0, f s’écrit ( y
2 2 si (x, y) 6= (0, 0)
f (x, y) = x +y
0 sinon
qui n’est pas continue en (0, 0) car f (x, 0) = 0 mais f (x, x) = 1/(2x) −−−→ ∞.
x→0
B Si m = 1, f s’écrit ( xy
x 2 +y 2
si (x, y) 6= (0, 0)
f (x, y) =
0 sinon
qui n’est pas continue en (0, 0) car f (x, 0) = 0 mais f (x, x) = 1/2.
B Si m > 1, passons en coordonnées polaires : x = 0 + r cos(ϑ), y = 0 + r sin(ϑ),

f˜(r, ϑ) = f (x(r, ϑ), y(r, ϑ)) = r m−1 cosm (ϑ) sin(ϑ).

Comme | f˜(r, ϑ)| ≤ r m−1 −−−→ 0 alors lim(x,y)→(0,0) f (x, y) = 0 donc f est continue en (0, 0).
r →0
2.2. Le gradient de f en (0, 0) est le vecteur de composantes

f (x, 0) − f (0, 0) f (0, y) − f (0, 0)


∂x f (0, 0) = lim = 0, ∂ y f (0, 0) = lim = 0.
x→0 x −0 y→0 y −0

© G. Faccanoni 105
4 Dérivabilité et différentiabilité, fonctions implicites Dernière mise à jour : Jeudi 5 juin 2014

2.3. f est différentiable en (0, 0) si et seulement si

f (x, y) − f (0, 0) − ∂x f (0, 0)(x − 0) − ∂ y f (0, 0)(y − 0)


lim p = 0.
(x,y)→(0,0) (x − 0)2 + (y − 0)2

De plus, toute fonction différentiable est continue.


B Si m = 0 ou m = 1, f n’est pas continue en (0, 0) donc elle n’est pas différentiable en (0, 0).
f (x,y)− f (0,0)−∂x f (0,0)(x−0)−∂ y f (0,0)(y−0) xm y
B Soit m > 1 et notons h(x, y) = p = 2 2 3/2 .
2 2
(x−0) +(y−0) (x +y )
1
B Si m = 2, comme h(y, y) = 23/2 6= 0, alors f n’est pas différentiable en (0, 0).
B Si m > 2, passons en coordonnées polaires : x = 0 + r cos(ϑ), y = 0 + r sin(ϑ),

h̃(r, ϑ) = h(x(r, ϑ), y(r, ϑ)) = r m−2 cosm (ϑ) sin(ϑ).

Comme |h̃(r, ϑ)| ≤ r m−2 −−−→ 0 alors lim(x,y)→(0,0) h(x, y) = 0 donc f est différentiable en (0, 0).
r →0
2.4. Une fonction est de classe C 1 si ses dérivées partielles sont continues, autrement dit si

lim ∂x f (x, y) = ∂x f (0, 0) et lim ∂ y f (x, y) = ∂ y f (0, 0).


(x,y)→(0,0) (x,y)→(0,0)

De plus, toute fonction de classe C 1 est différentiable.


B Si m = 0 ou m = 1 ou m = 2, f n’est pas différentiable en (0, 0) donc elle n’est pas de classe C 1 en (0, 0).
B Si m > 2, passons en coordonnées polaires : x = 0 + r cos(ϑ), y = 0 + r sin(ϑ),
(
∂˜ x f (r, ϑ) = ∂x f (x(r, ϑ), y(r, ϑ)) = r m−2 (m − cos2 (ϑ)) cosm−1 (ϑ) sin(ϑ),
∂˜ y f (r, ϑ) = ∂ y f (x(r, ϑ), y(r, ϑ)) = r m−2 (cos2 (ϑ) − sin2 (ϑ)) cosm (ϑ).

Comme |∂˜ x f (r, ϑ)| ≤ mr m−2 −−−→ 0 et |∂˜ y f (r, ϑ)| ≤ 2r m−2 −−−→ 0 alors
r →0 r →0
lim ∂x f (x, y) = 0 et lim ∂ y f (x, y) = 0 donc f est de classe C 1 en (0, 0).
(x,y)→(0,0) (x,y)→(0,0)

Exercice 4.39 Continuité, dérivabilité, différentiabilité


Soit m ∈ N et f : R2 → R la fonction définie comme suit :
m 2

 x y

si (x, y) 6= (0, 0),
f (x, y) = x + y 2
2

0 si (x, y) = (0, 0).

1. Étude de la fonction sur R2 \ { (0, 0) } :


1.1. montrer que f est continue sur R2 \ { (0, 0) } pour tout m ∈ N ;
1.2. calculer le gradient de f pour (x, y) ∈ R2 \ { (0, 0) } pour tout m ∈ N ;
1.3. montrer que f est de classe C 1 sur R2 \ { (0, 0) } pour tout m ∈ N ;
1.4. que peut-on conclure sur la différentiabilité de f sur R2 \ { (0, 0) } ?
2. Étude de la fonction en (0, 0) :
2.1. pour quelles valeurs de m ∈ N la fonction f est-elle continue en (0, 0) ?
2.2. calculer le gradient de f en (0, 0) pour tout m ∈ N ;
2.3. pour quelles valeurs de m ∈ N la fonction f est-elle différentiable en (0, 0) ?
2.4. pour quelles valeurs de m ∈ N la fonction f est-elle de classe C 1 en (0, 0) ?

C ORRECTION .
1. Étude de la fonction sur R2 \ { (0, 0) }.
1.1. f est continue sur R2 \ { (0, 0) } car quotient de fonctions continues dont le dénominateur ne s’annule pas.
1.2. Le gradient de f pour (x, y) ∈ R2 \ { (0, 0) } est le vecteur de composantes

(m − 1)x m−1 y 2 (x 2 + y 2 ) − x m y 2 (2x) x m−1 y 2 (mx 2 + m y 2 − 2x 2 )


∂x f (x, y) = = ,
(x 2 + y 2 )2 (x 2 + y 2 )2
2y(x 2 + y 2 ) − 2y 3 2x m+2 y
∂ y f (x, y) = x m 2 2 2
= 2 .
(x + y ) (x + y 2 )2

106 © G. Faccanoni
Dernière mise à jour : Jeudi 5 juin 2014 4 Dérivabilité et différentiabilité, fonctions implicites

1.3. f est continue et dérivable sur R2 \ { (0, 0) } ; ses dérivées partielles sont continues sur R2 \ { (0, 0) } car quotients de
fonctions continues dont les dénominateurs ne s’annulent pas. Alors f est de classe C 1 (R2 \ { (0, 0) }).
1.4. Comme f est de classe C 1 (R2 \ { (0, 0) }) alors f est différentiable sur R2 \ { (0, 0) }.
2. Étude de la fonction en (0, 0).
2.1. Pour que f soit continue en (0, 0) il faut que lim(x,y)→(0,0) f (x, y) = f (0, 0).
B Si m = 0, f s’écrit
( y2
2 2 si (x, y) 6= (0, 0)
f (x, y) = x +y
0 sinon
qui n’est pas continue en (0, 0) car f (x, 0) = 0 mais f (x, x) = 1/2.
B Si m > 0, passons en coordonnées polaires : x = 0 + r cos(ϑ), y = 0 + r sin(ϑ),

f˜(r, ϑ) = f (x(r, ϑ), y(r, ϑ)) = r m cosm (ϑ) sin2 (ϑ).

Comme | f˜(r, ϑ)| ≤ r m −−−→ 0 alors lim(x,y)→(0,0) f (x, y) = 0 donc f est continue en (0, 0).
r →0
2.2. Le gradient de f en (0, 0) est le vecteur de composantes

f (x, 0) − f (0, 0) f (0, y) − f (0, 0)


∂x f (0, 0) = lim = 0, ∂ y f (0, 0) = lim = 0.
x→0 x −0 y→0 y −0

2.3. f est différentiable en (0, 0) si et seulement si

f (x, y) − f (0, 0) − ∂x f (0, 0)(x − 0) − ∂ y f (0, 0)(y − 0)


lim p = 0.
(x,y)→(0,0) (x − 0)2 + (y − 0)2

De plus, toute fonction différentiable est continue.


B Si m = 0, f n’est pas continue en (0, 0) donc elle n’est pas différentiable en (0, 0).
f (x,y)− f (0,0)−∂x f (0,0)(x−0)−∂ y f (0,0)(y−0) xm y 2
B Soit m > 0 et notons h(x, y) = p = .
(x−0)2 +(y−0)2 (x 2 +y 2 )3/2
1
B Si m = 1, comme h(y, y) = 23/2 6= 0, alors f n’est pas différentiable en (0, 0).
B Si m > 1, passons en coordonnées polaires : x = 0 + r cos(ϑ), y = 0 + r sin(ϑ),

h̃(r, ϑ) = h(x(r, ϑ), y(r, ϑ)) = r m−1 cosm (ϑ) sin2 (ϑ).

Comme |h̃(r, ϑ)| ≤ r m−1 −−−→ 0 alors lim(x,y)→(0,0) h(x, y) = 0 donc f est différentiable en (0, 0).
r →0
2.4. Une fonction est de classe C 1 si ses dérivées partielles sont continues, autrement dit si

lim ∂x f (x, y) = ∂x f (0, 0) et lim ∂ y f (x, y) = ∂ y f (0, 0).


(x,y)→(0,0) (x,y)→(0,0)

De plus, toute fonction de classe C 1 est différentiable.


B Si m = 0 ou m = 1, f n’est pas différentiable en (0, 0) donc elle n’est pas de classe C 1 en (0, 0).
B Si m > 1, passons en coordonnées polaires : x = 0 + r cos(ϑ), y = 0 + r sin(ϑ),
(
∂x f˜(r, ϑ) = ∂x f (x(r, ϑ), y(r, ϑ)) = r m−1 (m − cos2 (ϑ)) cosm−1 (ϑ) sin2 (ϑ),
∂ y f˜(r, ϑ) = ∂ y f (x(r, ϑ), y(r, ϑ)) = 2r m−1 cosm−2 (ϑ) sin(ϑ).

Comme

|∂x f˜(r, ϑ)| ≤ mr m−1 −−−→ 0 |∂ y f˜(r, ϑ)| ≤ 2r m−1 −−−→ 0


r →0 r →0

alors lim(x,y)→(0,0) ∂x f (x, y) = 0 et lim(x,y)→(0,0) ∂ y f (x, y) = 0 donc f est de classe C 1 en (0, 0).

© G. Faccanoni 107
4 Dérivabilité et différentiabilité, fonctions implicites Dernière mise à jour : Jeudi 5 juin 2014

Polynôme de Taylor

Exercice 4.40
Calculer les polynômes de TAYLOR à l’ordre 1 (i.e. l’équation du plan tangent) et à l’ordre 2 au voisinage du point (x 0 , y 0 )
des fonctions f : R2 → R suivantes :
1. f (x, y) = 1 + x + y + x 3 + x y 3 au voisinage du point (−1, 1)
2. f (x, y) = 1 + x + y + x 3 + x y 3 au voisinage du point (0, 0)
3. f (x, y) = 1 + x + y + x 2 − x y + y 2 au voisinage du point (1, 2)
4. f (x, y) = 1 + x + y + x 2 − x y + y 2 au voisinage du point (0, 0)
5. f (x, y) = 1 + x + y au voisinage du point (1, 1)
6. f (x, y) = 1 + x + y au voisinage du point (0, 0)
7. f (x, y) = 1 − x + y − x 2 + x y 3 au voisinage du point (0, 0)

C ORRECTION .
1. f (x, y) = 1 + x + y + x 3 + x y 3 au voisinage du point (x 0 , y 0 ) = (−1, 1) :

f (x, y) = 1 + x + y + x 3 + x y 3 f (x 0 , y 0 ) = 0
2 2
∂x f (x, y) = 1 + 3x + y − y ∂x f (x 0 , y 0 ) = 4
∂ y f (x, y) = 1 + x − 2x y ∂ y f (x 0 , y 0 ) = 2
∂xx f (x, y) = 6x ∂xx f (x 0 , y 0 ) = −6
∂x y f (x, y) = 1 − 2y ∂x y f (x 0 , y 0 ) = −1
∂ y y f (x, y) = −2x ∂ y y f (x 0 , y 0 ) = 2

donc
B Polynôme de TAYLOR à l’ordre 1 (= plan tangent) : z = 4x + 2y + 2
B Polynôme de TAYLOR à l’ordre 2 : z = 4x + 2y + 2 − 3(x + 1)2 + (y − 1)2 − (x + 1)(y − 1) = −3x 2 − x y + y 2 − x − y + 1
2. f (x, y) = 1 + x + y + x 3 + x y 3 au voisinage du point (x 0 , y 0 ) = (0, 0)

f (x, y) = 1 + x + y + x 3 + x y 3 f (x 0 , y 0 ) = 1
2 2
∂x f (x, y) = 1 + 3x + y − y ∂x f (x 0 , y 0 ) = 1
∂ y f (x, y) = 1 + x − 2x y ∂ y f (x 0 , y 0 ) = 1
∂xx f (x, y) = 6x ∂xx f (x 0 , y 0 ) = 0
∂x y f (x, y) = 1 − 2y ∂x y f (x 0 , y 0 ) = 1
∂ y y f (x, y) = −2x ∂ y y f (x 0 , y 0 ) = 0

donc
B Polynôme de TAYLOR à l’ordre 1 (= plan tangent) : z = 1 + x + y
B Polynôme de TAYLOR à l’ordre 2 : z = 1 + x + y + x y
3. f (x, y) = 1 + x + y + x 2 − x y + y 2 au voisinage du point (x 0 , y 0 ) = (1, 2)

f (x, y) = 1 + x + y + x 2 − x y + y 2 f (x 0 , y 0 ) = 7
∂x f (x, y) = 1 + 2x − y ∂x f (x 0 , y 0 ) = 1
∂ y f (x, y) = 1 − x + 2y ∂ y f (x 0 , y 0 ) = 4
∂xx f (x, y) = 2 ∂xx f (x 0 , y 0 ) = 2
∂x y f (x, y) = −1 ∂x y f (x 0 , y 0 ) = −1
∂ y y f (x, y) = 2 ∂ y y f (x 0 , y 0 ) = 2

donc
B Polynôme de TAYLOR à l’ordre 1 (= plan tangent) : z = x + 4y − 2
B Polynôme de TAYLOR à l’ordre 2 : z = x + 4y − 2 + (x − 1)2 − (x − 1)(y − 2) + (y − 2)2 = x 2 − x y + y 2 + x + y + 1 = f (x, y)
4. f (x, y) = 1 + x + y + x 2 − x y + y 2 au voisinage du point (x 0 , y 0 ) = (0, 0)

f (x, y) = 1 + x + y + x 2 − x y + y 2 f (x 0 , y 0 ) = 1

108 © G. Faccanoni
Dernière mise à jour : Jeudi 5 juin 2014 4 Dérivabilité et différentiabilité, fonctions implicites

∂x f (x, y) = 1 + 2x − y ∂x f (x 0 , y 0 ) = 1
∂ y f (x, y) = 1 − x + 2y ∂ y f (x 0 , y 0 ) = 1
∂xx f (x, y) = 2 ∂xx f (x 0 , y 0 ) = 2
∂x y f (x, y) = −1 ∂x y f (x 0 , y 0 ) = −1
∂ y y f (x, y) = 2 ∂ y y f (x 0 , y 0 ) = 2

donc
B Polynôme de TAYLOR à l’ordre 1 (= plan tangent) : z = x + y + 1
B Polynôme de TAYLOR à l’ordre 2 : z = x 2 − x y + y 2 + x + y + 1 = f (x, y)
5. f (x, y) = 1 + x + y au voisinage du point (x 0 , y 0 ) = (1, 1)
B Polynôme de TAYLOR à l’ordre 1 (= plan tangent) : z = 1 + x + y = f (x, y)
B Polynôme de TAYLOR à l’ordre 2 : z = 1 + x + y = f (x, y)
6. f (x, y) = 1 + x + y + x 2 − x y + y 2 au voisinage du point (x 0 , y 0 ) = (0, 0)
B Polynôme de TAYLOR à l’ordre 1 (= plan tangent) : z = 1 + x + y = f (x, y)
B Polynôme de TAYLOR à l’ordre 2 : z = 1 + x + y = f (x, y)
7. f (x, y) = 1 − x + y − x 2 + x y 3 au voisinage du point (x 0 , y 0 ) = (0, 0)
B Polynôme de TAYLOR à l’ordre 1 (= plan tangent) : z = 1 − x + y = f (x, y)
B Polynôme de TAYLOR à l’ordre 2 : z = 1 − x + y − x 2

© G. Faccanoni 109
4 Dérivabilité et différentiabilité, fonctions implicites Dernière mise à jour : Jeudi 5 juin 2014

Fonctions implicites

Exercice 4.41
Calculer y 0 (x) à partir des relations suivantes
1. y cos(x) = x 2 + y 2
2. cos(x y) = 1 + sin(y)
3. e y sin(x) = x + x y

C ORRECTION . On réécrit chaque relation sous la forme d’une équation F (x, y) = 0. On suppose qu’il existe une unique
fonction y = y(x) telle que F (x, y(x)) = 0 pour tout x ∈ I . Alors, en dérivant par rapport à x cette équation, on a

∂F ∂x ∂F ∂y
+ = 0.
∂x |{z}
∂x ∂y |{z}
∂x
=1 y 0 (x)

∂F
Donc, si ∂y 6= 0 alors
∂F
∂x (x, y(x))
y 0 (x) = − ∂F ∀x ∈ I .
∂y (x, y(x))
2x+y(x) sin(x)
1. F (x, y) = y cos(x) − x 2 − y 2 ; ∂x F (x, y) = −y sin(y) − 2x, ∂ y F (x, y) = cos(x) − 2y ; y 0 (x) = cos(x)−2y(x) .
−y(x) sin(x y(x))
2. F (x, y) = cos(x y) − 1 − sin(y) ; ∂x F (x, y) = −y sin(x y), ∂ y F (x, y) = −x sin(x y) − cos(y) ; y 0 (x) = cos(y(x))+x sin(x y(x)) .
1+y(x)−cos(x)e y(x)
3. F (x, y) = e y sin(x) − x − x y ; ∂x F (x, y) = e y cos(x) − 1 − y, ∂ y F (x, y) = e y sin(x) − x ; y 0 (x) = e y(x) sin(x)−x
.

Exercice 4.42
On considère la courbe plane d’équation
ye x + e y sin(2x) = 0. (4.1)
1. Vérifier que l’équation (4.1) définie une et une seule fonction y = ϕ(x) au voisinage de (0, 0).
2. Calculer ϕ0 (0) et écrire l’équation de la droite tangente au graphe de la fonction ϕ en le point (0, ϕ(0)).
y
3. En déduire la limite de x quand (x, y) tend vers (0, 0) en étant sur la courbe.

C ORRECTION . Soit f : R2 → R la fonction définie par f (x, y) = ye x + e y sin(2x). La courbe plane donnée est la courbe de
niveau 0 de la fonction f .
1. On note que (0, 0) est une solution de l’équation f (x, y) = 0. On a

∂x f (x, y) = ye x + 2e y cos(2x), ∂x f (0, 0) = 2,


∂ y f (x, y) = e x + e y sin(2x), ∂ y f (0, 0) = 1.

Puisque ∂ y f (0, 0) 6= 0 il existe une et une seule fonction ϕ : [0−ε, 0+ε] → R définie au voisinage de 0 tel que f (x, ϕ(x)) =
0, i.e. ϕ(x)e x + e ϕ(x) sin(2x) = 0.
2. On a

∂x f (x, ϕ(x)) ϕ(x)e x + 2e ϕ(x) cos(2x) ∂x f (0, 0)


ϕ0 (x) = − =− , ϕ0 (0) = − = −2,
∂ y f (x, ϕ(x)) e x + e ϕ(x) sin(2x) ∂ y f (0, 0)

donc l’équation de la droite tangente à ϕ en x = 0 est y = −2x.


3. On a
y ϕ(x) ϕ(x) − ϕ(0)
lim = lim = lim = lim ϕ0 (x) = ϕ0 (0) = −2.
(x,y)→(0,0) x x→0 x x→0 x −0 x→0
f (x,y)=0

110 © G. Faccanoni
Dernière mise à jour : Jeudi 5 juin 2014 4 Dérivabilité et différentiabilité, fonctions implicites

z = f (x, y)

z =0

y = ϕ(x)

Exercice 4.43
On considère la courbe plane d’équation
xe y + e x sin(2y) = 0. (4.2)
1. Vérifier que l’équation (4.2) définie une et une seule fonction y = ϕ(x) au voisinage de (0, 0).
2. Calculer ϕ0 (0) et écrire l’équation de la droite tangente au graphe de la fonction ϕ en le point (0, ϕ(0)).
y
3. En déduire la limite de x quand (x, y) tend vers (0, 0) en étant sur la courbe.

C ORRECTION . On pose f (x, y) = xe y + e x sin(2y)


1. On note que (0, 0) est une solution de l’équation f (x, y) = 0. On a

∂x f (x, y) = e y + e x sin(2y), ∂x f (0, 0) = 1,


y x
∂ y f (x, y) = xe + 2e cos(2y), ∂ y f (0, 0) = 2.

Puisque ∂ y f (0, 0) 6= 0 il existe une et une seule fonction y = ϕ(x) définie au voisinage de 0 tel que f (x, ϕ(x)) = 0.
2. On a
∂x f (0, 0) 1
ϕ0 (0) = − =−
∂ y f (0, 0) 2
donc l’équation de la droite tangente à ϕ en x = 0 est y = − 21 x.
3. On a
y ϕ(x) ϕ(x) − ϕ(0) 1
lim = lim = lim = lim ϕ0 (x) = ϕ0 (0) = − .
(x,y)→(0,0) x x→0 x x→0 x −0 x→0 2
f (x,y)=0

Exercice 4.44
On considère la courbe plane d’équation
2x 3 y + 2x 2 + y 2 = 5. (4.3)
1. Vérifier que l’équation (4.3) définie une et une seule fonction y = ϕ(x) au voisinage de (1, 1).
2. Calculer ϕ0 (1) et écrire l’équation de la droite tangente au graphe de la fonction ϕ en le point (1, ϕ(1)).
3. Sachant que

∂xx f (x, ϕ(x))(∂ y f (x, ϕ(x)))2 − 2∂x f (x, ϕ(x))∂ y f (x, ϕ(x))∂x y f (x, ϕ(x)) + (∂x f (x, ϕ(x)))2 ∂ y y f (x, ϕ(x))
ϕ00 (x) = −
(∂ y f (x, ϕ(x)))3

en déduire le développement de Taylor de ϕ à l’ordre 2 centré en x = 1.

C ORRECTION . On pose f (x, y) = 2x 3 y + 2x 2 + y 2 − 5


1. On note que (1, 1) est une solution de l’équation f (x, y) = 0. On a

∂x f (x, y) = 6x 2 y + 4x ∂x f (1, 1) = 10,


3
∂ y f (x, y) = 2x + 2y, ∂ y f (1, 1) = 4.

Puisque ∂ y f (1, 1) 6= 0 il existe une et une seule fonction y = ϕ(x) définie au voisinage de 1 tel que f (x, ϕ(x)) = 1.

© G. Faccanoni 111
4 Dérivabilité et différentiabilité, fonctions implicites Dernière mise à jour : Jeudi 5 juin 2014

2. On a
∂x f (1, 1) 5
ϕ0 (1) = − =−
∂ y f (1, 1) 2

donc l’équation de la droite tangente à ϕ en x = 1 est y = − 52 (x − 1) + 1 = − 52 x + 72 .


3. On a
∂xx f (∂ y f )2 − 2∂x f ∂ y f ∂x y f + (∂x f )2 ∂ y y f
ϕ00 (x) = −
(∂ y f )3
et

∂xx f (x, y) = 4(3x y + 1) ∂xx f (1, 1) = 16,


2
∂x y f (x, y) = 6x ∂x y f (1, 1) = 6,
∂ y y f (x, y) = 2, ∂ y y f (1, 1) = 2,

ϕ00 (1)
donc ϕ00 (1) = −1 d’où ϕ(x) = ϕ(1) + ϕ0 (1)(x − 1) + 2 (x − 1)
2
= 1 − 25 (x − 1) − 12 (x − 1)2 .

Exercice 4.45
On considère l’équation xe y + ye x = 0.
1. Vérifier qu’elle définie une et une seule fonction y = ϕ(x) au voisinage de (0, 0).
2. Calculer le développement de Taylor de ϕ à l’ordre 2 centré en x = 0.

C ORRECTION .

z = g (x, y)

z =0

y = ϕ(x)

Soit g (x, y) = xe y + ye x ; elle est clairement de classe C 1 . De plus, g (0, 0) = 0 et g y (0, 0) = 1. Elle définie alors une et une seule
fonction y = ϕ(x) dans un intervalle qui contient x = 0. De plus, on sait que ϕ(0) = 0 et ϕ0 (0) = −1 car

g x (x, ϕ(x)) e ϕ(x) + ϕ(x)e x


ϕ0 (x) = − =− .
g y (x, ϕ(x)) xe ϕ(x) + e x

Comme ϕ est de classe C 1 , alors ϕ0 est de classe C 1 car composition de fonctions de classe C 1 , il s’en suit que ϕ est de classe
C 2 . Alors
ϕ00 (0) 2
ϕ(x) = ϕ(0) + ϕ0 (0)x + x + o(x 2 ).
2
Pour calculer ϕ00 on peut soit dériver l’expression de ϕ0 soit dériver l’équation qui la défini implicitement : comme

g (x, ϕ(x)) = xe ϕ(x) + ϕ(x)e x

en dérivant cette expression on obtient

e ϕ(x) + xe ϕ(x) ϕ0 (x) + ϕ0 (x)e x + ϕ(x)e x = 0

112 © G. Faccanoni
Dernière mise à jour : Jeudi 5 juin 2014 4 Dérivabilité et différentiabilité, fonctions implicites

et en la dérivant à nouveau (ce que l’on peut faire car ϕ est de classe C 2 ) on obtient

e ϕ(x) ϕ0 (x) + e ϕ(x) ϕ0 (x) + xe ϕ(x) (ϕ0 (x))2 + xe ϕ(x) ϕ00 (x) + ϕ00 (x)e x + ϕ0 (x)e x + ϕ0 (x)e x + ϕ(x)e x = 0.
¡ ¢ ¡ ¢ ¡ ¢

On obtient ϕ00 (0) = 4 et le développement de Taylor s’écrit

ϕ(x) = −x + 2x 2 + o(x 2 ).

Exercice 4.46
Montrer que l’équation x y 4 − x 3 + y = 0 définit implicitement au voisinage de 0 une fonction réelle de la variable réelle
y = ϕ(x) et calculer la tangente au graphe de ϕ au point (0, 0).

C ORRECTION . Soit g (x, y) = x y 4 − x 3 + y ; elle est clairement de classe C 1 . De plus, g (0, 0) = 0 et g y (0, 0) = 1. Elle définie alors
une et une seule fonction y = ϕ(x) dans un intervalle qui contient x = 0. De plus, on sait que ϕ(0) = 0 et ϕ0 (0) = car

g x (x, ϕ(x)) y 4 − 3x 2
ϕ0 (x) = − =− .
g y (x, ϕ(x)) 4x y 3 + 1

La droite tangente à y = ϕ(x) en (0, 0) a équation y = ϕ0 (0)(x − 0) + 0 = 0.

Exercice 4.47
Soit f l’application définie sur R2 par
x2 y y3
f (x, y) = + x2 + − 4y.
2 3
Soit l’équation f (x, y) = 3. Montrer qu’elle définit implicitement au voisinage de (0, −3) une fonction y = ϕ(x) et calculer
l’équation de la droite tangente à y = ϕ(x) en x = 0.

C ORRECTION . f est une application de classe C ∞ sur R2 . Comme f (0, −3) = 3 et ∂ y f (0, −3) = 5 6= 0, alors il existe un inter-
valle ouvert I contenant x = 0 et une fonction ϕ de I dans R de classe C ∞ sur I telle que ϕ(0) = −3 et pour tout x ∈ I

∂x f (x, ϕ(x)) ϕ(x) + 2


f (x, ϕ(x)) = 3 et ϕ0 (x) = − = −2x 2 .
∂ y f (x, ϕ(x)) x + 2ϕ2 (x) − 8
−3+2
L’équation de la droite tangente à y = ϕ(x) en x = 0 est y = mx + q avec m = ϕ0 (0) = −2 × 0 02 +2×(−3) 2 −8 = 0 et q = y 0 − mx 0 =
−3 − 0 = −3.

Exercice 4.48
Soit f l’application définie sur R2 par
x2 y y3
f (x, y) = − x2 + − 4y.
2 3
Soit l’équation f (x, y) = −3. Montrer qu’elle définit implicitement au voisinage de (0, 3) une fonction y = ϕ(x) et calculer
l’équation de la droite tangente à y = ϕ(x) en x = 0.

C ORRECTION . f est une application de classe C ∞ sur R2 . Comme f (0, 3) = −3 et ∂ y f (0, 3) = 5 6= 0, alors il existe un intervalle
ouvert I contenant x = 0 et une fonction ϕ de I dans R de classe C ∞ sur I telle que ϕ(0) = 3 et pour tout x ∈ I

∂x f (x, ϕ(x)) ϕ(x) − 2


f (x, ϕ(x)) = −3 et ϕ0 (x) = − = −2x 2 .
∂ y f (x, ϕ(x)) x + 2ϕ2 (x) − 8
3−2
L’équation de la droite tangente à y = ϕ(x) en x = 0 est y = mx + q avec m = ϕ0 (0) = −2 × 0 02 +2×3 2 −8 = 0 et q = y 0 − mx 0 =
3 − 0 = 3.

© G. Faccanoni 113
4 Dérivabilité et différentiabilité, fonctions implicites Dernière mise à jour : Jeudi 5 juin 2014

Exercice 4.49
Soit f l’application définie sur R2 par
x y 2 x3
f (x, y) = + − 4x + y 2 .
2 3
Soit l’équation f (x, y) = 7. Montrer qu’elle définit implicitement au voisinage de (3, 2) une fonction y = ϕ(x) et calculer
l’équation de la droite tangente à y = ϕ(x) en x = 3.

C ORRECTION . f est une application de classe C ∞ sur R2 . Comme f (3, 2) = 7 et ∂ y f (3, 2) = 5 6= 0, alors il existe un intervalle
ouvert I contenant x = 3 et une fonction ϕ de I dans R de classe C ∞ sur I telle que ϕ(3) = 2 et pour tout x ∈ I

∂x f (x, ϕ(x)) ϕ(x)2 + 2x 2 − 8


f (x, ϕ(x)) = 7 et ϕ0 (x) = − =− .
∂ y f (x, ϕ(x)) 2(x + 2)ϕ(x)

7 7 41
L’équation de la droite tangente à y = ϕ(x) en x = 3 est y = mx + q avec m = ϕ0 (3) = − 10 et q = y 0 − mx 0 = 2 + 10 3= 10 .

Exercice 4.50
Calculer ∂x z(x, y) et ∂ y z(x, y) à partir des relations suivantes
1. x 2 + 2y 2 + 3z 2 = 1
2. x 2 − y 2 + z 2 − 2z = 4
3. e z = x y z
4. y z + x ln(y) = z 2

C ORRECTION . On réécrit chaque relation sous la forme d’une équation F (x, y, z) = 0. On suppose qu’il existe une unique
fonction z = z(x, y) telle que F (x, y, z(x, y)) = 0 pour tout (x, y) ∈ Ω. Alors, en dérivant par rapport à x et à y cette équation, on
a les deux relations
∂F ∂x ∂F ∂y ∂F ∂z

 ∂x ∂x + ∂y ∂x + ∂z ∂x = 0,



 |{z} |{z} |{z}
=1 =0 ∂ z(x,y)

x
∂F ∂x ∂F ∂y ∂F ∂z

 ∂x ∂y + ∂y ∂y + ∂z ∂y = 0.

 |{z} |{z} |{z}

∂ y z(x,y)

=0 =1

∂F
Donc, si ∂z 6= 0, alors
∂F ∂F
(x, y, z(x, y)) ∂y (x, y, z(x, y))

∂x z(x, y) = − ∂Fx , ∂ y z(x, y) = − ∂F .
∂z (x, y, z(x, y)) ∂z (x, y, z(x, y))
x −2y
1. F (x, y, z) = x 2 +2y 2 +3z 2 −1 ; ∂x F (x, y, z) = 2x, ∂ y F (x, y, z) = 4y, ∂z F (x, y, z) = 6z ; ∂x z(x, y) = − 3z(x,y) , ∂ y z(x, y) = 3z(x,y) .
x
2. F (x, y, z) = x 2 −y 2 +z 2 −2z−4 ; ∂x F (x, y, z) = 2x, ∂ y F (x, y, z) = −2y, ∂z F (x, y, z) = 2z−2 ; ∂x z(x, y) = − z(x+y)−1 , ∂ y z(x, y) =
y
z(x+y)−1 .
y z(x,y)
3. F (x, y, z) = e z − x y z ; ∂x F (x, y, z) = −y z, ∂ y F (x, y, z) = −xz, ∂z F (x, y, z) = e z − x y ; ∂x z(x, y) = e z(x,y) −x y
, ∂ y z(x, y) =
xz(x,y)
e z(x,y) −x y
.
− ln(y)
4. F (x, y, z) = y z +x ln(y)−z 2 ; ∂x F (x, y, z) = ln(y), ∂ y F (x, y, z) = z + xy , ∂z F (x, y, z) = y −2z ; ∂x z(x, y) = y−2z(x,y) , ∂ y z(x, y) =
z(x,y)+ xy
y−2z(x,y) .

Exercice 4.51
Montrez que l’équation
x 3 + 4x y + z 2 − 3y z 2 − 3 = 0
permet d’exprimer z en fonction de (x, y) au voisinage de (1, 1, 1). Calculez alors ∂x z(1, 1) et ∂ y z(1, 1).

114 © G. Faccanoni
Dernière mise à jour : Jeudi 5 juin 2014 4 Dérivabilité et différentiabilité, fonctions implicites

C ORRECTION . Soit f la fonction définie sur R3 par f (x, y, z) = x 3 + 4x y + z 2 − 3y z 2 − 3. f possède des dérivées partielles
continues :

∂x f (x, y, z) = 3x 2 + 4y, ∂ y f (x, y, z) = 4x − 3z 2 , ∂z f (x, y, z) = 2z − 6y z,

et au point (1, 1, 1) on a

∂x f (1, 1, 1) = 7, ∂ y f (1, 1, 1) = 1, ∂z f (1, 1, 1) = −4.

Puisque f (1, 1, 1) = 0 et ∂z f (1, 1, 1) 6= 0, le théorème des fonctions implicites nous assure qu’on peut exprimer z en fonction
de (x, y) dans un voisinage de (1, 1, 1) et que

∂z ∂x f (1, 1, 1) 7 ∂z ∂ y f (1, 1, 1) 1
(1, 1) = − = , (1, 1) = − = .
∂x ∂z f (1, 1, 1) 4 ∂y ∂z f (1, 1, 1) 4

Pour (x, y) ' (1, 1) on a alors

∂z ∂z 7 1 7 1
z(x, y) ' z(1, 1) + (x − 1) (1, 1) + (y − 1) (1, 1) = 1 + (x − 1) + (y − 1) = x + y − 1.
∂x ∂y 4 4 4 4

© G. Faccanoni 115
5 Optimisation de fonctions de Rn dans R
Parmi toutes les sujets abordées dans ce cours, l’optimisation de fonctions, généralement de plusieurs variables, est sans
conteste celle qui apparaît le plus fréquemment dans la modélisation physique ou économique (maximiser le bénéfice, la
satisfaction des clients, la productivité ou minimiser les coûts, le risque, etc.).
Un optimum ou extremum est soit un maximum soit un minimum, c’est-à-dire la valeur la plus haute ou la plus faible que
prend la fonction sur son ensemble de définition ou tout sous-ensemble de son ensemble de définition. La détermination
des extrema dans le cas d’une seule variable a été détaillée lors du module M11. Logiquement, le présent chapitre en est un
prolongement qui fait intervenir les diverses notions spécifiques aux fonctions de plusieurs variables. Néanmoins, le thème
de l’optimisation à plusieurs variables exige un saut conceptuel important. En effet, plusieurs notions relatives aux fonctions
d’une variable sont loin de se généraliser de façon évidente. Par exemple, les fonctions d’une variable (suffisamment régu-
lières) admettent une seule dérivée première, une seule dérivée seconde, etc., tandis que, pour les fonctions de n variables,
le nombre de dérivées possibles croît avec l’ordre de dérivation : n dérivées partielles premières, n 2 dérivées partielles se-
condes, etc. Une conséquence directe de cet accroissement apparaîtra sous la forme de conditions nécessaires et suffisantes
plus lourdes à formuler. Par ailleurs, la prise en compte pratique des contraintes imposées aux variables doit être fondamen-
talement revue.
Prenons le cas de la maximisation de la fonction x 7→ f (x) dont l’unique variable est soumise à une contrainte de non-
négativité (x ≥ 0), qui traduit par exemple le fait que x est une quantité, une densité, un prix ou toute autre grandeur dépour-
vue de sens pour une valeur négative. L’optimisation s’effectue alors à l’aide de la procédure usuelle, la contrainte ayant pour
effet de restreindre le domaine d’étude à R+ et d’exiger un examen particulier pour le seul point qui en constitue le «bord»
(x = 0). Plongeons le même problème dans un cadre bivarié : maximiser f (x, y) sous © la double contrainte x ≥ 0 et y ≥ 0. À pré-
sent, le domaine admissible est donné par (R+ )2 dont le «bord», { (x, 0) | x ∈ R+ }∪ (0, y) ¯ y ∈ R+ , comporte évidemment une
¯ ª

infinité de points. Une étude individuelle des points de cet ensemble devient techniquement difficile, de sorte qu’il apparaît
indispensable de disposer de méthodes d’optimisation qui intègrent d’emblée la présence de contraintes qui font apparaître
des «bords». Cette approche (recherche d’extrema liés), spécifique aux fonctions de plusieurs variables, sera abordée dans
ce chapitre après l’exposé des principes de l’optimisation dite libre, qui vise la détermination des extrema dans un domaine
ouvert, donc «sans bords».

Exemple
Vous vous promenez sur un chemin de montagne.
B Vous êtes très sportif et vous désirez aller le plus haut possible dans la région dont vous avez la carte.
B Vous êtes un peu moins sportif, vous vous contentez de suivre le chemin indiqué. À la fin de la journée, vous désirez savoir quel est
le point le plus haut ou le plus bas où vous êtes passé.
L’altitude est une fonction h de deux variables, la position (x, y) sur la carte.
B Dans le premier cas, vous cherchez à trouver le maximum de la fonction h.
B Dans le deuxième cas, le chemin est représenté par une contrainte entre x et y donnée par une équation g (x, y) = 0. La question est
donc de trouver les points (x 0 , y 0 ) vérifiant g (x 0 , y 0 ) tels que h soit extrémale en (x 0 , y 0 ) parmi les points vérifiant g (x, y) = 0.

Définition
Soit f une fonction de D ⊂ Rn dans R. On dit que
B f est bornée dans D s’il existe un nombre réel M ≥ 0
tel que
∀x ∈ D, | f (x)| ≤ M ;
B f admet un maximum (resp. minimum) global (ou ab-
solu) en x0 ∈ D si

∀x ∈ D, f (x) ≤ f (x0 ) (resp. f (x) ≥ f (x0 )) ;

B f admet un maximum (resp. minimum) local (ou re-


latif) en x0 ∈ D s’il existe une boule de rayon non nul
B(x0 , r ) telle que

∀x ∈ D ∩ B(x0 , r ), f (x) ≤ f (x0 ) (resp. f (x) ≥ f (x0 )).

117
5 Extrema Dernière mise à jour : Jeudi 5 juin 2014

Un problème d’optimisation étant donné, deux questions se posent : existe-t-il des solutions ? Et comment calculer les solu-
tions éventuelles ? La théorie de l’optimisation affronte donc deux problèmes classiques en mathématiques : celui de l’exis-
tence et celui des méthodes de recherche.
À priori, un problème d’optimisation peut n’admettre aucune solution ou en admettre au moins une. En toute généralité,
aucun argument mathématique ne garantit l’existence de solution(s). On dispose cependant d’une condition suffisante
grâce au théorème de W EIERSTRASS, qui ne concerne que les fonctions continues sur un compact de Rn , i.e. un sous-
ensemble de Rn qui est fermé et borné. On rappelle qu’un ensemble est fermé s’il contient sa frontière et qu’un ensemble
2
©est borné 2s’il
¯ est contenuª dans une boule de rayon r ∈ R. Par exemple,
© le carré 2[−1;
¯ 21] est unª compact de R2 ; l’ensemble
2
(x, y) ∈ R x ≥ 0, y ≥© 0 est un ensemble
¯
ª fermé non borné ; le disque (x, y) ∈ R x + y < 1 est un ensemble borné non
¯
fermé ; le demi-plan (x, y) ∈ R2 ¯ x > 0 est un ensemble non fermé non borné.
¯

Théorème de W EIERSTRASS
Soit D un compact de Rn et soit f : D → R une fonction continue, alors f admet un maximum et un minimum globaux
atteints au moins une fois, autrement dit il existe xm ∈ D et xM ∈ D tels que

f (xm ) ≤ f (x) ≤ f (xM ), ∀(x) ∈ D.

Remarquons que les extrema peuvent appartenir soit à l’ouvert D̊ = D \ ∂D soit à la frontière ∂D.

Ce résultat donne une condition suffisante d’existence d’un minimum et d’un maximum globaux. Cependant, la restriction
relative au domaine compact est malheureusement trop forte pour la résolution de nombreux problèmes, en particulier ceux
pour lesquels les variables ne sont pas bornées. Beaucoup plus épineuse à n variables qu’à une seule variable, la question
du traitement des éventuels «bords» du domaine motive la scission entre extrema libres et liés. En effet, les restrictions du
domaine qui s’expriment sous la forme d’une égalité (g (x) = 0) ou d’une inégalité non stricte (g (x) ≤ 0) créent généralement
une infinité de points de bords, dont l’étude au cas par cas devient impraticable. Les conditions de L AGRANGE et de K UHN et
T UCKER visent à combler cette lacune.
Avant d’aborder ces nouveaux résultats, la section suivante présente la détermination des extrema dans un domaine ouvert.
La terminologie d’extrema «libres» résulte de l’absence de conditions introduisant des «bords» dans le domaine, que l’on
appelle aussi des «contraintes».

5.1 Extrema libres


Pour rechercher les extrema, la première idée qui vient en tête est de calculer les valeurs que prend la fonction f pour toutes
les valeurs prises par les arguments puis de repérer la plus grande et la plus petite valeurs prises par les images. Ce n’est
évidemment pas la bonne option si les arguments peuvent prendre une infinité de valeurs. Si la fonction à optimiser est une
fonction numérique d’une variable réelle, on peut toujours construire le tableau de variations ou tracer la fonction dans le
plan comme on l’a appris au lycée. C’est fastidieux dans la mesure où on ne s’intéresse qu’aux optima et à eux seuls. Si la
fonction à optimiser est à deux variables réelles, on peut à la rigueur demander à un logiciel approprié de tracer sa surface
représentative ou des courbes de niveau et conclure au vu des graphes. Ce peut être parfois utile mais c’est souvent frustrant
dans la mesure où on ignore a priori où se trouvent les optima, ce qui entraine qu’on est bien en peine de donner toutes
les indications utiles au traceur de fonction. Quoiqu’il en soit, dès que la fonction a plus de trois variables, les méthodes gra-
phiques ne sont d’aucun secours et il faut disposer d’une théorie solide pour déterminer les optima. L’objet de ce chapitre est
d’illustrer certaines résultats mathématiques qui permettent de répondre à certains de ces questions.
Il se trouve que la théorie mathématique de l’optimisation est très complète pour les fonctions au moins deux fois continu-
ment différentiables sur un ouvert. Résoudre un problème de maximisation (resp. de minimisation) au moyen des théorèmes
reposant sur la différentiabilité revient à rechercher les maxima (resp. les minima) locaux. En effet, la différentiabilité est une
propriété locale en ce sens qu’il est licite de remplacer une fonction par une fonction polynomiale au voisinage d’un point.
L’approximation résultant de la différentiation est de moins en moins bonne, voire franchement erronée, quand on s’éloigne
du point où elle a été calculée. En conséquence, la partie la plus élaborée de la théorie de l’optimisation donne les proprié-
tés locales d’une solution et se trouve démunie pour caractériser les optima globaux sauf si les fonctions ont des propriétés
particulières telles que la convexité ou la concavité.

118 © G. Faccanoni
Dernière mise à jour : Jeudi 5 juin 2014 5 Extrema

Théorème de F ERMAT : condition nécessaire du premier ordre


Soit D un sous-ensemble ouvert de Rn , x0 un point contenu dans D et f : D → R une fonction de classe C 1 en ce point.
Si f présente un extrémum local alors
∇ f (x0 ) = 0.

Définition Point stationnaire ou critique


À l’instar des fonctions d’une variable réelle, un point x0 vérifiant ∇ f (x0 ) = 0 est appelé point stationnaire ou point critique
de f .

Nature d’un point critique : étude directe


La condition du premier ordre signifie géométriquement que le plan tangent à la surface d’équation z = f (x, y) au point
(x 0 , y 0 ) de coordonnées (x 0 , y 0 , f (x 0 , y 0 )) est horizontal. Après avoir déterminé un point stationnaire x0 , on peut alors
déterminer sa nature en étudiant le signe de la différence

d (h) = f (x0 + h) − f (x0 ).

Si cette différence est de signe constant pour h voisin de 0, il s’agit d’un extrémum local (un maximum si d < 0, un
minimum si d > 0). Sinon, il s’agit d’un point-col (ou point-selle). Mieux, si le signe est constant pour h quelconque, alors
l’extrémum est global.
La figure à gauche illustre le cas d’un maximum et la figure au centre le cas d’un minimum. La figure à droite illustre le fait
que la condition nécessaire d’optimalité n’est pas une condition suffisante ; dans ce cas on dit que f présente un col en
(x 0 , y 0 ) ou que (x 0 , y 0 ) est un point-selle de f . Le mot col vient de l’exemple de la fonction altitude et de la configuration
(idéalisée) d’un col de montagne : minimum de la ligne de crête, maximum de la route, sans être un extremum du paysage.
Le mot selle vient de l’exemple d’une selle de cheval.

5,0 -1,5 5,24

4,9 -1,6 5,14

4,8 -1,7 5,04

4,7 -1,8 4,94

4,6 -1,9 4,84


1,5 1,5 1,5

4,5 -2,0 4,74


2,0 2,0
2,0 -0,5 -0,5 x
-0,5 x x

0,0 0,0
0,0
2,5 2,5
2,5 y 0,5
y 0,5 y 0,5

Exemple
©On cherche ¯ les extremaª de la fonction f (x, y) = x 2 + y 2 dans le disque ouvert centré en (0, 0) de rayon 1, représenté par D =
(x, y) ∈ R x + y 2 < 1 . Le seul candidat extremum est l’unique point critique (0, 0) qu’on trouve en résolvant ∂x f (x, y) = 0 et
2 ¯ 2

∂ y f (x, y) = 0. La définition implique de façon immédiate que f admet un minimum global en (0, 0). En effet

f (x, y) = x 2 + y 2 ≥ 0 = f (0, 0) ∀(x, y) ∈ D.

En revanche, la fonction n’admet aucun maximum.

Attention
La condition du premier ordre ne concerne que les fonctions différentiables définies sur un ensemble D ouvert. Ce-
pendant, il peut naturellement arriver qu’une fonction ne soit pas différentiable en un point et admette néanmoins un
extremum. Ou encore, la fonction peut être différentiable sur un compact, mais la condition (nécessaire) du premier
ordre s’applique aux points intérieurs et non aux points de la frontière où f peut pourtant passer par un optimum. On a
alors la «recette» suivante pour le calcul des extrema d’une fonction f continue dans un ensemble D compact.

© G. Faccanoni 119
5 Extrema Dernière mise à jour : Jeudi 5 juin 2014

«Recette» pour le calcul des extrema d’une fonction f continue dans un ensemble D compact
B On calcul la valeur de f en les points stationnaires de f dans l’ouvert D̊ = D \ ∂D ;
B on calcul la valeur de f en les points stationnaires de f sur le bord ∂D (i.e. il faut étudier la restriction de f à la courbe
qui définit le bord ∂D) ;
B on calcul la valeur de f en les points de non dérivabilité (s’il en existe).
La plus grande valeur donne le maximum global, la plus petite le minimum global.

Exemple
On veut trouver les extrema globaux de la fonction f définie par f (x, y) = (x − y)2 sur le carré fermé D = [0; 1]2 .
B On calcule la valeur de f en les points stationnaires de f dans l’ouvert D̊ =]0; 1[2 :

f (x, y) = (x − y)2 , ∂x f (x, y) = 2(x − y), ∂ y f (x, y) = −2(x − y),

et ∇ f = (0, 0) si et seulement si y = x et l’on a f (x, x) = 0.


B On calcule la valeur de f en les points stationnaires de f sur le bord ∂D :
B Arrête d’équation y = 0 : soit g :]0, 1[→ R telle que g (x) ≡ f (x, 0) = x 2 alors g 0 (x) 6= 0 pour tout x ∈]0, 1[
B Arrête d’équation y = 1 : soit g :]0, 1[→ R telle que g (x) ≡ f (x, 1) = (x − 1)2 alors g 0 (x) 6= 0 pour tout x ∈]0, 1[
B Arrête d’équation x = 0 : soit g :]0, 1[→ R telle que g (y) ≡ f (0, y) = (−y)2 alors g 0 (y) 6= 0 pour tout y ∈]0, 1[
B Arrête d’équation x = 1 : soit g :]0, 1[→ R telle que g (y) ≡ f (1, y) = (1 − y)2 alors g 0 (y) 6= 0 pour tout y ∈]0, 1[
donc il n’existe pas de points stationnaires sur les arrêtes du carré.
B On calcule la valeur de f en les points de non dérivabilité :
B f (0, 0) = 0
B f (1, 0) = 1
B f (0, 1) = 1
B f (1, 1) = 0
En résumé, les candidats extrema globaux sont les points (0, 0), (0, 1), (1, 0), (1, 1) et (k, k) avec k ∈]0; 1[. On conclut que
B les minima globaux sont atteints aux points (k, k) avec k ∈ [0; 1] et valent 0,
B le maximum global est atteint au point (1, 0) et (0, 1) et vaut 1.

Exemple
On veut trouver les extrema globaux de la fonction f définie par f (x, y) = −6y 2 − 4x y − 4y + x 2 + 6x − 6 sur la région triangulaire
compacte D de sommets A = (0, 3), B = (−4, 1) et C = (−3, −2).
y = 5x/3 + 3

A y = x/2 + 3

y = −3x − 11

B On calcule la valeur de f en les points stationnaires de f dans l’ouvert D̊ = D \ ∂D :

f (x, y) = −6y 2 − 4x y − 4y + x 2 + 6x − 6, ∂x f (x, y) = −4y + 2x + 6 ∂ y f (x, y) = −12y − 4x − 4

et ∇ f = (0, 0) si et seulement si (x, y) = (−11/5, 2/5) et l’on a f (−11/5, 2/5) = −67/5.


B On calcule la valeur de f en les points stationnaires de f sur le bord ∂D :
B Arrête AB : y = mx + q avec m = (y A − y B )/(x A − x B ) = 1/2 et q = y A − mx A = 3
B Arrête AC : y = mx + q avec m = (y A − yC )/(x A − xC ) = 5/3 et q = y A − mx A = 3
B Arrête BC : y = mx + q avec m = (y B − yC )/(x B − xC ) = −3 et q = y B − mx B = −11
donc
µ ¶
1 5
g AB (x) ≡ f x, y = x + 3 = − x 2 − 26x − 72, x ∈ [−4; 0] g 0AB (x) = −5x − 26 < 0;
2 2

120 © G. Faccanoni
Dernière mise à jour : Jeudi 5 juin 2014 5 Extrema

µ ¶ µ ¶
5x 67 218 134 218 109 109 2591
g AC (x) ≡ f x, y = + 3 = − x2 − x − 72, x ∈ [−3; 0] g 0AC (x) = − x− ⇐⇒ x = − et g AC − =− ;
3 3 3 3 3 67 67 201
g BC (x) ≡ f (x, y = −3x − 11) = −41x 2 − 334x − 688, x ∈ [−4; −3] 0
g BC (x) = −82x − 334 < 0.

En résumé, les candidats extrema globaux sont les points

x y f (x, y)

−11/5 2/5 −67/5 ∈ D̊


0 3 −72 ∈ ∂D
−4 1 −8 ∈ ∂D
−3 −2 −55 ∈ ∂D
−109/67 58/201 −2591/201 ∈ ∂D

On conclut que
B le minimum global est atteint au point (−11/5, 2/5) ∈ D̊ et vaut −67/5,
B le maximum global est atteint au point (−26/5, 4/5) ∈ ∂D et vaut −22/5.

-12

-32
3

-52 -1

-3

1
-72 -1
-3
-5

Exemple
On construit une canalisation de forme trapézoïdale avec une plaque de largeur L > 0 fixée. Quelles sont les valeurs optimales de la
longueur ` du côté incliné et de l’angle ϑ qu’il fait avec la verticale pour que le flux passant dans la gouttière soit maximal ?

ϑ ϑ
`

L − 2`
La surface à maximiser est décrite par la fonction

f (`, ϑ) = ` cos(ϑ) ` sin(ϑ) + L − 2` ,


¡ ¢

avec D = (`, ϑ) ¯ 0 ≤ ` ≤ L2 , 0 ≤ ϑ ≤ π2 .
n ¯ o
¯

B On cherche tout d’abord les points critiques de f qui se trouvent dans D̊ l’intérieur du domaine de D et on calcul la valeur de f en
ces points :

f (`, ϑ) = ` cos(ϑ) ` sin(ϑ) + L − 2` ,


¡ ¢

∂` f (`, ϑ) = cos(ϑ) 2` sin(ϑ) + L − 4` ,


¡ ¢

∂ϑ f (`, ϑ) = ` 2` cos2 (ϑ) + 2` sin(ϑ) − L sin(ϑ) − ` = ` − 2` sin2 (ϑ) + (2` − L) sin(ϑ) + ` ,


¡ ¢ ¡ ¢

p 2
et ∇ f (`, ϑ) = (0, 0) dans D̊ si et seulement si (`, ϑ) = (L/3, π/6) et l’on a f (L/3, π/6) = 3L /12 ≈ 0.14434L 2 .
B On cherche le maximum de f sur le bord ∂D de D :

2 p
g 1 (`) ≡ f (`, 0) = ` L − 2` , g 10 (`) = L − 4` = 0 ⇐⇒ ` = L4 et f ( L4 , 0) = L8 = 0.125L 2 < 3L 2 /12;
¡ ¢

g 2 (`) ≡ f (`, π/2) = 0;


g 3 (ϑ) ≡ f (0, ϑ) = 0;
2 p
g 4 (ϑ) ≡ f (L, ϑ) = L 2 cos(ϑ) sin(ϑ)/4, g 40 (ϑ) = L 2 (cos2 (ϑ) − sin2 (ϑ))/4 = 0 ⇐⇒ ϑ = π4 et f ( L2 , π4 ) = L8 = 0.125L 2 < 3L 2 /12.

Comme f (`, ϑ) ≤ f (L/3, π/6) pour tout (`, ϑ) ∈ D on conclut que le flux passant dans la gouttière sera maximal pour ` = L/3 et ϑ = π/6.

© G. Faccanoni 121
5 Extrema Dernière mise à jour : Jeudi 5 juin 2014

(a) Point de maximum (b) Point de selle

π π

L/

3
L/
6 6

3 L/3

Théorème Condition suffisante d’extrémum local dans un ouvert (cas de 2 variables)


Soit f une fonction de classe C 2 sur un ouvert D ⊂ R2 et (x 0 , y 0 ) un point stationnaire ; posons
¢2
dét(H f (x 0 , y 0 )) =≡ ∂xx f (x 0 , y 0 ) · ∂ y y f (x 0 , y 0 ) − ∂x y f (x 0 , y 0 ) ,
¡

le déterminant de la matrice hessienne de f évalué en (x 0 , y 0 ).


B Si dét(H f (x 0 , y 0 )) > 0, alors f présente un extrémum relatif en (x 0 , y 0 ) ; il s’agit
B d’un maximum si ∂xx f (x 0 , y 0 ) < 0
B d’un minimum si ∂xx f (x 0 , y 0 ) > 0 ;
B si dét(H f (x 0 , y 0 )) < 0, alors f présente un point-selle (ou point-col) en (x 0 , y 0 ) ; ce n’est pas un extrémum ;
B si dét(H f (x 0 , y 0 )) = 0, on ne peut pas conclure à partir des dérivées secondes.
En résumé, si ∂x f (x 0 , y 0 ) = 0 et ∂ y f (x 0 , y 0 ) = 0, la nature du point critique (x 0 , y 0 ) est déterminée par le tableaux suivant :

dét(H f (x 0 , y 0 )) ∂xx f (x 0 , y 0 ) Nature de (x 0 , y 0 )


+ + minimum local
+ − maximum local
− point-selle
0 on ne peut pas conclure

Exemple
On veut étudier la fonction f (x, y) = x 2 + y 2 − 2x − 4y sur R2 . Elle a pour dérivées partielles ∂x f (x, y) = 2x − 2 et ∂ y f (x, y) = 2y − 4 qui
ne s’annulent qu’en (1, 2), seul point où il peut donc y avoir un extremum local. On étudie directement le signe de la différence

d (h, k) = f (1 + h, 2 + k) − f (1, 2) = h 2 + k 2 > 0.

Comme cette différence est positive pour h et k voisins de 0 il s’agit d’un minimum. En effet, ∂xx f (1, 2) = 2 > 0, ∂ y y f (1, 2) = 2,
∂x y f (1, 2) = 0 donc dét(H f (1, 2)) = 4 > 0 et il s’agit bien d’un minimum.

Exemple
Pour déterminer les extrema libres de la fonction f (x, y) = x 2 + y 3 −2x y − y dans R2 , on constate d’abord que f est un polynôme, donc
différentiable dans l’ouvert R2 . Les seuls candidats extrema locaux sont les points critiques. Toutefois, nous ne disposons d’aucune
garantie à priori sur le fait que les éventuels extrema locaux soient globaux.
Recherche des points critiques On a

µ ¶ µ ¶ µ ¶
2x − 2y 0 1 1
∇f = 0 ⇐⇒ 2 = ⇐⇒ (x, y) = − , − ou (x, y) = (1, 1).
3y − 2x − 1 0 3 3

³ ´
Les deux candidats sont donc − 31 , − 13 et (1, 1).

122 © G. Faccanoni
Dernière mise à jour : Jeudi 5 juin 2014 5 Extrema

Classification La matrice hessienne de f en un point (x, y) ∈ R2 est

∂xx f (x, y) ∂x y f (x, y)


µ ¶ µ ¶
2 −2
H f (x, y) = = .
∂ y x f (x, y) ∂ y y f (x, y) −2 6y
³ ³ ´´
Comme dét H f − 13 , − 13 < 0 et D(1, 1) > 0, alors (− 13 , − 31 ) est un point-selle et f admet en (1, 1) un minimum local de valeur
f (1, 1) = −1. Ce minimum n’est cependant pas global puisque, par exemple, f (0, −2) = −6 < f (1, 1) = −1.

Exemple
Une firme produit deux types de biens A et B. Son coût total de fabrication C et la demande respective des deux biens q A et q B sont
donnés par

C = q 2A + 2q B2 + 10, (5.1)
q A = 40 − 2p A − p B , (5.2)
q B = 35 − p A − p B . (5.3)

Quels sont les niveaux de production qui maximisent le profit ? Quels sont les prix de A et B qui suscitent une demande correspondant
à ces niveaux optimaux ?
Pour répondre à ces questions, on écrit la fonction profit Π(q A , q B ) = q A p A + q B p B −C (q A , q B ). De (5.2) et (5.3) on peut déduire

p A = 5 − q A + pB , (5.4)
p B = 30 + q A − 2q B , (5.5)

et donc que
Π(q A , q B ) = 5q A − 2q 2A + 2q A q B − 4q B2 + 30q B − 10.
Pour maximiser le profit en fonction du niveau de production, on recherche d’abord les points stationnaires de Π :
µ ¶ µ ¶
5 − 4q A + 2q B 0
∇Π(q A , q B ) = , ∇Π(q A , q B ) = ⇐⇒ (q A , q B ) = (50/14, 65/14).
2q A − 8q B + 30 0

On étudie la nature de ce point critique en évaluant la matrice Hessienne :


µ ¶
−4 2
HΠ (q A , q B ) = ;
2 −8

pour le point stationnaire (50/14, 65/14) on a det(HΠ )(50/14, 65/14) > 0 et ∂q A q A Π(50/14, 65/14) < 0, donc le point (50/14, 65/14) est
un maximum local de Π. Comme HΠ (q A , q B ) ne dépend pas des variables q A et q B , on a det(HΠ )(q A , q B ) > 0 et ∂q A q A Π(q A , q B ) < 0
en tout point du domaine, autrement dit Π est une fonction concave et le point (50/14, 65/14) est un maximum global de Π. Comme
q A = 50/14 et q B = 65/14 maximisent le profit en fonction des niveaux de production, les prix associés à ce niveau de production sont
p A = 85/14 et p B = 340/14.

Exemple Lois de S NELL -D ESCARTES


Un rayon lumineux se propage à la vitesse v 1 dans le milieu M 1 et à la vitesse v 2 dans le milieu M 2 . Le principe de F ERMAT précise que
la lumière suit le trajet le plus économique en temps.
En dimension 2. En notant x l’abscisse du point où la trajectoire coupe l’interface, le temps nécessaire pour aller de (x 1 , z 1 ) à (x 2 , z 2 )
en passant par le point d’abscisse x vaut

q q
(x − x 1 )2 + z 12 (x 2 − x)2 + z 22
t (x) = + .
v1 v2

Cette quantité est (localement) optimale quand sa dérivée par rapport à x s’annule, c’est à dire lorsque

x − x1 x2 − x
q − q = 0,
v1 (x − x 1 )2 + z 12 v2 (x 2 − x)2 + z 22

ce qui conduit à la loi de S NELL -D ESCARTES

sin(ϑ1 ) sin(ϑ2 )
= .
v1 v2

© G. Faccanoni 123
5 Extrema Dernière mise à jour : Jeudi 5 juin 2014

M1

z1
ϑ1
x2
x1 x

ϑ2

z2
M2

En dimension 3. Dans le cas tridimensionnel, la situation est similaire. Il s’agit cette fois de déterminer les coordonnées (x, y) dans le
plan de l’interface où le rayon coupera celui-ci. Le temps de trajet vaut cette fois
q q
(x − x 1 )2 + (y − y 1 )2 + z 12 (x 2 − x)2 + (y 2 − y)2 + z 22
t (x, y) = + .
v1 v2

Il s’agit cette fois d’optimiser par rapport à x et y simultanément, ce qui revient à annuler simultanément les dérivées de t par
rapport à x et y, c’est à dire son gradient. Ceci conduit aux équations
x−x 1 x 2 −x

 
q q
2 (x 2 −x)2 +(y 2 −y)2 +z 22 
 v 1 (x−x1 )2 +(y−y 1 )2 +z1 v 2 d2 x − x1
µ ¶ µ ¶ µ ¶
v2 0 x2 − x
∇t = 0 ⇐⇒ y−y 1 y 2 −y = ⇐⇒ =
− 0 y2 − y v 1 d1 y − y 1

q q
v1 (x−x 1 )2 +(y−y 1 )2 +z 12 v2 (x 2 −x)2 +(y 2 −y)2 +z 22

autrement dit, le point du plan de coordonnées (x, y) se trouve dans le segment compris entre (x 1 , y 1 ) et (x 2 , y 2 ). On se ramène
donc à un problème bidimensionnel et le raisonnement ci-dessus s’applique directement.
On obtient de la même façon la loi de S NELL -D ESCARTES à la réflexion.

5.1.1 Méthode des moindres carrés

Lorsqu’un chercheur met au point une expérience (parce qu’il a quelques raisons © ªn croire que les deux grandeurs x et y
de
sont liées par une fonction f ), il récolte des données sous la forme de points (x i , y i ) i =0 mais en générale ces données sont
affectées par des erreurs de mesure. Lorsqu’il en fait une représentation graphique il cherche f pour qu’elle s’ajuste le mieux
possible aux points observés. Soit d i = y i − f (x i ) l’écart vertical du point (x i , y i ) par rapport à la fonction f . La méthode des
moindres carrés est celle qui choisit f de sorte que la somme des carrés de ces déviations soit minimale.
Fitting par une relation affine
Supposons que les deux grandeurs x et y sont liées approximativement par une relation affine, c’est-à-dire de la forme
y = mx + q pour certaines valeurs de m et q (autrement dit, lorsqu’on affiche ces points dans un plan cartésien, les
points ne sont pas exactement alignés mais cela semble être dû à des erreurs de mesure). On souhaite alors trouver les
constantes m et q pour que la droite d’équation y = mx + q s’ajuste le mieux possible aux points observés. Pour cela,
introduisons d i ≡ y i − (mx i + q) l’écart vertical du point (x i , y i ) par rapport à la droite.

La méthode des moindres carrés est celle qui choisit m et q de sorte que la somme des carrés de ces déviations soit mini-
male. Pour cela, on doit minimiser la fonction E : R2 → R+ définie par
n n
E (m, q) = d i2 = (y i − mx i − q)2 .
X X
i =0 i =0

∂E ∂E
Pour minimiser E on cherche d’abord les points stationnaires, i.e. les points (m, q) qui vérifient ∂m = ∂q = 0. Puisque
à ! à !
∂E X n ∂E X n
(m, q) = −2 (y i − (mx i + q))x i , (m, q) = −2 (y i − (mx i + q)) ,
∂m i =0 ∂q i =0

124 © G. Faccanoni
Dernière mise à jour : Jeudi 5 juin 2014 5 Extrema

alors

∂E
( (P
n
∂m (m, q) = 0 (y i − mx i − q)x i = 0
∂E ⇐⇒ Pin=0
∂q (m, q) = 0 i =0 (y i − mx i − q) = 0
 ¡Pn ¢ ¡ Pn ¢ ¡Pn ¢
i =0 x i i =0 y i − (n + 1) i =0 x i y i
m= ,


(¡P
n
 ¢2
x 2 m + ni=0 x i q = ni=0 y i x i
¢ ¡P ¢ P  ¡Pn ¡Pn 2
¢
 x i − (n + 1) x
⇐⇒ ¡Pin=1 i ¢ Pn ⇐⇒ ¡Pn ¢i =0
¡ Pn ¢ ¡Pn i =0 ¢ ¡i Pn 2
¢
i =1 x i m + (n + 1)q = i =0 y i
 i =0 x i i =0 x i y i − i =0 y i i =0 x i
q = .

 ¡Pn ¢2 ¡Pn 2
¢
i =0 x i − (n + 1) i =0 x i

On a trouvé un seul point stationnaire. On établi sa nature en étudiant la matrice Hessienne :


µPn Pn
x2

HE (m, q) = 2 Pin=1 i i =0 x i
i =0 x i (n + 1)
³ ¢2 ´
et det(HE (m, q)) = 4 (n + 1) ni=1 x i2 − ni=0 x i > 0 avec ∂mm E (m, q) = ni=1 x i2 > 0 donc il s’agit bien d’un minimum. La
P ¡P P

droite d’équation y = mx + q ainsi calculée s’appelle droite de régression de y par rapport à x.

Exemple y
5
Si on a le points suivantes 4
3
x 1 2 3 4 5 2
y 0.9 1.5 3.5 4.2 4.9 1
012345x
on trouve m = 1.07 et q = −0.21.

Fitting par un exponentiel


Soit a > 0 et considérons la fonction f (x) = ae kx : elle est non-linéaire mais si on prend son logarithme on obtient
ln( f (x)) = kx + ln(a) qui est linéaire en kª et a la forme mx + q avec m = k et q = ln(a). On peut alors faire une régres-
© n
sion linéaire sur l’ensemble (x i , ln(y i )) i =0 et obtenir ainsi k et ln(a). Cependant ceci n’est pas équivalent à faire un
ªn
fitting sur l’ensemble initial (x i , y i ) i =0 . En effet, si on note d i = y i − ae kxi et D i = ln(y i ) − (kx i + ln(a)), lorsqu’on fait
©
© ªn
une régression linéaire sur l’ensemble (x i , ln(y i )) i =0 on minimise D i et non d i .

Fitting par un polynôme


© ªn
On considère un ensemble de points expérimentaux (x i , y i ) i =0 et on suppose que les deux grandeurs x et y sont liées,
au moins approximativement, par une relation polynomiale, c’est-à-dire de la forme y = m j
P
j =0 a j x pour certaines valeurs
Pm
de a j . On souhaite alors trouver les m +1 constantes a j pour que le polynôme d’équation y = j =0 a j x j s’ajuste le mieux
³P ´
j
possible aux points observés. Soit d i = y i − m j =0 a j x i l’écart vertical du point (x i , y i ) par rapport au polynôme. La
méthode des moindres carrés est celle qui choisit les a j de sorte que la somme des carrés de ces déviations soit minimale.
Pour cela, on doit minimiser la fonction E : Rm+1 → R+ définie par
à !2
n n m
j
E (a 0 , a 1 , . . . , a m ) = d i2
X X X
= yi − a j xi .
i =1 i =0 j =0

∂E
Pour minimiser E on cherche d’abord ses points stationnaires, i.e. les points qui vérifient ∂a j = 0 pour j = 0, . . . , m.
Puisque
à à !!
∂E X n
0
Xm
j
(a 0 , a 1 , . . . , a m ) = −2 xi y i − a j xi ,
∂a 0 i =0 j =0
à à !!
∂E X n
1
Xm
j
(a 0 , a 1 , . . . , a m ) = −2 xi y i − a j xi ,
∂a 1 i =0 j =0
..
.

© G. Faccanoni 125
5 Extrema Dernière mise à jour : Jeudi 5 juin 2014

à à !!
∂E X n
m
Xm
j
(a 0 , a 1 , . . . , a m ) = −2 xi y i − a j xi ,
∂a m i =0 j =0

on obtient alors le système linéaire de (m + 1) équations en les (m + 1) inconnues a 0 , a 1 , . . . , a m suivant

∂E
  Pn Pn Pn
m
 ∂a0 (a 0 , a 1 , . . . , a m ) = 0
 a 0 (n + 1) + a 1 i =0 x i + · · · + a m i =0 x i = i =0 y i

 ∂E (a 0 , a 1 , . . . , a m ) = 0
 
a 0 n x i + a 1 n x 2 + · · · + a m n x m+1 = n y i x i

 
 P P P P
∂a 1 i =0 i =0 i i =0 i i =0
.. ⇐⇒ .

 . 
 ..

 
 P
 ∂E
a 0 ni=0 x im + a 1 ni=0 x im+1 + · · · + a m ni=0 x i2m = ni=0 y i x im
  P P P
∂a m (a 0 , a 1 , . . . , a m ) = 0

Pn Pn     Pn
x im
 
(n + 1) i =0 xi ... i =0 a0 i =0 yi
 Pn x Pn 2 Pn m+1     Pn
i =0 x i ... i =0 x i   a 1   i =0 y i x i 

 i =0 i
⇐⇒   .. .. ..   ..  = 
    .. 
. . .  .   .

 
Pn m Pn m+1 Pn 2m Pn m
i =0 ix i =0 i x . . . i =0 i x a m i =0 y i x i

Quand m = n, le polynôme des moindres carrés coïncide avec le polynôme d’interpolation de L AGRANGE, i.e. l’unique
polynôme L tel que L(x i ) = y i pour tout i = 0, . . . , n.

Exemple
À partir des données
x 1.0 2.5 3.5 4.0 1.1 1.8 2.2 3.7
y 6.008 15.722 27.130 33.772 5.257 9.549 11.098 28.828
on veut calculer la droite et la parabole de régression et comparer les erreurs des chaque régression.
1. La droite de régression a équation y = mx + q avec
P6
y i (x i − x̄)
m = P7i =0 ≈ 8.420042377, q = ȳ − m x̄ ≈ −3.37833528,
x (x − x̄)
i =0 i i

où x̄ = 71 6i =0 x i = 2.0125 et ȳ = 17 6i =0 y i = 13.567. L’erreur est


P P

6
(y i − (mx i + q))2 = 39.59960820.
X

i =0

2. La parabole de régression a équation y = a 0 + a 1 x + a 2 x 2 avec a 0 , a 1 , a 2 solution du système linéaire


P6 P6
x i2 a 0
     P6 
8 xi yi a0

8 16.1 44.79
  
108.536

 6 i =0 i =0 i =0
P P 6 2 P 6 3    6 P
 i =0 x i x x  a 1 =  i =0 y i x i  i.e. 16.1 44.79 141.311 a 1 = 322.7425 
     
P6 2 P6i =0 i3 P6i =0 i4 P6 2
x x x a 2 y x 44.79 141.311 481.5123 a2 1067.97905
i =0 i i =0 i i =0 i i =0 i i

et on obtient 
a 0 = 0.744611871628180655,

a 1 = 2.14480468957977077,


a 2 = 1.51926210146774388.
L’erreur est
6
(y i − (a 0 + a 1 x i + a 2 x i2 ))2 = 5.715921703.
X

i =0
y

40

30

20

10

1 2 3 4 x

126 © G. Faccanoni
Dernière mise à jour : Jeudi 5 juin 2014 5 Extrema

Exemple
Le tableau ci-dessous donne la conductivité thermique k du sodium pour différentes valeurs de la température. On veut calculer la
parabole de meilleur approximation.
T (◦C) 79 190 357 524 690
k 1.00 0.932 0.839 0.759 0.693
La parabole de régression a équation y = a 0 + a 1 x + a 2 x 2 avec a 0 , a 1 , a 2 solution du système linéaire
P4 P4
x i2 a 0 4 y
    P 
6 xi a0

8 16.1 44.79
  
108.536

i =0 i =0 i =0 i
 4
P P 4 2 P 4 3    4P
 i =0 x i x x  a 1 =  i =0 y i x i  i.e.  16.1 44.79 141.311  a 1  =  322.7425 

P4 2 P4i =0 i3 P4i =0 i4 P4 2
x x x a2 y i x 44.79 141.311 481.5123 a 2 1067.97905
i =0 i i =0 i i =0 i i =0 i

et on obtient 
a 0 = 0.744611871628180655,

a 1 = 2.14480468957977077,


a 2 = 1.51926210146774388.
L’erreur est
6
(y i − (a 0 + a 1 x i + a 2 x i2 ))2 = 5.715921703.
X

i =0

Exemple
La viscosité cinématique µ de l’eau varie en fonction de la température comme dans le tableau suivant :

T (◦C) 0 21.1 37.8 54.4 71.1 87.8 100


µ (10−3 m2 s−1 ) 1.79 1.13 0.696 0.519 0.338 0.321 0.296

On veut évaluer les valeurs µ(10◦ ), µ(30◦ ), µ(60◦ ), µ(90◦ ) par le polynôme de meilleur approximation de degré 3.
ª6
On a la famille de points (Ti , µi ) i =0 . Le polynôme de meilleur approximation de degré 3 s’écrit
©

r (T ) = a 0 + a 1 T + a 2 T 2 + a 3 T 3

où a 0 , a 1 , a 2 , a 3 sont solution du système linéaire


P6 P6
T2
P6
T 3 a 
 P6
µ
  
6 T
 P6 T P6i =0 i2 Pi6=0 i3 Pi6=0 i4  0  6
i =0 i
 P i =0 µi Ti 
P
 i =0 i Ti T T  a 1  
P6 i =0 P6i =0 i4 P6i =0 i5  
a 2  = 
 i =0 Ti2
P6
T3 Ti T 6 µ T 2
P6i =0 i4 i =0 P6i =0 i6 P6i =0 i i3
  
P6
T3
P6
i =0 i i =0 i
T i =0 i
T5 T
i =0 i
a3 µ T
i =0 i i

et on obtient 

 a 0 = 0.914534618675843625,


a = 0.914534618675843625,
1

 a
 2 = −0.000620138768106035594,

a 3 = −0.000620138768106035594.

On a alors
r (10◦ ) = 1.004300740 r (30◦ ) = 0.9114735501 r (60◦ ) = 0.9114735501 r (90◦ ) = 0.249145396

5.2 Extrema liés


La scission entre extrema libres et liés (ou «sous contraintes») est née de l’impossibilité de traiter l’optimisation dans les do-
maines non ouverts selon la procédure exposée dans la section précédente. En effet, la condition nécessaire ne s’applique
pas aux bords du domaine. Or, si de tels points existent, leur étude au cas par cas est généralement peu aisée. Dès lors, la
théorie mathématique propose des méthodes d’optimisation liée. Celles-ci incorporent directement dans la résolution les
contraintes qui définissent des domaines non ouverts. La nomenclature peut s’avérer trompeuse. En effet, au plan opéra-
tionnel, ce n’est pas la présence intrinsèque de contraintes dans l’optimisation qui conduit à délaisser l’optimisation libre
au profit de l’optimisation liée. Ce sont plutôt les conséquences de ces restrictions au niveau de la nature topologique du
domaine de définition de la fonction qui guident l’utilisateur vers l’une ou l’autre des techniques. Ainsi, dans un domaine
fort limité, mais ouvert, la recherche des extrema libres s’applique. À l’inverse, une contrainte sous forme d’inégalité non
stricte doit toujours être prise en compte pour déterminer les extrema liés.

© G. Faccanoni 127
5 Extrema Dernière mise à jour : Jeudi 5 juin 2014

Parmi les types de contraintes auxquelles le modélisateur peut se trouver confronté, deux classes se distinguent. D’une part,
celles qui lient les variables du problème au travers d’une ou plusieurs équations. Ces contraintes dites d’égalités sont ap-
préhendées grâce au théorème de L AGRANGE, qui fournit une condition de premier ordre formulée à partir d’une fonction
ad hoc, dénommée lagrangien. Dans cette approche, de nouvelles variables, dites multiplicateurs, apparaissent et offrent
une possibilité supplémentaire dans l’analyse des résultats. D’autre part, l’optimisation sous des contraintes d’inégalités non
strictes, généralement traitée grâce au théorème de K UHN et T UCKER, ne sera pas étudie lors de ce module.

Exemple
En économie, l’optimisation sous contraintes permet de déterminer une situation optimale (par exemple comment maximiser le pro-
fit, minimiser les dépenses, ou encore maximiser le bien-être) sous une contrainte quelconque (budget limité, bien-être minimum
requis. . .). Chaque jour, en tant que consommateur, nous résolvons dans notre tête un problème d’optimisation sous contraintes en
répondant à la question «quelle quantité de chaque bien vais-je acheter pour maximiser mon bien-être, étant donné ma contrainte
budgétaire ?». Chaque jour, les producteurs aussi résolvent un problème d’optimisation sous contraintes en se demandant par exemple
«pour produire un bien supplémentaire, vais-je acheter une machine ou embaucher un nouveau salarié, étant donné le rendement de
chacun et ma contrainte actuelle de trésorerie ?». Nous allons essayer de comprendre l’idée qui se cache derrière ce problème d’opti-
misation.
Supposons donc l’exemple d’un consommateur qui arrive au fast-food et a le choix entre deux biens : il peut acheter c canettes de
coca et/ou h hamburgers. Sachant qu’un hamburger coûte p h = 4 euros et une canette de coca coûte p c = 2 euros et que notre
consommateur a un budget de 16 euros par jour pour manger (i.e. cp c + hp h ≤ 16), il a donc le choix entre diverses combinaisons :
acheter 8 canettes de coca et 0 hamburger, 6 canettes de coca et 1 hamburger, 4 canettes de coca et 2 hamburgers, 2 canettes de coca
et 3 hamburgers, 0 canette de coca et 4 hamburgers. Pour trouver quelle est la consommation optimale pour lui, notre consommateur
va essayer de mesurer le bien-être (ce que l’on appelle en économie la fonction «utilité» u : (R+ )2 → R qui est une fonction des deux
biens c et h) des quatre options possibles et choisir l’option (i.e. c 0 et h 0 ) qui maximise son bien-être (u(c 0 , h 0 )) tout en respectant sa
contrainte de budget.
La première chose à bien comprendre dans ce problème, c’est que le bien-être apporté par la consommation d’une unité supplé-
mentaire d’un bien est décroissant. Qu’est ce que cela signifie ? Simplement que lorsque votre consommation de coca-cola augmente,
passant de 0 canette à 1 canette, alors le bien-être apporté par cette première canette sera très grand. Par contre, le bien-être addi-
tionnel que vous procure une hausse de consommation de 7 à 8 canettes de coca-cola sera très faible. Donc pour une même hausse
de quantité (+1 canette) et le même coût (2 euros en moins dans votre portefeuille), votre bien-être n’augmente pas dans la même
manière. D’un point de vue économique, on dit alors que l’utilité marginale d’un bien est décroissante ; le bien-être procuré par chaque
dose supplémentaire d’un bien consommé va en diminuant et devient nul à partir d’un certain seuil appelé «point de satiété».
Cette relation non-linéaire du bien-être permet d’introduire ce que l’on appelle les courbes d’indifférence, qui représentent toutes les
combinaisons de deux biens apportant le même bien-être final (i.e. les courbes de niveau de u). Sur le graphique ci-dessous, nous
voyons le cas d’un consommateur qui se dit que, pour lui, le bien-être apporté par la consommation de 8 bananes et de 2 pommes est
le même que le bien-être procuré par 4 bananes et 4 pommes, mais aussi le même que s’il mange 2 bananes et 8 pommes. Les trois
points sont situés sur cette même courbe d’indifférence.
Banane
8

4 Courbe
2 d’indifférence

2 4 8 Pomme
Pour un même individu, il existe donc une infinité de courbes d’indifférence, chaque courbe représentant l’ensemble des combinai-
sons de deux biens procurant au consommateur un niveau de bien-être identique. Plus on s’éloigne de l’ordonnée à l’origine, plus le
bien-être total est grand ; par exemple sur le graphique ci-dessous le bien être d’un consommateur sur la courbe bleue est supérieur
au bien-être sur la courbe rouge. C’est logique, il consomme davantage de quantité des deux biens sur cette courbe bleue.
Quantité
du bien h
Ar
h2 Ab
u(c 2 , h 1 ) < u(c 2 , h 2 )
u(c 2 , h 1 ) < u(c 3 , h 1 )

Br Bb
h1
u(c, h) = 4

u(c, h) = 2
c1 c2 c3 Quantité
du bien c

128 © G. Faccanoni
Dernière mise à jour : Jeudi 5 juin 2014 5 Extrema

Mais le but d’un consommateur n’est pas simplement de maximiser son utilité (car dans ce cas, il faut simplement consommer le plus
possible jusqu’à atteindre satiété pour l’ensemble des biens), mais de maximiser son utilité sous contrainte budgétaire. L’idée va donc
être d’aller chercher la courbe d’indifférence la plus éloignée de l’ordonnée à l’origine, tout en respectant son budget. Pour cela, il faut
donc introduire une droite de budget, représentant la quantité de chaque bien que l’on peut acheter étant donné son budget total.
Dans notre exemple coca/hamburgers, cela correspond aux différentes combinaisons possible : c’est la droite en rouge ci-dessous.
Quantité
du bien h
4

4 8 Quantité
du bien c
On commence à voir comment trouver la consommation optimale : le but est de maximiser l’utilité sous contrainte budgétaire, c’est
à dire en fait trouver la courbe d’indifférence la plus éloignée possible de l’ordonnée à l’origine mais qui reste tout de même sur la
ligne rouge de contrainte budgétaire. Graphiquement, le point optimale est donc trouvé lorsque la droite de contrainte budgétaire est
tangente à la courbe d’indifférence.
Quantité
du bien h

k1 < k2 < k3

4 B
D
A C
2
u(c, h) = k 3
u(c, h) = k 2
u(c, h) = k 1
4 8 Quantité
du bien c
Sur le graphique ci-dessus, on retrouve donc pour les biens c et h notre contrainte budgétaire (droite rouge) et 3 courbes d’indifférence.
Étant donné la contrainte, la consommation optimale se trouve au point C . Pourquoi ce n’est pas A, B ou D par exemple ? Car en B ,
l’ensemble du budget est utilisé, mais on se trouve sur une courbe d’indifférence plus basse qu’en C , donc le bien-être en B est inférieur
au bien-être de C pour un même niveau de dépense. Idem en A, on ne maximise pas l’utilité et on ne respecte pas au passage notre
contrainte budgétaire. En D, le bien être serait supérieur au bien-être en C , mais on ne respecte pas la contrainte budgétaire, donc ce
point n’est pas atteignable (vous n’avez pas assez d’argent pour atteindre la consommation des deux biens en D).
Voila donc comment trouver graphiquement et intuitivement la solution de notre problème. Mais lorsque cela devient plus complexe,
quelques notions de mathématiques et l’utilisation de la méthode de L AGRANGE permettent de répondre précisément à ce type de
questions concernant la maximisation ou minimisation d’une fonction sous contrainte quelconque.

Problème
Déterminer les extrema d’une fonction de n variables, notée f (x), mathématiquement définie dans un domaine ouvert
D ⊂ Rn , mais dont les variables sont soumises aux m contraintes g 1 (x) = 0, g 2 (x) = 0, . . ., g m (x) = 0, où les fonctions g j sont
également définies dans D. Ces contraintes délimitent le sous-ensemble A de D dans lequel s’effectue l’optimisation
A = x ∈ D ¯ g 1 (x) = 0, . . . , g m (x) = 0 .
© ¯ ª

La définition d’extremum lié résulte donc de celle des extrema libres.


Définition Optimisation liée
La fonction f , appelée fonction objectif, admet en x0 ∈ A un maximum lié (resp. un minimum lié) sous les contraintes
g 1 (x) = 0, g 2 (x) = 0, . . ., g m (x) = 0, si en ce point elle admet un maximum libre (resp. un minimum libre) dans le do-
maine A .

Généralement, une contrainte de type g j (x) = 0 définit une courbe. L’ensemble A est donc constitué de l’intersection de
m courbes dans D ⊂ Rn . Pour m > 1, il peut donc contenir fort peu de points. C’est pourquoi, en pratique, il est rare de
rencontrer plus d’une contrainte d’égalité. Et, dans tous les cas, il est exclu d’avoir plus de contraintes que de variables, de
sorte que la condition m < n sera systématiquement imposée.

© G. Faccanoni 129
5 Extrema Dernière mise à jour : Jeudi 5 juin 2014

Exemple
Le consommateur maximise une fonction d’utilité, notée U (x, y), qui dépend des quantités consommées de deux biens, x et y, sous
une contrainte budgétaire p 1 x +p 2 y = R, où p 1 > 0 et p 2 > 0 sont les prix des biens. Cette contrainte exprime que le montant alloué aux
dépenses relatives aux deux biens considérés est fixé à R. Dans ce cas simple qui comporte deux variables et une contrainte linéaire, on
peut aisément ramener l’optimisation liée à la recherche d’un maximum libre. En effet, la contrainte de budget permet d’expliciter la
p
quantité d’un bien en fonction de l’autre y = pR − p 1 x et Ũ (x) = U (x, y(x)). En vertu de la condition nécessaire pour la fonction d’une
2 2
variable obtenue après substitution de y en fonction de x, on a

dU (x, y(x)) p1
Ũ 0 (x) = = ∂x U (x, y) + ∂ y U (x, y)y 0 (x) = ∂x U (x, y) − ∂ y U (x, y)
dx p2

donc
∂x U (x, y) ∂ y U (x, y)
Ũ 0 (x) = 0 =⇒ = .
p1 p2
Ce résultat indique qu’à l’optimum, les utilités marginales (terme économique qui signifie tout simplement dérivées partielles de la
fonction utilité par rapport aux quantités consommées) pondérées par les inverses des prix s’égalisent et désignons par λ cette quantité
commune.
Par ailleurs, la contrainte de budget peut être reformulée par g (x, y) = 0, où g (x, y) = p 1 x + p 2 y − R, de sorte que p 1 = ∂x g (x, y) et
p 2 = ∂ y g (x, y) et finalement

∂x U (x, y) ∂ y U (x, y)
(
∂x U (x, y) − λ∂x g (x, y) = 0
= = λ ⇐⇒ ⇐⇒ ∇U = λ∇g .
p1 p2 ∂ y U (x, y) − λ∂ y g (x, y) = 0

Autrement dit, si on introduit la fonction


L(x, y, λ) ≡ U (x, y) − λg (x, y),
à l’optimum on a ∇L = 0, ce qui correspond à la condition nécessaire d’existence d’un extrema libre pour la fonction L.

Le théorème de L AGRANGE généralise la démarche adoptée dans cette résolution. Il représente une condition nécessaire pour
l’optimisation sous des contraintes d’égalité.
Théorème des multiplicateurs de L AGRANGE
Soit les fonctions f et g 1 , . . ., g m (m < n) de classe C 1 dans un ouvert D ⊂ Rn . Si f admet en x0 un extremum lié sous les
contraintes g 1 (x) = 0, . . ., g m (x) = 0 et si la jacobienne en x0 (i.e. la matrice (∂x j g i (x0 ))m×n ) est de rang m, alors

m
∃λ = (λ1 , . . . , λm ) ∈ Rm tel que ∇ f (x0 ) = λi ∇g i (x0 ).
X
i =1

Le théorème de L AGRANGE peut être vu comme la condition nécessaire d’existence d’extrema libres appliquée à la fonc-
tion de n + m variables appelée fonction lagrangienne (ou le lagrangien) :

L : D × Rm → R
m
(x, λ) 7→ f (x0 ) − λi g i (x0 )
X
i =1

où λ = (λ1 , . . . , λm ).

Attention
La méthode de L AGRANGE permet juste de trouver les candidats extrema x0 de f sous les contraintes g i (x) = 0 mais
ne
© permet pas de conclure. Ilª faut alors étudier le comportement de f au voisinage de chaque point critique dans
x ∈ D ¯ g 1 (x) = 0, . . . , g m (x) = 0 . Pour cela, on étudie le signe de la fonction distance définie par
¯

d (h) ≡ f (x0 + h) − f (x0 ),

pour h ' 0 et g i (x0 + h) = 0 pour i = 1, . . . , m.

D’une manière qui peut sembler paradoxale, la méthode de L AGRANGE permet de transformer un problème d’optimisa-
tion liée d’une fonction de n variables sous m contraintes en un problème d’optimisation libre d’une fonction de n + m
variables, alors qu’intuitivement, on aurait plutôt attendu une baisse de la dimension du problème due aux restrictions im-
pliquées par la présence de contraintes. Mais le paradoxe n’est qu’apparent. En effet, l’introduction des m multiplicateurs de
L AGRANGE, (λ1 , . . . , λm ), enrichit plus qu’elle n’alourdit l’optimisation, grâce à l’interprétation dont jouissent ces variables.
Précisément, la valeur prise par un multiplicateur traduit l’influence marginale du niveau de la contrainte correspondante

130 © G. Faccanoni
Dernière mise à jour : Jeudi 5 juin 2014 5 Extrema

sur la valeur de la fonction objective à l’optimum. On dit aussi que le multiplicateur de L AGRANGE mesure l’intensité de la
contrainte. Chaque multiplicateur exprime la sensibilité de la fonction d’objectif à la variation du niveau d’une contrainte.
Par exemple, une contrainte qui n’influence pas l’optimisation (contrainte inopérante ou superflue) est affectée d’un mul-
tiplicateur nul. À l’opposé, un multiplicateur élevé correspond à une contrainte qui pénalise de façon importante l’opti-
mum.
Explication (prérequis : algèbre linéaire)
On considère le problème d’optimisation suivant :
optimiser localement la fonction f : D ⊂ Rn → R sous les contraintes


 g 1 (x 1 , x 2 , . . . , x n ) = 0,


g 2 (x 1 , x 2 , . . . , x n ) = 0,

.. (x 1 , x 2 , . . . , x n ) ∈ D,



 .

g m (x 1 , x 2 , . . . , x n ) = 0,

c’est-à-dire qu’il y a m contraintes unidimensionnelles dans un espace à n dimensions (m < n). Ce que nous entendons
par optimiser localement, c’est chercher les points x0 ∈ Rn qui minimisent localement ou maximisent localement f tout
en satisfaisant
¯ la contrainte g(x0 ) = 0 ayant noté g ≡ (g 1 , g 2 , . . . , g m ) : D → Rm et A l’ensemble des contraintes défini par

A = x ∈ R g(x) = 0 . Soit x0 un point de A , c’est-à-dire tel que g(x0 ) = 0, et soit
© ª

γ : R → D,
t 7→ x(t ) ≡ (x 1 (t ), x 2 (t ), . . . , x n (t ))

une courbe paramétrée sur A qui passe par x0 à l’instant t = t 0 , c’est-à-dire telle que g(x(t )) = 0 pour tout t et x(t 0 ) = x0 .
On introduit la fonction

ϕ : R → R,
t 7→ f (x(t ))

Si x0 optimise f sur A , alors il optimise a fortiori f sur la courbe γ, puisque γ est contenue dans A . On en déduit que t 0
optimise ϕ, c’est-à-dire que ϕ atteint un minimum ou un maximum en t 0 . Or ϕ est une fonction habituelle, on doit donc
avoir annulation de sa dérivée en t 0 : ϕ(t 0 ) = 0. Compte tenu de la formule de dérivation d’une fonction composée, on
∂f dx ∂f dx
a ϕ0 (t ) = ni=1 ∂x (x(t )) dti (t ) ; par conséquent ni=1 ∂x (x0 ) dti (t 0 ) = 0. Ce raisonnement étant valable pour toute courbe
P P
i i
Pn ∂ f
γ contenue dans A passant par x0 en t = t 0 , on a i =1 ∂x (x0 )v i = 0 pour tout v = (v 1 , v 2 , . . . , v n ) appartenant au plan
i
tangent à f en x0 , c’est-à-dire que tout x optimisant f sous la contrainte g(x0 ) = 0 est tel que ∇ f (x0 ) appartient à l’espace
normal à l’ensemble A engendré par les vecteurs ∇g 1 (x0 ), ∇g 2 (x0 ), . . ., ∇g m (x0 ). Il existe donc des coefficients réels λ1 ,
λ2 , . . ., λm tels que ∇ f (x0 ) = ni=1 λi ∇g i (x0 ).
P

Exemple Inégalité des moyennes arithmétiques et géométriques


p
Cherchons le maximum de la fonction f (x 1 , x 2 , . . . , x n ) = n x 1 x 2 . . . x n sur Rn
+ sous la contrainte x 1 + x 2 + · · · + x n = n.
En un point où le maximum est atteint, on peut appliquer le théorème précédent avec g (x 1 , x 2 , . . . , x n ) = (x 1 + x 2 + · · · + x n ) − n. On
obtient    
1
 x1  1 
   
   
p
n x x ...x 
 1   
1 
n  x2 

1 2
∇ f = λ∇g ⇐⇒   = λ .
 
n  ..   .. 
 .  .
   
   
   
1
x 1
n

En particulier, on obtient que pour que (x 1 , x 2 , . . . , x n ) soit un extremum il faut que tous les x i soient égaux et dans ce cas la contrainte
implique qu’ils sont tous égaux à 1. Ainsi, on a f (x 1 , x 2 , . . . , x n ) ≤ f (1, 1, . . . , 1) = 1 = g (x, y)/n. On retrouve ainsi l’inégalité des moyennes
arithmétiques et géométriques
p
n
x1 + x2 + · · · + xn
x1 x2 . . . xn ≤ .
n

© G. Faccanoni 131
5 Extrema Dernière mise à jour : Jeudi 5 juin 2014

F IGURE 5.1: On cherche les extrema de la fonction f (x, y) sous la contrainte g (x, y) = k. Cela signifie qu’on cherche les ex-
trema de f (x, y) lorsque le point (x, y) appartient à la courbe de niveau g (x, y) = k. En figure on voit cette courbe
ainsi que plusieurs courbes de niveau de f . Celles-ci ont équation f (x, y) = c pour c = 7, 8, 9, 10, 11. Optimi-
ser f (x, y) sous la condition g (x, y) = k signifie chercher la plus grande (ou plus petite) valeur de c telle que
la courbe de niveau f (x, y) = c croise la courbe g (x, y) = k. Pour que cela ait lieu il faut que les deux courbes
aient la même droite tangente. Cela signifie que les gradients sont parallèles, c’est-à-dire qu’il existe λ ∈ R tel que
∇ f (x 0 , y 0 ) = λ∇g (x 0 , y 0 ).

Optimiser f (x, y) sous la contrainte g (x, y) = 0.


Réécrivons le théorème des multiplicateurs de L AGRANGE pour une fonction de deux variables et une seule contrainte
d’égalité (voir la figure 5.1 pour une interprétation géométrique) :
Soit les fonctions f et g de classe C 1 dans un ouvert D ⊂ R2 . Si f admet en (x 0 , y 0 ) un extremum lié sous la
contrainte g (x, y) = 0 et si ∇g (x 0 , y 0 ) 6= (0, 0), alors

∃λ ∈ R tel que ∇ f (x 0 , y 0 ) = λ∇g (x 0 , y 0 ).

On a alors deux méthodes pratiques pour déterminer les extrema liés de f sous la contrainte g :
Méthode 1 : Lagrangien. Formons le lagrangien

L: D ×R → R
(x, y, λ) 7→ L(x, y, λ) = f (x, y) − λg (x, y)

où λ (multiplicateur de L AGRANGE) est une inconnue. Pour que cette fonction ait un extremum il faut que le gra-
dient de L soit nul, autrement dit on cherche les triplets (x, y, λ) tels que

∂x f (x, y) = λ∂x g (x, y),




∂ y f (x, y) = λ∂ y g (x, y),

0 = g (x, y).

Notons (x 0 , y 0 , λ0 ) une solution de ce système. Si ∇g (x 0 , y 0 ) 6= 0, alors (x 0 , y 0 ) est un point critique de la fonction


f sous la contrainte g . Ces points critiques satisfont la contrainte, mais il s’agit à présent de classer ces candidats.
Comme dans le cas des extrema libres, on peut formuler des conditions du deuxième ordre relatives aux points
critiques, mais qui ne s’appliquent qu’à certains cas. Soit

∆(x 0 , y 0 , λ0 ) ≡ ∂xx L(x 0 , y 0 , λ0 )∂ y y L(x 0 , y 0 , λ0 ) − (∂x y L(x 0 , y 0 , λ0 ))2 ,

c’est-à-dire le détérminant de la sous matrice obtenue à partir de la hessienne de L en éliminant la dernière ligne
et la dernière colonne.
B Si ∆(x 0 , y 0 , λ0 ) > 0, ∂xx L(x 0 , y 0 , λ0 ) < 0 et ∂ y y L(x 0 , y 0 , λ0 ) < 0 on a un maximum local en (x 0 , y 0 ) ;
B si ∆(x 0 , y 0 , λ0 ) > 0, ∂xx L(x 0 , y 0 , λ0 ) > 0 et ∂ y y L(x 0 , y 0 , λ0 ) > 0 on a un minimum local en (x 0 , y 0 ) ;
B si ∆(x 0 , y 0 , λ0 ) ≤ 0 on ne peut pas conclure directement. On étudie alors le signe de la différence

d (h, k) ≡ f (x 0 + h, y 0 + k) − f (x 0 , y 0 ),

h et k étant liés par la relation φ(h, k) ≡ g (x 0 + h, y 0 + k) = 0. Si cette différence est de signe constant pour (h, k)
voisin de (0, 0), il s’agit d’un extrémum local (un maximum si d < 0, un minimum si d > 0). Sinon, f ne présent
pas d’extremum local en (x 0 , y 0 ).
Méthode 2 : Réduction. Cette méthode est basée sur la possibilité d’exprimer la contrainte sous forme paramétrique.
Par exemple

132 © G. Faccanoni
Dernière mise à jour : Jeudi 5 juin 2014 5 Extrema

B s’il existe une fonction h(x) telle que (x, y) ∈ R2 ¯ g (x, y) = 0 = x ∈ R ¯ h(x) = y , alors optimiser la fonction
© ¯ ª © ¯ ª

de deux variables f (x, y) sous la contrainte g (x, y) = 0 équivaut à optimiser la fonction d’une seule variable
f (x, y = h(x)) ;
B s’il existe une fonction h(y) telle que (x, y) ∈ R2 ¯ g (x, y) = 0 = y ∈ R ¯ h(y) = x , alors optimiser la fonction
© ¯ ª © ¯ ª

de deux variables f (x, y) sous la contrainte g (x, y) = 0 équivaut à optimiser la fonction d’une seule variable
f (x = h(x), y) ;
B s’il existe deux fonctions x = x(t ) et y = y(t ) telles que (x, y) ∈ R2 ¯ g (x, y) = 0 = t ∈ R ¯ g (x(t ), y(t )) = 0 , alors
© ¯ ª © ¯ ª

optimiser la fonction de deux variables f (x, y) sous la contrainte g (x, y) = 0 équivaut à optimiser la fonction
d’une seule variable f (x = x(t ), y = y(t )).

Exemple
Soient à déterminer les minima et maxima de la fonction objectif f (x, y) = 5x 2 + 6y 2 − x y sous la contrainte x + 2y = 24. Pour cela,
construisons la fonction de L AGRANGE

L : R2 × R → R
(x, y, λ) 7→ L(x, y, λ) = f (x, y) − λg (x, y) = 5x 2 + 6y 2 − x − λ(x + 2y − 24)

et annulons son gradient

10x − y + λ
 
0
   
0
∇L = 0 ⇐⇒  12y − x + 2λ  = 0 . f |g (x,y) = 0
(6, 9, 612)
0 −x − 2y − 24 0

On obtient x = 6, y = 9. Comme ∂xx L(6, 9) = 10 > 0, ∂ y y L(6, 9) =


12 > 0, ∂x y L(6, 9) = −1 et ∂xx L(6, 9)∂ y y L(6, 9) − ∂x y L 2 (6, 9) > 0, il
s’agit d’un minimum. On notera que, dans ce cas, la valeur de λ (6, 9, 0)
ne présente pas d’intérêt et n’est donc pas cherchée.
Dans cet exemple on peut expliciter une variable dans la
contrainte, par exemple y = 12− x2 . Alors on peut minimiser direc- g (x, y) = 0
tement la fonction f (x, 12− x2 ) = 7x 2 −84x +864 : comme f 0 (x, 12−
x 6
2 ) = 14x − 84, le minimum se trouve en x = 6, y = 12 − 2 = 9 et la
fonction vaut f (6, 9) = 612.

Exemple
On veut étudier l’existence d’extremum de la fonction f : R2 → R définie par f (x, y) = x y sous la contrainte g (x, y) = x + y − 6 = 0.
Première méthode Introduisons le lagrangien

L : R2 × R → R
(x, y, λ) 7→ L(x, y, λ) = f (x, y) − λg (x, y) = x y − λ(6 − x − y)

En appliquant la condition nécessaire des multiplicateurs de L AGRANGE, on doit calculer les triplets (x, y, λ) solutions du sys-
tème
 y + λ = 0,
  
0 
∇L(x, y, λ) = 0 =⇒ x + λ = 0, =⇒ (x, y, λ) = (3, 3, −3).
0


x + y = 6,
L’unique candidat extrémum de f sous la contrainte g est le point (3, 3) et f (3, 3) = 9. Comme ∆(3, 3, −3) = −1 < 0, on ne peut
pas établir directement la nature du point critique. Étudions alors le signe de la différence d (h, k) = f (3 + h, 3 + k) − f (3, 3) =
hk +3(h+k), h et k étant liés par la relation 0 = g (3+h, 3+k) = h+k, soit encore k = −h. On a par conséquente d (h, −h) = −h 2 < 0
pour tout h ∈ R : le point (3, 3) est alors un maximum local de f sous la contrainte g .
Seconde méthode Dans ce cas on peut écrire directement la restriction de f sur la contrainte et on obtient

h(x) = f (x, 6 − x) = x(6 − x).

Pour maximiser cette restriction on calcule sa dérivée première :

h 0 (x) = 6 − 2x = 0 ssi x = 3, h 0 (x) > 0 ssi x < 3, h 0 (x) < 0 ssi x > 3.

Le point (3, 3) est bien un maximum.

© G. Faccanoni 133
5 Extrema Dernière mise à jour : Jeudi 5 juin 2014

Exemple Trouver un champ rectangulaire d’aire maximale délimité par une clôture de longueur ` donnée.
Si x et y désignent les longueurs des côtés du champ, la longueur de la clôture est 2(x + y) et l’aire x y. Le problème est de trouver le
maximum atteint par l’aire, non pas parmi tous les points de (R∗ 2
+ ) , mais seulement parmi ceux vérifiant la contrainte 2(x + y) = `, où
` > 0 est fixé.
Méthode 1 : Lagrangien. Formons le lagrangien
2
L : ( R∗
+) × R → R
(x, y, λ) 7→ L(x, y, λ) = x y − λ(2(x + y) − `)

où λ (multiplicateur de L AGRANGE) est une inconnue. Pour que cette fonction ait un extremum il faut que le gradient de L soit
nul, autrement dit on cherche les triplets (x, y, λ) tels que

x = `/4

y − 2λ
   
0 
∇L = 0 ⇐⇒  x − 2λ  = 0 ⇐⇒ y = `/4
` − 2(x + y) 0

λ = `/8

Donc (`/4, `/4) est un point critique de la fonction f sous la contrainte g . Observons que ce point a été obtenu par une condition
nécessaire et comme ∆(`/4, `/4, `/8) < 0, rien dans le théorème ne permet de savoir directement si c’est un maximum, un
minimum ou ni l’un ni l’autre. Étudions alors le signe de la différence d (h, k) = f (`/4+h, `/4+k)− f (`/4, `/4) = hk +`(h +k)/4,
h et k étant liés par la relation 0 = g (`/4 + h, `/4 + k) = 2(h + k), soit encore k = −h. On a par conséquente d (h, −h) = −h 2 < 0
pour tout h ∈ R : le point (`/4, `/4) est alors un maximum local de f sous la contrainte g .
Méthode 2 : Réduction. Bien sûr, on peut utiliser la contrainte pour calculer une des variables en fonction des deux autres. Par
exemple pour y :
`
2(x + y) = ` =⇒ y= − x.
2
En reportant cette valeur de y dans l’expression de l’aire, on obtient

` `
µ ¶
A ` (x) = x −x et A 0` (x) = − 2x.
2 2

Le maximum de A ` (x) est atteint pour x = `/4, ce qui entraîne aussi y = x : à périmètre fixé, le rectangle d’aire maximale est le
carré.

Optimiser f (x, y, z) sous la contrainte g (x, y, z) = 0.


Réécrivons le théorème des multiplicateurs de L AGRANGE pour une fonction de trois variables et une seule contrainte
d’égalité :
Soit les fonctions f et g de classe C 1 dans un ouvert D ∈ R3 . Si f admet en (x 0 , y 0 , z 0 ) un extremum lié sous
la contrainte g (x, y, z) = 0 et si ∇g (x 0 , y 0 , z 0 ) 6= (0, 0, 0), alors

∃λ ∈ R tel que ∇ f (x 0 , y 0 , z 0 ) = λ∇g (x 0 , y 0 , z 0 ).

On a alors deux méthodes pratiques pour déterminer les extrema liés de f sous la contrainte g :
Méthode 1 : Lagrangien. Formons le lagrangien
L: D ×R → R
(x, y, z, λ) 7→ L(x, y, z, λ) = f (x, y, z) − λg (x, y, z)

où λ (multiplicateur de L AGRANGE) est une inconnue. Pour que cette fonction ait un extremum il faut que le gra-
dient de L soit nul, autrement dit on cherche les triplets (x, y, z, λ) tels que


 ∂x f (x, y, z) = λ∂x g (x, y, z),

∂ f (x, y, z) = λ∂ g (x, y, z),

y y

 z

 f (x, y, z) = λ∂ z g (x, y, z),

0 = g (x, y, z).

Notons (x 0 , y 0 , z 0 , λ0 ) une solution de ce système. Si ∇g (x 0 , y 0 , z 0 ) 6= 0, alors (x 0 , y 0 , z 0 ) est un point critique de la


fonction f sous la contrainte g . Ces points critiques satisfont la contrainte, mais il reste encore à déterminer s’il
s’agit effectivement d’extrema. On étudie alors le signe de la différence

d (h 1 , h 2 , h 3 ) ≡ f (x 0 + h 1 , y 0 + h 2 , z 0 + h 3 ) − f (x 0 , y 0 , z 0 ),

h 1 , h 2 et h 3 étant liés par la relation φ(h 1 , h 2 , h 3 ) ≡ g (x 0 + h 1 , y 0 + h 2 , z 0 + h 3 ) = 0. Si cette différence est de signe


constant pour (h 1 , h 2 , h 3 ) voisin de (0, 0, 0), il s’agit d’un extrémum local (un maximum si d < 0, un minimum si
d > 0). Sinon, f ne présent pas d’extremum local en (x 0 , y 0 , z 0 ).

134 © G. Faccanoni
Dernière mise à jour : Jeudi 5 juin 2014 5 Extrema

Méthode 2 : Réduction. Cette méthode est basée sur la possibilité d’exprimer la contrainte sous forme paramétrique.
Par exemple
B s’il existe une fonction h(x, y) telle que (x, y, z) ∈ R3 ¯ g (x, y, z) = 0 = (x, y) ∈ R2 ¯ h(x, y) = z , alors chercher
© ¯ ª © ¯ ª

les extrema liés de la fonction de trois variables f (x, y, z) sous la contrainte g (x, y, z) = 0 équivaut à chercher les
extrema libres de la fonction de deux variables f (x, y, z¯ = h(x, y)) ; ª ©
B s’il existe une fonction h(x, z) telle que (x, y, z) ∈ R3 ¯ g (x, y, z) = 0 = (x, z) ∈ R2 ¯ h(x, z) = y , alors chercher
© ¯ ª

les extrema liés de la fonction de trois variables f (x, y, z) sous la contrainte g (x, y, z) = 0 équivaut à chercher les
extrema libres de la fonction de deux variables f (x, y¯ = h(x, z), z) ;ª ©
B s’il existe une fonction h(y, z) telle que (x, y, z) ∈ R3 ¯ g (x, y, z) = 0 = (y, z) ∈ R2 ¯ h(y, z) = x , alors chercher les
© ¯ ª

extrema liés de la fonction de trois variables f (x, y, z) sous la contrainte g (x, y, z) = 0 équivaut à chercher les
extrema libres de la fonction de deux variables f (x = h(y, z), y, z).

Exemple Parmi les parallélépipèdes de surface S fixée, lesquels ont un volume maximal ?
Si x, y et z désignent les longueurs des côtés du parallélépipède, la surface est 2(x y + y z + xz) et le volume x y z. Le problème est
de trouver le maximum atteint par le volume x y z, non pas parmi tous les points de (R∗ 3
+ ) , mais seulement parmi ceux vérifiant la
contrainte 2(x y + y z + xz) = S, où S est fixé.
Lagrangien. Formons le lagrangien
3
L : ( R∗
+) × R → R
(x, y, z, λ) 7→ L(x, y, z, λ) = x y z − λ(2(x y + y z + xz) − S)

où λ (multiplicateur de L AGRANGE) est une inconnue. Si (x 0 , y 0 , z 0 ) ∈ (R∗ + ) est un extremum de f (x, y, z) = x y z sous la contrainte
g (x, y, z) = 0 avec g (x, y, z) = 2(x y + y z + xz) − S alors il existe λ0 ∈ R tel que ∇L(x 0 , y 0 , z 0 , λ0 ) = 0. Cherchons donc les points
stationnaires de L :
  p

y z − 2λ(y + z)
  
0  z(y − x) = 2λ(y − x)  x = S/6
 y = pS/6

 


 y(z − x) = 2λ(z − x) 
 xz − 2λ(x + z)  0
∇L = 0 ⇐⇒    =   ⇐⇒ ⇐⇒ p
x y − 2λ(y + x)  0  x(z − y) = 2λ(z − y)  z = S/6
p

 

S − 2(x y + y z + xz) 0 

2(x y + y z + xz) = S


λ = S/96
p p p
Donc ( S/6, S/6, S/6) est un point critique de la fonction f sous la contrainte g . Observons que ce point a été obtenu par
une condition nécessaire. Rien dans le théorème ne permet de savoir si c’est un maximum, un minimum ou ni l’un ni l’autre.
Pour cela, on essaye d’étudier directement le signe de la distance
p p p p p p
d (h 1 , h 2 , h 3 ) ≡ f ( S/6 + h 1 , S/6 + h 2 , S/6 + h 3 ) − f ( S/6, S/6, S/6)
p p p
avecp h 1 , h 2 pet h 3 proches de zéro et liés par l’équation g ( S/6 + h 1 , S/6 + h 2 , S/6 + h 3 ) = 0, soit encore h 3 =
S
2p−( S/6+h 1 )( S/6+h 2 ) p
p − S/6. On doit alors étudier le signe de la fonction
( S/6+h 1 )+( S/6+h 2 )

S
p p
p p 2 − ( S/6 + h 1 )( S/6 + h 2 )
p 1 N (h 1 , h 2 )
˜
d (h 1 , h 2 ) ≡ d (h 1 , h 2 , h 3 (h 1 , h 2 )) = ( S/6 + h 1 )( S/6 + h 2 ) p p − S 3 /63 = − p ,
( S/6 + h 1 ) + ( S/6 + h 2 ) 2 6S + 3(h 1 + h 2 )
p
avec N (h 1 , h 2 ) ≡ S(h 12 +h 22 )+6h 12 h 22 +Sh 1 h 2 +2 6Sh 1 h 2 (h 1 +h 2 ), pour (h 1 , h 2 ) ∈ B((0, 0), δ) et δ ' 0. Le signe de d˜ est l’opposé du
signe de N mais celui ci est difficile à étudier. Cependant, il est simple de vérifier que ∇N (0, 0) = (0, 0) et que la matrice Hessienne
H N (h 1 , h 2 ) est semi-définie positive, par conséquent p p convexe et donc N (h 1 , h 2 ) ≥ 0 pour (h 1 , h 2 ) ∈ B((0, 0), δ) et δ ' 0. On
p N est
trouve alors d˜(h 1 , h 2 ) ≤ 0 et on conclut que ( S/6, S/6, S/6) est un maximum de la fonction f sous la contrainte g .
Réduction. Dans ce cas, on peut utiliser la contrainte pour calculer une des variables en fonction des deux autres. Par exemple,
S
−xy
2(x y + y z + xz) = S =⇒ z= 2 .
x+y
En reportant cette valeur de z dans l’expression du volume, on obtient
S
−xy
VS (x, y) = f (x, y, z(x, y)) = x y 2 .
x+y
Maximiser f sous la contrainte g est équivalent à maximiser VS . Pour calculer le maximum de cette fonction il faut d’abord
trouver les points critiques de VS et ensuite en étudier la nature :
µ 2
y (S − 2x 2 − 4x y) p p

1 2
B ∇VS (x, y) = 2(x+y) donc, dans (R∗+ ) , le gradient de VS s’annule seulement en ( S/6, S/6) ;
2
x 2 (S − 2y 2 − 4x y)
µ p p
y 2 (S + 2y 2 ) x y(2x 2 + 6x y + 2y 2 − S) p p
µ ¶ ¶
−1 − S/6 − S/24
B HVS (x, y) = (x+y) donc H VS ( S/6, S/6) = p p et
3
x y(2x 2 + 6x y + 2y 2 − S) x 2 (S + 2x 2 ) − S/24 − S/6
p p
det(HVS ( S/6, S/6)) = S/8 > 0.
p p
Le maximum de VS (x, y) est donc atteint pour x = y = S/6, ce qui entraîne aussi z = S/6 : à surface fixée, le parallélépipède
de volume maximal est le cube.

© G. Faccanoni 135
5 Extrema Dernière mise à jour : Jeudi 5 juin 2014

Optimiser f (x, y, z) sous les contraintes g 1 (x, y, z) = 0 et g 2 (x, y, z) = 0.


Réécrivons le théorème des multiplicateurs de L AGRANGE pour une fonction de deux variables et deux contraintes d’éga-
lité :
Soit les fonctions f , g 1 et g 2 de classe C 1 dans un ouvert D ∈ R3 . Si f admet en (x 0 , y 0 , z 0 ) un extremum lié
sous les contraintes g 1 (x, y, z) = 0 et g 2 (x, y, z) = 0 et si ∇g 1 (x 0 , y 0 , z 0 ) 6= (0, 0, 0) et ∇g 2 (x 0 , y 0 , z 0 ) 6= (0, 0, 0), alors

∃λ, µ ∈ R tels que ∇ f (x 0 , y 0 , z 0 ) = λ∇g 1 (x 0 , y 0 , z 0 ) + µ∇g 2 (x 0 , y 0 , z 0 ).

On a alors deux méthodes pratiques pour déterminer les extrema liés de f sous les contraintes g 1 et g 2 :
Méthode 1 : Lagrangien. Formons le lagrangien

L : D × R2 → R
(x, y, z, λ, µ) 7→ L(x, y, z, λ, µ) = f (x, y, z) − λg 1 (x, y, z) − µg 2 (x, y, z)

où λ et µ (multiplicateurs de L AGRANGE) sont des inconnues. Pour que cette fonction ait un extremum il faut que
le gradient de L soit nul, autrement dit on cherche les 5-uplets (x, y, z, λ, µ) tels que

∂x f (x, y, z) = λ∂x g 1 (x, y, z) − µ∂x g 2 (x, y, z),


∂ y f (x, y, z) = λ∂ y g 1 (x, y, z) − µ∂ y g 2 (x, y, z),
∂z f (x, y, z) = λ∂z g 1 (x, y, z) − µ∂z g 2 (x, y, z),
0 = g 1 (x, y, z),
0 = g 2 (x, y, z).

Notons (x 0 , y 0 , z 0 , λ0 , µ0 ) une solution de ce système. Si rg([∇g 1 (x 0 , y 0 , z 0 )|∇g 2 (x 0 , y 0 , z 0 )]) = 2, alors (x 0 , y 0 , z 0 ) est


un point critique de la fonction f sous les contraintes g 1 et g 2 . Ces points critiques satisfont les contraintes, mais il
reste encore à déterminer s’il s’agit effectivement d’extrema. On étudie alors le signe de la différence

d (h 1 , h 2 , h 3 ) ≡ f (x 0 + h 1 , y 0 + h 2 , z 0 + h 3 ) − f (x 0 , y 0 , z 0 ),

h 1 , h 2 et h 3 étant liés par les relations φ1 (h 1 , h 2 , h 3 ) ≡ g 1 (x 0 +h 1 , y 0 +h 2 , z 0 +h 3 ) = 0 et φ2 (h 1 , h 2 , h 3 ) ≡ g 2 (x 0 +h 1 , y 0 +


h 2 , z 0 + h 3 ) = 0. Si cette différence est de signe constant pour (h 1 , h 2 , h 3 ) voisin de (0, 0, 0), il s’agit d’un extrémum
local (un maximum si d < 0, un minimum si d > 0). Sinon, f ne présent pas d’extremum local en (x 0 , y 0 , z 0 ).
Méthode 2 : Réduction. Cette méthode est basée sur la possibilité d’exprimer la contrainte sous forme paramétrique.
s’il existe deuxª fonctions h 1 (x) et h 2 (x) telles que (x, y, z) ∈ R3 ¯ g 1 (x, y, z) = 0 et g 2 (x, y, z) = 0 =
© ¯ ª
Par
© exemple,
x ∈ R ¯ y = h 1 (x) et z = h 2 (x) , alors optimiser la fonction de trois variables f (x, y, z) sous les contraintes
¯

g 1 (x, y, z) = 0 et g 2 (x, y, z) = 0 équivaut à optimiser la fonction d’une seule variable f (x, y = h 1 (x), z = h 2 (x)) ; etc.

Exemple
Dans R3 , on veut optimiser la fonction f (x, y, z) = x y z sous les contraintes g 1 (x, y, z) = x 2 + y 2 +z 2 −1 et g 2 (x, y, z) = x + y +z −1. Notons
que les deux contraintes décrivent un cercle de R 3 : la contrainte g 1 (x, y, z) = 0 est l’équation de la sphère de centre (0, 0, 0) et de rayon
1 ; la contrainte g 2 (x, y, z) = 0 est l’équation d’un plan. On cherche donc les extrema de f sur l’intersection de la sphère unité et d’un
plan, à savoir sur un cercle dans l’espace.
Avant d’oublier, commençons par chercher les points de la surface de contrainte qui ne sont pas réguliers, c’est à dire les (x, y, z) tels
que g 1 (x, y, z) = 0, g 2 (x, y, z) = 0 et les vecteurs ∇g 1 et ∇g 2 ne forment pas une famille libre de R3 . On a ∇g 1 (x, y, z) = 2(x, y, z) et
∇g 2 (x, y, z) = (1, 1, 1). Ils sont liés s’ils sont colinéaires, c’est-à-dire lorsque x = y = z. Ce qui, avec les contraintes, donne le système non
linéaire
 2 2 2
x + y + z = 1,

x + y + z = 1,


x = y = z.

Or, ce système n’admet pas de solution, donc tous les points de la surface de contrainte (le cercle) sont réguliers et on peut utiliser la
méthode de L AGRANGE : si un point (x, y, z) est un extremum de f sous les deux contraintes g 1 et g 2 , alors il existe deux multiplicateurs
λ0 , µ0 tels que ∇L(x 0 , y 0 , z 0 , λ0 , µ0 ) = 0 où

L : R3 × R2 → R
(x, y, z, λ, µ) 7→ L(x, y, z, λ, µ) = x y z − λ(x 2 + y 2 + z 2 − 1) − µ(x + y + z − 1)

136 © G. Faccanoni
Dernière mise à jour : Jeudi 5 juin 2014 5 Extrema

On cherche donc à résoudre le système


y z − λ(2x) − µ
   
0
 xz − λ(2y) − µ  0
   
∇L = 0 ⇐⇒  x y − λ(2z) − µ  = 0 .
   
2 2 2
1 − (x + y + z ) 0
   

1 − (x + y + z) 0
On obtient un système de 5 équations et 5 inconnues : x, y, z, λ, µ. L’étude de ce système montre qu’il y a 6 solutions, données dans le
tableau ci-dessous :
x0 y0 z0 λ0 µ0
1 0 0 0 0
0 1 0 0 0
0 0 1 0 0
−1/3 2
/3 2
/3 −1/3 2/9
2
/3 −1/3 2
/3 −1/3 2/9
2
/3 2
/3 −1/3 −1/3 2/9
Observons que ces points ont été obtenus par une condition nécessaire. Rien dans le théorème ne permet de savoir si ce sont des
maxima, des minima ou ni l’un ni l’autre.

© G. Faccanoni 137
5 Extrema Dernière mise à jour : Jeudi 5 juin 2014

............... Exercices ..............


Optimisation libre dans un ouvert

Exercice 5.1
Soit f : R2 → R une fonction de classe C 2 (R2 ) et (a, b) un point de R2 . On suppose que

f (a, b) = 0, ∂x f (a, b) = 0, ∂ y f (a, b) = 0, ∂xx f (a, b) = 1, ∂ y y f (a, b) = 2, ∂x y f (a, b) = 3.

Le point (a, b) est-il un point critique ? Si oui, de quelle nature ?

C ORRECTION . Il est un point critique et plus particulièrement il s’agit d’un point-selle car dét(H f (a, b)) < 0.

Exercice 5.2
On suppose que (1, 1) est un point critique d’une fonction f dont les dérivées secondes sont continues. Dans chaque cas,
que peut-on dire au sujet de f ?

1. ∂xx f (1, 1) = 4, ∂x y f (1, 1) = 1, ∂ y y f (1, 1) = 2 ; 2. ∂xx f (1, 1) = 4, ∂x y f (1, 1) = 3, ∂ y y f (1, 1) = 2.

C ORRECTION .
1. D’abord on calcule dét(H f (1, 1)) = ∂xx f (1, 1)∂ y y f (1, 1) − (∂x y f (1, 1))2 = 7. Comme dét(H f (1, 1)) > 0 et ∂xx f (1, 1) > 0, f a
un minimum local en (1, 1).
2. D’abord on calcule dét(H f (1, 1)) = ∂xx f (1, 1)∂ y y f (1, 1) − (∂x y f (1, 1))2 = −1. Comme dét(H f (1, 1)) < 0, f a un point-selle
en (1, 1).

Exercice 5.3
À partir de la carte des courbes de niveau de la figure ci-
contre, localiser les points critiques de f : R2 → R et pré-
ciser pour chacun de ces points s’il s’agit d’un point-selle
ou d’un maximum ou d’un minimum local.
Vérifier ensuite le raisonnement sachant que

f (x, y) = 4 + x 3 + y 3 − 3x y.

C ORRECTION . Dans la figure, le point (1, 1) est entouré par des courbes de niveau qui sont de forme ovale et qui indiquent
que si nous nous éloignons du point dans n’importe quelle direction les valeurs de f augmentent. Ainsi on pourrait s’attendre
à un minimum local en ou à proximité de (1, 1).
Les courbes de niveau proches du point (0, 0) ressemblent à des hyperboles, et si nous nous éloignons de l’origine, les valeurs
de f augmentent dans certaines directions et diminuent dans d’autres, donc nous nous attendons à trouver un point selle.
Vérifions cette analyse :
Points critiques : ∂x f (x, y) = 3x 2 − 3y, ∂ y f (x, y) = 3y 2 − 3x. On a un point critique si les deux dérivées partielles s’annulent
en même temps ; on trouve deux points critiques : (1, 1) et (0, 0).
Études des points critiques : les dérivées secondes sont ∂xx f (x, y) = 6x, ∂x y f (x, y) = −3, ∂ y y f (x, y) = 6y, ainsi dét(H f (x, y)) =
∂xx f (x, y)∂ y y f (x, y)−(∂x y f (x, y))2 = 36x y −9. Comme dét(H f (1, 1)) > 0 et ∂xx f (1, 1) > 0, f a un minimum local en (1, 1).
Comme dét(H f (0, 0)) < 0, f a un point-selle en (0, 0).

138 © G. Faccanoni
Dernière mise à jour : Jeudi 5 juin 2014 5 Extrema

Exercice 5.4
À partir de la carte des courbes de niveau de la figure ci-
contre, localiser les points critiques de f : R2 → R et pré-
ciser pour chacun de ces points s’il s’agit d’un point-selle
ou d’un maximum ou d’un minimum local.
Vérifier ensuite le raisonnement sachant que

f (x, y) = 3x − x 3 − 2y 2 + y 4 .

C ORRECTION . Dans la figure, les points (−1, −1) et (−1, 1) sont entourés par des courbes de niveau qui sont de forme ovale
et qui indiquent que si nous nous éloignons du point dans n’importe quelle direction les valeurs de f augmentent. Ainsi on
pourrait s’attendre à des minima locaux en ou à proximité de (−1, ±1).
De la même manière, le point (1, 0) est entouré par des courbes de niveau qui sont de forme ovale et qui indiquent que si
nous nous éloignons du point dans n’importe quelle direction les valeurs de f diminuent. Ainsi on pourrait s’attendre à un
maximum local en ou à proximité de (1, 0).
Les courbes de niveau proche des points (−1, 0), (1, 1) et (1, −1) ressemblent à des hyperboles, et si nous nous éloignons de
ces points, les valeurs de f augmentent dans certaines directions et diminuent dans d’autres, donc nous nous attendons à
trouver des points de selle.
Vérifions cette analyse : (
3 − 3x 2 = 0
∇ f = 0 ⇐⇒
−4y + 4y 3 = 0
donc les points critiques sont (1, 0), (1, 1), (1, −1), (−1, 0), (−1, 1), (−1, −1). Les dérivées secondes sont ∂xx f (x, y) = −6x,
∂x y f (x, y) = 0, ∂ y y f (x, y) = 12y 2 − 4, ainsi dét(H f (x, y)) = ∂xx f (x, y)∂ y y f (x, y) − (∂x y f (x, y))2 = −72x y 2 + 24x.

Point critique (x 0 , y 0 ) dét(H f (x 0 , y 0 )) ∂xx f (x 0 , y 0 ) Conclusion


(1, 0) 24 > 0 −6 < 0 f a un maximum local en (1, 0)
(1, 1) −48 < 0 f a un point-selle en (1, 1)
(1, −1) −48 < 0 −6 < 0 f a un point-selle en (1, −1)
(−1, 0) −24 < 0 f a un point-selle en (−1, 0)
(−1, 1) 48 > 0 6>0 f a un minimum local en (−1, 1)
(−1, −1) 48 > 0 6>0 f a un minimum local en (−1, −1)

Exercice 5.5
Une montagne a la forme de la surface z(x, y) = 2x y − 2x 2 − y 2 − 8x + 6y + 4 (l’unité de mesure est de 100 mètres). Si le
niveau de la mer correspond à z = 0, quelle est la hauteur de la montagne ?

C ORRECTION . Il s’agit d’évaluer z(x, y) dans le point de maximum. Cherchons d’abord les points critiques :
µ ¶
~ 2y − 4x − 8
∇z(x, y) =
2x − 2y + 6

et ~
∇z(x, y) =~0 ssi (x, y) = (−1, 2). On établie la nature du point critique en étudiant le déterminant de la matrice hessienne :

∂xx f (x, y) = −4 < 0, ∂ y y f (x, y) = −2, ∂x y f (x, y) = 2,

et ∂xx f (−1, 2)∂ y y f (−1, 2) − (∂x y f (−1, 2))2 = 4 > 0 donc (−1, 2) est un maximum. Comme z(−1, 2) = 14, la montagne est haute
1400 mètre.

© G. Faccanoni 139
5 Extrema Dernière mise à jour : Jeudi 5 juin 2014

Exercice 5.6
Déterminer et établir la nature des points critiques des fonction f : R2 → R définies par
2 −y 2 )
1. f (x, y) = x 2 + x y + y 2 + y 15. f (x, y) = (x 2 − y 2 )e (−x
2. f (x, y) = x y − 2x − 2y − x 2 − y 2 16. f (x, y) = (y 2 − x 2 )e (−x
2 −y 2 )

3. f (x, y) = (x − y)(1 − x y) 17. f (x, y) = x 4 + y 4 − 2(x − y)2


3 2 2 2
4. f (x, y) = y + 3x y − 6x − 6y + 2 18. f (x, y) = x 4 + y 4 − 4(x − y)2
3 3
5. f (x, y) = x + y − 3x y + 3 x2
19. f (x, y, z) = 2 + xyz − z + y
6. f (x, y) = x y(1 − x − y)
20. f (x, y) = (x − 1)2 + 2y 2
3 3
7. f (x, y) = x − 12x y + 8y
21. f (x, y) = x 2 + x y + y 2 − 2x − y
1 1
8. f (x, y) = x y + + x y 22. f (x, y) = x 3 y 2 (6 − x − y)
x
9. f (x, y) = e cos(y)
23. f (x, y) = e x−y (x 2 − 2y 2 )
10. f (x, y) = y cos(x)
24. f (x, y) = 8
x + xy + y
11. f (x, y) = y 2 + x y ln(x)
25. f (x, y) = x 2 − cos(y)
x2 y y3
12. f (x, y) = 2 + x2 + 3 − 4y 2 +y 2 )
26. f (x, y) = (x 2 + y 2 )e −(x
x2 y y3
13. f (x, y) = 2 − x2 + 3 − 4y 27. f (x, y) = x 3 + y 2 − 6(x 2 − y 2 )
x y2 x3
14. f (x, y) = 2 + 3 − 4x + y 2 28. f (x, y) = (x 2 + y 2 − y 3 )e −y

C ORRECTION .
1. f (x, y) = x 2 + x y + y 2 + y
B f est de classe C 2 dans son domaine de définition, l’ouvert R2 .
B Recherche de points critiques :
(
2x + y = 0
µ ¶ µ ¶
0 1 2
∇ f (x, y) = ⇐⇒ ⇐⇒ (x, y) = ,− .
0 x + 2y + 1 = 0 3 3
¡1 2
¢
On a un unique point critique : 3,−3 .
B Nature du point critique : µ ¶
2 1
H f (x, y) = , dét(H f (x, y)) = 3.
1 2

dét H f 13 , − 23 > 0 et ∂xx f 31 , − 23 > 0 donc 13 , − 23 est un minimum.


¡ ¡ ¢¢ ¡ ¢ ¡ ¢

2. f (x, y) = x y − 2x − 2y − x 2 − y 2
B f est de classe C 2 dans son domaine de définition, l’ouvert R2 .
B Recherche de points critiques :
(
y − 2 − 2x = 0
µ ¶
0
∇ f (x, y) = ⇐⇒ ⇐⇒ (x, y) = (−2, −2).
0 x − 2 − 2y = 0

On a un unique point critique : (−2, −2).


B Nature du point critique : µ ¶
−2 1
H f (x, y) = , dét(H f (x, y)) = 3.
1 −2
dét(H f (−2, −2)) > 0 et ∂xx f (−2, −2) < 0 donc (−2, −2) est un maximum.
3. f (x, y) = (x − y)(1 − x y)
B f est de classe C 2 dans son domaine de définition, l’ouvert R2 .
B Recherche de points critiques :
(
1 − 2x y + y 2 = 0
µ ¶
0
∇ f (x, y) = ⇐⇒ ⇐⇒ (x, y) ∈ { (−1, −1), (1, 1) } .
0 −1 − x 2 + 2x y = 0

On a deux points critiques : (−1, −1) et (1, 1).

140 © G. Faccanoni
Dernière mise à jour : Jeudi 5 juin 2014 5 Extrema

B Nature des points critiques :


µ ¶
−2y −2x + 2y
H f (x, y) = , dét(H f (x, y)) = −4x y − 4(y − x)2 .
−2x + 2y 2x

dét(H f (−1, −1)) < 0 donc (−1, −1) est un point-selle ;


dét(H f (1, 1)) < 0 donc (1, 1) est un point-selle.
4. f (x, y) = y 3 + 3x 2 y − 6x 2 − 6y 2 + 2
B f est de classe C 2 dans son domaine de définition, l’ouvert R2 .
B Recherche de points critiques :
(
6x y − 12x = 0
µ ¶
0
∇ f (x, y) = ⇐⇒ ⇐⇒ (x, y) ∈ { (0, 0), (0, 4), (2, 2), (−2, 2) } .
0 3y 2 + 3x 2 − 12y = 0

On a quatre points critiques : (0, 0), (0, 4), (2, 2) et (−2, 2).
B Nature des points critiques :
µ ¶
6y − 12 6x ³ ´
H f (x, y) = , dét(H f (x, y)) = (6y − 12)2 − 36x 2 = 36 (y − 2)2 − x 2 .
6x 6y − 12

dét(H f (0, 0)) > 0 et ∂xx f (0, 0) < 0 donc (0, 0) est un maximum ;
dét(H f (0, 4)) > 0 et ∂xx f (0, 4) > 0 donc (0, 4) est un minimum ;
dét(H f (2, 2)) < 0 donc (2, 2) est un point-selle ;
dét(H f (−2, 2)) < 0 donc (−2, 2) est un point-selle.
5. f (x, y) = x 3 + y 3 − 3x y + 3.
B f est de classe C 2 dans son domaine de définition, l’ouvert R2 .
B Recherche de points critiques :
(
3(x 2 − y) = 0
µ ¶
0
∇ f (x, y) = ⇐⇒ ⇐⇒ (x, y) ∈ { (0, 0), (1, 1) } .
0 3(y 2 − x) = 0

On a deux points critiques : (0, 0) et (1, 1).


B Nature des points critiques :
µ ¶
6x −3
H f (x, y) = , dét(H f (x, y)) = 36x y − 9.
−3 6y

dét(H f (0, 0)) = −9 < 0 donc (0, 0) est un point-selle ;


dét(H f (1, 1)) = 27 > 0 et ∂xx f (1, 1) = 6 > 0, donc (1, 1) est un minimum.
6. f (x, y) = x y(1 − x − y)
B f est de classe C 2 dans son domaine de définition, l’ouvert R2 .
B Recherche de points critiques :
(
y − 2x y − y 2 = 0
µ ¶ ½ µ ¶¾
0 1 1
∇ f (x, y) = ⇐⇒ ⇐⇒ (x, y) ∈ (0, 0), (1, 0), (0, 1), , .
0 x − x 2 − 2x y = 0 3 3

On a quatre points critiques : (0, 0), (1, 0), (0, 1) et 31 , 31 .


¡ ¢

B Nature des points critiques :


µ ¶
−2y 1 − 2x − 2y
H f (x, y) = , dét(H f (x, y)) = 4x y − (1 − 2x − 2y)2
1 − 2x − 2y −2x

dét(H f (0, 0)) < 0 donc (0, 0) est un point-selle ;


dét(H f (1, 0)) < 0 donc (1, 0) est un point-selle ;
dét(H f (0, 1)) < 0 donc (0, 1) est un point-selle ;
dét H f 13 , 13 > 0 et ∂xx f 31 , 31 < 0 donc 13 , 31 < 0 est un maximum.
¡ ¡ ¢¢ ¡ ¢ ¡ ¢

7. f (x, y) = x 3 − 12x y + 8y 3
B f est de classe C 2 dans son domaine de définition, l’ouvert R2 .
B Recherche de points critiques :
(
3x 2 − 12y = 0
µ ¶
0
∇ f (x, y) = ⇐⇒ ⇐⇒ (x, y) ∈ { (0, 0), (2, 1) } .
0 −12x + 24y 2 = 0

On a deux points critiques : (0, 0), et (2, 1).

© G. Faccanoni 141
5 Extrema Dernière mise à jour : Jeudi 5 juin 2014

B Nature des points critiques :


µ ¶
6x −12
H f (x, y) = , dét(H f (x, y)) = 144(2x y − 1)
−12 48y

dét(H f (0, 0)) < 0 donc (0, 0) est un point-selle ;


dét(H f (2, 1)) > 0 et ∂xx f (2, 1) > 0 donc (2, 1) est un minimum.
8. f (x, y) = x y + x1 + 1y
B f est de classe C 2 dans son domaine de définition, l’ouvert R2 \ { (0, κ) | κ ∈ R } \ { (κ, 0) | κ ∈ R }.
B Recherche de points critiques :

y − x12 = 0
µ ¶ (
0
∇ f (x, y) = ⇐⇒ ⇐⇒ (x, y) = (1, 1).
0 x − y12 = 0

On a un unique point critique : (1, 1).


B Nature du point critique :
2
à !
x3
1 4
H f (x, y) = 2 , dét(H f (x, y)) = −1
1 y3 (x y)3
dét(H f (1, 1)) > 0 et ∂xx f (1, 1) > 0 donc (1, 1) est un minimum.
9. f (x, y) = e x cos(y)
B f est de classe C 2 dans son domaine de définition, l’ouvert R2 .
B Recherche de points critiques :
(
e x cos(y) = 0
µ ¶
0
∇ f (x, y) = ⇐⇒ ⇐⇒ 6 ∃(x, y) ∈ R2 .
0 −e x sin(y) = 0

Cette fonction n’admet aucun point critique.


10. f (x, y) = y cos(x)
B f est de classe C 2 dans son domaine de définition, l’ouvert R2 .
B Recherche de points critiques :
(
µ ¶
0 −y sin(x) = 0 ³π ´
∇ f (x, y) = ⇐⇒ ⇐⇒ (x, y) = + κπ, 0 , κ ∈ Z.
0 cos(x) = 0 2

π
On a une infinité de points critiques alignés sur la droite d’équation y = 0 et qui ont ordonnée x = 2 +κπ avec κ ∈ Z.
B Nature des points critiques :
µ ¶
−y cos(x) − sin(x)
H f (x, y) = , dét(H f (x, y)) = − sin2 (x)
− sin(x) 0

dét H f π2 + κπ, 0 < 0 donc ils sont tous des points-selle.


¡ ¡ ¢¢

11. f (x, y) = y 2 + x y ln(x)


B f est de classe C 2 dans son domaine de définition, l’ouvert D = (x, y) ∈ R2 ¯ x > 0 .
© ¯ ª

B Recherche de points critiques :


(
y(ln(x) + 1) = 0
µ ¶ ½ µ ¶¾
0 1 1
∇ f (x, y) = ⇐⇒ ⇐⇒ (x, y) ∈ (1, 0); ,
0 2y + x ln(x) = 0 e 2e
¡1 1
¢
On a deux points critiques : (1, 0) et e , 2e .
B Nature des points critiques :
y
y
µ ¶
1 + ln(x)
H f (x, y) = x , dét(H f (x, y)) = 2 − (1 + ln(x))2
1 + ln(x) 2 x

dét(H f (1, 0)) < 0 donc (1, 0) est un point-selle ;


dét H f 1e , 2e
¡ ¡ 1 ¢¢
> 0 et ∂xx f 1e , 2e
¡ 1¢
> 0 donc 1e , 2e
¡ 1¢
est un minimum.
x2 y y3
12. f (x, y) = 2 + x 2 + 3 − 4y
B f est de classe C 2 dans son domaine de définition, l’ouvert R2 .

142 © G. Faccanoni
Dernière mise à jour : Jeudi 5 juin 2014 5 Extrema

B Recherche de points critiques :


(
x y + 2x = 0
µ ¶
0
∇ f (x, y) = ⇐⇒ x 2 2
⇐⇒ (x, y) ∈ { (0, −2), (0, 2) } .
0
2 + y −4 = 0

On a deux points critiques : (0, −2) et (0, 2).


B Nature des points critiques :
µ ¶
y +2 x
H f (x, y) = , dét(H f (x, y)) = 2y(y + 2) − x
x 2y

dét(H f (0, 2)) > 0 et ∂xx f (0, 2) = 4 > 0 donc (0, 2) est un minimum pour f ;
comme dét(H f (0, −2)) = 0, on ne peut pas conclure en utilisant la matrice hessienne (l’étude du signe de la distance
dans ce cas est trop compliquée).
x2 y y3
13. f (x, y) = 2 − x 2 + 3 − 4y
B f est de classe C 2 dans son domaine de définition, l’ouvert R2 .
B Recherche de points critiques :
(
x y − 2x = 0
µ ¶
0
∇ f (x, y) = ⇐⇒ x 2 ⇐⇒ (x, y) ∈ { (0, −2), (0, 2) } .
0 + y2 − 4 = 0 2

On a deux points critiques : (0, −2) et (0, 2).


B Nature des points critiques :
µ ¶
y −2 x
H f (x, y) = , dét(H f (x, y)) = 2y(y − 2) − x
x 2y

dét(H f (0, −2)) > 0 et ∂xx f (0, −2) < 0 donc (0, −2) est un maximum pour f ;
comme dét(H f (0, 2)) = 0, on ne peut pas conclure en utilisant la matrice hessienne (l’étude du signe de la distance
dans ce cas est trop compliquée).
x y2 3
14. f (x, y) = 2 + x3 − 4x + y 2
B f est de classe C 2 dans son domaine de définition, l’ouvert R2 .
B Recherche de points critiques :
( 2
y
+ x2 − 4 = 0
µ ¶
0
∇ f (x, y) = ⇐⇒ 2 ⇐⇒ (x, y) ∈ { (−2, 0), (2, 0) } .
0 x y + 2y = 0

On a deux points critiques : (0, −2) et (0, 2).


B Nature des points critiques :
µ ¶
2x y
H f (x, y) = , dét(H f (x, y)) = 2x(x + 2) − y
y x +2

dét(H f (2, 0)) > 0 et ∂xx f (2, 0) = 4 > 0 donc (2, 0) est un minimum pour f ;
comme dét(H f (−2, 0)) = 0, on ne peut pas conclure en utilisant la matrice hessienne (l’étude du signe de la distance
dans ce cas est trop compliquée).
2 2
15. f (x, y) = (x 2 − y 2 )e (−x −y )
B f est de classe C 2 dans son domaine de définition, l’ouvert R2 .
B Recherche de points critiques :
( 2 2
2x(1 − x 2 + y 2 )e (−x −y ) = 0
µ ¶
0
∇ f (x, y) = ⇐⇒ 2 2 ⇐⇒ (x, y) ∈ { (0, 0), (0, 1), (0, −1), (1, 0), (−1, 0) } .
0 2y(−1 − x 2 + y 2 )e (−x −y ) = 0

On a 5 points critiques : (0, 0), (0, 1), (0, −1), (1, 0) et (−1, 0).
B Nature des points critiques :
2 −y 2 )
∂xx f (x, y) = 2e (−x (1 − 5x 2 + y 2 + 2x 4 − 2x 2 y 2 ),
2 −y 2 )
∂x y f (x, y) = 4x y(x 2 − y 2 )e (−x ,
(−x 2 −y 2 )
∂ y y f (x, y) = 2e (−1 − x 2 + 5y 2 + 2x 2 y 2 − 2y 4 ).

© G. Faccanoni 143
5 Extrema Dernière mise à jour : Jeudi 5 juin 2014

∂xx f (x, y) ∂x y f (x, y)


µ ¶
H f (x, y) = , dét(H f (x, y)) = ∂xx f (x, y)∂ y y f (x, y) − (∂x y f (x, y))2
∂x y f (x, y) ∂ y y f (x, y)
On a alors

(x 0 , y 0 ) ∂xx f (x 0 , y 0 ) ∂x y f (x 0 , y 0 ) ∂ y y f (x 0 , y 0 ) dét(H f (x 0 , y 0 ))
(0, 0) 2 0 −2 −4 c’est un point-selle
(1, 0) − 4e 0 − 4e 16
e2
c’est un maximum
(−1, 0) − 4e 0 − 4e 16
e2
c’est un maximum
4 4 16
(0, 1) e 0 e e2
c’est un minimum
4 4 16
(0, −1) e 0 e e2
c’est un minimum

2 2
16. f (x, y) = (y 2 − x 2 )e (−x −y )
B f est de classe C 2 dans son domaine de définition, l’ouvert R2 .
B Recherche de points critiques :
( 2 2
2x(−1 + x 2 − y 2 )e (−x −y ) = 0
µ ¶
0
∇ f (x, y) = ⇐⇒ 2 2 ⇐⇒ (x, y) ∈ { (0, 0), (0, 1), (0, −1), (1, 0), (−1, 0) } .
0 2y(1 + x 2 − y 2 )e (−x −y ) = 0

On a 5 points critiques : (0, 0), (0, 1), (0, −1), (1, 0) et (−1, 0).
B Nature des points critiques :
2 −y 2 )
∂xx f (x, y) = −2e (−x (1 − 5x 2 + y 2 + 2x 4 − 2x 2 y 2 ),
2 −y 2 )
∂x y f (x, y) = −4x y(x 2 − y 2 )e (x ,
(−x 2 −y 2 )
∂ y y f (x, y) = −2e (−1 − x 2 + 5y 2 + 2x 2 y 2 − 2y 4 ).

∂xx f (x, y) ∂x y f (x, y)


µ ¶
H f (x, y) = , dét(H f (x, y)) = ∂xx f (x, y)∂ y y f (x, y) − (∂x y f (x, y))2
∂x y f (x, y) ∂ y y f (x, y)
On a alors

(x 0 , y 0 ) ∂xx f (x 0 , y 0 ) ∂x y f (x 0 , y 0 ) ∂ y y f (x 0 , y 0 ) dét(H f (x 0 , y 0 ))
(0, 0) −2 0 2 −4 c’est un point-selle
4 4 16
(1, 0) e 0 e e2
c’est un minimum
4 4 16
(−1, 0) e 0 e e2
c’est un minimum
(0, 1) − 4e 0 − 4e 16
e2
c’est un maximum
(0, −1) − 4e 0 − 4e 16
e2
c’est un maximum

17. f (x, y) = x 4 + y 4 − 2(x − y)2


B f est de classe C 2 dans son domaine de définition, l’ouvert R2 . Comme la restriction f (x, 0) = x 4 −2x 2 tend vers +∞
pour x qui tend vers ±∞, il n’y a pas de maximum global sur R2 . Comme R2 est ouvert, un extrémum relatif de f
vérifie la condition nécessaire ∇ f (x, y) = 0.
B Recherche de points critiques :
(
4(x 3 − x + y) = 0 p p p p o
µ ¶
0 n
∇ f (x, y) = ⇐⇒ ⇐⇒ (x, y) ∈ (0, 0), ( 2, − 2), (− 2, 2) .
0 4(y 3 + x − y) = 0
p p p p
On a 3 points critiques : 1 (0, 0), ( 2, − 2) et (− 2, 2) (on note que f (x, y) = f (−x, −y)).
B Nature des points critiques :

12x 2 − 4
µ ¶
4 ³ ´
H f (x, y) = 2 , dét(H f (x, y)) = 16 (3x 2 − 1)(3y 2 − 1) − 1 .
4 12y − 4
p p p p p p
dét(H f ( 2, − 2)) = 384 > 0 et ∂xx f ( 2, − 2) = 20 > 0 donc ( 2, − 2) est un minimum pour f ;
( ( (
4x 3 − 4x + 4y = 0 x3 + y 3 = 0 x = −y
1. =⇒ =⇒
4y 3 + 4x − 4y = 0 y3 + x − y = 0 (y 2 − 2)y = 0

144 © G. Faccanoni
Dernière mise à jour : Jeudi 5 juin 2014 5 Extrema

p p p p p p
dét(H f (− 2, 2)) = 384 > 0 et ∂xx f (− 2, 2) = 20 > 0 donc (− 2, 2) est un minimum pour f ;
comme dét(H f (0, 0)) = 0, on ne peut pas conclure en utilisant la matrice hessienne.
Pour connaître la nature du point (0, 0) on étudie le signe de d (h, k) = f (h, k) − f (0, 0) pour h et k voisins de 0 :

d (h, k) = h 4 + k 4 − 2(h − k)2 ;

comme d (h, 0) = (h 2 − 2)h 2 < 0 lorsque h est voisin de 0 mais d (h, h) = 2h 4 > 0, alors (0, 0) est un point-selle.
Remarquons qu’avec des transformations algébriques, on peut réécrire la fonction sous la forme

f (x, y) = (x 2 − 2)2 + (y 2 − 2)2 + 2(x + y)2 − 8 ≥ 8 ∀(x, y) ∈ R2 .


p p p p p p p p
Comme f ( 2, − 2) = f (− 2, 2) = −8, les points ( 2, − 2) et (− 2, 2) sont des minima globaux.
18. f (x, y) = x 4 + y 4 − 4(x − y)2
B f est de classe C 2 dans son domaine de définition, l’ouvert R2 . Comme la restriction f (x, 0) = x 4 −4x 2 tend vers +∞
pour x qui tend vers ±∞, il n’y a pas de maximum global sur R2 . Comme R2 est ouvert, un extrémum relatif de f
vérifie la condition nécessaire ∇ f (x, y) = 0.
B Recherche de points critiques :
(
4(x 3 − 2x + 2y) = 0
µ ¶
0
∇ f (x, y) = ⇐⇒ ⇐⇒ (x, y) ∈ { (0, 0), (2, −2), (−2, 2) } .
0 4(y 3 + 2x − 2y) = 0

On a 3 points critiques : 2 (0, 0), (2, −2) et (−2, 2) (on note que f (x, y) = f (−x, −y)).
B Nature des points critiques :
12x 2 − 8
µ ¶
8 ³ ´
H f (x, y) = 2 , dét(H f (x, y)) = 48 3x 2 y 2 − 2(x 2 + y 2 ) .
8 12y − 8

B dét(H f (2, −2)) = 1536 > 0 et ∂xx f (2, −2) = 40 > 0 donc (2, −2) est un minimum local pour f ;
B dét(H f (−2, 2)) = 1536 > 0 et ∂xx f (−2, 2) = 40 > 0 donc (−2, 2) est un minimum local pour f ;
B comme dét(H f (0, 0)) = 0, on ne peut pas conclure en utilisant la matrice hessienne. Pour connaître la nature du
point (0, 0) on étudie le signe de d (h, k) = f (h, k) − f (0, 0) pour h et k voisins de 0 :

d (h, k) = h 4 + k 4 − 4(h − k)2 ;

comme d (h, 0) = (h 2 − 4)h 2 < 0 lorsque h est voisin de 0 mais d (h, h) = 2h 4 > 0, alors (0, 0) est un point-selle.
Remarquons qu’avec des transformations algébriques, on peut réécrire la fonction sous la forme

f (x, y) = (x 2 − 4)2 + (y 2 − 4)2 + 4(x + y)2 − 32 ≥ −32 ∀(x, y) ∈ R2 .

Comme f (2, −2) = f (−2, 2) = −32, les points (2, −2) et (−2, 2) sont des minima globaux.
2
19. f (x, y, z) = x2 + x y z − z + y
B f est définie sur R3 à valeur dans R ; comme la restriction f (0, 0, z) = −z tend vers ±∞ pour z qui tend vers ∓∞, il
n’y a pas d’extremum global sur R3 . Comme R3 est ouvert, un extrémum relatif de f vérifie la condition nécessaire
∇ f (x, y, z) = 0.
B Recherche de points critiques :

µ ¶ x + y z = 0
0

∇ f (x, y, z) = ⇐⇒ xz + 1 = 0 ⇐⇒ (x, y, z) = (1, 1, −1).
0 
xy −1 = 0

Il n’y a qu’un point critique : (1, 1, −1).


B Nature du point critique : on étudie le signe de ∆ f (h, k, l ) ≡ f (1+h, 1+k, −1+l ) pour h, k et l voisins de 0 (les termes
de degré 1 en h, k et l doivent disparaître) :

h 2 + 1 + 2h 3 h2
∆ f (h, k, l ) = + (1 + h)(1 + k)(−1 + l ) − (−1 + l ) + (1 + k) − = + hkl + hl − hk + kl .
2 2 2
Il ne reste que transformer ∆ f si on pense qu’il s’agit d’un extrémum ou fournir des restrictions qui se contredisent
si on pense que ce n’est pas un extrémum. Comme les deux restrictions à deux courbes continues passant par
l’origine ∆ f (h, 0, h) = 32 h 2 > 0 et ∆ f (h, h, 0) = − 21 h 2 < 0 donnent des signes différents, on conclut que ce n’est pas
un extrémum.
( ( ( ( (
4x 3 − 8x + 8y = 0 x 3 − 2(x − y) = 0 x3 + y 3 = 0 x = −y x = −y
2. =⇒ =⇒ =⇒ =⇒
4y 3 + 8x − 8y = 0 y 3 + 2(x − y) = 0 y 3 + 2x − 2y = 0 (y 2 − 4)y = 0 y = 0 ou y = 2 ou y = −2

© G. Faccanoni 145
5 Extrema Dernière mise à jour : Jeudi 5 juin 2014

20. f (x, y) = (x − 1)2 + 2y 2


B f est de classe C 2 dans son domaine de définition, l’ouvert R2 .
B Recherche de points critiques :
(
2x − 2 = 0
µ ¶
0
∇ f (x, y) = ⇐⇒ ⇐⇒ (x, y) = (1, 0).
0 4y = 0

On a un seul point critique : (1, 0).


B Nature du point critique : µ ¶
2 0
H f (x, y) = , dét(H f (x, y)) = 8.
0 4
dét(H f (1, 0)) = 8 > 0 et ∂xx f (1, 0) = 2 > 0 donc (1, 0) est un minimum pour f .
21. f (x, y) = x 2 + x y + y 2 − 2x − y
B f est de classe C 2 dans son domaine de définition, l’ouvert R2 .
B Recherche de points critiques :
(
2x + y − 2 = 0
µ ¶
0
∇ f (x, y) = ⇐⇒ ⇐⇒ (x, y) = (1, 0).
0 x + 2y − 1 = 0

On a un seul point critique : (1, 0).


B Nature du point critique : µ ¶
2 1
H f (x, y) = , dét(H f (x, y)) = 7.
1 4
dét(H f (1, 0)) = 8 > 0 et ∂xx f (1, 0) = 2 > 0 donc (1, 0) est un minimum pour f .
22. f (x, y) = x 3 y 2 (6 − x − y)
B f est de classe C 2 dans son domaine de définition, l’ouvert R2 .
B Recherche de points critiques :
(
3x 2 y 2 (6 − x − y) − x 3 y 2 = 0
µ ¶
0
∇ f (x, y) = ⇐⇒ ⇐⇒ (x, y) ∈ { (3, 2), (t , 0), (0, t ) | t ∈ R } .
0 2x 3 y(6 − x − y) − x 3 y 2 = 0

On a une infinité de points critiques : les points (t , 0) et (0, t ) pour t ∈R sont des points critiques ainsi que le point
(3, 2).
B Nature des points critiques :

∂xx f (x, y) = 6x y 2 (6 − x − y) − 6x 2 y 2 ,
∂x y f (x, y) = 6x 2 y(6 − x − y) − 3x 2 y 2 − 2x 3 y,
∂ y y f (x, y) = 2x 3 (6 − x − y) − 4x 3 y.

∂xx f (x, y) ∂x y f (x, y)


µ ¶
H f (x, y) = , dét(H f (x, y)) = ∂xx f (x, y)∂ y y f (x, y) − (∂x y f (x, y))2
∂x y f (x, y) ∂ y y f (x, y)
dét(H f (3, 2)) > 0 et ∂xx f (3, 2) < 0 donc (3, 2) est un maximum pour f .
dét(H f (t , 0)) = 0 pour tout t ∈ R : l’étude de la matrice hessienne ne permet pas de conclure pour les points sur
l’axe d’équation y = 0. Pour connaître la nature de ces points on étudie le signe de d (h, k) = f (t + h, 0 + k) − f (t , 0) =
(t +h)3 k 2 (6− t −h −k) pour h et k proches de 0. On conclut que les points (t , 0) pour t < 0 ou t > 6 sont des maxima,
les points (t , 0) pour 0 < t < 6 sont des minima et les points (0, 0) et (6, 0) sont des points-selle.
dét(H f (0, t )) = 0 pour tout t ∈ R : l’étude de la matrice hessienne ne permet pas de conclure pour les points sur les
axes. Pour connaître la nature de ces points on étudie le signe de d (h, k) = f (0 + h, t + k) − f (0, t ) = h 3 (t + k)2 (6 − t −
h − k) pour h et k proches de 0. On conclut que les points (0, t ) sont des points-selle pour tout t ∈ R.
23. f (x, y) = e x−y (x 2 − 2y 2 )
B f est de classe C 2 dans son domaine de définition, l’ouvert R2 .
B Recherche de points critiques :
(
(x 2 − 2y 2 + 2x)e x−y = 0
µ ¶
0
∇ f (x, y) = ⇐⇒ ⇐⇒ (x, y) ∈ { (0, 0), (−4, −2) } .
0 (−x 2 + 2y 2 − 4y)e x−y = 0

On a deux points critiques : (0, 0) et (−4, −2).

146 © G. Faccanoni
Dernière mise à jour : Jeudi 5 juin 2014 5 Extrema

B Nature des points critiques :

∂xx f (x, y) = e x−y (x 2 −2y 2 +4x +2), ∂x y f (x, y) = e x−y (−x 2 +2y 2 −2x −4y), ∂ y y f (x, y) = e x−y (x 2 −2y 2 +8y −4);

∂xx f (x, y) ∂x y f (x, y)


µ ¶
H f (x, y) = , dét(H f (x, y)) = ∂xx f (x, y)∂ y y f (x, y) − (∂x y f (x, y))2 .
∂x y f (x, y) ∂ y y f (x, y)
On en déduit que

(x 0 , y 0 ) ∂xx f (x 0 , y 0 ) ∂x y f (x 0 , y 0 ) ∂ y y f (x 0 , y 0 ) dét(H f (x 0 , y 0 ))
(−4, −2) −6e −2 8e −2 −12e −2 8e −4 maximum
(0, 0) 2 0 −4 −8 point-selle

24. f (x, y) = 8
x + xy + y
B f est de classe C 2 dans son domaine de définition, l’ouvert R2 \ (x, y) ¯ x y = 0 .
© ¯ ª

B Recherche de points critiques :


(1 8
y − x2 = 0
µ ¶
0
∇ f (x, y) = ⇐⇒ ⇐⇒ (x, y) = (4, 2).
0 1 − yx2 = 0

On a un unique point critique : (4, 2).


B Nature du point critique :
16
− y12
à ! µ ¶
x3 1 16 1
H f (x, y) = , dét(H f (x, y)) = − .
− y12 2x
y3 y 3 x2 y

dét(H f (4, 2)) > 0 et ∂xx f (4, 2) > 0 donc (4, 2) est un minimum pour f .
25. f (x, y) = x 2 − cos(y)
B f est de classe C 2 dans son domaine de définition, l’ouvert R2 .
B Recherche de points critiques :
(
2x = 0
µ ¶
0
∇ f (x, y) = ⇐⇒ ⇐⇒ ⇐⇒ (x, y) ∈ { (0, κπ) | κ ∈ Z } .
0 sin(y) = 0

On a une infinité de points critiques qui s’écrivent (0, κπ) avec κ ∈ Z.


B Nature du point critique : µ ¶
2 0
H f (x, y) = , dét(H f (x, y)) = 2 cos(y).
0 cos(y)
dét(H f (0, κπ)) = (−1)κ et ∂xx f (0, κπ) > 0 pour toutκ ∈ Z donc (0, κπ) est un minimum si κ est pair et un point-selle
si κ est impair.
2 +y 2 )
26. f (x, y) = (x 2 + y 2 )e −(x .
2
On peut remarquer que si on passe aux coordonnées polaire on obtient w(r ) ≡ f (r cos(ϑ), r sin(ϑ)) = r 2 e −r , autrement-
2
dit on obtient une fonction de la seule variable r > 0 et on a w 0 (r ) = 2r (1−r 2 )e −r qui s’annule pour r = 1 et dont l’étude
des variations montre qu’il s’agit d’un minimum. Il faut étudier séparément le cas (x = 0, y = 0) car il n’est pas pris en
compte lorsqu’on passe aux coordonnées polaire. Si on n’a pas remarqué cette symétrie, on étudie la fonction comme
dans les cas précédents :
B f est de classe C 2 dans son domaine de définition, l’ouvert R2 .
B Recherche de points critiques :
( 2 2
2x(1 − x 2 − y 2 )e −(x +y ) = 0
µ ¶
0
∇ f (x, y) = ⇐⇒ 2 2
0 2y(1 − x 2 − y 2 )e −(x +y ) = 0

On a une infinité de points critiques : le point (0, 0) et les points (x, y) qui appartiennent au cercle x 2 + y 2 = 1.
B Nature du point critique : comme f (x, y) ≥ 0 pour tout (x, y) ∈ R2 et f (x, y) = 0 ssi (x, y) 6= (0, 0) ou (x, y) est tel que
x 2 + y 2 − 1 = 0, on en déduit qu’ils sont des minima (le calcul des dérivées secondes porte à des calculs très longues
et inutiles dans ce cas).
27. f (x, y) = x 3 + y 2 − 6(x 2 − y 2 )
B f est de classe C 2 dans son domaine de définition, l’ouvert R2 .

© G. Faccanoni 147
5 Extrema Dernière mise à jour : Jeudi 5 juin 2014

B Recherche de points critiques :


(
3x(x − 4) = 0
µ ¶
0
∇ f (x, y) = ⇐⇒ ⇐⇒ (x, y) ∈ { (0, 0), (0, −4), (4, 0), (4, −4) } .
0 3y(y + 4) = 0

On a quatre points critiques : (0, 0), (0, −4), (4, 0) et (4, −4).
B Nature des points critiques :
µ ¶
6(x − 2) 0
H f (x, y) = , dét(H f (x, y)) = 36(x − 2)(y + 2).
0 6(y + 2)

dét(H f (0, 0)) < 0 donc (0, 0) est un point-selle ;


dét(H f (0, −4)) > 0 et ∂xx f (0, −4) < 0 donc (0, −4) est un maximum ;
dét(H f (4, 0)) > 0 et ∂xx f (4, 0) > 0 donc (4, 0) est un minimum ;
dét(H f (4, −4)) < 0 (4, −4) est un point-selle.
28. f (x, y) = (x 2 + y 2 − y 3 )e −y
B f est de classe C 2 dans son domaine de définition, l’ouvert R2 .
B Recherche de points critiques :
(
2xe −y = 0 p p o
µ ¶
0 n
∇ f (x, y) = ⇐⇒ ⇐⇒ (x, y) ∈ (0, 0), (0, 2 − 2), (0, 2 + 2) .
0 (−x 2 + 2y − 4y 2 + y 3 )e −y = 0
p p
On a quatre trois critiques : (0, 0), (0, 2 − 2) et (0, 2 + 2).
B Nature des points critiques :
µ −y
−2xe −y

2e
dét(H f (x, y)) = 4 − 2x 2 − 20y + 14y 2 − 2y 3 e −2y .
¡ ¢
H f (x, y) = 2 2 3 ,
−2xe −y (2 + x − 10y + 7y − y )e −y

dét(H f (0, 0)) > 0 et ∂xx f (0, 0) > 0 donc (0, 0) est un minimum ;
p p
dét(H f (0, 2 − 2)) < 0 donc (0, 2 − 2) est un point-selle ;
p p p
dét(H f (0, 2 + 2)) > 0 et ∂xx f (0, 2 + 2) > 0 donc (0, 2 + 2) est un minimum.

Exercice 5.7
La société d’Adèle produit deux types d’ampoules : E17 et E24. Indiquons par x le nombre de milliers d’ampoules de
type E17 produites et supposons que la demande pour ce type de lampes est donnée par p 1 = 50 − x, où p 1 est le prix
de vente en euros. De même, indiquons par y le nombre de milliers d’ampoules de type E24 produites et supposons
que la demande pour ce type est donnée par p 2 = 60 − 2y, où p 2 est aussi le prix de vente en euros. Les coûts communs
de production de ces ampoules est C = 2x y (en milliers d’euros). Par conséquent, le bénéfice de la société d’Adèle (en
milliers d’euros) est une fonction de deux variables x et y. Déterminer le profit maximal d’Adèle.

C ORRECTION . La fonction profit en milliers d’euros est p(x, y) = p 1 x + p 2 y −C (x, y) = 50x − x 2 + 60y − 2y 2 − 2x y. Pour maxi-
miser le profit, on cherche d’abord les points stationnaires :
(
x = 20,
µ ¶ µ ¶
50 − 2x − 2y 0
∇p = 0 ⇐⇒ = ⇐⇒
60 − 4y − 2x 0 y = 5.

Pour établir la nature de ces points, on étudie la matrice hessienne :

∂xx p(x, y) = −2, ∂xx p(20, 5) = −2 < 0,


∂x y p(x, y) = −2, ∂x y p(20, 5) = −2,
∂ y y p(x, y) = −4, ∂ y y p(20, 5) = −4,

et dét(H f (20, 5)) = (−2)(−4) − (−2)2 = 4 > 0 donc (20, 5) est un point de maximum pour p et le profit maximal vaut p(20, 5) =
650. La société d’Adèle réalise le profit maximal de 650000 euros lorsqu’elle vend 20000 ampoules E17 à 30 euros l’une et 5000
ampoules E24 à 50 euros l’une.

148 © G. Faccanoni
Dernière mise à jour : Jeudi 5 juin 2014 5 Extrema

Exercice 5.8
Vous êtes le directeur financier de la firme S ANBON & F ILS. Cette entreprise a investi 3000 euros pour mettre au point un
nouveau parfum. Le coût de la production est de 3 euros par flacon de 100 mL. L’expert consulté par M. S ANBON père a
établi que si la firme consacre x euros en publicité pour son parfum et que le prix de vente d’un flacon est de y euros, la
p
firme vendra exactement 300+6 x −10y pièces. La firme S ANBON & F ILS fixe évidemment x et y de manière à maximiser
son profit. En tant que directeur financier, il vous incombe de déterminer ces valeurs.

C ORRECTION .
p
B Revenu de la vente : y(300 + 6 x − 10y)
p
B Coût de production : 3(300 + 6 x − 10y)
B Coût de développement et de publicité : 3000 + x
B Profit = (Revenu de la vente) - (Coût de production) - (Coût de développement et de publicité)
Le profit de la firme à maximiser est donc la fonction

f : (R∗+ )2 → R
p
x 7→ f (x, y) = (y − 3)(300 + 6 x − 10y) − x − 3000

La condition nécessaire s’écrit


( 3(y−3)
∂x f (x, y) = p−1 = 0
x
p =⇒ (x 0 , y 0 ) = (164025, 138).
∂ y f (x, y) = 330 + 6 x − 20y = 0

La hessienne en ce point est définie négative :


 3(y−3)
∂ f (x, y) = − p 3
 xx

 2 x
241
∂x y f (x, y) = p3x =⇒ dét(H f (x 0 , y 0 )) = − .
∂ f (x, y) = 30(y−3)
 32805
− p3

yy p
x3 x

Comme ∂xx f (x 0 , y 0 ) = −20, on a bien un maximum. La firme S ANBON & F ILS va donc consacrer 164025 euros à la pro-
motion de son nouveau parfum et vendre le flacon de 100 mL à 138 euros. Elle réalisera de la sorte le profit maximal de
f (164025, 138) = 15225 euros.

Exercice 5.9 Une fabrication optimale


Votre société s’occupe de la fabrication d’une pièce mécanique. Celle-ci dépend de deux paramètres réels x et y (à priori
non-contraints) de la façon suivante : le coût unitaire de fabrication d’une pièce est égal à

c(x, y) = x 2 + 2y 2

tandis que le taux de pièces défectueuses (compris entre 0 et 1) est égal à

1
t (x, y) = .
1 + (x y)2

On cherche à maximiser la rentabilité totale du processus de fabrication. On prendra pour fonction objectif le coût uni-
taire moyen d’une pièce non-défectueuse, qui est égal au coût de fabrication d’une pièce divisé par le taux de pièces
non-défectueuses, et on tentera de le simplifier autant que possible.

c(x,y) x 2 +2y 2 (x 2 +2y 2 )(1+x 2 y 2 ) 1


C ORRECTION . La fonction à minimiser s’écrit f (x, y) = = = = + x 2 + x22 +2y 2 . La condition
1−t (x,y) 1− 1 2 x2 y 2 y2
1+(x y)
nécessaire s’écrit
4
∂x f (x, y) = 2 x x−2
(
3 =0 p
4
p
4
2y 4 −1 =⇒ (x 0 , y 0 ) = ( 2, 1/ 2).
∂y f (x, y) = 2 y 3 =0

La hessienne en ce point est définie positive :


x 4 +6

∂xx f (x, y) = 2 x 4


2+6 1+3
∂x y f (x, y) = 0 =⇒ dét(H f (x 0 , y 0 )) = 4 > 0.
∂ f (x, y) = 2 2y 4 +3

 2 1/2
yy y4

© G. Faccanoni 149
5 Extrema Dernière mise à jour : Jeudi 5 juin 2014

p
4
p
4
Comme ∂xx f (x 0 , y 0 ) > 0, on a bien un
p minimum. En choisissant (x, y) = ( 2, 1/ 2), le coût unitaire moyen d’une pièce non-
défectueuse est minimale et égal à 4 2.

Exercice 5.10
Une boîte a la forme d’un parallélépipède surmonté par un demi-cylindre comme dans la figure ci-dessous

y
x
On cherche les valeurs x, y, z ∈ R∗+ qui minimisent la surface totale S de la boîte pour un volume V égal à C .
1. Écrire S(x, y, z)
2. Écrire V (x, y, z)
3. Exprimer z(x, y) comme solution de l’équation V (x, y, z) = C
4. Écrire S̃(x, y) = S(x, y, z(x, y)). Calculer et établir la nature des points critiques de S̃(x, y)

C ORRECTION .
¡ y ¢2
1. S(x, y, z) = x y + 2xz + 2y z + π 2 + π 2 x = 1 + π2 x y + π4 y 2 + 2(x + y)z
y ¡ ¢
¡ y ¢2
2. V (x, y, z) = x y z + 12 π 2 x = x y z + π8 x y 2
C − π8 x y 2
3. V (x, y, z) = C ⇐⇒ z = xy donc z(x, y) = xCy − π8 y
³ ´ ¡
4. S̃(x, y) = S(x, y, z(x, y)) = 1 + π2 x y + π4 y 2 + 2(x + y) xCy − π8 y = 1 + π4 x y + 2C 2C
¡ ¢ ¢
x + y
B Calcul des points critiques :

1 + π4 y − 2C
¡ ¢  Ãs s !
x2
µ ¶
0 3
2C 3 2C
∇S̃(x, y) = ¡ donc ∇S̃(x, y) = ⇐⇒ (x, y) = ,
1 + π4 1 + π4

1 + π4 x − 2C 0
¢
y2
³q q ´
2C 2C
Il existe un seul point critique qui est 3
1+ π4
, 3
1+ π4
.
B Nature des points critiques :
4C
1 + π4
à !
x3 16C 2 ³ π ´2
HS̃ (x, y) = π 4C et det(HS̃ (x, y)) = − 1 +
1+ 4 y3 x3 y 3 4

donc
2 1 + π4 π ¶ π ´2
µr r ¶ µ ¡ µ µr ¢ r ¶¶
¡1 + 4π ¢
³
3 2C 3 2C 2C 2C
HS̃ , = det HS̃ 3 1+ π ,
3
= 3 1 + et .
1+ π4 1+ π4 1 + π4
π
4 1+ 4
2 1+ 4 4
³q ´ ³ ´2/3
= µ C ¶2/3 − π8 1+
q
2C 2C 2C
On conclut que l’unique point critique est bien un minimum et l’on a z 3 1+ π ,
3
1+ π π
4 4 2C 4
1+ π4

Exercice 5.11 Courbe de meilleure approximation


© ªn
On considère un ensemble de points expérimentaux (x i , y i ) i =0 et on suppose que les deux grandeurs x et y sont
liées, au moins approximativement, par une relation de la forme y = a sin( π2 x) + b cos( π2 x). On souhaite alors trouver les
constantes a et b pour que la courbe d’équation y = a sin( π2 x) + b cos( π2 x) s’ajuste le mieux possible aux points observés
(on parle de courbe de meilleure approximation).
Soit d i = y i − (a sin( π2 x i ) + b cos( π2 x i )) l’écart vertical du point (x i , y i ) par rapport à la courbe. La méthode de régression
(ou des moindres carrés) est celle qui choisit a et b de sorte que la somme des carrés de ces déviations soit minimale.

150 © G. Faccanoni
Dernière mise à jour : Jeudi 5 juin 2014 5 Extrema

Pour cela, on doit minimiser la fonction E définie par

E : R2 → R+
n
(a, b) 7→ E (a, b) = d i2 .
X
i =0

Écrire et résoudre le système linéaire qui permet de calculer a et b.

C ORRECTION . Pour minimiser E on cherche ses points stationnaires. Puisque


à !
∂E X n ¡ ¡ ¡π ¢ ¡ π ¢¢¢ ¡π ¢
(a, b) = −2 y i − a sin 2 x i + b cos 2 x i sin 2 x i ,
∂a i =0
à !
∂E X n ¡ ¡ ¡π ¢ ¡ π ¢¢¢ ¡π ¢
(a, b) = −2 y i − a sin 2 x i + b cos 2 x i cos 2 x i ,
∂b i =0

on obtient

∂E
y i − a sin π2 x i + b cos π2 x i sin( π2 x i ) = 0
( (P ¡
n
¡ ¡ ¢ ¡ ¢¢¢
∂a (a, b) = 0 i =0
⇐⇒ Pn ¡
∂E
¡π ¢ ¡ π ¢¢¢
cos( π2 x i ) = 0
¡
∂b (a, b) = 0 i =0 y i − a sin 2 x i + b cos 2 x i

a sin π2 x i + b cos π2 x i sin( π2 x i ) = ni=0 y i sin( π2 x i )


(P ¡¡
n
¡ ¢ ¡ ¢¢¢ P
⇐⇒ Pn ¡¡ i =0 ¡π ¢ ¡ π ¢¢¢
cos( π2 x i ) = ni=0 y i cos( π2 x i )
P
i =0 a sin 2 x i + b cos 2 x i
sin2 π x i
· Pn ¡ ¢ Pn ¡π ¢ ¡ π ¢¸ · ¸ ·Pn ¡ π ¢¸
⇐⇒ Pn i =0¡ π ¢ 2 ¡ π ¢ i =0Psin 2 x i ¡cos ¢2 x i a
= i =0 y i sin ¡ 2 x i ¢ .
n 2 π P n π
i =0 sin 2 x i cos 2 x i i =0 cos 2 x i b i =0 y i cos 2 x i

Si on note
n ¡π n ¡π ¡π n ¡π n ¡π n ¡π
sin2 cos2
X ¢ X ¢ ¢ X ¢ X ¢ X ¢
U≡ 2 xi , V≡ sin 2 xi cos 2 xi , W≡ 2 xi , P≡ y i sin 2 xi , Q≡ y i cos 2 xi ,
i =0 i =0 i =0 i =0 i =0

on doit résoudre le système linéaire µ ¶µ ¶ µ ¶


U V a P
=
V W b Q
dont la solution est
W P −V Q UQ − V P
a= , b= .
UW − V 2 UW − V 2

Exercice 5.12
La méthode de régression s’étend facilement à desªdonnées qui dépendent de deux ou plusieurs variables. On considère
© n
un ensemble de points expérimentaux (x i , y i , z i ) i =0 et on suppose que les trois grandeurs x, y et z sont liées, au moins
approximativement, par une relation affine de la forme z = a + bx + c y. On souhaite alors trouver les constantes a, b et c
pour que le plan d’équation z = a + bx + c y s’ajuste le mieux possible aux points observés (on parle de plan de meilleure
approximation).
Soit d i = z i − (a + bx i + c y i ) l’écart vertical du point (x i , y i , z i ) par rapport au plan. La méthode de régression (ou des
moindres carrés) est celle qui choisit a, b et c de sorte que la somme des carrés de ces déviations soit minimale. Pour
cela, on doit minimiser la fonction E définie par

E : R3 → R+
n
(a, b, c) 7→ E (a, b, c) = d i2 .
X
i =0

1. Écrire le système linéaire qui permet de calculer a, b et c

© G. Faccanoni 151
5 Extrema Dernière mise à jour : Jeudi 5 juin 2014

© ª5
2. Calculer l’équation du plan de meilleure approximation pour l’ensemble (x i , y i , z i ) i =0 où

i 0 1 2 3 4 5
xi 0 0 1 2 2 2
yi 0 1 0 0 1 2
3 1 1
zi 2 2 2 0 2 1

On utilisera la méthode du pivot de G AUSS pour la résolution du système linéaire.

C ORRECTION .
1. Pour minimiser E on cherche ses points stationnaires. Puisque
à !
∂E X n
(a, b, c) = −2 (z i − (a + bx i + c y i )) ,
∂a i =0
à !
∂E X n
(a, b, c) = −2 (z i − (a + bx i + c y i ))x i ,
∂b i =0
à !
∂E X n
(a, b, c) = −2 (z i − (a + bx i + c y i ))y i ,
∂c i =0

on obtient
 ∂E Pn Pn Pn
 ∂a (a, b, c) = 0
 Pi =0 (z i − (a + bx i + c y i )) = 0
 Pi =0 (a + bx i + c y i ) = i =0P
 zi
∂E n n 2 n
∂b (a, b, c) = 0 i =0 (z i − (a + bx i + c y i ))x i = 0 i =0 (ax i + bx i + c y i x i ) = i =0 z i x i
⇐⇒ ⇐⇒
 ∂E
 
Pn 
Pn 2 Pn
∂c (a, b, c) = 0 i =0 (z i − (a + bx i + c y i ))y i = 0 i =0 (a y i + bx i y i + c y i ) = i =0 z i y i
 Pn Pn     Pn 
(n + 1) x y a z
Pn Pni =0 2i Pni =0 i Pni =0 i
⇐⇒  i =0 x i x x i y i  b  =  i =0 z i x i  .
Pn Pni =0 i Pi =0
n 2 Pn
y
i =0 i x
i =0 i i y i =0 y i c i =0 z i y i

2. Dans notre cas,


n
X n
X n
X 11
xi = 7 yi = 4 zi =
i =0 i =0 i =0 2
n
X n
X 7 n
X 9
xi y i = 6 xi zi = y i zi =
i =0 i =0 2 i =0 2
n n
x i2 = 13 y i2 = 6
X X
n +1 = 6
i =0 i =0

donc on a le système linéaire


    11 
6 7 4 a /2
7 13 6 b  =  7/2 
4 6 6 c 9
/2
qu’on peut résoudre par la méthode de G AUSS

  L 2 ←L 2 − 76 L 1    
6 7 4 11
/2 6 7 4 /2
11
6 7 4 11
/2
L 3 ←L 3 − 23 L 1 8
L 3 ←L 3 − 29 L2
 7 13 6 7
/2  −−−−−−−−→  0 29
/6 4
/3 − /12  −−−−−−−−−→  0
35
/6
29 4
/3 − /12 
35

4 6 6 9
/2 0 4
/3 10
/3 5
/6 0 0 86
/29 95
/58

dont la solution est    123   


a /86 1.430232557
b  = −65/86 ≈ −0.7558139503 .
c 95
/172 0.5523255766

Exercice 5.13 Fonctions implicites, extrema libres


On considère l’équation x 2 + 4y 2 + 2y 4 + z 2 + sin(z) = 0.
1. Vérifier qu’elle définie une et une seule fonction z = ϕ(x, y) au voisinage de (0, 0, 0).

152 © G. Faccanoni
Dernière mise à jour : Jeudi 5 juin 2014 5 Extrema

2. Montrer que le point (0, 0) est un point stationnaire pour z = ϕ(x, y) et en établir sa nature.

C ORRECTION . Soit g (x, y, z) = x 2 + 4y 2 + 2y 4 + z 2 + sin(z). Elle est clairement de classe C ∞ (R3 ) et on a

∂x g (x, y, z) = 2x, ∂ y g (x, y, z) = 8y + 8y 3 , ∂z g (x, y, z) = 2z + cos(z),


∂x g (0, 0, 0) = 0, ∂ y g (0, 0, 0) = 0, ∂z g (0, 0, 0) = 1.

Comme ∂z g (0, 0, 0) 6= 0 on peut conclure que l’équation ∂x g (x, y, z) = 0 définit implicitement une et une seule fonction z =
ϕ(x, y) au voisinage de (0, 0) et ϕ(0, 0) = 0. De plus

∂x g (x, y, ϕ(x, y)) −2x


∂x ϕ(x, y) = − = , ∂x ϕ(0, 0) = 0,
∂z g (x, y, ϕ(x, y)) 2ϕ(x, y) + cos(ϕ(x, y))
∂ y g (x, y, ϕ(x, y)) −8y(1 + y 2 )
∂ y ϕ(x, y) = − = , ∂ y ϕ(0, 0) = 0,
∂z g (x, y, ϕ(x, y)) 2ϕ(x, y) + cos(ϕ(x, y))

donc (0, 0) est un point stationnaire pour z = ϕ(x, y). Comme

−2 ϕ(x, y)(2 − sin(ϕ(x, y)))


∂xx ϕ(x, y) = + 2x , ∂xx ϕ(0, 0) = −2,
2ϕ(x, y) + cos(ϕ(x, y)) (2ϕ(x, y) + cos(ϕ(x, y)))2
2∂ y ϕ(x, y) − sin(ϕ(x, y))∂ y ϕ(x, y)
∂x y ϕ(x, y) = 2x , ∂x y ϕ(0, 0) = 0
(2ϕ(x, y) + cos(ϕ(x, y)))2
−8(1 + 3y 2 ) 8(y + y 3 )∂ y ϕ(x, y)(2 − sin(ϕ(x, y)))
∂ y y ϕ(x, y) = + ∂ y y ϕ(x, y) = −8.
2ϕ(x, y) + cos(ϕ(x, y)) (2ϕ(x, y) + cos(ϕ(x, y)))2

le déterminant de la matrice hessienne est 16 : le point (0, 0) est un point de maximum local pour la fonction z = ϕ(x, y).

© G. Faccanoni 153
5 Extrema Dernière mise à jour : Jeudi 5 juin 2014

Optimisation liée (sous contraintes d’égalité)

Exercice 5.14
On cherche les extrema de la fonction f : R2 → R définie par f (x, y) = x + y sous la contrainte x 2 + y 2 = 1. Faire d’abord une
étude des courbes de niveau de f et de leurs intersections avec le graphe de la contrainte. En déduire les extrema ainsi
que leur nature. Vérifier ensuite que cette étude est correcte en étudiant la nature des points critiques du lagrangien.

C ORRECTION .
B f (x, y) = k ssi y = k − x : les
k courbes de niveau de f sont des
=
5
y droites parallèles de pente −1.
B x 2 + y 2 = 1 est l’équation du cercle
k
=
de centre (0, 0) et rayon 1.
k
=
3

p 2
B Les courbes de niveau de f qui
sont tangentes à la contraintes p
sont les droites y = −x − 2 et
k

k p
=

k = x
p 2

=

1 y = −x³ + 2 respectivement ´ aux
3
´ ³
points − p1 , − p1 et p1 , p1 .
2 2 2 2
k B En analysant le sens de croissance
k =
=


1 de f le long ³de la contrainte, on en
5
´
déduit que − p1 , − p1 est un mi-
³ 2 ´ 2
nimum et p1 , p1 un maximum.
2 2

Introduisons le lagrangien :

L : R2 × R → R
(x, y, λ) 7→ x + y − λ(x 2 + y 2 − 1).

On cherche les points critiques de L :



1 − 2λx = 0,
 
0  ½µ ¶ µ ¶¾
1 1 1 1 1 1
∇L(x, y, λ) = 0 ⇐⇒ 1 − 2λy = 0, ⇐⇒ (x, y, λ) ∈ −p ,−p ,−p , p , p , p
0
 2 2 2 2 2 2
1 − x 2 − y 2 = 0,

On étudie la nature de ces points critiques :

∂xx L(x, y, λ) = −2λ ∂ y y L(x, y, λ) = −2λ ∂x y L(x, y, λ) = 0 ∆(x, y, λ) = 4λ2


³ ´ p ³ ´ p ³ ´ ³ ´
∂xx L − p1 , − p1 , − p1 = 2 > 0 ∂ y y L − p1 , − p1 , − p1 = 2 ∂x y L − p1 , − p1 , − p1 = 0 ∆ − p1 , − p1 , − p1 = 2 > 0
2 2 2 2 2 2 2 2 2 2 2 2
³
1 p1 p1
´ p ³
1 p1 p1
´ p ³
1 p1 p1
´ ³
1 p1 p1
´
∂xx L p , , =− 2<0 ∂y y L p , , =− 2 ∂x y L p , , =0 ∆ p , , =2>0
2 2 2 2 2 2 2 2 2 2 2 2
³ ´ ³ ´
donc − p1 , − p1 est un minimum et p1 , p1 un maximum pour f sous la contrainte x 2 + y 2 = 1.
2 2 2 2

Exercice 5.15
Utiliser la méthode des multiplicateurs de L AGRANGE pour calculer le maximum et le minimum de la fonction f sous la
(les) contrainte(s) indiquée(s) :
1. f (x, y) = x 2 + y 2 sous la contrainte x y = 1
2. f (x, y) = 3x + y sous la contrainte x 2 + y 2 = 10
3. f (x, y) = y 2 − x 2 sous la contrainte 14 x 2 + y 2 = 1
4. f (x, y) = e x y sous la contrainte x 3 + y 3 = 16
5. f (x, y, z) = 2x + 2y + z sous la contrainte x 2 + y 2 + z 2 = 9
6. f (x, y, z) = x 2 + y 2 + z 2 sous la contrainte x + y + z = 12

154 © G. Faccanoni
Dernière mise à jour : Jeudi 5 juin 2014 5 Extrema

7. f (x, y, z, t ) = x + y + z + t sous la contrainte x 2 + y 2 + z 2 + t 2 = 1


8. f (x 1 , x 2 , . . . , x n ) = x 1 + x 2 + · · · + x n sous la contrainte x 12 + x 22 + · · · + x n2 = c
9. f (x, y, z) = x + 2y sous les contraintes x + y + z = 1 et y 2 + z 2 = 4
10. f (x, y, z) = 3x − y − 3z sous les contraintes x + y − z = 0 et x 2 + 2z 2 = 1

C ORRECTION .
1. Formons le lagrangien
L(x, y, λ) = x 2 + y 2 − λ(x y − 1)
où λ (multiplicateur de L AGRANGE) est une inconnue. Pour que cette fonction ait un extremum il faut que le gradient
de L soit nul, autrement dit on cherche les triplets (x, y, λ) tels que
2x − λy
   
0
∇L = 0 ⇐⇒ 2y − λx  = 0 ⇐⇒ (x, y, λ) ∈ { (1, 1, 2), (−1, −1, 2) }
1−xy 0
Donc (1, 1) et (−1, −1) sont des points critiques de la fonction f sous la contrainte x y = 1. Observons que ces points ont
été obtenus par une condition nécessaire et comme ∆(1, 1, 2) = ∆(−1, −1, 2) = 0, rien dans le théorème ne permet de
savoir s’il s’agit de maxima, minima ou ni l’un ni l’autre. Étudions alors le signe de la différence d (h, k) = f (1 + h, 1 +
k) − f (1, 1) = (1 + h)2 + (1 + k)2 − 2, h et k étant liés par la relation 0 = g (1 + h, 1 + k) = (1 + h)(1 + k) − 1, soit encore
1 + k = 1/(1 + h). On a par conséquente d (h, −1 + 1/(1 + h)) = h 2 (2 + h)2 /(1 + h)2 ≥ 0 pour tout h ∈ R : le point (1, 1)
est alors un minimum local de f sous la contrainte g . De la même manière on montre que le point (−1, −1) est un
minimum local de f sous la contrainte g . En observant les courbes de niveau ci-dessous on constate qu’il s’agit bien
de minima.
y
k%

f (x, y) = k
xy = 1

x
xy = 1

f (x, y) = 2

2. Formons le lagrangien
L(x, y, λ) = 3x + y − λ(x 2 + y 2 − 10)
où λ (multiplicateur de L AGRANGE) est une inconnue. Pour que cette fonction ait un extremum il faut que le gradient
de L soit nul, autrement dit on cherche les triplets (x, y, λ) tels que
   
3 − 2λx 0
∇L = 0 ⇐⇒  1 − 2λy  = 0 ⇐⇒ (x, y, λ) ∈ { (3, 1, 1/2), (−3, −1, −1/2) }
10 − x 2 − y 2 0

Donc (3, 1) et (−3, −1) sont des points critiques de la fonction f sous la contrainte x 2 +y 2 = 10. Comme ∆(3, 1, 1/2) = 1 > 0
et ∂xx L(3, 1, 1/2) = −1 < 0, le point (3, 1) est un maximum ; puisque ∆(−3, −1, −1/2) = 1 > 0 et ∂xx L(−3, −1, −1/2) = 1 > 0,
le point (−3, −1) est un minimum.
y

f (x, y) = 10 x 2 + y 2 = 10

f (x, y) = k
k%
x

f (x, y) = 10

© G. Faccanoni 155
5 Extrema Dernière mise à jour : Jeudi 5 juin 2014

3. Formons le lagrangien µ ¶
1 2
L(x, y, λ) = y 2 − x 2 − λ x + y2 − 1
4
où λ (multiplicateur de L AGRANGE) est une inconnue. Pour que cette fonction ait un extremum il faut que le gradient
de L soit nul, autrement dit on cherche les triplets (x, y, λ) tels que

−2x − λx/2
   
0
∇L = 0 ⇐⇒  2y − 2λy  = 0 ⇐⇒ (x, y, λ) ∈ { (0, 1, 1), (0, −1, 1), (2, 0, −4), (−2, 0, −4) }
1 − 14 x 2 − y 2 0

Donc (0, 1), (0, −1), (2, 0) et (−2, 0) sont des points critiques de la fonction f sous la contrainte 41 x 2 + y 2 = 1. Observons
que ces points ont été obtenus par une condition nécessaire et comme ∆(x, y, λ) = −(4 + λ)(1 − λ), alors ∆(0, 1, 1) =
∆(0, −1, 1) = ∆(2, 0, −4) = ∆(−2, 0, −4) = 0, rien dans le théorème ne permet de connaître leur nature.
Étudions alors directement le comportement de la fonction au voisinage de ces points en regardant le signe de la
fonction
d (h, k) = f (x 0 + h, y 0 + k) − f (x 0 , y 0 ) = (y 0 + k)2 − (x 0 + h)2 − y 02 + x 02 ,
h et k étant liés par (x 0 + h)2 + 4(y 0 + k)2 = 4, i.e. g (x 0 + h, y 0 + k) = 0.
Si (x 0 , y 0 ) = (0, ±1) alors ∂ y g (x 0 , y 0 ) 6= 0. Dans ce cas l’équation g (x 0 + h, y 0 + k) = 0 définit implicitement k en fonction
de h au voisinage de h = 0 : en résolvant g (x 0 + h, y 0 + k) = 0 on trouve (y 0 + k)2 = 1 − (x 0 + h)2 /4 ; en remplaçant
l’expression ainsi trouvée dans d (h, k) on trouve la fonction d’une seule variable

5
d˜(h) = − h 2 ≤ 0,
4
donc les points (0, ±1) sont des maxima locaux.
Si (x 0 , y 0 ) = (±2, 0) alors ∂ y g (x 0 , y 0 ) = 0 mais ∂x g (x 0 , y 0 ) 6= 0. Dans ce cas l’équation g (x 0 + h, y 0 + k) = 0 définit implici-
tement h en fonction de k au voisinage de k = 0 : en résolvant g (x 0 + h, y 0 + k) = 0 on trouve (x 0 + h)2 = 4 − 4(y 0 + k)2 ;
en remplaçant l’expression ainsi trouvée dans d (h, k) on trouve la fonction d’une seule variable

d˜(k) = 5k 2 ≥ 0,

donc les points (±2, 0) sont des minima locaux.


En observant les courbes de niveau ci-dessous on constate que les points (0, 1) et (0, −1) sont des minima et les points
(2, 0) et (−2, 0) sont des maxima.
k%
y

f (x, y) = 1

f (x, y) = k
x 2 + 4y 2 = 4
f (x, y) = −4 f (x, y) = −4

x
k% k%

f (x, y) = 1

k%

4. Formons le lagrangien
L(x, y, λ) = e x y − λ x 3 + y 3 − 16
¡ ¢

156 © G. Faccanoni
Dernière mise à jour : Jeudi 5 juin 2014 5 Extrema

où λ (multiplicateur de L AGRANGE) est une inconnue. Pour que cette fonction ait un extremum il faut que le gradient
de L soit nul, autrement dit on cherche les triplets (x, y, λ) tels que

ye x y − 3λx 2
   
0
∇L = 0 ⇐⇒ xe x y − 3λy 2  = 0 ⇐⇒ (x, y, λ) = (2, 2, e 4 /6)
16 − x 3 − y 3 0

Donc (2, 2) est un point critique de la fonction f sous la contrainte x 3 + y 3 = 16. Observons que ces points ont été ob-
tenus par une condition nécessaire et comme ∆(2, 2, e 4 /6) < 0, on ne peut pas conclure sur la nature du point critique.
Cependant, en analysant les courbes de niveau, on voit qu’il s’agit d’un maximum.
4

y 2

K1 0 1 2 3 4
x

K1

5. Formons le lagrangien
L(x, y, z, λ) = 2x + 2y + z − λ x 2 + y 2 + z 2 − 9
¡ ¢

où λ (multiplicateur de L AGRANGE) est une inconnue. Pour que cette fonction ait un extremum il faut que le gradient
de L soit nul, autrement dit on cherche les points (x, y, z, λ) tels que

2 − 2λx 0
   
2 − 2λy  0
(x, y, z, λ) ∈ { (2, 2, 1, 1/2), (−2, −2, −1, −1/2) }

∇L = 0 ⇐⇒  =  ⇐⇒
 1 − 2λz  0
2 2 2
16 − x − y − z 0

Donc (2, 2, 1) et (−2, −2, −1) sont des points critiques de la fonction f sous la contrainte x 2 + y 2 + z 2 = 9. Observons que
ces points ont été obtenus par une condition nécessaire et on ne peut pas conclure sur leur nature. Cependant, comme
f (2, 2, 1) = 9 et f (−2, −2, −1) = −9, le premier est un maximum tandis que le dernier un minimum.
6. Formons le lagrangien
L(x, y, z, λ) = x 2 + y 2 + z 2 − λ x + y + z − 12
¡ ¢

où λ (multiplicateur de L AGRANGE) est une inconnue. Pour que cette fonction ait un extremum il faut que le gradient
de L soit nul, autrement dit on cherche les points (x, y, z, λ) tels que

2x − λ 0
   
2y − λ  0
(x, y, z, λ) = (4, 4, 4, 2).

∇L = 0 ⇐⇒  =  ⇐⇒
 2z − λ  0
12 − x − y − z 0

Donc (4, 4, 4) est un point critique de la fonction f sous la contrainte x + y + z = 12. Observons que ce point a été
obtenu par une condition nécessaire et on ne peut pas conclure sur sa nature. Pour cela, on peut expliciter une variable
de la contrainte et étudier l’optimisation libre de la fonction de deux variables ainsi obtenue. Par exemple, on pose
z = 12−x−y et on obtient la fonction h(x, y) = f (x, y, z = 12−x−y) = x 2 +y 2 +(12−x−y)2 = 2(x 2 +y 2 +x y −12x−12y +72).
On a µ ¶ µ ¶
4x + 2y − 24 0
∇h = 0 ⇐⇒ = ⇐⇒ (x, y) = (4, 4).
4y + 2x − 24 0
De plus, µ ¶
2 1
Hh (x, y) = ∀(x, y) ∈ R2
1 2
donc ∆Hh (4, 4) > 0 et ∂xx h(4, 4) = 2 > 0 : le point (4, 4) est un minimum pour h donc le point (4, 4, 12 − 4 − 4 = 4) est un
minimum de f sous la contrainte x + y + z = 12.

© G. Faccanoni 157
5 Extrema Dernière mise à jour : Jeudi 5 juin 2014

7. Formons le lagrangien
L(x, y, z, λ) = x + y + z + t − λ x 2 + y 2 + z 2 + t 2 − 1
¡ ¢

où λ (multiplicateur de L AGRANGE) est une inconnue. Pour que cette fonction ait un extremum il faut que le gradient
de L soit nul, autrement dit on cherche les points (x, y, z, t , λ) tels que

1 − 2λx 0
   
 1 − 2λy  0 
(x, y, z, t , λ) ∈ { (1/2, 1/2, 1/2, 1/2, 1), (−1/2, −1/2, −1/2, −1/2, −1) } .
   
∇L = 0 ⇐⇒ 
 1 − 2λz  = 0 
   ⇐⇒
 1 − 2λt  0 
2 2 2 2
1−x − y −z −t 0

Donc (1/2, 1/2, 1/2, 1/2) et (−1/2, −1/2, −1/2, −1/2) sont des points critiques de la fonction f sous la contrainte x 2 + y 2 +
z 2 + t 2 = 1. Observons que ces points ont été obtenus par une condition nécessaire et on ne peut pas conclure sur leur
nature.
8. Formons le lagrangien à !
n n
L(x 1 , x 2 , . . . , x n , λ) = xi − λ x i2 − c
X X
i =1 i =1

où λ (multiplicateur de L AGRANGE) est une inconnue. Pour que cette fonction ait un extremum il faut que le gradient
de L soit nul, autrement dit on cherche les points (x 1 , x 2 , . . . , x n , λ) tels que
   
1 − 2λx 1 0
 1 − 2λx  0
 2   
 ..  .
 =  .  ⇐⇒ (x 1 , x 2 , . . . , x n , λ) ∈ { (c/n, c/n, . . . , c/n, 1), (−c/n, −c/n, . . . , −c/n, −1) } .
∇L = 0 ⇐⇒   .  .
   
 1 − 2λx n  0
¡Pn
c − i =1 x i2
¢
0

Donc (c/n, c/n, . . . , c/n) et (−c/n, −c/n, . . . , −c/n) sont des points critiques de la fonction f sous la contrainte ni=1 x i2 =
P

c. Observons que ces points ont été obtenus par une condition nécessaire et on ne peut pas conclure sur leur nature.
9. Formons le lagrangien
L(x, y, z, λ, µ) = x + 2y − λ x + y + z − 1 − µ y 2 + z 2 − 4
¡ ¢ ¡ ¢

où λ et µ (multiplicateurs de L AGRANGE) sont des inconnues. Pour que cette fonction ait un extremum il faut que le
gradient de L soit nul, autrement dit on cherche les points (x, y, z, λ, µ) tels que

1−λ 0
   
 2 − λ − 2µy  0 n p p p p p p o
(x, y, z, λ, µ) ∈ (1, 2, − 2, 1, 1/ 8), (1, − 2, 2, 1, −1/ 8) .
   
∇L = 0 ⇐⇒  −λ − 2µz  = 0 ⇐⇒
   
1 − x − y − z  0
4 − y 2 − z2 0
p p p p
Donc (1, 2, − 2) et (1, − 2, 2) sont des points critiques de la fonction f sous les contraintes x +y +z = 1 et y 2 +z 2 = 4.
Observons que ces points ont été obtenus par une condition nécessaire et on ne peut pas conclure sur leur nature.
10. Formons le lagrangien
L(x, y, z, λ, µ) = 3x − y − 3z − λ x + y − z − µ x 2 + 2z 2 − 1
¡ ¢ ¡ ¢

où λ et µ (multiplicateurs de L AGRANGE) sont des inconnues. Pour que cette fonction ait un extremum il faut que le
gradient de L soit nul, autrement dit on cherche les points (x, y, z, λ, µ) tels que

3 − λ − 2µx 0
   
 −1 − λ  0 ½µ ¶ µ ¶¾
−3 + λ − 2µz  = 0 1 2 1 2 1 2 1 2
(x, y, z, λ, µ) ∈ − p , p , p , −1, − p , p , − p , − p , −1, p
   
∇L = 0 ⇐⇒ ⇐⇒ .
3 3 3 3 3 3 3 3
   
 −x − y + z  0
1 − x 2 − 2z 2 0
p p p p p p
Donc (−1/ 3, 2/ 3, 1/ 3) et (1/ 3, −2/ 3, −1/ 3) sont des points critiques de la fonction f sous les contraintes x+y =
z et x 2 +2z 2 = 1. Observons que ces points ont été obtenus par une condition nécessaire et on ne peut pas conclure sur
leur nature.

158 © G. Faccanoni
Dernière mise à jour : Jeudi 5 juin 2014 5 Extrema

Exercice 5.16
Trouver et classer les points stationnaires des fonctions suivantes
1. f (x, y) = x y dans R2 sous la contrainte x 2 + y 2 = 1,
2. f (x, y) = x 2 + y 2 + 2 dans R2 sous la contrainte x 2 − 2x y + y 2 = 0,
3. f (x, y) = (x − y) ln(4 + x − y) sous la contrainte x 2 + y 2 = 1.

C ORRECTION .

1. On a ∇g (x, y) = (2x, 2y)T = (0, 0) si et seulement si (x, y) = (0, 0) mais (0, 0) ne vérifie pas la contrainte donc on peut
appliquer la méthode des multiplicateurs de L AGRANGE. On introduit le lagrangien L(x, y, λ) = x y − λ(x 2 + y 2 − 1) et on
cherche ses points critiques :
   
y − 2λx 0 ½µ ¶ µ ¶ µ ¶ µ ¶¾
1 1 1 1 1 1 1 1 1 1 1 1
∇L(x, y, λ) = x − 2λy = 0 ⇐⇒ (x, y, λ) ∈
    p , p , , −p ,−p , , p ,−p ,− , −p , p ,− .
1 − x2 − y 2 0 2 2 2 2 2 2 2 2 2 2 2 2

Soit
∆(x, y, λ) ≡ ∂xx L(x, y, λ)∂ y y L(x, y, λ) − (∂xx L(x, y, λ))2 = 4λ2 − 1.
³ ´
Comme ∆ ± p1 , ± p1 , ± 21 = 0 on ne peut pas établir directement la nature de ces points critiques.
2 2
2. On cherche les possibles extrema par la méthode des multiplicateurs de L AGRANGE. Si on note g (x, y) = x 2 − 2x y + y 2
la contrainte, on a ∂x g (x, y) = 2x − 2y et ∂ y g (x, y) = −2x + 2y donc le point (0, 0) est une singularité de la contrainte.
Cherchons maintenant les points stationnaires du lagrangien :

L(x, y, λ) = x 2 + y 2 + 2 − λ(x 2 − 2x y + y 2 ).

On a
L x (x, y, λ) = 2x − λ(2x − 2y), L y (x, y, λ) = 2y − λ(−2x + 2y), L λ (x, y, λ) = −(x 2 − 2x y + y 2 ),
donc le gradient de L s’annule seulement en (0, 0) et f (0, 0) = 2. Comme f (x, y) ≥ 2 en tout point, il s’agit d’un mi-
nimum. En revanche il n’y a pas de maximum (ceci ne contredit pas le théorème de W EIERSTRASS car la contrainte
g (x, y) = (x − y)2 = 0, i.e. la droite d’équation y = x, n’est pas un ensemble borné). En effet, f (x, y)|g (x,y)=0 = f (x, x) =
2x 2 + 2 −−−−−→ +∞.
x→+∞
3. Remarquons tout d’abord que la fonction n’est définie que si y < x + 4, ce qui est toujours vérifié si on réalise la
contrainte. Comme la contrainte g (x, y) = x 2 + y 2 −1 a gradient ∇g (x, y) = (2x, 2y) qui ne s’annule pas lorsque g (x, y) =
0, on peut utiliser la méthode des multiplicateurs de L AGRANGE : on défini le lagrangien L(x, y, `) = (x − y) ln(4+ x − y)−
`(1 − x 2 − y 2 ) et on cherche les points stationnaires pour L, comme
 x−y 
ln(4 + x − y) + 4+x−y − 2`x
x−y
∇L(x, y, `) = − ln(4 + x − y) − 4+x−y − 2`y 
 

x2 + y 2 − 1
p p p p p p p
∇L(x, y, `) =p
et p p ssi (x, y)p= (1/ 2, −1/
(0, 0, 0) p 2) ou (x, y) = (−1/ 2, 1/ 2).
p Commep f (1/ 2, −1/ 2) = 2/ 2 ln(4 +
2/ 2)p et f p
(−1/ 2, 1/ 2) = −2/ 2 ln(4 − 2/ 2), on conclut que (x, y) = (1/ 2, −1/ 2) est un maximum lié et (x, y) =
(−1/ 2, 1/ 2) un minimum lié.

Exercice 5.17
Trouver le rectangle de périmètre minimale parmi ceux qui ont surface fixée.

© G. Faccanoni 159
5 Extrema Dernière mise à jour : Jeudi 5 juin 2014

C ORRECTION . Appelons x > 0 et y > 0 respectivement la base et la hauteur d’un rectangle quelconque. Le périmètre est la
2
∗ ) → R∗ définie par
fonction 2x + 2y tandis que la surface est la fonction s(x, y) = x y. Il s’agit de minimiser la fonction p : (R+ +
+ 2
p(x, y) = 2(x + y) sous la contrainte s(x, y) = 0 où s : (R∗ ) → R∗ est définie par s(x, y) = x y − K avec K > 0 une constante.
+

Première méthode Soit L(x, y, λ) = p(x, y) − λs(x, y). En introduisant le système de L AGRANGE, il s’agit de chercher les so-
2
lutions (x, y, λ) ∈ (R+
∗ ) × R de

2 − λy = 0,
  
0  µ
p p

2
∇L(x, y, λ) = 0 =⇒ 2 − λx = 0, =⇒ (x, y, λ) = K, K, p .
0
 K
K − x y = 0,

p p p p p p p
On obtient que le seul point critique est ( K , K ) et p( K , K ) = 4 K . Comme ∆( K , K , p2 ) = − p4 < 0, on ne peut
K K p p
pas établir
p p directement la nature du point critique. Étudions alors
p le signe
p de la différence
p dp(h, k) = p( K + h, K +
p K, p
k) − p( K ) = 2(h + k), h et k étant liés par la relation 0 = p
g ( K + h, K + k) = ( K + h)( K + k) p− Kp, soit encore
k = − K h/( K + h). On a par conséquente d (h, k(h)) = 2h 2 /( K + h) > 0 pour tout h ' 0 : le point ( K , K ) est alors
un minimum local de f sous la contrainte g .
Seconde méthode La contrainte se réécrit y = Kx donc il s’agit de chercher les extrema de la fonction réelle de variable réelle
h : (R∗+ ) → R+ définie par
K K
µ ¶ µ ¶
h(x) = f x, =2 x+ .
x x
Cherchons d’abord les points critiques :
K
h 0 (x) = 2 − 2
x2
p
et h 0 (x) = 0 ssi x = K . Comme h 00 (x) = 4 xK3 > 0, il s’agit d’un minimum.

Exercice 5.18
Trouver les dimensions d’une boîte rectangulaire ouverte de surface 12 de volume maximale.

C ORRECTION . Notons x, y et z les dimensions de la boîte. Il s’agit de maximiser la fonction f (x, y, z) = x y z sous la contrainte
g (x, y, z) = 2xz + 2y z + x y = 12 avec x, y, z > 0 (la face manquante a cotés x et y).
1. Méthode des multiplicateurs de L AGRANGE : on cherche les solutions du système 3

 y z = λ(2z + y),


xz = λ(2z + x),



 x y = λ(2x + 2y),

2xz + 2y z + x y = 12.

L’unique solution est le point (x, y, z, λ) = (2, 2, 1, 1/2) et f (2, 2, 1) = 4. Il s’agit bien d’un point critique mais pour établir
s’il est un extremum il faut étudier le signe de la différence

d (h 1 , h 2 , h 3 ) = f (2 + h 1 , 2 + h 2 , 1 + h 3 ) − f (2, 2, 1) = (2 + h 1 )(2 + h 2 )(1 + h 3 ) − 4,

h 1 , h 2 et h 3 étant liés par la relation 0 = g (2 + h 1 , 2 + h 2 , 1 + h 3 ) − 12, soit encore

12 − 2(2 + h 1 )(2 + h 2 )
(1 + h 3 ) = .
2(2 + h 1 ) + (2 + h 2 )

On a par conséquente
¡ ¢ ¡ ¢
¢ (2 + h 1 )(2 + h 2 ) 12 − 2(2 + h 1 )(2 + h 2 ) − 4 2(2 + h 1 ) + (2 + h 2 )
d˜(h 1 , h 2 ) = d h 1 , h 2 , h 3 (h 1 , h 2 ) =
¡
2(2 + h 1 ) + (2 + h 2 )


 x y z = λ(2xz + x y),

x y z = λ(2y z + x y),

3. =⇒ 2xz + x y = 2y z + x y = 2xz + 2y z =⇒ x = y = 2z


 x y z = λ(2xz + 2y z),

2xz + 2y z + x y = 12.

160 © G. Faccanoni
Dernière mise à jour : Jeudi 5 juin 2014 5 Extrema

1
4h 1 h 2 + 4h 22 + 4h 1 h 22 + 4h 12 + 4h 12 h 2 + h 12 h 22
¡ ¢
=−
2(4 + h 1 + h 2 )

pour tout h 1 , h 2 ' 0. Le signe de d˜ est l’opposé du signe de N (h 1 , h 2 ) = 4h 1 h 2 + 4h 22 + 4h 1 h 22 + 4h 12 + 4h 12 h 2 + h 12 h 22 mais


celui ci est difficile à étudier. Cependant, il est simple de vérifier que ∇N (0, 0) = (0, 0) et que la matrice Hessienne
H N (h 1 , h 2 ) est semi-définie positive, par conséquent N est convexe et donc N (h 1 , h 2 ) ≥ 0 pour h 1 , h 2 ' 0. On trouve
alors d˜(h 1 , h 2 ) ≤ 0 et on conclut que (2, 2, 1) est un maximum de la fonction f sous la contrainte g .
12−x y
2. Méthode de réduction : la contrainte se réécrit z(x, y) = 2(x+y) ; il s’agit alors de maximiser la fonction h : (R∗+ )2 → R+
12−x y
définie par h(x, y) = x y 2(x+y) . Cherchons d’abord les points critiques :

−y 2 (x 2 +2x y−12)
 
2
∇h(x, y) =  −x 2 (y2(x+y)
2 +2x y−12)

2(x+y) 2

et ∇h(x, y) = (0, 0) ssi (x, y) = (2, 2). Étudions maintenant ce point en calculant le déterminant de la matrice hessienne
de la fonction h :
−y 2 (12 + y 2 )
h xx (x, y) = h xx (2, 2) = −1 < 0,
(x + y)3
−x y(x 2 + y 2 + 3x y − 12) 1
h x y (x, y) = h x y (2, 2) = − ,
(x + y)3 2
−x 2 (12 + x 2 )
h y y (x, y) = h y y (2, 2) = −1,
(x + y)3
3
D(2, 2) = h xx (2, 2)h y y (2, 2) − (h x y (2, 2))2 = > 0.
4
On a donc que (2, 2) est bien un maximum.

Exercice 5.19
Une firme produit des appareils dans deux usines différentes. Les coûts totaux de production pour les deux usines sont
respectivement :

C 1 (q) = 200 + 6q + 0.03q 2 , C 2 (q) = 150 + 10q + 0.02q 2 ,

où q représente le nombre d’appareils produits dans l’usine. La firme s’est engagée à livrer 100 appareils à une entreprise.
Les frais de transport par appareil sont de 4 euros pour les livraisons à partir de la première usine et de 2 euros pour les
livraisons à partir de la seconde usine. Les frais de transport sont supportés par la firme productive. Calculer le nombre
d’appareils que doit produire la firme dans chaque usine afin de minimiser le coût total de production compris le coût
de transport.

C ORRECTION . Deux méthodes possibles :


Méthode de Lagrange : soit qi le nombre d’appareils produits dans l’usine i .
B Contrainte :
q 1 + q 2 = 100
B Coût totale :
C (q 1 , q 2 ) = C 1 (q 1 ) + 4q 1 +C 2 (q 2 ) + 2q 2 = 0.03q 12 + 0.02q 22 + 10q 1 + 12q 2 + 350.
B Lagrangien :

L(q 1 , q 2 , `) = C (q 1 , q 2 ) − `(100 − q 1 − q 2 ) = 0.03q 12 + 0.02q 22 + 10q 1 + 12q 2 + 350 − `(100 − q 1 − q 2 ).

B Gradient du lagrangien :
0.06q 1 + 10 + `
 

∇L(q 1 , q 2 , `) = 0.04q 2 + 12 + `
q 1 + q 2 − 100
B Points critiques du lagrangien :

∇L(q 1 , q 2 , `) = (0, 0, 0) ⇐⇒ (q 1 , q 2 , `) = (60, 40, 13.6).

© G. Faccanoni 161
5 Extrema Dernière mise à jour : Jeudi 5 juin 2014

B Nature du point critique :

∂q1 q1 L(60, 40, 13.6) = 0.06 > 0, ∂q2 q2 L(60, 40, 13.6) = 0.04 > 0, ∂q1 q2 L(60, 40, 13.6) = 0,

et (∂q1 q1 L∂q2 q2 L − ∂q1 q2 L 2 )(60, 40, 13.6) = 0.0024 > 0 : il s’agit d’un minimum.
Méthode de réduction : soit qi le nombre d’appareils produits dans l’usine i .
B Contrainte :
q 1 + q 2 = 100 =⇒ q 2 = 100 − q 1 .
B Coût total :

C̃ (q 1 ) = C (q 1 , q 2 (q 1 )) = C 1 (q 1 ) + 4q 1 +C 2 (q 2 (q 1 )) + 2q 2 (q 1 ) = 0.03q 12 + 0.02(q 2 (q 1 ))2 + 10q 1 + 12q 2 (q 1 ) + 350 =


0.03q 12 + 0.02(100 − q 1 )2 + 10q 1 + 12(100 − q 1 ) + 350 =
0.03q 12 + 0.02(10000 − 400q 1 + q 12 ) + 10q 1 + 1200 − 12q 1 + 350 = 0.05q 12 − 6q 1 + 1050.

B Dérivée du coût total :


C̃ 0 (q 1 ) = 0.1q 1 − 6
B Points critiques du coût total :
C̃ 0 (q 1 ) = 0 ⇐⇒ q 1 = 60.
B Nature du point critique :
C̃ 00 (180) = 0.1 > 0,
donc il s’agit d’un minimum.
Par conséquent, quand la firme livre 60 appareils de sa première usine et 100 − q 1 = 40 appareils de sa deuxième, le coût total
est minimal sous la contrainte d’une livraison de 100 appareils.

Exercice 5.20
Trouver le parallélépipède (i.e. une boîte fermée) de volume 8 dont la surface est minimale.

C ORRECTION .
3
1. Méthode des multiplicateurs de L AGRANGE : on doit minimiser la fonction f : (R+ ∗ ) → R∗ définie par f (x, y, z) = 2(x y +
+
+ 3
xz + y z) sous la contrainte g (x, y, z) = 0 où g : (R∗ ) → R∗ est définie par g (x, y, z) = x y z − 8. En introduisant le système
+
3
de L AGRANGE, il s’agit de chercher les solutions (x, y, z, λ) ∈ (R+∗ ) × R de

2y + 2z − λy z = 0, (5.6a)



 2x + 2z − λxz = 0,

(5.6b)
 2x + 2y − λx y = 0,
 (5.6c)


x y z − 8 = 0. (5.6d)

On obtient 4 que le seul point critique est (2, 2, 2). Il s’agit bien d’un point critique mais pour établir s’il est un extremum
il faut étudier le signe de la différence
¡ ¢
d (h 1 , h 2 , h 3 ) = f (2 + h 1 , 2 + h 2 , 2 + h 3 ) − f (2, 2, 2) = 2 (2 + h 1 )(2 + h 2 ) + (2 + h 1 )(2 + h 3 ) + (2 + h 2 )(2 + h 3 ) − 24,

h 1 , h 2 et h 3 étant liés par la relation 0 = g (2 + h 1 , 2 + h 2 , 2 + h 3 ) − 8, soit encore


8
(2 + h 3 ) = .
(2 + h 1 )(2 + h 2 )
On a par conséquente
2 ¡ 2
d˜(h 1 , h 2 ) = d h 1 , h 2 , h 3 (h 1 , h 2 ) = 4h 2 + 4h 1 h 2 + 4h 1 h 22 + 4h 12 + 4h 12 h 2 + h 12 h 22
¡ ¢ ¢
(2 + h 1 )(2 + h 2 )

pour tout h 1 , h 2 ' 0. Le signe de d˜ est le même que celui de N (h 1 , h 2 ) = 4h 22 + 4h 1 h 2 + 4h 1 h 22 + 4h 12 + 4h 12 h 2 + h 12 h 22


mais celui ci est difficile à étudier. Cependant, il est simple de vérifier que ∇N (0, 0) = (0, 0) et que la matrice Hessienne
H N (h 1 , h 2 ) est semi-définie positive, par conséquent N est convexe et donc N (h 1 , h 2 ) ≥ 0 pour h 1 , h 2 ' 0. On trouve
alors d˜(h 1 , h 2 ) ≥ 0 et on conclut que (2, 2, 2) est un minimum de la fonction f sous la contrainte g .
4. Avec les trois soustractions (5.6b)-(5.6a), (5.6c)-(5.6a) et (5.6c)-(5.6b) on obtient x = y = z, on insère ce résultat dans (5.6d) et on trouve la solution.

162 © G. Faccanoni
Dernière mise à jour : Jeudi 5 juin 2014 5 Extrema

8
2. Méthode de réduction : la contrainte se réécrit z = xy donc il s’agit de chercher les extrema de la fonction de deux
variables h : (R∗+ )2 → R définie par
+
µ ¶ µ ¶
8 8 8
h(x, y) = f x, y, = 2 xy + + .
xy y x
Cherchons d’abord les points critiques :  ³ ´
2 y − x82
∇h(x, y) =  ³ ´
2 x − y82

et ∇h(x, y) = (0, 0) ssi (x, y) = (2, 2). Étudions maintenant ce point en calculant le déterminant de la matrice hessienne
de la fonction h :
32
h xx (x, y) = h xx (2, 2) = 4 > 0,
x3
h x y (x, y) = 2 h x y (2, 2) = 2,
32
h y y (x, y) = 3 h y y (2, 2) = 4,
y
D(2, 2) = ∂xx h(2, 2)∂ y y h(2, 2) − (∂x y h(2, 2))2 = 12 > 0.

On a donc que (2, 2) est bien un minimum.

Exercice 5.21
Soit un cylindre de rayon r > 0 et hauteur h > 0.
1. Maximiser le volume du solide pour une surface fixée égale à sπ, s étant une constante positive.
2. Minimiser la surface du solide pour un volume fixé égale à vπ, v étant une constante positive.

C ORRECTION . La surface et le volume du cylindre en fonction de h et r sont donnés respectivement par

S(r, h) = π(2r 2 + 2hr ), V (r, h) = πr 2 h.

1. Maximiser le volume du solide pour une surface fixée égale à sπ.


1.1. Méthode des multiplicateurs de L AGRANGE : on introduit la fonction lagrangienne

L (r, h, µ) = V (r, h) − µ (sπ − S(r, h)) = π r 2 h − µ(2r 2 + 2hr − s) .


¡ ¢

On cherche les points critiques de la fonction lagrangienne :


 q p
s 6s
r = = 6 ,
π ¡2r h − 2µ(2r
 ¡ ¢
+ h) = 0,

 6
p
 
 q
∇L = 0 ⇐⇒ π r 2 − 2µr = 0, ⇐⇒ h = 2 6s = 36s ,
¢
  p
π(s − 2r 2 − 2hr ) = 0,
  q
µ = 1 s = 6s .

2 6 12
³p p p ´
6s 6s 6s
Notons (r 0 , h 0 , µ0 ) ≡ 6 , 3 , 12 et étudions la nature de ce point critique :

∆(r, h, µ) = (∂r r L )(∂hh L ) − (∂r h L )2 = −(∂r h L )2 ≤ 0, ∀(r, h, µ)

donc on ne peut pas conclure directement. Étudions alors le signe de la différence d (ξ1 , ξ2 ) = V (r 0 + ξ1 , h 0 + ξ2 ) −
V (r 0 , h 0 ), ξ1 et ξ2 étant liés par la relation sπ = S(r 0p+ξ1 , h 0 +ξ2 ), soit encore ξ2 = −h 0 −(r 0 +ξ1 )+ s/(2(r 0 +ξ1 )). On
p
a par conséquente d (ξ1 , ξ2 (ξ1 )) = −πξ21 (ξ1 + 3 s/ 6) < 0 lorsque ξ1 ' 0 : le point (r 0 , h 0 ) est alors un maximum
local de V sous la contrainte S = sπ.
1.2. Méthode de réduction : on élimine h de la contrainte :
s
π(2r 2 + 2hr ) = sπ ⇐⇒ h = − r.
2r
Il s’agit alors de minimiser la fonction
³s ´
g (r ) ≡ V (r, h(r )) = π h(r )r 2 = π r − r 3 .
¡ ¢
2
On a p
³s ´ 6s
0 2
g (r ) = π − 3r = 0 ⇐⇒ r =
2 6
p p
6s 6s
et g 00 (r ) = −6πr < 0 donc r = 6 est un maximum et h(r 0 ) = 3 .

© G. Faccanoni 163
5 Extrema Dernière mise à jour : Jeudi 5 juin 2014

2. Minimiser la surface du solide pour un volume fixé égale à vπ


2.1. Méthode des multiplicateurs de L AGRANGE : on introduit la fonction lagrangienne

L (r, h, µ) = S(r, h) − µ (vπ − V (r, h)) = π (2r 2 + 2hr ) − µ(v − hr 2 ) .


¡ ¢

On cherche les points critique de la fonction lagrangienne :


 q
3 v
r = ,
2π ¡2r + h ¢+ µhr = 0,
 ¡ ¢ 
q2
 


3 v
∇L = 0 ⇐⇒ πr 2 + µr = 0, ⇐⇒ h = 2 2 ,
 
πhr 2 = vπ,
  q
µ = −2 3 2 .

v

On étudie la nature du point critique trouvé :

∆(r, h, µ) = (∂r r L )(∂hh L ) − (∂r h L )2 = −π2 (2 + 2µr )2 =⇒ ∆(r 0 , h 0 , µ0 ) < 0,

on ne peut pas conclure directement. Étudions alors le signe de la différence d (ξ1 , ξ2 ) = S(r 0 +ξ1 , h 0 +ξ2 )−S(r 0 , h 0 ),
ξ1 et ξ2 étant liés par la relation vπ = V (r 0 +ξ1 , h 0 +ξ2 ), soit encore ξ2 = −h 0 + v/(r 0 +ξ1 )2 . Sachant que v = h 0 r 02 =
2r 03 et que h 0 r 0 = 2r 02 , on trouve d (ξ1 , ξ2 (ξ1 )) = 2πξ21 (3r 0 + ξ1 )/(r 0 + ξ1 ) > 0 lorsque ξ1 ' 0 : le point (r 0 , h 0 ) est alors
un minimum local de S sous la contrainte V = vπ.
2.2. Méthode de réduction : on élimine h de la contrainte :
v
πhr 2 = vπ ⇐⇒ h = .
r2
Il s’agit alors de minimiser la fonction
³ v´
g (r ) = S(r, h(r )) = π(2r 2 + 2h(r )r ) = 2π r 2 + .
r
On a r
0
³ v ´ v
g (r ) = 2π 2r − 2 = 0 ⇐⇒ r = 3
r 2
³ ´ q ³q ´ q
v v v v
et g 00 (r ) = 2π 2 + r 3 > 0 donc r = 3 2 est un minimum et h 3 2 = 2 3 2 .

164 © G. Faccanoni
Soit le cylindre de rayon r > 0 et hauteur h > 0.

V : (R∗+ )2 → R S : (R∗+ )2 → R

© G. Faccanoni
h (r, h) 7→ πr 2 h (r, h) 7→ 2π(r 2 + r h)
Dernière mise à jour : Jeudi 5 juin 2014

Maximiser V sous la contrainte S(r, h) = sπ. Minimiser S sous la contrainte V (r, h) = vπ.
Lagrangien : L : (R∗+ )2 × R → R défini par L (r, h, λ) = V (r, h) − λ(vπ − S(r, h)) Lagrangien : L : (R∗+ )2 × R → R défini par L (r, h, λ) = S(r, h) − λ(vπ − V (r, h))
Pts critiques
p du lagrangien : ∇L (r, h, λ) = (0, 0, 0) ssi (r, h, λ) = (κ, 2κ, κ/2) où Pts critiques
p du lagrangien : ∇L (r, h, λ) = (0, 0, 0) ssi (r, h, λ) = (κ, 2κ, −2κ) où
3
κ= s/6 κ= v/2
Étude des pts critiques du lagrangien : d (ξ1 , ξ2 ) = V (κ + ξ1 , 2κ + ξ2 ) − Étude des pts critiques du lagrangien : d (ξ1 , ξ2 ) = S(κ + ξ1 , 2κ + ξ2 ) −
V (κ, 2κ) < 0 au voisinage de (0, 0), ξ1 et ξ2 étant liés par la relation S(κ, 2κ) > 0 au voisinage de (0, 0), ξ1 et ξ2 étant liés par la relation
S(κ + ξ1 , 2κ + ξ2 ) = vπ V (κ + ξ1 , 2κ + ξ2 ) = vπ
Conclusion : (κ, 2κ) maximise V sous la contrainte S = sπ. Conclusion : (κ, 2κ) minimise S sous la contrainte V = vπ.

Contrainte : S(r, h) = sπ ssi h = s/(2r ) − r Contrainte : V (r, h) = vπ ssi h = v/r 2


Fonction à maximiser : g : R∗+ × R → R définip par g (r ) = V (r, h(r )) Fonction à minimiser : g : R∗+ × R → R défini p par g (r ) = S(r, h(r ))
Pts critiques : g 0 (r ) = 0 ssi r = κ où κ = s/6 Pts critiques : g 0 (r ) = 0 ssi r = κ où κ = 3 v/2
Étude des pts critiques : g 00 (κ) < 0 Étude des pts critiques : g 00 (κ) > 0
Conclusion : r = κ maximise g et h(κ) = 2κ donc (κ, 2κ) maximise V sous la Conclusion : r = κ minimise g et h(κ) = 2κ donc (κ, 2κ) minimise S sous la
contrainte S = sπ. contrainte V = vπ.

Méthode de réduction Méthode de L AGRANGE

165
5 Extrema
5 Extrema Dernière mise à jour : Jeudi 5 juin 2014

Exercice 5.22
Soit le solide constitué d’un cylindre de rayon r > 0 et
hauteur h > 0 limité par deux demi-sphères de rayon
r
r > 0 comme dans la figure ci-contre.
1. Maximiser le volume du solide pour une surface
fixée égale à 16π.
h
2. Minimiser la surface du solide pour un volume fixé
égale à 32
3 π.

Rappel : un cylindre de rayon de base R et de hauteur H a


volume πR 2 H et surface totale 2(πR 2 )+2πR H , une sphère
de rayon R a volume 43 πR 3 et surface 4πR 2 .

C ORRECTION . La surface du solide est la somme de la surface latérale du cylindre 2πr h et de la surface des deux demi-
sphères 4πr 2 :
S(r, h) = π(4r 2 + 2hr ).
Le volume du solide est la somme du volume du cylindre πr 2 h et du volume des deux demi-sphères 43 πr 3 :
µ ¶
4
V (r, h) = π hr 2 + r 3 .
3

1. Maximiser le volume du solide pour une surface fixée égale à 16π.


1.1. Méthode des multiplicateurs de L AGRANGE : on introduit la fonction lagrangienne
µ ¶
4
L (r, h, µ) = V (r, h) − µ (16π − S(r, h)) = π hr 2 + r 3 − µ 16π − π(4r 2 + 2hr ) .
¡ ¢
3

On cherche les points critiques du lagrangien :

2 2
π(2hr + 4r ) − µ (−π(8r + 2h)) = 0, 2hr + 4r + µ(2h + 8r ) = 0,
  
  r = 2,

∇L = 0 ⇐⇒ π(r 2 ) − µ (−π2r ) = 0, ⇐⇒ r 2 + µ2r = 0, ⇐⇒ h = 0,
  
π(4r 2 + 2hr ) = 16π, 4r 2 + 2hr = 16, µ = −1.
  

Comme
∆(r, h, µ) ≡ (∂r r L )(∂hh L ) − (∂r h L )2 = −4π2 (r + µ)2 , ∆(2, 0, −1) < 0,
on ne peut pas établir la nature du point critique. Étudions alors le signe de la différence d (ξ1 , ξ2 ) = V (2 + ξ1 , ξ2 ) −
V (2, 0), ξ1 et ξ2 étant liés par la
¡ relation 0 = S(2 + ξ1 , ξ2 ) −¢ 16π, soit encore ξ2 = (8 − 2(2 + ξ1 )2 )/(2 + ξ1 ). On a par
2
conséquente d (ξ1 , ξ2 (ξ1 )) = π − 3 (2 + ξ1 )3 + 8(2 + ξ1 ) − 32
3 < 0 lorsque ξ1 ' 0 (étudier la cubique d’équation y =
−x 3 + 24x − 16 pour x ' 2) : le point (2, 0) est alors un maximum local de V sous la contrainte S = 16π.
1.2. Méthode de réduction : on élimine h de la contrainte :
8
π(4r 2 + 2hr ) = 16π ⇐⇒ h = − 2r.
r
Il s’agit alors de maximiser la fonction
µ ¶
4 2 ¡
g (r ) = V (r, h(r )) = π hr 2 + r 3 = π 12r − r 3
¢
3 3

On a
2 ¡
g 0 (r ) = π 12 − 3r 2 = 0 ⇐⇒ r = 2
¢
3
et g 00 (r ) = 23 π (−6r ) < 0, donc r = 2 est un maximum et h(2) = 0.
Le solide qui maximise le volume pour une surface donnée est donc une sphère !
32
2. Minimiser la surface du solide pour un volume fixé égale à 3 π.

166 © G. Faccanoni
Dernière mise à jour : Jeudi 5 juin 2014 5 Extrema

2.1. Méthode des multiplicateurs de L AGRANGE : on introduit la fonction lagrangienne


µ ¶ · µ µ ¶¶¸
32 32 4
L (r, h, µ) = S(r, h) − µ π − V (r, h) = π (4r 2 + 2hr ) − µ − hr 2 + r 3 .
3 3 3

On cherche les points critiques de la fonction lagrangienne :

2 2
π £(8r + 2h)¡ − µ ¢¤−(2hr + 4r ) = 0, 2h + 8r + µ(2hr + 4r ) = 0,
 £ ¡ ¢¤  
  r = 2,

∇L = 0 ⇐⇒ π (2r ) − µ −r 2 = 0, ⇐⇒ 2r + µr 2 = 0, ⇐⇒ h = 0,

 ¡ 2 4 3 ¢ 32 
 2 4 3 32 
π hr + 3 r = 3 π, hr + 3 r = 3 , µ = −1.

On étudie la nature du point critique trouvé :

∆(r, h, µ) = (∂r r L )(∂hh L ) − (∂r h L )2 = −4π2 (1 + µr )2 =⇒ ∆(2, 0, −1) < 0,

on ne peut pas conclure directement. Étudions alors le signe de la différence d (ξ1 , ξ2 ) = S(2 + ξ1 , ξ2 ) − S(2, 0),
ξ1 et ξ2 étant liés par la relation 0 =¡ V (2 + ξ1 , ξ2 ) − 32π/3, soit¢ encore ξ2 = 4(8 − (2 + ξ1 )3 )/(3(2 + ξ1 )2 ). On a par

conséquente d (ξ1 , ξ2 (ξ1 )) = 3(2+ξ )2
3(2 + ξ1 )4 − 2(2 + ξ1 )3 + 16 > 0 lorsque ξ1 ' 0 (étudier la fonction d’équation
1
y = 3x 4 − 2x 3 + 16 pour x ' 2) : le point (2, 0) est alors un minimum local de S sous la contrainte V = 32π/3.
2.2. Méthode de réduction : on élimine h de la contrainte :
µ ¶ µ ¶
4 32 4 8
π hr 2 + r 3 = π ⇐⇒ h = − r .
3 3 3 r2

Il s’agit alors de maximiser la fonction


µ ¶
4 16
g (r ) = S(r, h(r )) = π(4r 2 + 2hr ) = π +r2
3 r

On a µ ¶
0 8 8
g (r ) = π r − 2 = 0 ⇐⇒ r = 2
3 r
et g 00 (r ) = 38 π 1 + 16/r 3 > 0 donc r = 2 est un minimum et h(2) = 0.
¡ ¢

Le solide qui minimise la surface pour un volume donné est donc une sphère !

© G. Faccanoni 167
Soit le solide constitué d’un cylindre de rayon r > 0 et hauteur h > 0 limité par deux demi-sphères de rayon r > 0

168
5 Extrema

V : (R∗+ )2 → R S : (R∗+ )2 → R
r µ ¶
4
(r, h) 7→ π r 2 h + r 3 (r, h) 7→ π(4r 2 + 2r h)
3
h

Maximiser V sous la contrainte S(r, h) = 16π. Minimiser S sous la contrainte V (r, h) = 32π/3.

Lagrangien : L : (R∗+ )2 × R → R défini par L (r, h, λ) = V (r, h) − λ(12π − S(r, h)) Lagrangien : L : (R∗+ )2 × R → R défini par L (r, h, λ) = S(r, h) − λ(32π/3 −V (r, h))
Pts critiques du lagrangien : ∇L (r, h, λ) = (0, 0, 0) ssi (r, h, λ) = (2, 0, −1) Pts critiques du lagrangien : ∇L (r, h, λ) = (0, 0, 0) ssi (r, h, λ) = (2, 0, −1)
Étude des pts critiques du lagrangien : d (ξ1 , ξ2 ) = V (2+ξ1 , ξ2 )−V (2, 0) < 0 au Étude des pts critiques du lagrangien : d (ξ1 , ξ2 ) = S(2 + ξ1 , ξ2 ) − S(2, 0) > 0 au
voisinage de (0, 0), ξ1 et ξ2 étant liés par la relation S(2 + ξ1 , ξ2 ) = 16π voisinage de (0, 0), ξ1 et ξ2 étant liés par la relation V (2 + ξ1 , ξ2 ) = 32π/3
Conclusion : (2, 0) maximise V sous la contrainte S = 12π. Conclusion : (2, 0) minimise S sous la contrainte V = 32π/3.

Contrainte : S(r, h) = 12π ssi h = 8/r − 2r Contrainte : V (r, h) = 32π/3 ssi h = 4(8/r 2 − r )/3
Fonction à maximiser : g : R∗+ × R → R défini par g (r ) = V (r, h(r )) Fonction à minimiser : g : R∗+ × R → R défini par g (r ) = S(r, h(r ))
Pts critiques : g 0 (r ) = 0 ssi r = 2 Pts critiques : g 0 (r ) = 0 ssi r = 2
Étude des pts critiques : g 00 (2) < 0 Étude des pts critiques : g 00 (2) > 0
Conclusion : 2 maximise g et h(2) = 0 donc (2, 0) maximise V sous la contrainte Conclusion : 2 minimise g et h(2) = 0 donc (2, 0) minimise S sous la contrainte
S = 12π. V = 32π/3.

Méthode de réduction Méthode de L AGRANGE

© G. Faccanoni
Dernière mise à jour : Jeudi 5 juin 2014
Dernière mise à jour : Jeudi 5 juin 2014 5 Extrema

Exercice 5.23

Soit le solide constitué d’un cylindre de rayon r > 0 et hau-


teur h > 0 surmonté par une demi-sphère de rayon r > 0
comme dans la figure ci-contre.
1. Maximiser le volume du solide pour une surface fixée r
égale à 5π.
2. Minimiser la surface du solide pour un volume fixé
égale à 35 π h

Rappel : un cylindre de rayon de base R et de hauteur H a


volume πR 2 H et surface totale 2(πR 2 ) + 2πR H , une sphère
de rayon R a volume 43 πR 3 et surface 4πR 2 .

C ORRECTION . La surface du solide est la somme de la surface du disque de base πr 2 avec la surface latérale du cylindre
2πr h et la surface de la demi-sphère 2πr 2 :
S(r, h) = π(3r 2 + 2hr ).
Le volume du solide est la somme du volume du cylindre πr 2 h et du volume de la demi-sphère 23 πr 3 :
µ ¶
2
V (r, h) = π hr 2 + r 3 .
3

1. Maximiser le volume du solide pour une surface fixée égale à 5π.


1.1. Méthode des multiplicateurs de L AGRANGE : on introduit la fonction lagrangienne
µ ¶
2 3
L (r, h, µ) = V (r, h) − µ (5π − S(r, h)) = π hr + r − µ 5π − π(3r 2 + 2hr ) .
2
¡ ¢
3

On cherche les points critiques de la fonction lagrangienne :

2 2
π(2hr + 2r ) − µ (−π(6r + 2h)) = 0, 2hr + 2r + µ(6r + 2h) = 0,
 
  (
r = h = 1,
∇L = 0 ⇐⇒ πr 2 − µ (−π(2r )) = 0, ⇐⇒ r 2 + µ(2r ) = 0, ⇐⇒
  µ = − 12 .
π(3r 2 + 2hr ) = 5π,
 2
3r + 2hr = 5,

On étudie la nature du point critique trouvé :

∆(r, h, µ) = (∂r r L )(∂hh L ) − (∂r h L )2 = −4π2 (r + µ)2 =⇒ ∆(1, 1, −1/2) < 0,

on ne peut pas conclure directement. Étudions alors le signe de la différence d (ξ1 , ξ2 ) = V (1 + ξ1 , 1 + ξ2 ) − V (1, 1),
ξ1 et ξ2 étant liés par la relation 5π = S(1+ξ1 , 1+ξ2 ), soit encore ξ2 = −ξ1 (8+3ξ1 )/(2(1+ξ1 )). On a par conséquente
d (ξ1 , ξ2 (ξ1 )) = −5πξ21 (3 + ξ1 ) < 0 lorsque ξ1 ' 0 : le point (1, 1) est alors un maximum local de V sous la contrainte
S = 5π.
1.2. Méthode de réduction : on élimine h de la contrainte :
5 3
π(3r 2 + 2hr ) = 5π ⇐⇒ h = − r.
2r 2
Il s’agit alors de minimiser la fonction
µ ¶ µ ¶ µ ¶
2 5 3 2 5 5
g (r ) = V (r, h(r )) = π h(r )r 2 + r 3 = π r − r 3 + r 3 = π r − r 3 .
3 2 2 3 2 6

On a µ ¶
5 5 2
g 0 (r ) = π − r = 0 ⇐⇒ r = 1
2 2
et g 00 (r ) = π (−5r ) < 0 donc r = 1 est un maximum et h(1) = 1.

2. Minimiser la surface du solide pour un volume fixé égale à 35 π

© G. Faccanoni 169
5 Extrema Dernière mise à jour : Jeudi 5 juin 2014

2.1. Méthode des multiplicateurs de L AGRANGE : on introduit la fonction lagrangienne


µ ¶ µ µ ¶¶
5 5 2
L (r, h, µ) = S(r, h) − µ π − V (r, h) = π(3r 2 + 2hr ) − µ π − π hr 2 + r 3 .
3 3 3

On cherche les points critique de la fonction lagrangienne :

π(6r + 2h)¡ − µ −π(2hr + 2r 2 ) = 0, 2


6r + 2h + µ(2hr + 2r ) = 0,
 ¡ ¢ 
  (
r = h = 1,
∇L = 0 ⇐⇒ π(2r ) − µ −πr 2 = 0, ⇐⇒ 2r + µr 2 = 0,
¢
⇐⇒

 ¡ 2 2 3¢ 5 
¡ 2 2 3 ¢ 5 µ = −2.
π hr + 3 r = 3 π, hr + 3 r = 3 ,

On étudie la nature du point critique trouvé :

∆(r, h, µ) = (∂r r L )(∂hh L ) − (∂r h L )2 = −4π2 (1 + µr )2 =⇒ ∆(1, 1, −2) < 0,

on ne peut pas conclure directement. Étudions alors le signe de la différence d (ξ1 , ξ2 ) = S(1 + ξ1 , 1 + ξ2 ) − S(1, 1),
ξ1 et ξ2 étant liés par la relation 5π = 3V (1 + ξ1 , 1 + ξ2 ), soit encore ξ2 = −(−10 + 12ξ1 + 9ξ21 + 2ξ3 )/(3(1 + ξ1 )2 ). On a
par conséquente d (ξ1 , ξ2 (ξ1 )) = 5π(4 + 3ξ21 + ξ31 )/(3(1 + ξ1 )) > 0 lorsque ξ1 ' 0 : le point (1, 1) est alors un minimum
local de S sous la contrainte V = 5π/3.
2.2. Méthode de réduction : on élimine h de la contrainte :
µ ¶
2 5 5 2
π hr 2 + r 3 = π ⇐⇒ h = 2 − r.
3 3 3r 3

Il s’agit alors de minimiser la fonction


µ ¶ µ ¶
10 4 2 10 5 2
g (r ) = S(r, h(r )) = π(3r 2 + 2h(r )r ) = π 3r 2 + − r =π + r .
3r 3 3r 3

On a µ ¶
10 10
g 0 (r ) = π − 2 + r = 0 ⇐⇒ r = 1
3r 3
³ ´
20
et g 00 (r ) = π 3r 3
+ 10
3 > 0 donc r = 1 est un minimum et h(1) = 1.

170 © G. Faccanoni
Soit le solide constitué d’un cylindre de rayon r > 0 et hauteur h > 0 surmonté par une demi-sphère de rayon r > 0

V : (R∗+ )2 → R S : (R∗+ )2 → R
µ ¶

© G. Faccanoni
r 2
(r, h) 7→ π r 2 h + r 3 (r, h) 7→ π(3r 2 + 2r h)
3

h
Dernière mise à jour : Jeudi 5 juin 2014

Maximiser V sous la contrainte S(r, h) = 5π. Minimiser S sous la contrainte V (r, h) = 5π/3.

Lagrangien : L : (R∗+ )2 × R → R défini par L (r, h, λ) = V (r, h) − λ(5π − S(r, h)) Lagrangien : L : (R∗+ )2 × R → R défini par L (r, h, λ) = S(r, h) − λ(5π/3 − V (r, h))
Pts critiques du lagrangien : ∇L (r, h, λ) = (0, 0, 0) ssi (r, h, λ) = (2, 1, −1/2) Pts critiques du lagrangien : ∇L (r, h, λ) = (0, 0, 0) ssi (r, h, λ) = (1, 1, −2)
Étude des pts critiques du lagrangien : d (ξ1 , ξ2 ) = V (1+ξ1 , 1+ξ2 )−V (1, 1) < 0 Étude des pts critiques du lagrangien : d (ξ1 , ξ2 ) = S(1 + ξ1 , 1 + ξ2 ) − S(1, 1) > 0
au voisinage de (0, 0), ξ1 et ξ2 étant liés par la relation S(1 + ξ1 , 1 + ξ2 ) = 5π au voisinage de (0, 0), ξ1 et ξ2 étant liés par la relation V (1+ξ1 , 1+ξ2 ) = 5π/3
Conclusion : (1, 1) maximise V sous la contrainte S = 5π. Conclusion : (1, 1) minimise S sous la contrainte V = 5π/3.

Contrainte : S(r, h) = 5π ssi h = 5/(2r ) − 3r /2 Contrainte : V (r, h) = 5π/3 ssi h = (5/r 2 − 2r )/3
Fonction à maximiser : g : R∗+ × R → R défini par g (r ) = V (r, h(r )) Fonction à minimiser : g : R∗+ × R → R défini par g (r ) = S(r, h(r ))
Pts critiques : g 0 (r ) = 0 ssi r = 1 Pts critiques : g 0 (r ) = 0 ssi r = 1
Étude des pts critiques : g 00 (1) < 0 Étude des pts critiques : g 00 (1) > 0
Conclusion : 1 maximise g et h(1) = 1 donc (1, 1) maximise V sous la contrainte Conclusion : 1 minimise g et h(1) = 1 donc (1, 1) minimise S sous la contrainte
S = 5π. V = 5π/3.

Méthode de réduction Méthode de L AGRANGE

171
5 Extrema
5 Extrema Dernière mise à jour : Jeudi 5 juin 2014

Exercice 5.24
Trouver le cylindre de volume maximal inscrit dans une sphère de rayon R > 0 fixé. . .
y
1. . . . avec la méthode des multiplicateurs de L AGRANGE (i.e. maximisation d’une R

x
fonction f (x, y) sous une contrainte g (x, y) = 0) [NB : on se contentera de
trouver le point critique] ;
2. . . . avec la méthode des extrema libres en éliminant une variable de la
contrainte (par exemple en maximisant une fonction h(x) = f (x, y(x))) [NB :
après avoir trouvé le point critique, établir sa nature].
Suggestion : voir la figure ci-contre pour déduire la contrainte.

C ORRECTION . Notons y > 0 le rayon de base du cylindre et x > 0 sa demi-hauteur.


1. Avec la méthode des multiplicateurs de L AGRANGE il s’agit de maximiser la fonction f (x, y) = 2πx y 2 sous la contrainte
g (x, y) = R 2 − x 2 − y 2 = 0. On écrit la fonction de L AGRANGE

F (x, y, λ) = f (x, y) − λg (x, y) = 2πx y 2 − λ(R 2 − x 2 − y 2 )

et on cherche les points critiques de F :


2πy 2 + 2λx
 
~
∇F (x, y, λ) =  4πx y + 2λy  .
x2 + y 2 − R2
³ q ´
On a ~
∇F (x, y, λ) =~0 ssi (x, y, λ) = p1 R, 23 R, π p2 R . On étudie la nature du point critique trouvé :
3 3
µ q ¶
∆(x, y, λ) = (∂xx F )(∂ y y F ) − (∂x y F )2 = 4λ(2πx + λ) − 16π2 y 2 =⇒ ∆ p1 R, 2 p2
3 R, π 3 R = 0,
3

³ q ´
on ne peut pas conclure directement. Étudions alors le signe de la différence d (ξ1 , ξ2 ) = f p1 R + ξ1 , 23 R + ξ2 −
3
³ q ´ ³q ´2
1 2 2 1 2 2
f p R, 3 R , ξ1 et ξ2 étant liés par la relation R = ( p R + ξ1 ) + 3 R + ξ2 . On a par conséquente d (ξ1 , ξ2 (ξ1 )) =
3 q ´ 3
p ³
− 3Rξ21 − ξ31 < 0 lorsque ξ1 ' 0 : le point p1 R, 23 R est alors un maximum local de f sous la contrainte g = 0.
3
p
2. Avec la méthode des extrema libres il s’agit d’éliminer une variable de la contrainte, par exemple en posant y = R 2 − x 2
et on maximise la fonction h(x) = f (x, y(x)) :
³ p ´
h(x) = f x, y(x) = R 2 − x 2 = 2πx(R 2 − x 2 ).

On cherche d’abord les points critiques :


h 0 (x) = 2π(R 2 − 3x 2 )
R
et h 0 (x) = 0 ssi x = p . On étudie la nature du point critique en étudiant la dérivée seconde :
3
³ ´
R
h 00 (x) = −12πx, h 00 p <0
3

R
donc x = p est un maximum.
3

Exercice 5.25
Trouver trois nombres réels positifs dont le produit vaut 1728 et dont la somme est minimale :
1. avec la méthode des multiplicateurs de L AGRANGE (i.e. minimisation d’une fonction f (x, y, z) sous une contrainte
g (x, y, z) = 0) ;
2. avec la méthode des extrema libres en «éliminant» une variable de la contrainte (en minimisant une fonction
h(x, y) = f (x, y, z(x, y))) ; après avoir trouvé le point critique, établir sa nature.

172 © G. Faccanoni
Dernière mise à jour : Jeudi 5 juin 2014 5 Extrema

C ORRECTION . Il s’agit de minimiser la fonction f (x, y, z) = x + y + z sous la contrainte g (x, y, z) = x y z − 1728 = 0.


1. On écrit la fonction de L AGRANGE

F (x, y, z, λ) = f (x, y, z) − λg (x, y, z) = x + y + z − λ(x y z − 1728)

et on cherche le(s) point(s) critique(s) de F :

1 − λy z
 
 1 − λxz 
~
∇F (x, y, z, λ) = 
 1 − λx y  .

1728 − x y z

On a ~ ∇F (x, y, z, λ) =~0 ssi (x, y, z, λ) = (12, 12, 12, 1/144).


Il s’agit bien d’un point critique mais pour établir s’il est un extremum il faut étudier le signe de la différence

d (h 1 , h 2 , h 3 ) = f (12 + h 1 , 12 + h 2 , 12 + h 3 ) − f (12, 12, 12) = h 1 + h 2 + h 3 ,

h 1 , h 2 et h 3 étant liés par la relation 0 = g (12 + h 1 , 12 + h 2 , 12 + h 3 ) − 1728, soit encore

12(12h 1 + 12h 2 + h 1 h 2 )
h3 = .
(12 + h 1 )(12 + h 2 )

On a par conséquente

¢ 12h 1 h 2 + 12h 22 + h 1 h 22 + 12h 12 + h 12 h 2


d˜(h 1 , h 2 ) = d h 1 , h 2 , h 3 (h 1 , h 2 ) =
¡
(12 + h 1 )(12 + h 2 )

pour tout h 1 , h 2 ' 0. Le signe de d˜ est le signe de N (h 1 , h 2 ) = 12h 1 h 2 + 12h 22 + h 1 h 22 + 12h 12 + h 12 h 2 mais celui ci est
difficile à étudier. Cependant, il est simple de vérifier que ∇N (0, 0) = (0, 0) et que la matrice Hessienne H N (h 1 , h 2 ) est
semi-définie positive, par conséquent N est convexe et donc N (h 1 , h 2 ) ≥ 0 pour h 1 , h 2 ' 0. On trouve alors d˜(h 1 , h 2 ) ≥ 0
et on conclut que (12, 12, 12) est un minimum de la fonction f sous la contrainte g .
1728
2. On réécrit la contrainte sous la forme z = xy et on l’injecte dans la fonction à minimiser :

1728
h(x, y) = f (x, y, z(x, y)) = x + y + z(x, y) = x + y − .
xy

On cherche d’abord le(s) point(s) critique(s) :

1 − 1728
à !
~
∇h(x, y) = x2 y
1 − 1728
x y2

et ~
∇h(x, y) = ~0 ssi (x, y) = (12, 12). On établie la nature du point critique en étudiant le déterminant de la matrice
hessienne :
3456 3456 1728
∂xx h(x, y) = , ∂ y y h(x, y) = , ∂x y h(x, y) = ,
x3 y x y3 x2 y 2
1 1 1
∂xx h(12, 12) = > 0, ∂ y y h(12, 12) = , ∂x y h(12, 12) =
6 6 12
1
et ∂xx h(12, 12)∂ y y h(12, 12) − (∂x y h(12, 12))2 = 48 > 0 donc (12, 12) est un minimum.

Exercice 5.26
Trouver le parallélépipède de volume 1 dont la somme des longueurs des arrêtes est minimale.

© G. Faccanoni 173
5 Extrema Dernière mise à jour : Jeudi 5 juin 2014

C ORRECTION .
1. Avec la méthode des multiplicateurs de L AGRANGE il s’agit de minimiser la fonction f (x, y, z) = 4x + 4y + 4z sous la
contrainte g (x, y, z) = x y z − 1 = 0. On écrit la fonction de L AGRANGE

F (x, y, z, λ) = f (x, y, z) − λg (x, y, z) = 4(x + y + z) − λ(x y z − 1)

et on cherche le(s) point(s) critique(s) de F :

4 − λy z
 
 4 − λxz 
~
∇F (x, y, z, λ) = 
4 − λx y  .

1− xyz

On a ~
∇F (x, y, z, λ) = ~0 ssi (x, y, z, λ) = (1, 1, 1, 4). Pour établir s’il est un extremum et de quelle nature, il faut étudier le
signe de la différence
d (h 1 , h 2 , h 3 ) = f (1 + h 1 , 1 + h 2 , 1 + h 3 ) − f (1, 1, 1) = 4(h 1 + h 2 + h 3 ),
h 1 , h 2 et h 3 étant liés par la relation 0 = g (1 + h 1 , 1 + h 2 , 1 + h 3 ) − 1, soit encore

h1 + h2 + h1 h2
h3 = − .
(1 + h 1 )(1 + h 2 )

On a par conséquente

h 1 h 2 + h 22 + h 1 h 22 + h 12 + h 12 h 2
d˜(h 1 , h 2 ) = d h 1 , h 2 , h 3 (h 1 , h 2 ) = 4
¡ ¢
(1 + h 1 )(1 + h 2 )

pour tout h 1 , h 2 ' 0. Le signe de d˜ est le signe de N (h 1 , h 2 ) = h 1 h 2 + h 22 + h 1 h 22 + h 12 + h 12 h 2 mais celui ci est difficile à
étudier. Cependant, il est simple de vérifier que ∇N (0, 0) = (0, 0) et que la matrice Hessienne H N (h 1 , h 2 ) est semi-définie
positive, par conséquent N est convexe et donc N (h 1 , h 2 ) ≥ 0 pour h 1 , h 2 ' 0. On trouve alors d˜(h 1 , h 2 ) ≥ 0 et on conclut
que (1, 1, 1) est un minimum de la fonction f sous la contrainte g .
1
2. Avec la méthode des extrema libres il s’agit d’éliminer une variable de la contrainte, par exemple en posant z = xy , et
de minimiser ensuite la fonction h(x, y) = f (x, y, z(x, y)) :
µ ¶ µ ¶
1 1
h(x, y) = f x, y, =4 x+y+ .
xy xy

On cherche d’abord le(s) point(s) critique(s) :

4 − x 42 y
à !
~
∇h(x, y) =
4 − x 4y 2

et ~
∇h(x, y) = ~0 ssi (x, y) = (1, 1). On établie la nature du point critique en étudiant le déterminant de la matrice hes-
sienne :
8 8 4
∂xx h(x, y) = , ∂ y y h(x, y) = , ∂x y h(x, y) = ,
x3 y x y3 x2 y 2
∂xx h(1, 1) = 8 > 0, ∂ y y h(1, 1) = 8, ∂x y h(1, 1) = 4

et ∂xx h(1, 1)∂ y y h(1, 1) − (∂x y h(1, 1))2 = 48 > 0 donc (1, 1) est un minimum.

Exercice 5.27
Si un courant électrique I traverse un circuit électrique de résistance R, la quantité de chaleur émise en une unité de
temps est proportionnelle à I 2 R. Calculer la décomposition du courant I en trois courants I 1 , I 2 , I 3 à l’aide de trois résis-
tances R 1 , R 2 , R 3 pour que la chaleur émise soit minimale. . .
1. . . . avec la méthode des multiplicateurs de L AGRANGE, i.e. en minimisant la fonction f (I 1 , I 2 , I 3 ) = I 12 R 1 +I 22 R 2 +I 32 R 3
sous la contrainte g (I 1 , I 2 , I 3 ) = I 1 +I 2 +I 3 −I , (on se contentera de trouver le point critique sans en étudier la nature)
2. . . . en réduisant le problème à une minimisation libre par élimination d’une variable dans la contrainte I = I 1 + I 2 +
I3.

174 © G. Faccanoni
Dernière mise à jour : Jeudi 5 juin 2014 5 Extrema

C ORRECTION .
1. On maximise le lagrangien L(I 1 , I 2 , I 3 , λ) = f (I 1 , I 2 , I 3 ) − λg (I 1 , I 2 , I 3 ).
2I 1 R 1 − λ 0
   
 2I 2 R 2 − λ  0 
Points critiques : ∇L(I 1 , I 2 , I 3 , λ) = 
 2I 3 R 3 − λ  donc ∇L(I 1 , I 2 , I 3 , λ) = 0 si et seulement si
  

I − I1 − I2 − I3 0

R2 R3 R1 R3 R1 R2 2R 1 R 2 R 3
µ ¶
(I 1 , I 2 , I 3 , λ) = I, I, I, I .
R1 R2 + R1 R3 + R2 R3 R1 R2 + R1 R3 + R2 R3 R1 R2 + R1 R3 + R2 R3 R1 R2 + R1 R3 + R2 R3
³ ´
R2 R3 R1 R3 R1 R2
Conclusion : (I 1 , I 2 , I 3 ) = R 1 R 2 +R 1 R 3 +R 2 R 3 I , R 1 R 2 +R 1 R 3 +R 2 R 3 I , R 1 R 2 +R 1 R 3 +R 2 R 3 I
est un extremum de f sous la contrainte
g (I 1 , I 2 , I 3 ) = 0.
2. On maximise f˜(I 1 , I 2 ) = f (I 1 , I 2 , I − I 1 − I 2 ) = I 12 R 1 + I 22 R 2 + (I − I 1 − I 2 )2 R 3 .
µ ¶ µ ¶
˜ 2I 1 R 1 − 2(I − I 1 − I 2 )R 3 ˜ 0
Points critiques : ∇ f (I 1 , I 2 ) = et l’on a ∇ f (I 1 , I 2 ) = si et seulement si
2I 2 R 2 − 2(I − I 1 − I 2 )R 3 0

R2 R3 R1 R3
µ ¶
(I 1 , I 2 ) = I, I .
R1 R2 + R1 R3 + R2 R3 R1 R2 + R1 R3 + R2 R3

Classification : ∂I 1 I 1 f˜(I 1 , I 2 ) = 2(R 1 + R 3 ) > 0, ∂I 2 I 2 f˜(I 1 , I 2 ) = 2(R 2 + R 3 ) et ∂I 1 I 2 f˜(I 1 , I 2 ) = 2R 3 , donc ∂I 1 I 1 f˜(I 1 , I 2 ) ·


¢2
∂I 2 I 2 f˜(I 1 , I 2 ) − ∂I 1 I 2 f˜(I 1 , I 2 ) = 4(R 1 + R 3 )(R 2 + R 3 ) − 4R 32 > 0 pour tout (I 1 , I 2 ) ∈ R2 .
¡
³ ´
Conclusion : (I 1 , I 2 ) = R1 R2 +RR21 RR33 +R2 R3 I , R1 R2 +RR11 RR33 +R2 R3 I est un minimum de f˜ donc

R2 R3 R1 R3 R1 R2
µ ¶
(I 1 , I 2 , I 3 ) = I, I, I
R1 R2 + R1 R3 + R2 R3 R1 R2 + R1 R3 + R2 R3 R1 R2 + R1 R3 + R2 R3

est un minimum de f sous la contrainte g (I 1 , I 2 , I 3 ) = 0.

© G. Faccanoni 175
5 Extrema Dernière mise à jour : Jeudi 5 juin 2014

Optimisation dans un fermé : optimisation libre et liée

Exercice 5.28
Déterminer les extrema de la fonction f : R2 → R définie par f (x, y) = 4x 2 + y 2 dans le disque x 2 + y 2 ≤ 4.

C ORRECTION .
1. Étude graphique :
y Dans la figure ci-contre on a tracé les courbes de niveau
0, 1, 4, 9, 16 de f ainsi que le disque. Il semblerait que
16
B le point (0, 0) est un minimum global,
B le point (2, 0) est un maximum global,
9
B le point (−2, 0) est un maximum global,
B le point (0, 2) est un minimum local,
4
B le point (0, −2) est un minimum local.
1

0 x

2. Étude de f dans l’ouvert, i.e. l’ensemble (x, y) ∈ R2 ¯ x 2 + y 2 < 4 :


© ¯ ª

Recherche des points critiques de f


(
8x = 0
µ ¶ µ ¶
8x 0
∇ f (x, y) = donc ∇ f (x, y) = ⇐⇒
2y 0 2y = 0

L’unique point critique de f est le point (x 0 , y 0 ) = (0, 0).


Étude de la nature du point critique par matrice Hessienne
∂xx f (x, y) = 8 ∂ y y f (x, y) = 2 ∂x y f (x, y) = 0

donc µ ¶
8>0 0
H f (x 0 , y 0 ) = et dét(H f (x 0 , y 0 )) = 16 > 0.
0 2>0
Conclusion : le point (0, 0) est un minimum local de f .
Étude directe de la nature du point critique On n’est pas obligé d’utiliser la matrice Hessienne pour établir la na-
ture du point (x 0 , y 0 ). En effet, il suffit d’étudier le signe de la fonction distance au voisinage du point critique :

d (h, k) ≡ f (x 0 + h, y 0 + k) − f (x 0 , y 0 ) = 4(x 0 + h)2 + (y 0 + k)2 − 4x 02 − y 02 = 4h 2 + k 2 ≥ 0 pour (h, k) ' (0, 0).

Puisque d (h, k) ≥ 0, on conclut que le point (x 0 , y 0 ) est un minimum local de f .


Conclusion : (0, 0) est un minimum local de f et f (0, 0) = 0 ; comme f (x, y) > f (0, 0) pour tout (x, y) 6= (0, 0), on conclut
que (0, 0) est un minimum global de f .
3. Étude de f sur le bord, i.e. dans l’ensemble (x, y) ∈ R2 ¯ x 2 + y 2 = 4 :
© ¯ ª

Soit g (x, y) = x 2 + y 2 − 4 et L (x, y, λ) = f (x, y) − λg (x, y) = 4x 2 + y 2 − λ(x 2 + y 2 − 4).


Recherche des points critiques de L

2x(4 − λ) = 0,
   
8x − 2λx 0 
∇L (x, y, λ) =  2y − 2λy  donc ∇L (x, y, λ) = 0 ⇐⇒ 2y(1 − λ) = 0,
 
4 − x2 − y 2 0

 2
x + y 4 = 4.

Les points critiques de L sont les points

(x 1 , y 1 , λ1 ) = (2, 0, 4) , (x 2 , y 2 , λ2 ) = (−2, 0, 4) , (x 3 , y 3 , λ3 ) = (0, 2, 1) , (x 4 , y 4 , λ4 ) = (0, −2, 1) .

176 © G. Faccanoni
Dernière mise à jour : Jeudi 5 juin 2014 5 Extrema

Étude de la nature des points critiques On a

∂xx L (x, y, λ) = 8 − 2λ ∂ y y L (x, y, λ) = 2 − 2λ ∂x y L (x, y, λ) = 0

donc

∂xx L (x 1 , y 1 , λ1 ) × ∂ y y L (x 1 , y 1 , λ1 ) − (∂x y L (x 1 , y 1 , λ1 ))2 = 0,


∂xx L (x 2 , y 2 , λ2 ) × ∂ y y L (x 2 , y 2 , λ2 ) − (∂x y L (x 2 , y 2 , λ2 ))2 = 0,
∂xx L (x 3 , y 3 , λ3 ) × ∂ y y L (x 3 , y 3 , λ3 ) − (∂x y L (x 3 , y 3 , λ3 ))2 = 0,
∂xx L (x 4 , y 4 , λ4 ) × ∂ y y L (x 4 , y 4 , λ4 ) − (∂x y L (x 4 , y 4 , λ4 ))2 = 0.

Conclusion : la méthode du lagrangien permet seulement de trouver les candidats extrema de f sous la contrainte
g mais ne permet pas de conclure s’ils sont effectivement des extrema.
Étude directe de la nature des points critiques On n’est pas obligé d’utiliser la sous-matrice Hessienne de L pour
établir la nature des points critiques. En effet, il suffit d’étudier le signe de la fonction distance

d i (h, k) ≡ f (x i + h, y i + k) − f (x i , y i ), i = 1, 2, 3, 4

pour (h, k) ' (0, 0) et g (x i + h, y i + k) = 0 :


B di (h, k) = 4(x i + h)2 + (y i + k)2 − 4x i2 − y i2
B g i (x i + h, y i + k) = (x i + h)2 + (y i + k)2 − 4
B si ∂ y g (x i , y i ) 6= 0 alors l’équation g (x i + h, y i + k) = 0 définit implicitement k en fonction de h au voisinage de
h = 0 : en résolvant g (x i + h, y i + k) = 0 on trouve (y i + k)2 = 4 − (x i + h)2 ; en remplaçant l’expression ainsi
trouvée dans d i (h, k) on trouve la fonction d’une seule variable

d˜i (h) = 3(x i + h)2 + 4 − 4x i2 − y i2

B si ∂x g (x i , y i ) 6= 0 alors l’équation g (x i + h, y i + k) = 0 définit implicitement h en fonction de k au voisinage de


k = 0 : en résolvant g (x i + h, y i + k) = 0 on trouve (x i + h)2 = 4 − (y i + k)2 ; en remplaçant l’expression ainsi
trouvée dans d i (h, k) on trouve la fonction d’une seule variable

d˜i (k) = 16 − 3(y i + k)2 − 4x i2 − y i2

Conclusion :
B d˜1 (k) = −3k 2 ≤ 0 donc le point (x 1 , y 1 ) est un minimum local de f sous la contrainte g ,
B d˜2 (k) = −3k 2 ≤ 0 donc le point (x 2 , y 2 ) est un minimum local de f sous la contrainte g ,
B d˜3 (h) = 3h 2 ≥ 0 donc le point (x 3 , y 3 ) est un maximum local de f sous la contrainte g ,
B d˜4 (h) = 3h 2 ≥ 0 donc le point (x 4 , y 4 ) est un maximum local de f sous la contrainte g .
Conclusion : les points (±2, 0) sont des minima locaux de f sous la contrainte g et f (±2, 0) = 16 ; les points (0, ±2) sont
des maxima locaux (et donc globaux) de f sous la contrainte g et f (0, ±2) = 4.

Exercice 5.29
On
© se propose¯ 2de déterminer les extrema de la fonction f : R2 → R définie par f (x, y) = x 2 + y 2 dans l’ensemble E =
2¯ 2
(x, y) ∈ R 4x + y ≤ 1 .
ª

1. Étudier la nature des points critiques de f lorsque (x, y) ∈ E˚ = (x, y) ∈ R2 ¯ 4x 2 + y©2 < 1 .
© ¯ ª

2. Étudier la nature des points critiques du lagrangien, i.e. de f lorsque (x, y) ∈ ∂E = (x, y) ∈ R2 ¯ 4x 2 + y 2 = 1 .
¯ ª

3. En déduire les maxima et minima globaux de f dans E .

C ORRECTION . La fonction étant de classe C ∞ (somme de fonctions C ∞ ) sur E fermé borné, on peut appliquer le théorème
de W EIERSTRASS et affirmer que f atteint son minimum global (au moins en un point du compact E ) et son maximum global
(au moins en un point du compact E ). Ces points sont à chercher parmi les minima et maxima locaux de f sur E , c’est-à-dire
parmi les minima et maxima locaux de f dans l’ouvert E˚ (i.e. optimisation libre d’une fonction C ∞ sur un ouvert) et parmi
les minima et maxima locaux de f sur le bord ∂E (i.e. optimisation liée d’une fonction C ∞ sous une contrainte C ∞ ).

© G. Faccanoni 177
5 Extrema Dernière mise à jour : Jeudi 5 juin 2014

y Dans la figure ci-contre on a tracé les courbes de niveau 0,


0.25, 1 et 4 de f ainsi que l’ensemble E . Il semblerait que
4 B le point (0, 0) est un minimum global,
1 B le point (0, 1) est un maximum global,
1/4 B le point (0, −1) est un maximum global,
0 x B le point (1/2, 0) est un minimum local,
B le point (−1/2, 0) est un minimum local.

1. Considérons l’ensemble E˚ = (x, y) ∈ R2 ¯ 4x 2 + y 2 < 1 .


© ¯ ª

Recherche des points critiques de f


(
2x = 0,
µ ¶ µ ¶
2x 0
∇ f (x, y) = donc ∇ f (x, y) = ⇐⇒
2y 0 2y = 0.

L’unique point critique de f est le point (x 0 , y 0 ) = (0, 0).


Étude de la nature du point critique
Méthode 1 :

∂xx f (x, y) = 2 ∂ y y f (x, y) = 2 ∂x y f (x, y) = 0

donc µ ¶
2 0
H f (x 0 , y 0 ) = et dét(H f (x 0 , y 0 )) = 4 > 0.
0 2

Conclusion : (0, 0) est un minimum local de f dans E̊ .


Méthode 2 : On n’est pas obligé d’utiliser la matrice Hessienne pour établir la nature du point (x 0 , y 0 ). En effet,
il suffit d’étudier le signe de la fonction distance au voisinage du point critique :

d (h, k) ≡ f (x 0 + h, y 0 + k) − f (x 0 , y 0 ) = h 2 + k 2 pour (h, k) ' (0, 0).

Puisque d (h, k) ≤ 0 pour (h, k) ' (0, 0), on conclut que le point (x 0 , y 0 ) est un minimum local.
Conclusion f (x, y) > f (0, 0) pour tout (x, y) ∈ E \ { (0, 0) } donc (0, 0) est un minimum global et le maximum global,
dont l’existence est assurée par le théorème de W EIERSTRASS, est atteint en un ou plusieurs points du bord du
domaine.
2. Considérons l’ensemble ∂E = (x, y) ∈ R2 ¯ 4x 2 + y 2 = 1 . Pour chercher les candidats extrema de f sur le bord du do-
© ¯ ª

maine, on utilise la méthode du lagrangien : soit g (x, y) = 4x 2 + y 2 −1 et L (x, y, λ) = f (x, y)−λg (x, y) = x 2 + y 2 −λ(4x 2 +
y 2 − 1).
Recherche des points critiques de L

x(1 − 4λ) = 0,
   
2x − 8λx 0 
∇L (x, y, λ) =  2y − 2λy  donc ∇L (x, y, λ) = 0 ⇐⇒ y(1 − λ) = 0,
 
1 − 4x 2 − y 2 0

 2
4x + y 2 = 1.

Les points critiques de L sont les points


µ ¶ µ ¶
1 1 1 1
(x 1 , y 1 , λ1 ) = (0, 1, 1) , (x 2 , y 2 , λ2 ) = (0, −1, 1) , (x 3 , y 3 , λ3 ) = , 0, , (x 4 , y 4 , λ4 ) = − , 0, .
2 4 2 4

Conclusion partielle : étant donné que f (0, ±1) = 1, f (±1/2, 0) = 1/4 et que le maximum global, dont l’existence est
assurée par le théorème de W EIERSTRASS, est atteint en un ou plusieurs points du bord du domaine, on peut
conclure que les points (0, 1) et (0, −1) sont des maxima globaux. En revanche, il faut encore étudier les points
(0, 1/2) et (0, −1/2) qui peuvent être des minima locaux, des maxima locaux ou ne pas être des extrema.
Étude de la nature des points critiques À la lumière de la conclusion précédente, il suffit d’étudier la nature des
deux points critiques (0, 1/2) et (0, −1/2). Cependant, dans un souci d’exhaustivité, on effectue l’analyse pour
chacun des quatre points critiques du lagrangien. On a

∂xx L (x, y, λ) = 2(1 − 4λ) ∂ y y L (x, y, λ) = 2(1 − λ) ∂x y L (x, y, λ) = 0

178 © G. Faccanoni
Dernière mise à jour : Jeudi 5 juin 2014 5 Extrema

donc ∂xx L (x i , y i , λi )×∂ y y L (x i , y i , λi )−(∂x y L (x i , y i , λi ))2 = 0 pour i = 1, 2, 3, 4 : la méthode du lagrangien permet


juste de trouver les candidats extrema de f sous la contrainte g mais ne permet pas de conclure sur leur nature.
On étudie alors le comportement de f au voisinage de chaque point critique en restant sur le bord ∂E . Pour cela,
on étudie le signe de la fonction distance définie par

d i (h, k) ≡ f (x i + h, y i + k) − f (x i , y i ), i = 1, 2, 3, 4

pour (h, k) ' (0, 0) et g i (x i + h, y i + k) = 0 :


B di (h, k) = (x i + h)2 + (y i + k)2 − x i2 − y i2
B g i (x i + h, y i + k) = 4(x i + h)2 + (y i + k)2 − 1
Pour étudier le signe de la fonction d i avec h et k liés par la relation g i (x i + h, y i + k) = 0, on essaye d’exprimer h
en fonction de k ou réciproquement k en fonction de h à partir de la relation g i (x i + h, y i + k) = 0. Cela nécessite
l’emploi du théorème des fonctions implicites dont il faudra vérifier les hypothèses avant d’exprimer une variable
en fonction de l’autre. Ainsi :
B si ∂ y g i (x i , y i ) 6= 0 alors l’équation g i (x i + h, y i + k) = 0 définit implicitement k en fonction de h au voisinage de
h = 0 : en résolvant g i (x i + h, y i + k) = 0 on trouve (y i + k)2 = 1 − 4(x i + h)2 ; en remplaçant l’expression ainsi
trouvée dans d i (h, k) on trouve la fonction d’une seule variable

d˜i (h) = 1 − 3(x i + h)2 − x i2 − y i2 ;

on remarque que si ∂ y g i (x i , y i ) = 0, on peut toujours résoudre l’équation g i (x i + h, y i + k) = 0 et on trouve


(x i + h)2 = (1 − (y i + k)2 )/4 ; en remplaçant l’expression ainsi trouvée dans d i (h, k) on trouve la fonction d’une
seule variable δi (k) ≡ d i (h(k), k) = 41 + 43 (y i + k)2 − x i2 − y i2 , cependant l’étude du signe de cette fonction n’est
d’aucune utilité car on a violé les hypothèses du théorème des fonctions implicites ;
B si ∂x g i (x i , y i ) 6= 0 alors l’équation g i (x i + h, y i + k) = 0 définit implicitement h en fonction de k au voisinage de
k = 0 : en résolvant g i (x i + h, y i + k) = 0 on trouve (x i + h)2 = (1 − (y i + k)2 )/4 ; en remplaçant l’expression ainsi
trouvée dans d i (h, k) on trouve la fonction d’une seule variable
1 3
d˜i (k) = + (y i + k)2 − x i2 − y i2 ;
4 4
on remarque que si ∂x g i (x i , y i ) = 0, on peut toujours résoudre l’équation g i (x i + h, y i + k) = 0 et on trouve
(y i + h)2 = 1 − 4(x i + k)2 ; en remplaçant l’expression ainsi trouvée dans d i (h, k) on trouve la fonction d’une
seule variable δi (h) ≡ d i (h, k(h)) = 1 − 3(x i + h)2 − x i2 − y i2 , cependant l’étude du signe de cette fonction n’est
d’aucune utilité car on a violé les hypothèses du théorème des fonctions implicites.
Conclusion :
B d˜1 (h) = −3h 2 ≤ 0 donc le point (x 1 , y 1 ) est un maximum,
(on remarque que δ1 (k) = 3k(k + 2)/4 qui change de signe) ;
B d˜2 (h) = −3h 2 ≤ 0 donc le point (x 2 , y 2 ) est un maximum,
(on remarque que δ2 (k) = 3k(k − 2)/4 qui change de signe) ;
B d˜3 (k) = 34 k 2 ≥ 0 donc le point (x 3 , y 3 ) est un minimum,
(on remarque que δ3 (h) = 3h(h + 1) qui change de signe) ;
B d˜4 (k) = 34 k 2 ≥ 0 donc le point (x 4 , y 4 ) est un minimum,
(on remarque que δ4 (h) = 3h(h − 1) qui change de signe).
Conclusion : les points (0, 1) et (0, −1) sont des maxima de f sous la contrainte g et f (0, ±1) = 1, les points (1/2, 0) et
(−1/2, 0) sont des minima de f sous la contrainte g et f (±1/2, 0) = 1/4.
3. (0, 0) est le minimum global et les points (0, 1) et (0, −1) sont les maxima globaux, autrement dit

f (0, 0) ≤ f (x, y) ≤ f (0, ±1) ∀(x, y) ∈ E .

Exercice 5.30
Dans le plan, on considère l’ensemble

E = (x, y) ∈ R2 ¯ 3x 2 + 3y 2 − 2x y ≤ 1 .
© ¯ ª

Parmi les points de cet ensemble, déterminer ceux qui sont les plus éloignés de l’origine (c’est-à-dire les points qui maxi-
misent leur distance au point (x, y) = (0, 0)).

C ORRECTION . Il s’agit de trouver le maximum de la fonction f : R2 → R définie par f (x, y) = (x −0)2 +(y −0)2 dans le compact
E . Comme E est un fermé borné, le théorème de W EIERSTRASS garantit que f atteint son maximum et son minimum dans E .
1. Étude graphique :

© G. Faccanoni 179
5 Extrema Dernière mise à jour : Jeudi 5 juin 2014

y Dans la figure ci-contre on a colorié l’ensemble E qui est


une ellipse de centre (0, 0) et d’équation
¶2 ¶2
X Y
¡ ¢ µ µ
1 1
¡ 2 , 2¢ p + =1
1 1
f 2,2 = 21 1/ 2 1/2
avec (
¡
−1 p1
¢ x = X cos(π/4) − Y sin(π/4),
p ,
8 8
f
¡ ¢
p , p1 = p1
−1 y = X sin(π/4) + Y cos(π/4).
8 8 2
k 2 = 0.1 Parmi les courbes de niveau k de f (qui sont des cercles de
p
k 2 = 0.4
¡ ¢ x centre (0, 0) et rayon k) on cherche celles qui sont tan-
p1 , p
−1
¡ 8 ¢ 8
p à l’ensemble E ; il s’agit des courbes de niveau 1/2
gentes
f p , p = p1
1 −1
8 8 2 et 1/ 2.
k 2 = 0.9 On peut alors voir que
¡
1 1
¢ B le point (0,
¡ 0)¢est un minimum global,
¡ − 2 ,− 2¢ B le point 12 , 12 est un maximum global,
f − 12 ,− 12 = 21
B le point ³− 21 , − 12´ est un maximum global,
¡ ¢

B le point p18 , p −1

est un minimum local,
³
−1 1
B le point p , p est un minimum local.
8 8

2. Considérons l’ensemble E˚ = (x, y) ∈ R2 ¯ 3x 2 + 3y 2 − 2x y < 1 .


© ¯ ª

Recherche des points critiques de f


(
2x = 0,
µ ¶ µ ¶
2x 0
∇ f (x, y) = donc ∇ f (x, y) = ⇐⇒
2y 0 2y = 0.

L’unique point critique de f est le point (x 0 , y 0 ) = (0, 0) qui appartient bien à E˚ . Comme f (x, y) > f (0, 0) pour tout
(x, y) 6= (0, 0), ce point est un minimum global de f .
Conclusion : le maximum de f appartient au bord de l’ensemble E˚ .
3. Considérons l’ensemble ∂E = (x, y) ∈ R2 ¯ 3x 2 + 3y 2 − 2x y = 1 . Soit g (x, y) = 3x 2 +3y 2 −2x y −1 et L (x, y, λ) = f (x, y)−
© ¯ ª

λg (x, y) = x 2 + y 2 − λ(3x 2 + 3y 2 − 2x y − 1).


Recherche des points critiques de L

2x − λ(6x − 2y)
 

∇L (x, y, λ) =  2y − λ(6y − 2x) 


1 − 3x 2 − 3y 2 + 2x y

donc  
x(1 − 3λ) = −λy, (x − y)(1 − 4λ) = 0,
 
0  
∇L (x, y, λ) = 0 ⇐⇒ y(1 − 3λ) = −λx, ⇐⇒ (x + y)(1 − 2λ) = 0,
0

 2 
3x + 3y 2 − 2x y = 1, 3x 2 + 3y 2 − 2x y = 1.

Les points critiques de L sont les points


µ ¶ µ ¶ µ ¶ µ ¶
1 1 1 1 1 1 1 1 1 1 1 1
(x 1 , y 1 , λ1 ) = , , , (x 2 , y 2 , λ2 ) = − , − , , (x 3 , y 3 , λ3 ) = p ,− p , , (x 4 , y 4 , λ4 ) = − p , p , .
2 2 2 2 2 2 2 2 2 2 4 2 2 2 2 4

Étude de la nature des points critiques On a

∂xx L (x, y, λ) = 2 − 6λ ∂ y y L (x, y, λ) = 2 − 6λ ∂x y L (x, y, λ) = 2λ

donc ∂xx L (x i , y i , λi ) × ∂ y y L (x i , y i , λi ) − (∂x y L (x i , y i , λi ))2 = 32(λi − 1/2)(λi − 1/4) = 0 pour i = 1, 2, 3, 4. Conclu-


sion : la méthode du lagrangien permet juste de trouver les candidats extrema de f sous la contrainte g mais ne
permet pas de conclure.
On étudie alors le signe de la fonction distance

d i (h, k) ≡ f (x i + h, y i + k) − f (x i , y i ) = (x i + h)2 + (y i + k)2 − x i2 − y i2 , i = 1, 2, 3, 4

180 © G. Faccanoni
Dernière mise à jour : Jeudi 5 juin 2014 5 Extrema

pour (h, k) ' (0, 0) et g (x i + h, y i + k) = 0, i.e. 3(x i + h)2 + 3(y i + k)2 − 2(x i + h)(y i + k) = 1. Si ∂ y g (x i , y i ) 6= 0 alors
l’équation g (x i + h, y i + k) = 0 définit implicitement k en fonction de h au voisinage de h = 0 :

∂ y g (x i , y i ) = 2(3y i − x i ) 6= 0, i = 1, 2, 3, 4.

En résolvant g (x i + h, y i + k) = 0 on trouve
 p
 (xi +h)+ 3−8(xi +h)2 si i = 1, 4,
k = −y i + p3
 (xi +h)− 3−8(xi +h)2
3 si i = 2, 3.

En remplaçant l’expression ainsi trouvée dans d i (h, k) on trouve la fonction d’une seule variable
 µ p ¶2
2 (x i +h)+ 3−8(x i +h)2
(x i + h) + si i = 1, 4;


3
d˜i (h) = µ p ¶2
(x +h)− 3−8(x i +h)2
(x i + h)2 + i


3 si i = 2, 3.

En étudiant les quatre fonctions d˜i lorsque h ' 0 on a :


B d˜1 (h) ≤ 0 si h ' 0 donc le point (x 1 , y 1 ) est un maximum,
B d˜2 (h) ≤ 0 donc le point (x 2 , y 2 ) est un maximum,
B d˜3 (h) ≥ 0 donc le point (x 3 , y 3 ) est un minimum,
B d˜4 (h) ≥ 0 donc le point (x 4 , y 4 ) est un minimum.
Conclusion
p : les p 1/2)pet (−1/2, −1/2) sont des maxima de f sous la contrainte p
p points (1/2, g et f vaut 1/2, les points
(−1/ 8, 1/ 8) et (1/ 8, −1/ 8) sont des minima de f sous la contrainte g et f vaut 1/ 2.
4. (0, 0) est le minimum global et les points (1/2, 1/2) et (−1/2, −1/2) sont les maxima globaux, autrement dit

f (0, 0) ≤ f (x, y) ≤ f (1/2, 1/2) = f (−1/2, −1/2) ∀(x, y) ∈ E .

Exercice 5.31
Soit E = (x, y) ∈ R2 ¯ 16x 4 + y 4 ≤ 1 . On se propose de déterminer les extrema de la fonction
© ¯ ª

f : E →R
(x, y) 7→ x 4 + 16y 4

1. Étudier la nature des points critiques de f lorsque (x, y) ∈ E˚ = (x, y) ∈ R2 ¯ 16x 4 + ©y 4 < 1 .
© ¯ ª

2. Étudier la nature des points critiques du lagrangien, i.e. de f lorsque (x, y) ∈ ∂E = (x, y) ∈ R2 ¯ 16x 4 + y 4 = 1 .
¯ ª

3. En déduire les maxima et minima globaux de f dans E .

C ORRECTION . La fonction étant de classe C ∞ (somme de fonctions C ∞ ) sur E fermé borné, on peut appliquer le théorème
de W EIERSTRASS et affirmer que f atteint son minimum global (au moins en un point du compact E ) et son maximum global
(au moins en un point du compact E ). Ces points sont à chercher parmi les minima et maxima locaux de f sur E , c’est-à-dire
parmi les minima et maxima locaux de f dans l’ouvert E˚ (i.e. optimisation libre d’une fonction C ∞ sur un ouvert) et parmi
les minima et maxima locaux de f sur le bord ∂E (i.e. optimisation liée d’une fonction C ∞ sous une contrainte C ∞ ).

1. Considérons l’ensemble E˚ = (x, y) ∈ R2 ¯ 16x 4 + y 4 < 1 .


© ¯ ª

Recherche des points critiques de f


(
4x 3 x 3 = 0,
µ ¶ µ ¶
0
∇ f (x, y) = donc ∇ f (x, y) = ⇐⇒
64y 3 0 y 3 = 0.

L’unique point critique de f est le point (x 0 , y 0 ) = (0, 0).


Étude de la nature du point critique

∂xx f (x, y) = 12x 2 ∂ y y f (x, y) = 192y 2 ∂x y f (x, y) = 0

donc µ ¶
0 0
H f (x 0 , y 0 ) = et dét(H f (x 0 , y 0 )) = 0.
0 0

© G. Faccanoni 181
5 Extrema Dernière mise à jour : Jeudi 5 juin 2014

Comme l’étude de la matrice Hessienne ne permet pas d’établir la nature du point (x 0 , y 0 ), on étudie directement
le signe de la fonction distance au voisinage du point critique :
d (h, k) ≡ f (x 0 + h, y 0 + k) − f (x 0 , y 0 ) = h 4 + 16k 4 pour (h, k) ' (0, 0).
Puisque d (h, k) ≤ 0 pour (h, k) ' (0, 0), on conclut que le point (x 0 , y 0 ) est un minimum local.
Conclusion sur l’ouvert : f (x, y) > f (0, 0) pour tout (x, y) ∈ E \ { (0, 0) } donc (0, 0) est un minimum global.
2. Considérons l’ensemble ∂E = (x, y) ∈ R2 ¯ 16x 4 + y 4 = 1 . Pour chercher les candidats extrema de f sur le bord du
© ¯ ª

domaine, on utilise la méthode du lagrangien : soit g (x, y) = 16x 4 + y 4 − 1 et L (x, y, λ) = f (x, y) − λg (x, y) = x 4 + 16y 4 −
λ(16x 4 + y 4 − 1).
Recherche des points critiques de L
 3  3
4x − 64λx 3 4x (1 − 16λ) = 0,
  
0 
∇L (x, y, λ) = 192y 3 − 4λy 3  donc ∇L (x, y, λ) = 0 ⇐⇒ 4y 3 (16 − λ) = 0,
1 − 16x 4 − y 4 0

16x 4 + y 4 = 1.

Les points critiques de L sont les points


µ ¶ µ ¶
1 1 1 1
(x 1 , y 1 , λ1 ) = (0, 1, 16) , (x 2 , y 2 , λ2 ) = (0, −1, 16) , (x 3 , y 3 , λ3 ) = , 0, , (x 4 , y 4 , λ4 ) = − , 0, .
2 16 2 16
Conclusion partielle : étant donné que f (0, ±1) = 16, f (±1/2, 0) = 1/16 et que le maximum global, dont l’existence
est assurée par le théorème de W EIERSTRASS, est atteint en un ou plusieurs points du bord du domaine, on peut
conclure que les points (0, 1) et (0, −1) sont des maxima globaux. En revanche, il faut encore étudier les points
(0, 1/2) et (0, −1/2) qui peuvent être des minima locaux, des maxima locaux ou ne pas être des extrema.
Étude de la nature des points critiques À la lumière de la conclusion partielle précédente, il suffit d’étudier la na-
ture des deux points critiques (0, 1/2) et (0, −1/2). Cependant, dans un souci d’exhaustivité, on effectue l’analyse
pour chacun des quatre points critiques du lagrangien. On a
∂xx L (x, y, λ) = 12x 2 (1 − 16λ) ∂ y y L (x, y, λ) = 12y 2 (16 − λ) ∂x y L (x, y, λ) = 0
donc ∂xx L (x i , y i , λi )×∂ y y L (x i , y i , λi )−(∂x y L (x i , y i , λi ))2 = 0 pour i = 1, 2, 3, 4 : la méthode du lagrangien permet
juste de trouver les candidats extrema de f sous la contrainte g mais ne permet pas de conclure. On étudie alors
le comportement de f au voisinage de chaque point critique en restant sur le bord ∂E . Pour cela, on étudie le
signe de la fonction distance définie par
d i (h, k) ≡ f (x i + h, y i + k) − f (x i , y i ), i = 1, 2, 3, 4
pour (h, k) ' (0, 0) et g (x i + h, y i + k) = 0 :
B di (h, k) = (x i + h)4 + 16(y i + k)4 − x i2 − 16y i2
B g i (x i + h, y i + k) = 16(x i + h)4 + (y i + k)4 − 1
Pour étudier le signe de la fonction d i avec h et k liés par la relation g i (x i + h, y i + k) = 0, on essaye d’exprimer h
en fonction de k ou réciproquement k en fonction de h à partir de la relation g i (x i + h, y i + k) = 0. Cela nécessite
l’emploi du théorème des fonctions implicites dont il faudra vérifier les hypothèses avant d’exprimer une variable
en fonction de l’autre. Ainsi :
B si ∂ y g i (x i , y i ) 6= 0 alors l’équation g i (x i + h, y i + k) = 0 définit implicitement k en fonction de h au voisinage de
h = 0 : en résolvant g i (x i + h, y i + k) = 0 on trouve (y i + k)4 = 1 − 16(x i + h)4 ; en remplaçant l’expression ainsi
trouvée dans d i (h, k) on trouve la fonction d’une seule variable
d˜i (h) = 16 − 15(x i + h)4 − x i4 − 16y i4 ;
on remarque que si ∂ y g i (x i , y i ) = 0, on peut toujours résoudre l’équation g i (x i + h, y i + k) = 0 et on trouve
(x i + h)2 = (1 − (y i + k)2 )/4 ; en remplaçant l’expression ainsi trouvée dans d i (h, k) on trouve la fonction d’une
1
seule variable δi (k) ≡ d i (h(k), k) = 16 + 255 4 4 4
16 (y i + k) − x i − 16y i , cependant l’étude du signe de cette fonction
n’est d’aucune utilité car on a violé les hypothèses du théorème des fonctions implicites ;
B si ∂x g i (x i , y i ) 6= 0 alors l’équation g i (x i + h, y i + k) = 0 définit implicitement h en fonction de k au voisinage de
k = 0 : en résolvant g i (x i +h, y i +k) = 0 on trouve (x i +h)4 = (1−(y i +k)4 )/16 ; en remplaçant l’expression ainsi
trouvée dans d i (h, k) on trouve la fonction d’une seule variable
1 255
d˜i (k) = + (y i + k)4 − x i4 − 16y i4 ;
16 16
on remarque que si ∂x g i (x i , y i ) = 0, on peut toujours résoudre l’équation g i (x i + h, y i + k) = 0 et on trouve
(y i + h)4 = 1 − 16(x i + k)4 ; en remplaçant l’expression ainsi trouvée dans d i (h, k) on trouve la fonction d’une
seule variable δi (h) ≡ d i (h, k(h)) = 16 − 15(x i + h)4 − x i4 − 16y i4 , cependant l’étude du signe de cette fonction
n’est d’aucune utilité car on a violé les hypothèses du théorème des fonctions implicites.

182 © G. Faccanoni
Dernière mise à jour : Jeudi 5 juin 2014 5 Extrema

Conclusion partielle :
B d˜1 (h) = −15h 4 ≤ 0 donc le point (x 1 , y 1 ) est un maximum,
B d˜2 (h) = −15h 4 ≤ 0 donc le point (x 2 , y 2 ) est un maximum,
B d˜3 (k) = 255 4
16 k ≥ 0 donc le point (x 3 , y 3 ) est un minimum,
B d˜4 (k) = 16 k 4 ≥ 0 donc le point (x 4 , y 4 ) est un minimum.
255

Conclusion sur le bord : les points (0, 1) et (0, −1) sont des maxima de f sous la contrainte g et f (0, ±1) = 16, les
points (1/2, 0) et (−1/2, 0) sont des minima de f sous la contrainte g et f (±1/2, 0) = 1/16.
3. (0, 0) est le minimum global et les points (0, 1) et (0, −1) sont les maxima globaux, autrement dit

0 = f (0, 0) ≤ f (x, y) ≤ f (0, ±1) = 16 ∀(x, y) ∈ E .

Exercice 5.32
p
Quel point du cercle d’équation x 2 + y 2 ≤ 1 minimise la distance à la droite d’équation y = 4 − 3x ?

C ORRECTION . En se rappelant la formule pour la distance d’un point (x, y) d’une droite d’équation ax + b y + c = 0, il s’agit
de minimiser la fonction p
|ax + b y + c| | 3x + y − 4|
f (x, y) = p =
a2 + b2 2
¯ 2 2
p
dans D = (x, y) x + y ≤ 1 . Comme 3x + y − 4 < 0 pour tout point de D, il s’agit alors de minimiser la fonction
© ª
¯
p
4 − 3x − y
d (x, y) =.
2
p
Les extrema se trouvent soit dans D̊ soit sur ∂D. Comme ∇d (x, y) = (− 3, −1) 6= (0, 0), il n’y a pas d’extrema dans D̊. Il faut
alors les chercher sur ∂D. Pour cela on exprime ∂D sous forme paramétrique x = cos(ϑ) et y = sin(ϑ) pour ϑ ∈ [0; 2π]. La
restriction de d à cette courbe donne la fonction d’une seule variable
p
4 − 3 cos(ϑ) − sin(ϑ)
h(ϑ) = d (x(ϑ), y(ϑ)) = .
2

Comme h 0 (ϑ) = sin ϑ − π6 on a que h 0 (ϑ) = 0 ssi ϑ = π6 ou ϑ = 7π


¡ ¢
6 . Il faut comparer la valeur de h en ces points avec la valeur
en 0 et en 2π : p
4− 3 ³π´ µ ¶

h(0) = h(2π) = , h = 1, h = 3.
2 6 6
p
On conclut que le point qui minimise la distance correspond à ϑ = π6 , c’est-à-dire le point ( 12 , 3
2 ).

Remarquons que, une fois établit que les extrema se trouvent sur la courbe d’équation x 2 + y 2 = 1, cela revient à calculer le
y) = x 2 + y 2 − 1. On peut alors utiliser la méthode des multiplicateurs
minimum de d sous la contrainte g (x, y) = 0 avec g (x, p
4− 3x−y
de L AGRANGE : on introduit le lagrangien L(x, y, `) = 2 − `(x 2 + y 2 − 1), on calcul son gradient
 p 
− 23 − 2`x
∇L =  − 1 − 2`y  ,
 
2
1 − x2 − y 2
p p p p p
3 3 3 3 3
comme ∇L(x, y, `) = (0, 0, 0) ssi (x, y) = ( 12 , 2 ) ou (x, y) = (− 21 , − 2 ), et comme d ( 21 , 1
2 ) < d (− 2 , − 2 ), on conclut que ( 21 , 2 )
est le minimum.

p
3
2

1 x
2

p
y = 4 − 3x

© G. Faccanoni 183
5 Extrema Dernière mise à jour : Jeudi 5 juin 2014

Exercice 5.33
On cherche les extrema de la fonction f : R2 → R définie par f (x, y) = x 2 + 2y 2 dans le disque x 2 + y 2 ≤ 1.
Étude dans l’ouvert x 2 + y 2 < 1 : déterminer le point critique de f dans x 2 + y 2 < 1 et en établir la nature.
déterminer les points critiques du lagrangien L (x, y, λ) ≡ f (x, y)−λ(x 2 +y 2 −1) et en établir
Étude sur le bord x 2 + y 2 = 1 :
la nature.
Quels sont les minimum et maximum absolus de f dans le disque x 2 + y 2 ≤ 1 ?

C ORRECTION . Le disque x 2 + y 2 ≤ 1 est un ensemble fermé borné et la fonction f est continue, pour le théorème de W EIERS -
TRASS il existe un maximum et un minimum absolus de f dans le disque.

1. Étude de f (x, y) = x 2 + 2y 2 dans le disque x 2 + y 2 < 1 :


¡ 2x ¢
Points critiques ∇ f (x, y) = 4y et
(
2x = 0,
µ ¶
0
∇ f (x, y) = ⇐⇒ ⇐⇒ (x, y) = (0, 0)
0 4y = 0,

Nature des points critiques D(x, y) = ∂xx f (x, y)∂ y y f (x, y)−(∂x y f (x, y))2 = 2×4−02 = 8. Comme D(0, 0) > 0, le point
(0, 0) est un minimum local.
2. Étude sur le bord x 2 + y 2 = 1 :
2x−2λx
µ ¶
Points critiques ∇L (x, y, λ) = 4y−2λy et
1−x 2 −y 2


2x − 2λx = 0,
 
0 
∇L (x, y, λ) = 0 ⇐⇒ 4y − 2λy = 0,
  ⇐⇒ (x, y, λ) ∈ { (0, 1, 2), (0, −1, 2), (1, 0, 1), (−1, 0, 1) }
0

 2
x + y2 = 1

Nature des points critiques D(x, y, λ) = ∂xx L (x, y, λ)∂ y y L (x, y, λ) − (∂x y L (x, y, λ))2 = 2(1 − λ) × 2(2 − λ) − 02 = 4(1 −
λ)(2 − λ). Comme D(0, 1, 2) = D(0, −1, 2), D(1, 0, 1) = D(−1, 0, 1) = 0, on ne peut pas établir la nature de ces points
critiques en utilisant le lagrangien. Étudions alors directement le comportement de la fonction au voisinage de
ces points en regardant le signe de la fonction

d (h, k) = f (x 0 + h, y 0 + k) − f (x 0 , y 0 ) = (x 0 + h)2 + 2(y 0 + k)2 − x 02 − 2y 02 ,

h et k étant liés par (x 0 + h)2 + (y 0 + k)2 = 1.


Si (x 0 , y 0 ) = (0, ±1) alors ∂ y g (x 0 , y 0 ) 6= 0. Dans ce cas l’équation g (x 0 + h, y 0 + k) = 0 définit implicitement k en
fonction de h au voisinage de h = 0 : en résolvant g (x 0 + h, y 0 + k) = 0 on trouve (y 0 + k)2 = 1 − (x 0 + h)2 ; en
remplaçant l’expression ainsi trouvée dans d (h, k) on trouve la fonction d’une seule variable

d˜(h) = −h 2 ≤ 0,

donc les points (0, ±1) sont des maxima locaux.


Si (x 0 , y 0 ) = (±1, 0) alors ∂ y g (x 0 , y 0 ) = 0 mais ∂x g (x 0 , y 0 ) 6= 0. Dans ce cas l’équation g (x 0 + h, y 0 + k) = 0 définit
implicitement h en fonction de k au voisinage de k = 0 : en résolvant g (x 0 + h, y 0 + k) = 0 on trouve (x 0 + h)2 =
1 − (y 0 + k)2 ; en remplaçant l’expression ainsi trouvée dans d (h, k) on trouve la fonction d’une seule variable

d˜(k) = k 2 ≥ 0,

donc les points (±1, 0) sont des minima locaux.


Étant donné que f (0, 0) = 0, f (1, 0) = f (−1, 0) = 1 et f (0, 1) = f (0, −1) = 2, on conclut que (0, 0) est le minimum absolu et
(0, ±1) sont les maxima absolus de f dans le disque x 2 + y 2 ≤ 1.

Exercice 5.34
Étudier et classer les
p points stationnaires des fonctions suivantes
1. f (x, y) = x y 1 − x 2 − y 2 dans son domaine de définition,
2. f (x, y) = x 2 + y 2 − x − y dans l’ensemble D = {(x, y) | |x| ≤ 1, |y| ≤ 1}.

184 © G. Faccanoni
Dernière mise à jour : Jeudi 5 juin 2014 5 Extrema

C ORRECTION .
1. f (x, y) = x y 1 − x 2 − y 2 est définie sur le disque D = (x, y) ∈ R2 ¯ x 2 + y 2 ≤ 1 à valeur dans R ; les extrema peuvent
p © ¯ ª

être soit dans D̊ soit sur le bord ∂D = (x, y) ∈ R2 ¯ x 2 + y 2 = 1 .


© ¯ ª

Cherchons d’abord les points critiques dans D̊ :

y(1−2x 2 −y 2 )
 
p
 1−x 2 −y 2 
∇ f (x, y) =  x(1−x 2 −2y 2 ) 
p 2 2
1−x −y

et ∇ f (x, y) = (0, 0) ssi


n p p p p p p p p o
(x, y) ∈ (0, 0), (1/ 3, 1/ 3), (1/ 3, −1/ 3), (−1/ 3, 1/ 3), (−1/ 3, −1/ 3) .

Étudions maintenant séparément chacun de ces points en calculant au préalable le déterminant de la matrice hes-
sienne de la fonction f en un point quelconque :

−x y(3 − 2x 2 − 3y 2 )
∂xx f (x, y) = p ,
(1 − x 2 − y 2 )3
1 − 3x 2 + 3x 2 y 2 − 3y 2 + 2x 4 + 2y 4
∂x y f (x, y) = p ,
(1 − x 2 − y 2 )3
−x y(3 − 3x 2 − 2y 2 )
∂ y y f (x, y) = p ,
(1 − x 2 − y 2 )3
D(x, y) = ∂xx f (x, y)∂ y y f (x, y) − (∂x y f (x, y))2 .

On a alors
(x 0 , y 0 ) ∂xx f (x 0 , y 0 ) ∂x y f (x 0 , y 0 ) ∂ y y f (x 0 , y 0 ) D(x 0 , y 0 )
(0, 0) 0 1 0 −1 SELLE
( p1 , p1 ) − p4 − p2 − p4 4 MAX
3 3 3 3 3
(− p1 , p1 ) p4 − p2 p4 4 MIN
3 3 3 3 3
( p1 , − p1 ) p4 − p2 p4 4 MIN
3 3 3 3 3
(− p1 , − p1 ) − p4 − p2 − p4 4 MAX
3 3 3 3 3

La fonction n’est pas dérivable sur ∂D mais on peut étudier la nature de ces points en observant que f est nulle sur les
axes, positive dans le premier et le troisième quadrant, négative p dans le deuxième et quatrième quadrant p donc (1, 0),
(0, 1), (−1, 0) et (0, −1) sont des points de SELLE, les points (x, 1 − x 2 ) avec x > 0 et les points (x, − 1 − x 2 ) avec x < 0
p p
sont des MIN, les points (x, 1 − x 2 ) avec x < 0 et les points (x, − 1 − x 2 ) avec x > 0 sont des MAX.
2. L’ensemble D un fermé borné et la fonction f est continue, pour le théorème de W EIERSTRASS il existe un maximum
et un minimum de f dans D. On suit la «recette» pour le calcul de ces valeurs :
B on cherche dans D̊ : les points critiques de f doivent annuler le gradient de f ; on a ∇ f (x, y) = (2x − 1, 2y − 1) = (0, 0)
ssi (x, y) = (1/2, 1/2) et f (1/2, 1/2) = −1/2 ;
B on cherche © sur¯ ∂D : on doit considérer quatre segments
B S 1 = (x, y) ¯ x = 1, |y| ≤ 1 : h 1 (y) = f |S 1 = y 2 − y pour y ∈ [−1; 1], h 10 (y) = 2y − 1 = 0 ssi y = 1/2 et h 1 (1/2) = −1/4 ;
ª

© h 1 (−1)
de plus, ¯ = 2 et h 1 (1)ª= 0 ; donc le MAX de h 1 vaut 2 et le MIN vaut −1/4 ;
B S 2 = (x, y) ¯ x = −1, |y| ≤ 1 : h 2 (y) = f |S 1 = 2+y 2 −y pour y ∈ [−1; 1], h 20 (y) = 2y−1 = 0 ssi y = 1/2 et h 2 (1/2) = 7/4 ;

© h 2 (−1)
de plus, ¯ = 4 et h 2 (1)ª = 2 ; donc le MAX de h 2 vaut 4 et le MIN vaut 7/4 ;
B S 3 = (x, y) ¯ y = 1, |x| ≤ 1 : h 3 (x) = f |S 3 = x 2 − x pour x ∈ [−1; 1], h 30 (x) = 2x − 1 = 0 ssi x = 1/2 et h 3 (1/2) = −1/4 ;

© h 3 (−1)
de plus, ¯ = 2 et h 3 (1)ª= 0 ; donc le MAX de h 3 vaut 2 et le MIN vaut −1/4 ;
B S 4 = (x, y) ¯ y = −1, |x| ≤ 1 : h 4 (y) = f |S 4 = 2+x 2 −x pour x ∈ [−1; 1], h 40 (y) = 2x−1 = 0 ssi x = 1/2 et h 2 (1/2) = 7/4 ;
de plus, h 4 (−1) = 4 et h 4 (1) = 2 ; donc le MAX de h 4 vaut 4 et le MIN vaut 7/4.
En résumé, les candidats sont :

(x 0 , y 0 ) (1/2, 1/2) (1, −1) (−1, 1) (1, 1/2) (1/2, 1) (−1, 1/2) (1/2, −1) (−1, −1)
f (x 0 , y 0 ) −1/2 2 2 −1/4 −1/4 7/4 7/4 4

On conclut que le MAX absolu est 4 obtenu en (−1, −1) et le MIN absolu est −1/2 obtenu en (1/2, 1/2).

© G. Faccanoni 185
5 Extrema Dernière mise à jour : Jeudi 5 juin 2014

Max pour h 2 et h 4

Min pour h 2 Min pour h 4


Max pour h 3 Max pour h 1

Min libre

Min pour h 3 Min pour h 1

186 © G. Faccanoni
Dernière mise à jour : Jeudi 5 juin 2014 5 Extrema

Problèmes d’optimisation sur un fermé : optimisation libre et liée

Exercice 5.35
Une société d’expédition doit construire un entrepôt de la forme d’un parallélépipède et de capacité 4000 m3 . Cette
construction ne nécessite pas de fondations ni de sol, son coût est de 20 ¤ le mètre carré pour le toit et de 160 ¤ le
mètre carré pour les parois. Quelles dimensions de l’entrepôt minimisent le coût de construction ?

C ORRECTION . Notons x > 0, y > 0 et z > 0 les dimensions de l’entrepôt (la largeur, la longueur et la hauteur respectivement).
Il s’agit de minimiser la fonction f (x, y, z) = 160(2xz + 2y z) + 20x y sous la contrainte g (x, y, z) = x y z − 4000 avec x, y, z > 0 (la
face manquante du parallélépipède est le planché qui a pour cotés x et y). Pour trouver les dimensions optimales on peut
utiliser l’une des deux méthodes suivantes.
Méthode de Lagrange. Introduisons le lagrangien :

L : (R∗+ )2 × R → R
(x, y, z, λ) 7→ f (x, y, z) − λg (c, y, z)

Il s’agit de chercher les solutions (x, y, z, λ) de ∇L(x, y, z, λ) = (0, 0, 0, 0)T :

20y + 320z − λy z = 0, (5.7a)





 20x + 320z − λxz = 0,

(5.7b)


 320x + 320y − λx y = 0, (5.7c)

x y z = 4000. (5.7d)

Avec les trois soustractions (5.7b)-(5.7a), (5.7c)-16×(5.7a) et (5.7c)-16×(5.7b) on obtient x = y = 16z, on insère ce résul-
tat dans (5.7d) et on trouve que le seul point critique est (40, 40, 5/2, 16).
Il reste à étudier sa nature. Pour cela, étudions le signe de la différence
µ ¶ µ ¶
5 5
d (u, v, w) ≡ f 40 + u, 40 + v, + w − f 40, 40, = 1600u + 1600v + 25600w + 320uw + 320v w + 20uv,
2 2

u, v et w étant liés par la relation (40 + u)(40 + v) 52 + w = 4000, soit encore


¡ ¢

5 40u + 40v + uv
w(u, v) = − .
2 (40 + u)(40 + v)
On a alors
20
d˜(u, v) ≡ d (u, v, w(u, v)) = h(u, v)
(40 + u)(40 + v)
avec
h(u, v) ≡ 1600u 2 + 1600v 2 + 1600uv + 80uv 2 + 80u 2 v + u 2 v 2 .
On s’intéresse au signe de d˜ lorsque (u, v) ' (0, 0), i.e. au signe de h lorsque (u, v) ' (0, 0). On vérifie facilement que
(0, 0) est un minimum local de h car
µ ¶ µ ¶
0 3200 1600
∇h(0, 0) = , Hh (0, 0) = , det(Hh (0, 0)) > 0.
0 1600 3200

Comme (0, 0) est un minimum local de h, alors h(0, 0) ≤ h(u, v) pour tout (u, v) ' (0, 0). Comme h(0, 0) = 0, cela signifie
que h(u, v) ≥ 0 pour tout (u, v) ' (0, 0) et donc d˜(u, v) ≥ d˜(0, 0) pour tout (u, v) ' (0, 0) : le point (40, 40, 5/2) est alors un
minimum local de f sous la contrainte x y z = 4000.
Méthode de réduction. La contrainte se réécrit z = 4000
x y donc il s’agit de chercher les extrema libres de la fonction de deux
variables h : (R∗+ )2 → R+ définie par
µ ¶ µ ¶
4000 64000 64000
h(x, y) = f x, y, = 20 x y + + .
xy y x

Cherchons d’abord les points critiques de h :


³
 ´
20 y − 64000
x 2
∇h(x, y) =  ³ ´
20 x − 64000
y2

© G. Faccanoni 187
5 Extrema Dernière mise à jour : Jeudi 5 juin 2014

et ∇h(x, y) = (0, 0)T ssi (x, y) = (40, 40). Étudions maintenant ce point en calculant le déterminant de la matrice hes-
sienne de la fonction h :
2560000
h xx (x, y) = h xx (40, 40) = 40 > 0,
x3
h x y (x, y) = 20 h x y (40, 40) = 20,
2560000
h y y (x, y) = h y y (40, 40) = 40,
y3
D(40, 40) ≡ ∂xx h(40, 40)∂ y y h(40, 40) − (∂x y h(40, 40))2 = 1200 > 0.

On a donc que (40, 40) est bien un minimum et z(40, 40) = 5/2.

Exercice 5.36
Un glacier produit deux types de glace, la glace A et la glace B . Si en une semaine il produit x kilogrammes de glace de
type A et y kilogrammes de glace de type B , le coût total est

c(x, y) = 600 + 200x + 200y − 2x 2 y.

Peut-il minimiser le coût hebdomadaire en produisant les deux types de glace en une quantité inférieure ou égale à 300
kilogrammes ? Pour répondre à cette question on analyse d’abord le cas d’une utilisation à pleine capacité du laboratoire
et on étudie ensuite si cette stratégie est la meilleure possible.
1. En supposant qu’il désire utiliser à pleine capacité son laboratoire, trouver la répartition de la production permet-
tant de minimiser le coût. Prouver qu’il s’agit bien d’un minimum absolu.
2. Le glacier s’interroge sur la pertinence de vouloir utiliser à pleine capacité son laboratoire. Il se demande si la
solution qu’il obtiendrait sans cette contrainte serait plus intéressante. Aidez-le à répondre à cette question en
trouvant s’il existe une solution qui minimise le coût sans cette contrainte. La solution obtenue est-elle réalisable ?
3. En exploitant les résultats obtenus aux point précédents, suggériez-vous au glacier de diminuer la production totale
hebdomadaire ou d’utiliser à pleine capacité son laboratoire ?

C ORRECTION . Il s’agit de minimiser la fonction

c : E → R+
(x, y) 7→ 600 + 200x + 200y − 2x 2 y


E = (x, y) ∈ (R+ )2 ¯ x + y ≤ 300 .
© ¯ ª

Recherche sur le bord.


B Si on ne produit que de la glace de type A, i.e. y = 0, alors le coût totale est a(x) ≡ c(x, 0) = 600+200x qui est minimal
pour x = 0 avec c(0, 0) = 600 et maximale pour x = 300 avec c(300, 0) = 1200.
B Si on ne produit que de la glace de type B , i.e. x = 0, alors le coût totale est b(y) ≡ c(0, y) = 600+200y qui est minimal
pour y = 0 avec c(0, 0) = 600 et maximale pour y = 300 avec c(0, 300) = 1200.
B Il s’agit maintenant de résoudre le problème

minimiser c(x, y) sous la contrainte x + y = 300.

Pour cela, on peut utiliser l’une des deux méthodes suivantes :


Méthode 1 On maximise c̃(x) = c(x, 300 − x) = 600 + 200x + 200(300 − x) − 2x 2 (300 − x) = 60600 − 600x 2 + 2x 3 .
Points critiques c̃ 0 (x) = −1200x + 6x 2 = 6x(x − 200), c̃ 0 (x) = 0 si et seulement si x = 0 ou x = 200.
Classification c̃ 00 (x) = −1200 + 12x, c̃ 00 (0) = −1200 < 0 et c̃ 00 (200) = 1200 > 0.
Conclusion x = 0 est un maximum de c̃ ; x = 200 est un minimum de c̃ et y(x) = 300 − x = 100. Pour minimiser
le coût le glacier doit produire 200 kilogrammes de glace de type A et 100 kilogrammes de glace de type B
pour un coût hebdomadaire de −7 939 400 ¤.
Méthode 2 On maximise le lagrangien L(x, y, λ) = c(x, y)−λ(x + y −300) = 600+200x +200y −2x 2 y −λ(x + y −300).
200 − 4x y − λ
   
0
2
Points critiques ∇L(x, y, λ) =  200 − 2x − λ , ∇L(x, y, λ) = 0 si et seulement si (x, y, λ) = (200, 100, −79800).
300 − x − y 0

188 © G. Faccanoni
Dernière mise à jour : Jeudi 5 juin 2014 5 Extrema

Classification Soit D(x, y, λ) ≡ ∂xx L(x, y, λ)∂ y y L(x, y, λ) − (∂x y L(x, y, λ))2 = −16x 2 . On a D(200, 100, −79800) <
0 : on ne peut pas conclure directement.
Étudions alors le signe de la différence

d (h, k) ≡ c(200 + h, 100 + k) − c(200, 100) = 8000000 + 200(h + k) − 2(200 + h)2 (100 + k)2

h et k étant liés par la relation (200 + h) + (100 + k) = 300, soit encore k = −h. On a alors d˜(h) = d (h, −h) =
2h 2 (300 + h) ≥ 0 lorsque h ' 0 : le point (200, 100) est alors un minimum local de c sous la contrainte
x + y = 300.
Conclusion (x, y) = (200, 100) est un minimum de c sous la contrainte x+y = 300 avec c(200, 100) = −7939400¤
Recherche dans l’ouvert. On minimise d’abord c(x, y) sans contraintes et on vérifie à posteriori si la solution obtenue
appartient à l’ensemble E˚ .
µ ¶ µ ¶
200 − 4x y 0
Points critiques ∇c(x, y) = , ∇c(x, y) = si et seulement si (x, y) = (10, 5).
200 − 2x 2 0
Classification D(x, y) = ∂xx c(x, y)∂ y y c(x, y) − (∂x y c(x, y))2 = (−4y) × (0) − (−4x)2 = −16x 2 donc D(10, 5) < 0.
Conclusion (x, y) = (10, 5) est un point-selle de c. La solution obtenue, bien que réalisable car 10+5 < 300 (i.e. (10, 5) ∈
E˚ ), ne minimise pas le coût.
Conclusion Pour une rente optimale on conseil au glacier d’utiliser à pleine capacité son laboratoire en produisant 200
kilogrammes de glace de type A et 100 kilogrammes de glace de type B .

Exercice 5.37
Une entreprise fabrique deux modèles de vélos de montagne : le modèle X est plus abordable et se vend 500¤ l’unité,
tandis que le modèle Y se vend 1000¤ l’unité. Les coûts totaux de fabrication (en ¤) sont exprimés par la fonction
c(x, y) = 5x 2 + 5y 2 − 52 x y + 10000 où x est le nombre de vélos du modèle X et y est le nombre de vélos du modèle Y ,
produits mensuellement. On suppose que chaque vélo produit peut être vendu sur le marché.
1. Écrire (x, y) 7→ p la fonction des profits totaux mensuels.
2. La capacité de production de l’entreprise est de 150 vélos par mois. En supposant que l’entreprise désire utiliser
à pleine capacité son usine, trouver la répartition de la production mensuelle permettant de maximiser les profits
(on utilisera la méthode que l’on préfère). Prouvez qu’il s’agit bien d’un maximum absolu et donnez la valeur du
profit mensuel.
3. Le patron de l’entreprise s’interroge sur la pertinence de vouloir produire à pleine capacité. Il se demande si la
solution qu’il obtiendrait sans cette contrainte serait plus intéressante. Aidez-le à répondre à cette question en
trouvant la solution qui maximise les profits sans cette contrainte. Prouvez qu’il s’agit bien d’un maximum absolu
et donnez la valeur du profit mensuel. La solution obtenue est-elle réalisable pour l’entreprise ?

C ORRECTION .
1. La fonction des profits totaux mensuels est p(x, y) = 500x + 1000y − c(x, y) = 500x + 1000y − 5x 2 − 5y 2 + 52 x y − 10000.
2. On veut produire 150 vélos par mois ce qui donne la contrainte x + y = 150. Il s’agit alors de résoudre le problème

maximiser p(x, y) sous la contrainte x + y = 150.

Pour cela, on peut utiliser l’une de deux méthodes suivantes :


Méthode 1 On maximise p̃(x) = p(x, 150−x) = −5x 2 −5(150−x)2 +500x +1000(150−x)+ 52 x(150−x)−10000 = − 25 2
2 x +
1375x + 47500.
Points critiques p̃ 0 (x) = −25x + 1375, p̃ 0 (x) = 0 si et seulement si x = 55.
Classification p̃ 00 (x) = −25, p̃ 00 (55) < 0.
Conclusion x = 55 est un maximum de p̃ donc (x, y) = (55, 95) est un maximum de p sous la contrainte x+y = 155
avec p(55, 95) = 65312, 50¤
Méthode 2 On maximise le lagrangien L(x, y, λ) = p(x, y) − λ(x + y − 150) = 500x + 1000y − c(x, y) = 500x + 1000y −
5x 2 − 5y 2 + 25 x y − 10000 − λ(x + y − 150).
500 − 10x + 52 y − λ
   
0
Points critiques ∇L(x, y, λ) = 1000 − 10y + 2 x − λ, ∇L(x, y, λ) = 0 si et seulement si (x, y, λ) = (55, 95, 375
5
2 ).
150 − x − y 0
Classification Soit D(x, y, λ) ≡ ∂xx L(x, y, λ)∂ y y L(x, y, λ)−(∂x y L(x, y, λ))2 = (−10)×(−10)−(5/2)2 . On a D(55, 95, 375
2 )>
0, ∂xx L(55, 95, 375 375
2 ) < 0 et ∂ y y L(55, 95, 2 ) < 0.

© G. Faccanoni 189
5 Extrema Dernière mise à jour : Jeudi 5 juin 2014

Conclusion (x, y) = (55, 95) est un maximum de p sous la contrainte x + y = 155 avec p(55, 95) = 65312, 50¤
3. On maximise le profit p(x, y) sans contraintes.
500 − 10x + 52 y
µ ¶ µ ¶
0
Points critiques ∇p(x, y) = , ∇p(x, y) = si et seulement si (x, y) = (80, 120).
1000 − 10y + 52 x 0
Classification D(x, y) = ∂xx p(x, y)∂ y y p(x, y)−(∂x y p(x, y))2 = (−10)×(−10)−(5/2)2 , D(80, 120) > 0 et ∂xx p(80, 120) < 0.
Conclusion (x, y) = (80, 120) est un maximum de p avec p(80, 120) = 70000¤
La solution obtenue n’est pas réalisable pour l’entreprise car elle dépasse la capacité de 150 vélos.

Exercice 5.38
Au ministère de l’agriculture, on a établi que le profit annuel (en ¤) pour les fermes cultivant des germes de soja et des
pistaches est exprimé par la fonction
p(x, y) = 600x + 800y − x 2 − 2y 2 − 2x y
où x représente le nombre d’acres plantés en germes de soja et y le nombre d’acres planté en pistaches.
1. Un fermier possède une terre de 500 acres. En supposant qu’il désire utiliser à pleine capacité ses terres, trouver la
répartition de la production permettant de maximiser son profit. Prouver qu’il s’agit bien d’un maximum absolu.
2. Le fermier s’interroge sur la pertinence de vouloir cultiver à pleine capacité ses terres. Il se demande si la solution
qu’il obtiendrait sans cette contrainte serait plus intéressante. Aidez-le à répondre à cette question en trouvant la
solution qui maximise le profit sans cette contrainte. Prouvez qu’il s’agit bien d’un maximum absolu. La solution
obtenue est-elle réalisable ?
3. En exploitant les résultats obtenus aux point précédent, suggériez-vous au fermier de diminuer la surface totale
consacrée à ces deux cultures ou d’utiliser à pleine capacité ses terres ?

C ORRECTION .
1. On veut cultiver 500 acres ce qui donne la contrainte x + y = 500. Il s’agit alors de résoudre le problème

maximiser p(x, y) sous la contrainte x + y = 500.

Pour cela, on peut utiliser l’une des deux méthodes suivantes :


Méthode 1 On maximise p̃(x) = p(x, 500−x) = 600x +800(500−x)−x 2 −2(500−x)2 −2x(500−x) = 800x −100000−x 2 .
Points critiques p̃ 0 (x) = 800 − 2x, p̃ 0 (x) = 0 si et seulement si x = 400.
Classification p̃ 00 (x) = −2, p̃ 00 (400) < 0.
Conclusion x = 400 est un maximum de p̃ donc (x, y) = (400, 100) est un maximum de p sous la contrainte
x + y = 500.
Méthode 2 On maximise le lagrangien L(x, y, λ) = p(x, y)−λ(x + y −500) = 600x +800y −x 2 −2y 2 −2x y −λ(x + y −500).
600 − 2x − 2y − λ
   
0
Points critiques ∇L(x, y, λ) =  800 − 4y − 2x − λ , ∇L(x, y, λ) = 0 si et seulement si (x, y, λ) = (400, 100, −400).

500 − x − y 0
Classification Soit D(x, y, λ) ≡ ∂xx L(x, y, λ)∂ y y L(x, y, λ) − (∂x y L(x, y, λ))2 = (−10) × (−10) − (5/2)2 .
On a D(400, 100, −400) > 0, ∂xx L(400, 100, −400) < 0 et ∂ y y L(400, 100, −400) < 0.
Conclusion (x, y) = (400, 100) est un maximum de p sous la contrainte x + y = 500 avec p(400, 100) = 60000¤
2. On maximise le profit p(x, y) sans contraintes.
µ ¶ µ ¶
600 − 2x − 2y 0
Points critiques ∇p(x, y) = , ∇p(x, y) = si et seulement si (x, y) = (200, 100).
800 − 4y − 2x 0
Classification D(x, y) = ∂xx p(x, y)∂ y y p(x, y) − (∂x y p(x, y))2 = (−2) × (−4) − (−2)2 , D(200, 100) > 0 et ∂xx p(200, 100) < 0.
Conclusion (x, y) = (200, 100) est un maximum de p. La solution obtenue est réalisable pour le fermier car elle ne
dépasse pas les 500 acres.
3. Comme p(400, 100) = 60000¤ < p(200, 100) = 100000¤, il est plus rentable pour le fermier de diminuer la surface totale
consacrée à ces deux cultures plutôt que d’utiliser à pleine capacité ses terres.

190 © G. Faccanoni
Dernière mise à jour : Jeudi 5 juin 2014 5 Extrema

Exercice 5.39
Un industriel produit simultanément 2 biens A et B dont il a le monopole de la production et de la vente dans un pays.
Soit x la quantité produite du premier bien et y la quantité produite du second. Les prix p A et p B auxquels il vend res-
pectivement les bien A et B sont fonction des quantités écoulées selon les relations p A (x) = 28 − 3x et p B (y) = 22 − 2y.
Le coût de production total des quantités x et y est la fonction c(x, y) = x 2 + 3y 2 + 4x y. Le bénéfice de l’entreprise si elle
vend les quantités x et y est donc la fonction

b(x, y) = xp A (x) + y p B (y) − c(x, y).

1. L’industriel peut produire simultanément au mieux 5 biens. En supposant que l’entreprise désire utiliser à pleine
capacité son usine, trouver la répartition de la production mensuelle permettant de maximiser le bénéfice de l’en-
treprise (on utilisera la méthode d’élimination ou réduction et on prouvera qu’il s’agit bien d’un maximum absolu).
Donner la valeur maximale du bénéfice ainsi que les prix de vente de chacun des biens.
2. L’industriel s’interroge sur la pertinence de vouloir produire à pleine capacité. Il se demande si la solution qu’il ob-
tiendrait sans cette contrainte serait plus intéressante. Aidez-le à répondre à cette question en trouvant la solution
qui maximise le bénéfice sans cette contrainte. Prouvez qu’il s’agit bien d’un maximum absolu et donnez la valeur
du bénéfice. La solution obtenue est-elle réalisable pour l’industriel ?

C ORRECTION .
1. On veut produire 5 biens ce qui donne la contrainte x + y = 5. Il s’agit alors de résoudre le problème

maximiser b(x, y) sous la contrainte x + y = 5.

On maximise b̃(x) = b(x, 5 − x) = −4x 2 − 5(5 − x)2 + 28x + 22(5 − x) − 4x(5 − x) = −5x 2 + 36x − 15.
Points critiques b̃ 0 (x) = −10x + 36, b̃ 0 (x) = 0 si et seulement si x = 18
5 .
¡ 18 ¢
Classification b̃ 00 (x) = −10, b̃ 00 5 < 0.
Conclusion x = 18
¡ 18
, 75 est un maximum de b sous la contrainte x + y = 5 avec
¢
est un maximum de b̃ donc (x, y) =
¡ 18 7 ¢ 2495 5
b 5 , 5 = 5 = 49, 8
2. On maximise le profit b(x, y) sans contraintes.
µ ¶ µ ¶
−8x − 4y + 28 0
Points critiques ∇b(x, y) = , ∇b(x, y) = si et seulement si (x, y) = (3, 1).
−10y − 4x + 22 0
Classification det(Hb )(x, y) = ∂xx b(x, y)∂ y y b(x, y)−(∂x y b(x, y))2 = (−8)×(−10)−(−4)2 = 64, det(Hb )(3, 1) > 0 et ∂xx b(3, 1) <
0.
Conclusion (x, y) = (3, 1) est un maximum de b avec b(3, 1) = 53
La solution obtenue ¡ est7 ¢réalisable pour l’industriel car elle ne dépasse pas la capacité de 5 biens et elle est préférable
car b(3, 1) = 53 > b 18 ,
5 5 = 249
5 = 49, 8.

Exercice 5.40
Un industriel produit simultanément 2 biens A et B dont il a le monopole de la production et de la vente dans un pays.
Soit x la quantité produite du premier bien et y la quantité produite du second. Les prix p A et p B auxquels il vend res-
pectivement les bien A et B sont fonction des quantités écoulées selon les relations p A (x) = 33 − 4x et p B (y) = 27 − y. Le
coût de production total des quantités x et y est la fonction c(x, y) = x 2 + 3y 2 + x y. Le bénéfice de l’entreprise si elle vend
les quantités x et y est donc la fonction

b(x, y) = xp A (x) + y p B (y) − c(x, y).

1. L’industriel peut produire simultanément au mieux 5 biens. En supposant que l’entreprise désire utiliser à pleine
capacité son usine, trouver la répartition de la production mensuelle permettant de maximiser le bénéfice de l’en-
treprise (on utilisera la méthode d’élimination ou réduction et on prouvera qu’il s’agit bien d’un maximum absolu).
Donner la valeur maximale du bénéfice ainsi que les prix de vente de chacun des biens.
2. L’industriel s’interroge sur la pertinence de vouloir produire à pleine capacité. Il se demande si la solution qu’il ob-
tiendrait sans cette contrainte serait plus intéressante. Aidez-le à répondre à cette question en trouvant la solution
qui maximise le bénéfice sans cette contrainte. Prouvez qu’il s’agit bien d’un maximum absolu et donnez la valeur
du bénéfice. La solution obtenue est-elle réalisable pour l’industriel ?

© G. Faccanoni 191
5 Extrema Dernière mise à jour : Jeudi 5 juin 2014

C ORRECTION .
1. On veut produire 5 biens ce qui donne la contrainte x + y = 5. Il s’agit alors de résoudre le problème

maximiser b(x, y) sous la contrainte x + y = 5.

Pour cela, on peut utiliser l’une de deux méthodes suivantes :


Méthode 1 On maximise b̃(x) = b(x, 5 − x) = −5x 2 − 4(5 − x)2 − x(5 − x) + 33x + 27(5 − x) = −8x 2 + 41x + 35.
41
Points critiques b̃ 0 (x) = −16x + 41, b̃ 0 (x) = 0 si et seulement si x = 16 .
¡ 41 ¢
Classification b̃ 00 (x) = −16, b̃ 00
16 < 0.
41
¡ 41 39
¢
Conclusion x = 16 est un maximum de b̃ donc (x, y) = 16 , 16 est un maximum de b sous la contrainte x + y = 5
avec b 41 39
¡ ¢ 2801
16 , 16 = 32 = 87, 53125
Méthode 2 On maximise le lagrangien L(x, y, λ) = b(x, y) − λ(x + y − 5) = −5x 2 − 4y 2 − x y + 33x + 27y − λ(x + y − 5).
−10x − y + 33 − λ
   
0
Points critiques ∇L(x, y, λ) =  −8y − x + 27 − λ , ∇L(x, y, λ) = 0 si et seulement si (x, y, λ) = 41 39 495
¡ ¢
16 , 16 , − 16 .
5−x − y 0
¡ 41 39 495 ¢
Classification Soit D(x, y, λ) ≡ ∂xx L(x, y, λ)∂ y y L(x, y, λ)−(∂x y L(x, y, λ))2 = (−10)×(−8)−(−1)2 . On a D 16 , 16 , − 16 >
¡ 41 39 495 ¢ ¡ 41 39 495 ¢
0, ∂xx L 16 , 16 , − 16 < 0 et ∂ y y L 16 , 16 , − 16 < 0.
¡ 41 39 ¢
, 16 est un maximum de b sous la contrainte x + y = 5 avec b 41 39 2801
¡ ¢
Conclusion (x, y) = 16 16 , 16 ) = 32 = 87, 53125
2. On maximise le profit b(x, y) sans contraintes.
µ ¶ µ ¶
−10x − y + 33 0
Points critiques ∇b(x, y) = , ∇b(x, y) = si et seulement si (x, y) = (3, 3).
−8y − x + 27 0
Classification det(Hb )(x, y) = ∂xx b(x, y)∂ y y b(x, y)−(∂x y b(x, y))2 = (−10)×(−8)−(−1)2 = 79, det(Hb )(3, 3) > 0 et ∂xx b(3, 3) <
0.
Conclusion (x, y) = (3, 3) est un maximum de b avec b(3, 3) = 90
La solution obtenue n’est pas réalisable pour l’industriel car elle dépasse la capacité de 5 biens.

192 © G. Faccanoni
6 Intégrales multiples
Dans ce chapitre nous allons étendre la notion d’intégrale définie aux intégrales doubles et triples des fonctions de deux et
trois variables. Ces notions sont ensuite exploitées pour calculer des volumes, des aires de surfaces, des masses et des centres
de gravité.

6.1 Intégrale double d’une fonction continue


Dans cette section, nous définissons l’intégrale d’une fonction de deux variables, appelée intégrale double, et nous montrons
comment l’évaluer. Elle nous permettra, entre autres, de calculer l’aire d’un domaine d’intégration, ainsi que le volume d’un
solide limité par les graphes de fonctions de deux variables.

Théorème de F UBINI ψ(x)


Soit x 7→ ϕ et x 7→ ψ deux fonctions continues sur [a, b] y
avec ϕ ≤ ψ ; notons Ω l’ensemble des points (x, y) ∈ R2
tels que a ≤ x ≤ b et ϕ(x) ≤ y ≤ ψ(x). Alors

Ï Z bZ ψ(x)
f (x, y) dx dy = f (x, y) dy dx.
Ω a ϕ(x)
ϕ(x)
Si le domaine le permet, on peut permuter les rôles de x a x
b
et de y : soit y 7→ ϕ et y 7→ ψ deux fonctions continues
sur [c, d ] avec ϕ ≤ ψ ; notons Ω l’ensemble des points
(x, y) ∈ R2 tels que c ≤ y ≤ d et ϕ(y) ≤ x ≤ ψ(y). Alors
y
Ï Z d Z ψ(y) ϕ(y)
f (x, y) dx dy = f (x, y) dx dy. d
Ω c ϕ(y)

Quelquefois, en inversant l’ordre de l’intégration sur un
domaine élémentaire, une intégrale double extrêmement c
difficile à évaluer devient relativement facile à résoudre. ψ(y)
x

193
6 Intégrales multiples Dernière mise à jour : Jeudi 5 juin 2014

Exemple
Soit R = [0; 1] × [0; 2] ; on veut calculer l’intégrale double
Ï
xe x y dx dy.
R

On a
Z 1Z 2 Z 1 µZ 2 Z 1 · x y ¸ y=2
e
Ï ¶
xe x y dx dy = xe x y dy dx = x e x y dy dx = x dx
R 0 0 0 0 0 x y=0
Z 1 Ã 2x ! " #x=1
e 2x e2 e2 3
Z 1
e 1 2x 1
= x − dx = e − 1 dx = −x = −1− +0 = − .
0 x x 0 2 2 2 2 2
x=0

Exemple
On veut calculer le volume du solide qui s’élève sur le domaine Ω du plan Ox y délimité par la droite d’équation y = 2x et la parabole
y = x 2 et couverte par le paraboloïde z = x 2 + y 2 .
Première méthode. Le domaine Ω peut être décrit par
n ¯ o
Ω = (x, y) ∈ R2 ¯ 0 ≤ x ≤ 2, x 2 ≤ y ≤ 2x .
¯

Le volume se calcule alors par


Z 2" # y=2x Z 2Ã !
y3 (2x)3 (x 2 )3
Ï Z 2 Z 2x
2 2 2 2 2 2
(x + y ) dx dy = (x + y ) dy dx = x y+ dx = x (2x) + − x 2 (x 2 ) − dx
Ω 0 x2 0 3 0 3 3
y=x 2
Z 2Ã 7 x=2
! " #
14x 3 4x6 7x 4 x 5 x 216
= −x + dx = − + = .
0 3 3 6 5 21 35
x=0

Deuxième méthode. Le domaine Ω peut être décrit par


n ¯ p o
Ω = (x, y) ∈ R2 ¯ 0 ≤ y ≤ 4, y ≤ x ≤ y/2 .
¯

Le volume se calcule alors par

Z 4" 3 #x=y/2 Z 4Ãp 3 !


(y/2)3
Z 4 Z y/2
x y
Ï
p
(x 2 + y 2 ) dx dy = p (x 2 + y 2 ) dx dy = + x y2 dy = + y y2 − − (y/2)y 2 dy
Ω 0 y 0 3 p 0 3 3
x= y
Z 4 3/2 Ã ! " # y=4
y 13y 3 2y 5/2 2y 7/2 13y 4 216
= + y 5/2 − dy = + − = .
0 3 24 15 7 96 35
y=0

y y p
y = x2 x= y

4 4
2 y
2x

x= 1
y=

2 x 2 x

Remarque Cas particulier


Si Ω est le rectangle Ω = (x, y) ∈ R2 ¯ a ≤ x ≤ b, c ≤ y ≤ d et si f est le produit d’une fonction qui ne dépend que de x et
© ¯ ª

d’une fonction qui ne dépend que de y, i.e. f (x, y) = h(x)g (y), alors
Z bZ d µZ b ¶ µZ d ¶
f (x, y) dy dx = h(x) dx g (y) dy .
a c a c

194 © G. Faccanoni
Dernière mise à jour : Jeudi 5 juin 2014 6 Intégrales multiples

Exemple
Î
Soit R = [0; 1] × [0; 2] ; on veut calculer l’intégrale double R x y dx dy. On a

Z 1Z 2 µZ 1 ¶ µZ 2 ¶ " 2 # y=1 " 2 # y=2


x y 1
Ï
x y dx dy = x y dy dx = x dx y dy = = .
R 0 0 0 0 2 2 2
y=0 y=0

Exemple

Soit Ω ⊂ R2 l’ensemble délimité par les courbes d’équation x = 12 , y


x y = 1 et y = x 2 . On veut calculer l’intégrale double Ω 1 dx dy. x = 1/2
Î

Cette intégrale mesure l’aire de la zone coloriée et on peut la


calculer par plusieurs méthodes. Pour cela, il faut tout d’abord
calculer les points d’intersection des courbes qui délimitent Ω : y = x2
Ω xy = 1

( ( (
x y = 1, x = 1/2, x = 1/2,
⇐⇒ (x, y) = (1, 1); ⇐⇒ (x, y) = (1/2, 1/4); ⇐⇒ (x, y) = (1/2, 2).
y = x2, y = x2, x y = 1,

Ω f (x, y) dx dy avec f (x, y) = 1 pour tout (x, y) ∈ Ω :


Î
1. On calcule l’intégrale double
B soit par
Z 1 Z 1/x Z 1 Z 1
1 1 1 7
Ï
£ ¤1/x
f (x, y) dx dy = 1 dy dx = y x 2 dx = − x 2 dx = ln(1) − + ln(2) + = ln(2) − ;
Ω 1/2 x 2 1/2 1/2 x 3 24 8
B soit par
Ï Z 1 Z py Z 2 Z 1/y Z 1 p Z 2
y 1/y
f (x, y) dx dy = 1 dx dy + 1 dx dy = [x]1/2 dy + [x]1/2 dy
Ω 1/4 1/2 1 1/2 1/4 1
" #1
y 3/2 y
Z 1 Z 2
p 1 1 1 h y i2 7
= y− dy + − dy = − + ln(y) − = ln(2) −
1/4 2 1 y 2 3/2 2
1/4
2 1 24

2. On calcul l’aire comme au lycée :


" #1
x3
Z 1 Z 1
1 2 1 1 1 7
dx − x dx = [ln(x)]1/2 − = ln(1) + ln(2) − + = ln(2) − .
1/2 x 1/2 3
1/2
3 24 24

Proposition Changement de variables


Soient ϕ : R2 → R et ψ : R2 → R deux bijections de classe C 1 . Considérons la fonction de changement de variables

F : R2 → R2
u, v 7→ x = ϕ(u, v), y = ψ(u, v)
¡ ¢ ¡ ¢

Soit f (x, y) une fonction continue sur un domaine D fermé et borné, alors
Ï Z
f (x, y) dx dy = f (ϕ(u, v), ψ(u, v)) · J du dv,
D D

∂u (ϕ) ∂v (ϕ)
· ¸
où J = ∂u (ϕ) · ∂v (ψ) − ∂v (ϕ) · ∂u (ψ), appelé jacobien, est le déterminant de la matrice appelée jacobienne
∂u (ψ) ∂v (ψ)
de la fonction F .

© G. Faccanoni 195
6 Intégrales multiples Dernière mise à jour : Jeudi 5 juin 2014

Cas des coordonnées polaires


Le changement de variables en coordonnées polaires est donné par l’application

r, ϑ 7→ x = a + r cos(ϑ), y = b + r sin(ϑ)
¡ ¢ ¡ ¢

où (r, ϑ) ∈ R∗+ × [0; 2π[ sont les coordonnées du point (x, y) ∈ R2 \ { (a, b) }.

y r
b ϑ

a x

Cette application a jacobien

∂r x ∂ϑ x
µ ¶ µ ¶
cos(ϑ) −r sin(ϑ)
J = det = det = r cos2 (ϑ) + r sin2 (ϑ) = r
∂r y ∂ϑ y sin(ϑ) r cos(ϑ)

donc Ï Ï
f (x, y) dx dy = f (a + r cos(ϑ), b + r sin(ϑ)) · r dr dϑ.
D D

Exemple

1 p
On veut intégrer la fonction f (x, y) = sur l’ensemble y= 3x
1 + x2 + y 2

n ¯ p o x2 + y 2 = 4
Ω = (x, y) ∈ R ¯ 0 < y < 3x et 1 < x 2 + y 2 < 4 .

y =0
Si on passe en coordonnées polaires on a
x2 + y 2 = 1
n ¯ π o
Ω = (r, ϑ) ∈ R∗
+ × [0; 2π[ ¯ 0 < ϑ < et 1 < r < 2 .
¯
3

On doit alors calculer


Z π/3 Z 2 µZ π/3 " #2
π ln(1 + r 2 ) π ln(5/2)
¶ µZ 2
r r

1
Z
2
r dr dϑ = dr dϑ = dϑ dr = = .
Ω 1+r 0 1 1+r2 0 1 1+r2 3 2
1
6

6.2 Intégrale triple


Principe
f étant continue sur un domaine fermé et borné D de R3 , l’intégrale triple
Ñ
f (x, y, z) dx dy dz
D

se définit de façon analogue aux intégrales doubles et se calcule par intégrations successives.

196 © G. Faccanoni
Dernière mise à jour : Jeudi 5 juin 2014 6 Intégrales multiples

Proposition Changement de variables


Le théorème est analogue au cas précédent : soit f (x, y, z) une fonction continue sur le domaine D fermé et borné, soit
ϕ, ψ et ξ trois bijections de D de classe C 1

x = ϕ(u, v, w), y = ψ(u, v, w), z = ξ(u, v, w)


¡ ¢ ¡ ¢
u, v, w →
7

alors Ñ Ñ
f (x, y, z) dx dy dz = f (ϕ(u, v, w), ψ(u, v, w), ξ(u, v, w)) · J du dv dw,
D D

J = ∂u ϕ · ∂v ψ · ∂w ξ − ∂u ϕ · ∂w ψ · ∂v ξ + ∂v ϕ · ∂w ψ · ∂u ξ − ∂v ϕ · ∂u ψ · ∂w ξ + ∂w ϕ · ∂u ψ · ∂v ξ − ∂w ϕ · ∂w ψ · ∂u ξ,
∂u ϕ ∂ v ϕ ∂ w ϕ
 

appelé jacobien, est le déterminant de la matrice ∂u ψ ∂v ψ ∂w ψ.


∂u ξ ∂v ξ ∂w ξ

Cas des coordonnées cylindriques


Un changement de variables en coordonnées cylindriques est donné par l’application

r, ϑ, z →
¡ ¢ ¡ ¢
7 x = a + r cos(ϑ), y = b + r sin(ϑ), z = z

où (r, ϑ, z) ∈ R∗+ × [0; 2π[×R sont les coordonnées du point (x, y, z) ∈ R3 \ { (0, 0, z) }. Cette application a jacobien J = r donc
Ñ Ñ
f (x, y, z) dx dy dz = f (a + r cos(ϑ), b + r sin(ϑ), z) · r dr dϑ dz.
D D

De la même manière on peut obtenir


Ñ Ñ
f (x, y, z) dx dy dz = f (a + r cos(ϑ), y, b + r sin(ϑ)) · r dr dy dϑ,
ÑD ÑD
f (x, y, z) dx dy dz = f (x, a + r cos(ϑ), b + r sin(ϑ)) · r dx dr dϑ.
D D

Cas des coordonnées sphériques


Le changement de variables en coordonnées sphériques est donné par l’application

r, ϕ, ϑ 7→ x = a + r cos(ϕ) cos(ϑ), y = b + r cos(ϕ) sin(ϑ), z = c + r sin(ϕ)


¡ ¢ ¡ ¢

où (r, ϑ, ϕ) ∈ R∗+ × [0; 2π[×[− π2 ; π2 [ sont les coordonnées du point (x, y, z) ∈ R3 \ { (0, 0, 0) }. Cette application a jacobien J =
r 2 cos(ϕ) donc
Ñ Ñ
f (x, y, z) dx dy dz = f (a + r cos(ϕ) cos(ϑ), b + r cos(ϕ) sin(ϑ), c + r sin(ϕ)) · r 2 cos(ϕ) dr dϑ dϕ.
D D

6.3 Applications

Définition Aire
Î
L’intégrale double A 1 dx dy mesure l’aire de A.

© G. Faccanoni 197
6 Intégrales multiples Dernière mise à jour : Jeudi 5 juin 2014

Exemple Aire d’un disque


Calculons l’aire d’un disque D R de rayon R > 0 : on se place dans un système de coordonnées centré sur le centre du disque, qui a
donc pour équation x 2 + y 2 ≤ R 2 . Alors
n ¯ o n ¯ o
D R = (x, y) ∈ R2 ¯ x 2 + y 2 ≤ R 2 = (r, ϑ) ∈ R∗
+ × [0; 2π[ ¯ r ≤ R
2
¯ ¯

et
Z 2π " 2 #R
R 2 2π
Z 2π Z R
r
Ï Z
1 dx dy = r dr dϑ = dϑ = 1 dϑ = πR 2 .
DR 0 0 0 2 2 0
0
Remarquons que dans ce cas on aurait pu utiliser directement ce qu’on a appris au premier semestre : si nous ne conservons que les
valeurs positives de y, on obtiendra un demi-disque, ensemble des points :
n ¯ p o
(x, y) ∈ R2 ¯ −R ≤ x ≤ R et 0 ≤ y ≤ R 2 − x 2 .
¯

p
C’est la surface sous la courbe de la fonction x 7→ f (x) = R 2 − x 2 dans l’intervalle [—R; R]. Comme c’est une fonction positive, l’aire
RR p
correspondante égale l’intégrale −R R 2 − x 2 dx. Pour calculer celle-ci, effectuons le changement de variable x = R cos(t ). On sou-
prend donc t variant dans l’intervalle [0; π] par exemple. Là, la fonction ϕ(t ) = R cos(t )
haite que x varie dans l’intervalle [−R; R], on p
est continûment dérivable sur [0; π]. De plus R 2 − x 2 = R sin(t ) car le sinus est positif ou nul sur [0; π]. On a donc, en effectuant ce
changement de variable
Z R p
R 2 − x 2 dx
−R x = R cos(t )
Z πq
dx = −R sin(t ) dt
= R 2 − R 2 cos( t )(−R sin(t ) dt
0
Z π
= − R2 sin2 t dt
0
Z π cos(2t ) = 1 − 2 sin2 t
1 − cos(2t )
= − R2 dt
0 2
¸π
t π
·
= − R2 − sin(2t ) = −R 2
2 0 2
C’est l’aire d’un demi-disque, donc l’aire du disque entier est πR 2 .

Définition Volume
Ð
L’intégrale triple V 1 dx dy dz mesure le volume de V .

Exemple Volume d’une sphère


Calculons le volume d’une boule B R de rayon R > 0 :

Z π/2 Z 2π Z R µZ π/2 " #R


¶ µZ 2π ¶ µZ R iπ/2 3 R3
2π r
Ñ ¶ h
2 2
1 dx dy dz = r cos(ϕ) dr dϑ dϕ = cos(ϕ) dϕ 1 dϑ r dr = sin(ϕ) [ϑ]0 = 4π .
BR −π/2 0 0 −π/2 0 0 −π/2 3 3
0

Remarquons que dans ce cas on aurait pu utiliser directement ce qu’on a appris pour les intégrales double : le volume d’une boule de
rayon R (sans restreindre la généralité nous la supposerons centrée à l’origine) est le double du volume contenu entre le plan z = 0 et
l’hémisphère nord de la sphère de rayon R centrée en (0, 0, 0). En résolvant l’équation 2 2 2 2
q de cette sphère x + y + z = R pour z, nous
pouvons décrire son hémisphère nord comme le graphe de la fonction f (x, y) = R 2 − x 2 − y 2 dont le domaine est le disque D R de
p
rayon R, intersection
p de la boule avec le plan z = 0. Ce disque est un domaine limité par les graphes des fonctions ϕ(x) = − R 2 − x 2
et ψ(x) = R 2 − x 2 sur l’intervalle [−R, R]. Le volume de la boule est donc
p
Z R Z pR 2 −x 2 q Z R · ¸ y= R 2 −x 2
1 y
Z
2 f (x, y) dy dx = p R 2 − x 2 − y 2 dy dx = y + (R 2 − x 2 ) arctan 2 p dx
DR −R − R 2 −x 2 −R 2 R − x 2 y=− R 2 −x 2
" #x=R
π 2 x3
Z R
2 2 4
= (R − x ) dx = R x − = πR 3 .
−R 2 3 3
x=−R

198 © G. Faccanoni
Dernière mise à jour : Jeudi 5 juin 2014 6 Intégrales multiples

Définition Masse
Î
Si f (x, y) est la densité au point (x, y), l’intégrale double AÐ f (x, y) dx dy est la masse de la partie A. De manière analogue,
si f (x, y, z) est la densité au point (x, y, z), l’intégrale triple V f (x, y, z) dx dy dz est la masse de la partie A.

Exemple
Si une charge électrique est répartie sur une région A et si la densité de charge (en unités par unités carrées) est donnée par σ(x, y) en
un point (x, y) de A, alors la charge totale Q est donnée par
Ï
σ(x, y) dx dy.
A

Par exemple, supposons qu’une charge est distribuée sur le do- y


maine A de la figure ci-contre de telle sorte que la densité de
charge en (x, y) est σ(x, y) = x y, mesurée en coulombs par mètre y =1
1
carré (C m−2 ). Alors la charge totale est
Z 1 " 2 # y=1 y

x =1
Z 1Z 1
y
Ï
=
Q= σ(x, y) dx dy = x y dy dx = x dx 1−
A 0 1−x 0 2 x
y=1−x
1 1 ³ 1 1³ 2 1 2 3 1 4 1
· ¸
5
Z ´ Z ´
= x 1 − (1 − x)2 dx = 2x − x 3 dx = x − x = . 1 x
2 0 2 0 2 3 4 0 24
5
La charge totale est donc de 24 C.

Définition Centre de gravité


Si f (x, y) est la densité au point (x, y), le centre de gravité de la partie A se trouve en (xG , yG ) ainsi définis
Î Î
A x f (x, y) dx dy y f (x, y) dx dy
xG = Î , yG = ÎA .
A f (x, y) dx dy A f (x, y) dx dy

Physiquement cela signifie que la plaque A se comporte comme si toute sa masse était concentrée en son centre d’inertie.
Par exemple, elle est en équilibre horizontal lorsqu’elle repose sur son centre d’inertie.
De manière analogue, si f (x, y, z) est la densité au point (x, y, z), le centre de gravité de la partie A se trouve en (xG , yG , zG )
ainsi définis
Ð Ð Ð
A x f (x, y, z) dx dy dz A y f (x, y, z) dx dy dz z f (x, y, z) dx dy dz
xG = Ð , yG = Ð , zG = ÐA .
A f (x, y, z) dx dy dz A f (x, y, z) dx dy dz A f (x, y, z) dx dy dz

Exemple
On veut déterminer la masse et le centre d’inertie d’une fine plaque de métal triangulaire dont les sommets sont en (0, 0), (1, 0) et (0, 2),
sachant que la fonction densité est f (x, y) = 1 + 3x + y.

Le triangle est représenté dans la figure ci-contre (l’équation de la y


frontière supérieure est y = 2−2x). La masse de la plaque de métal
est 2
Ï Z 1 Z 2−2x
m= f (x, y) dx dy = (1 + 3x + y) dy dx
y=

A 0 0
2−

Z 1" # y=2−2x Z 1" #


y2 (2 − 2x)2
2x

= y + 3x y + dx = 2 − 2x + 3x(2 − 2x) + dx
0 2 0 2
y=0
#1
"
x3
Z 1
2 8
=4 (1 − x ) dx = 4 x − = . x
0 3 3 1
0

© G. Faccanoni 199
6 Intégrales multiples Dernière mise à jour : Jeudi 5 juin 2014

Le moment de la plaque de métal entière par rapport à l’axe Ox est


Z 1" # y=2−2x " #1
x y2 x2 x4
Ï Z 1 Z 2−2x Z 1
x f (x, y) dx dy = x(1 + 3x + y) dy dx = x y + 3x 2 y + dx = 4 (x − x 3 ) dx = 4 − =1
A 0 0 0 2 0 2 4
y=0 0

et le moment de la plaque de métal entière par rapport à l’axe O y est


Z 1" 2 # y=2−2x
y2 y3
Z 1 Z 2−2x
y 2 1
Ï Z
y f (x, y) dx dy = y(1 + 3x + y) dy dx = + 3x + dx = (7 − 9x − 3x 2 + 5x 3 ) dx
A 0 0 0 2 2 3 3 0
y=0
· ¸1
2 9 2 5 11
= 7x − x − x 3 + x 4 = .
3 2 4 0 6
11
µ ¶ ³ ´
Les coordonnées du centre d’inertie sont donc 18 , 68 = 38 , 11
16 .
3 3

6.4 Quelques intégrales remarquables

sµZ sZ s
Z +∞ +∞ ¶ µZ +∞ ¶ +∞ Z +∞ Z +∞ Z π/2
−x 2 −x 2 −y 2 −x 2 −y 2 2
e dx = e dx e dy = e dy dx = r e −r dθ dr
0 0 0 0 0 0 0
· −r 2 ¸+∞ p
s s
π +∞ π e π
Z
2
= r e −r dr = − = .
2 0 2 2 0 2
Z +∞ p 2
e −x dx =
π.
−∞
π
Z +∞ r
2
e −αx dx = .
−∞ α

200 © G. Faccanoni
Dernière mise à jour : Jeudi 5 juin 2014 6 Intégrales multiples

............... Exercices ..............


Intégrales doubles, coordonnées cartésiennes

Exercice 6.1
Soit D la région comprise entre le graphe de la fonction d’équation y = sin(x) et le graphe de la fonction
Î y = cos(x) pour
x entre 0 et π4 . Décrire le domaine d’intégration D en coordonnées cartésiennes et calculer l’intégrale D (y + 1) dx dy.

C ORRECTION . D = (x, y) ∈ R2 ¯ 0 ≤ x ≤ π4 , sin(x) ≤ y ≤ cos(x)


© ¯ ª

Ï Z π/4 Z cos(x) Z π/4 · y 2 ¸cos(x)


(y + 1) dx dy = (y + 1) dy dx = +y dx
D 0 sin(x) 0 2 sin(x) y
Z π/4 µ 2
cos (x) − sin (x) 2 ¶
= + cos(x) − sin(x) dx sin(x)
0 2 cos(x)
Z π/4 µ ¶
cos(2x)
= + cos(x) − sin(x) dx 0 π x
0 2 4
¸π/4
p
·
sin(2x) 3
= + sin(x) + cos(x) = 2− .
4 0 4

y
Exercice 6.2
Calculer
2
Ï
y dx dy D
D 1
où l’ensemble d’intégration est
esquissé dans la figure ci-contre. x
−3 −2 −1 0 1 2 3 4

C ORRECTION .
½ ¯ ¾
2 2
D = (x, y) ∈ R2 ¯ −3 ≤ x ≤ −2, −2x − 4 ≤ y ≤ 2 ∪ (x, y) ∈ R2 ¯ −2 ≤ x ≤ 1, 0 ≤ y ≤ 2 ∪ (x, y) ∈ R2 ¯¯ 1 ≤ x ≤ 4, x − ≤ y ≤ 2
© ¯ ª © ¯ ª ¯
3 3

ou encore, ce qui est plus simple,


½ ¯ ¾
1 3
D = (x, y) ∈ R2 ¯¯ 0 ≤ y ≤ 2, − y − 2 ≤ x ≤ y + 1 ,
¯
2 2

donc
3
2 y+1
Z 2Z Z 2h ix= 3 y+1 2 34
Ï Z
2
y dy dx = y dx dy = yx dy = y(2y + 3) dy = .
D 0 − 12 y−2 0 x=− 12 y−2 0 3

y
Exercice 6.3
Calculer 1
Ï
x y dx dy
D −2 −1 0 1 x

où l’ensemble d’intégration est −1


colorié dans la figure ci-contre.

−2

© G. Faccanoni 201
6 Intégrales multiples Dernière mise à jour : Jeudi 5 juin 2014

C ORRECTION .
D = (x, y) ∈ R2 ¯ −1 ≤ x ≤ 0, 0 ≤ y ≤ x + 1 ∪ (x, y) ∈ R2 ¯ 0 ≤ x ≤ 1, x − 1 ≤ y ≤ 0
© ¯ ª © ¯ ª

donc
Ï Z 0 Z x+1 Z 1Z 0
y dy dx = x y dy dx + x y dy dx =
D −1 0 0 x−1
y 2 ix+1 1 h y 2 i0
Z 0h Z
= x dx + x dx
−1 2 0 0 2 x−1
1 0 3
Z 1
1
Z
= x + 2x 2 + x dx − x 3 − 2x 2 + x dx
2 −1 2 0
1 h x 4 2x 3 x 2 i0 1 h x 4 2x 3 x 2 i1 1
= + + − − + =− .
2 4 3 2 −1 2 4 3 2 0 12

y
Exercice 6.4 La baleine
Calculer l’intégrale double y = |x + 1|
Ï x = −2 x2 + y2 = 1
x 2 y dx dy.
D x
y = −x − 2 D

y = −1

C ORRECTION . L’ensemble d’intégration peut être décrit comme la réunion des trois domaines disjoints suivants :

D1 = (x, y) ∈ R2 ¯ −2 ≤ x < −1, −x − 2 ≤ y < −x − 1 ,


© ¯ ª
y = |x + 1|
D2 = (x, y) ∈ R2 ¯ −1 ≤ x < 0, −1 ≤ y < x + 1 ,
© ¯ ª
x = −2 x2 + y2 = 1
D1 D3
D3 = (x, y) ∈ R2 ¯ x ≥ 0, x 2 + y 2 ≤ 1
© ¯ ª
x
n ¯ p p o y = −x − 2 D2
= (x, y) ∈ R2 ¯ 0 ≤ x ≤ 1, − 1 − x 2 ≤ y ≤ 1 − x 2
¯
½ ¯ q ¾ y = −1
= (x, y) ∈ R2 ¯¯ 0 ≤ y ≤ 1, 0 ≤ x ≤ 1 − y 2
¯

n ¯ π πo
= (r, ϑ) ∈ R+ × [−π; π[ ¯ 0 < r ≤ 1, − ≤ ϑ ≤ .
¯
2 2

Par définition on a Ï Ï Ï Ï
x 2 y dx dy = x 2 y dx dy + x 2 y dx dy + x 2 y dx dy.
D D1 D2 D3

Calculons chaque intégrale :


B D1 x 2 y dx dy :
Î

¸ y=−x−1
−1 Z −x−1 −1 y2 −1 (−x − 1)2 (−x − 2)2
Ï Z Z · Z µ ¶
x 2 y dx dy = x 2 y dy dx = x2 dx = x2 − dx
D1 −2 −x−2 −2 2 y=−x−2 −2 2 2
· 4 ¸x=−1
x3
Z −1
3 x 1
=− x 3 + x 2 dx = − + =
−2 2 4 2 x=−2 4

x 2 y dx dy :
Î
B D2

¸ y=x+1
0 x+1 0 y2 0 (x + 1)2 (−1)2
Ï Z Z Z · Z µ ¶
x 2 y dx dy = x 2 y dy dx = x2 dx = x2 − dx
D2 −1 −1 −1 2 y=−1 −1 2 2
¸x=0
1 0 4 1 x5 x4
·
3
Z
= x + 2x 3 dx = + =−
2 −1 2 5 2 x=−1 20

202 © G. Faccanoni
Dernière mise à jour : Jeudi 5 juin 2014 6 Intégrales multiples

x 2 y dx dy = 0 car le domaine est symétrique par rapport à l’axe des abscisses et la fonction à intégrer est impaire par
Î
B D3
rapport à y. Si on n’a pas remarqué la symétrie, il suffit d’utiliser les coordonnées polaires :

π/2 π/2 ¸1 · ¸π/2


r5 cos3 (ϑ)
Ï Z 1Z µZ 1 ¶ µZ ¶ ·
2
¡ 4
r cos2 (ϑ) sin(ϑ) dϑ dr = 4 2
¢
x y dx dy = r dr cos (ϑ) sin(ϑ) dϑ = − =0
D3 0 −π/2 0 −π/2 5 0 3 −π/2
En conclusion,
1 3 1
Ï
x 2 y dx dy. = − +0 = .
D 4 20 10

Exercice 6.5 y
Calculer l’intégrale double
Ï y = |x|
x y 2 dx dy. (x + 1)2 + y 2 = 1 D (x − 1)2 + y 2 = 1
D
−2 −1 0 1 2 x
y = −|x|

C ORRECTION . Il s’agit de la réunion des quatre domaines :

D1 = (x, y) ¯ x ≤ −1, (x + 1)2 + y 2 ≤ 1 ,


© ¯ ª

D2 = (x, y) ¯ −1 ≤ x ≤ 0, x ≤ y ≤ −x ,
© ¯ ª
D1 D2 D3 D4
D3 = (x, y) ¯ 0 ≤ x ≤ 1, −x ≤ y ≤ x , 2 x
© ¯ ª
−2 −1 0 1
D4 = (x, y) ¯ x ≥ 0, (x − 1)2 + y 2 ≤ 1 .
© ¯ ª

Par définition on a
Ï Ï Ï Ï Ï
2 2 2 2
x y dx dy = x y dx dy + x y dx dy + x y dx dy + x y 2 dx dy.
D D1 D2 D3 D4

Calculons chaque intégrale :

3π/2 Z 1 3π/2 ¶ · 3 ¸ϑ=3π/2 · 5 ¸r =1


1r
µZ ¶ µZ
sin (ϑ) 2
Ï Z
x y 2 dx dy = r 4 cos(ϑ) sin2 (ϑ) dr dϑ = cos(ϑ) sin2 (ϑ) dϑ
r 4 dr = =−
D1 π/2 0 π/2 0 3 ϑ=π/2 5 r =0 15
Z 0 Z −x Z 0 · 3 ¸ y=−x Z 0 µ 3 3¶ Z 0 · 5 ¸x=0
y (−x) (x) 2 2 x 2
Ï
x y 2 dx dy = x y 2 dy dx = x dx = x − dx = − x 4 dx = − =−
D2 −1 x −1 3 y=x −1 3 3 3 −1 3 5 x=−1 15
Z 1 · 3 ¸ y=x Z 1 µ 3 · ¸x=1
(−x)3 2 x5
Z 1Z x
y x 2 1 4

2
Ï Z
x y 2 dx dy = x y 2 dy dx = x dx = x − dx = x dx = =
D3 0 −x 0 3 y=−x 0 3 3 3 0 3 5 x=0 15
Z π/2 Z 1 µZ π/2 ¶ µZ 1 ¶ · 3 ¸ϑ=π/2 · 5 ¸r =1
sin (ϑ) r 2
Ï
x y 2 dx dy = r 4 cos(ϑ) sin2 (ϑ) dr dϑ = cos(ϑ) sin2 (ϑ) dϑ r 4 dr = =
D4 −π/2 0 −π/2 0 3 ϑ=−π/2 5 r =0 15

En conclusion, Ï
x y 2 dx dy = 0.
D
Ce résultat était prévisible car le domaine est symétrique par rapport à l’axe des ordonnées et la fonction à intégrer est impaire
par rapport à x.

Exercice 6.6 Distance moyenne parcourue à pied pour rejoindre une porte d’embarquement
Dans les aéroports, les portes d’embarquements dans un terminal sont souvent alignées. Si on arrive à une porte d’em-
barquement et on doit rejoindre une autre porte d’embarquement pour une correspondance, quelle proportion de la
longueur du terminal doit-on parcourir en moyenne ? Un manière de modéliser cette situation est de choisir au hasard

© G. Faccanoni 203
6 Intégrales multiples Dernière mise à jour : Jeudi 5 juin 2014

R1R1
deux nombres x ∈ [0; 1], y ∈ [0; 1] et de calculer la valeur moyenne de |x − y|, i.e. 0 0 |x − y| dx dy. Calculer cette intégrale.

C ORRECTION .
¸ y=1 ¸ y=x
x y2 y2
Z 1Z 1 Z 1Z Z 1Z 1 Z 1· Z 1·
|x − y| dx dy = (x − y) dy dx + (y − x) dy dx = xy −
−xy dx dx +
0 0 0 0 0 x 0 y=0 0 2 2y=x
Z 1µ 2¶ ¸x=1
x2 x2 x3
Z 1µ Z 1µ
x
¶ ¶ ·
1 1 2 1 1
= dx + −x + dx = − x + x dx = x− + = .
0 2 0 2 2 0 2 2 2 3 x=0 3

Exercice 6.7
xy
f (x, y) dx dy où D = {(x, y) ∈ R2 , 0 ≤ x ≤ 1, 0 ≤ y ≤ 1, 1 ≤ x 2 + y 2 } et f (x, y) =
Î
Calculer l’intégrale double D 1+x 2 +y 2
.

C ORRECTION . La partie D de R2 est l’intersection du carré [0; 1] × [0; 1] et de l’extérieur du cercle de centre (0, 0) et de rayon
1. La fonction f est continue sur D.

1 x

À l’aide du théorème de Fubini (et du dessin pour ne pas se tromper sur les bornes) on peut écrire
Z 1Z 1 Z 1 Z 1
xy xy x 2y
Ï
I= dx dy = p dy dx = p dy dx
D 1 + x2 + y 2 0
2
1−x 2 1 + x + y
2
0 2
2
1−x 2 1 + x + y
2

x2
Z 1 Z 1 Z 1
x£ x£ x
µ ¶
2 2 1 2
¤ ¤
= p
ln(1 + x + y ) 1−x 2 dx = ln(2 + x ) − ln(2) dx = ln 1 + dx
0 2 0 2 0 2 2
Z 1
1 2 1 1 3 3 1
= ln(1 + u) du = [(1 + u) ln(1 + u) − u]02 = ln − .
2 0 2 4 2 4

204 © G. Faccanoni
Dernière mise à jour : Jeudi 5 juin 2014 6 Intégrales multiples

Calcul d’aires et centres de gravité, coordonnées cartésiennes


Exercice 6.8 Population de lapins y [km]

Une forêt est à coté d’une route (voir la figure ci-contre). Route
La densité de la population de lapins qui vivent dans la 5
forêt est proportionnelle à la distance de la route. Plus
précisément, elle vaut 0 au niveau de la route et vaut 10 Forêt
lapins par kilomètre carré au coté opposé de la forêt. Es-
timer la population totale de lapins dans la forêt.
−2 0 6 8 x [km]

C ORRECTION .
B La forêt (domaine d’intégration) s’écrit
½ ¯ ¾ ½ ¯ ¾
5 5
(x, y) ∈ R2 ¯¯ −2 ≤ x ≤ 0, − x ≤ y ≤ 5 ∪ (x, y) ∈ R2 ¯ 0 ≤ x ≤ 6, 0 ≤ y ≤ 5 ∪ (x, y) ∈ R2 ¯¯ 6 ≤ x ≤ 8, x − 15 ≤ y ≤ 5
¯ © ¯ ª ¯
2 2
ou encore, ce qui est plus simple, ½ ¯ ¾
2 2 2
(x, y) ∈ R ¯ 0 ≤ y ≤ 5, − y ≤ x ≤ (15 + y)
¯
¯
5 5
B La densité est une fonction affine de la seule variable y (comme la route est horizontale, la distance ne dépend que de la
coordonnées y), i.e. δ(y) = m y + q avec m et q à déterminer. On sait que δ(5) = 0 et δ(0) = 10 donc δ(y) = 2(5 − y).
B La population totale de lapins qui vivent dans la forêt est
Z 5Z 2 2
5 (15+y) 5 5 (15+y) 5 2 4 5 550
Z Z Z Z
δ(y) dx dy = 2 (5 − y) 1 dx dy = 2 (5 − y) (15 + 2y) dy = 75 − 5y − 2y 2 dy = .
0 − 25 y 0 − 25 y 0 5 5 0 3

Exercice 6.9
y

4
Soit D la plaque homogène représentée dans la figure. Sans
faire de calculs d’intégrales, en déduire les valeurs de 3
Ï Ï Ï 2
1 dx dy, x dx dy, y dx dy.
D D D 1
0 x
0 1 2 3 4

C ORRECTION . L’aire mesure 7 et le centre de gravité a coordonnées (5/2, 5/2) donc


Ï Ï Ï
1 dx dy = 7, x dx dy = 35/2, y dx dy = 35/2.
D D D

Exercice 6.10

y
Soit D la plaque homogène représentée dans la figure. Sans 6
faire de calculs d’intégrales, en déduire les valeurs de 5
4

Ï Ï Ï 3

1 dx dy, x dx dy, y dx dy. 2


D D D 1
0
0 1 2 3 4 5 6x

© G. Faccanoni 205
6 Intégrales multiples Dernière mise à jour : Jeudi 5 juin 2014

C ORRECTION . L’aire mesure 9 + π et le centre de gravité a coordonnées (7/2, 7/2) donc


Ï Ï Ï
1 dx dy = 9 + π, x dx dy = (9 + π)7/2, y dx dy = (9 + π)7/2.
D D D

Exercice 6.11
Calculer l’aire de la région coloriée en figure
y = x+2
2y = x + 3

3y = −x − 6

C ORRECTION . Soit D le domaine. Pour calculer l’aire on peut soit utiliser un simple calcul géométrique soit se rappeler que
l’aire est donnée par l’intégrale double Ï
1 dx dy.
D

3 2y = x + 3
4
4 2

1
2
36 − 4 − 4 − 2 − 6 = 20 −3 −2 −1 0 1 2 3
−1

−2
6 y = x+2
−3 3y = −x − 6

Pour ce calcul on décompose la région en deux parties et on obtient


1 3
Ï Z −1 Z x+2 Z 3 Z
2 x+ 2
1 dx dy = 1 dy dx + 1 dy dx
D −3 − 13 x−2 −1 − 31 x−2
Z −1 1
Z 3 1 3 1
Z −1 4
Z 3 5 7
= x + 2 + x + 2 dx + x + + x + 2 dx = x + 4 dx + x + dx = 20.
−3 3 −1 2 2 3 −3 3 −1 6 2

Exercice 6.12
Calculer l’aire de la région coloriée en figure.
y = −x + 2
2y = −x + 3

3y = x − 6

206 © G. Faccanoni
Dernière mise à jour : Jeudi 5 juin 2014 6 Intégrales multiples

C ORRECTION . Soit D le domaine. Pour calculer l’aire on peut soit utiliser un simple calcul géométrique soit se rappeler que
l’aire est donnée par l’intégrale double Ï
1 dx dy.
D

2y = −x + 3 3
4
4 2

1
2
36 − 4 − 4 − 2 − 6 = 20 −3 −2 −1 0 1 2 3
−1

−2
6 y = −x + 2
3y = x − 6 −3

Pour la calculer on décompose la région en deux parties et on obtient


Ï Z 1 Z − 21 x+ 32 Z 3Z −x+2
1 dx dy = 1 dy dx + 1 dy dx
1 1
D −3 3 x−2 1 3 x−2
Z
11 3 1
Z 3 1
Z 1 5 7
Z 3 4
= − x + − x + 2 dx + −x + 2 − x + 2 dx = − x + dx + − x + 4 dx = 20.
−3 2 2 3 1 3 −3 6 2 1 3

Exercice 6.13
Calculer l’aire de la région coloriée en figure.

3y = x + 6

2y = −x − 3
y = −x − 2

C ORRECTION . Soit D le domaine. Pour calculer l’aire on peut soit utiliser un simple calcul géométrique soit se rappeler que
l’aire est donnée par l’intégrale double Ï
1 dx dy.
D

3 3y = x + 6
6 y = −x − 2
2

1
36 − 4 − 4 − 2 − 6 = 20
2 −3 −2 −1 0 1 2 3
−1

4 −2
4
−3 2y = −x − 3

Pour la calculer on décompose la région en deux parties et on obtient


1 1
−1 Z 3 x+2 3 3 x+2
Ï Z Z Z
1 dx dy = 1 dy dx + 1 dy dx
D −3 −x−2 −1 − 21 x− 32
Z −1 1
Z 3 1 1 3
Z −1 4
Z 3 5 7
= x + 2 + x + 2 dx + x + 2 + x + dx = x + 4 dx + x + dx = 20.
−3 3 −1 3 2 2 −3 3 −1 6 2

© G. Faccanoni 207
6 Intégrales multiples Dernière mise à jour : Jeudi 5 juin 2014

Exercice 6.14
Calculer l’aire de la région coloriée en figure.

3y = −x + 6

2y = x − 3
y = x−2

C ORRECTION . Soit D le domaine. Pour calculer l’aire on peut soit utiliser un simple calcul géométrique soit se rappeler que
l’aire est donnée par l’intégrale double Ï
1 dx dy.
D

3y = −x + 6 3
6 y = x−2
2

1
36 − 4 − 4 − 2 − 6 = 20
2 −3 −2 −1 0 1 2 3
−1

4 −2
4
2y = x − 3 −3

Pour la calculer on décompose la région en deux parties et on obtient


Ï Z −1 Z − 31 x+2 Z 3 Z − 13 x+2
1 dx dy = 1 dy dx + 1 dy dx
1 3
D −3 2 x− 2 −1 x−2
Z −1 1 1 3
Z 3 1 1 3
Z −1 5 7
Z 3 4
= − x + 2 − x + dx + − x + 2 − x + dx = − x + dx + − x + 4 dx = 20.
−3 3 2 2 −1 3 2 2 −3 6 2 −1 3

Exercice 6.15
Calculer l’aire de la région coloriée en figure.
y
3y = −x + 6

2y = x − 3

y = x−2

C ORRECTION . Soit D le domaine. Pour calculer l’aire on peut soit utiliser un simple calcul géométrique soit se rappeler que
l’aire est donnée par l’intégrale double Ï
1 dx dy.
D

208 © G. Faccanoni
Dernière mise à jour : Jeudi 5 juin 2014 6 Intégrales multiples

y
3y = −x + 6 3

6 2

1 2y = x − 3
48 − 8 − 8 − 1 − 6 = 25
−3 −2 −1 0 1 2 3 x
1 −1

−2

−3

−4
8
8 −5

y = x−2

Pour la calculer on décompose la région en deux parties et on obtient


Ï Z −1 Z − 31 x+2 Z 3 Z − 13 x+2
1 dx dy = 1 dy dx + 1 dy dx
1 3
D −3 x−2 −1 2 x− 2
Z −1 1
Z 31 1 3
Z −1 4
Z 3
5 7
= − x + 2 − x + 2 dx + − x + 2 − x + dx = − x + 4 dx + − x + dx = 25.
−3 3 −1 3 2 2 −3 3 −1 6 2

Exercice 6.16
y
1

Calculer les coordonnées du centre de gravité de la plaque supposée homo-


−1 1x gène dessinée ci-contre (vous pouvez éviter le calcul d’intégrales, mais justifiez
−1 votre raisonnement).

−2

C ORRECTION . Pour calculer les coordonnées (xG , yG ) du centre de gravité on doit calculer d’abord les trois intégrales doubles
suivantes :
Ï Z 0 Z −x Z 1Z x Z 0 Z 1
1 dy dx = dy dx + dy dx = 2 dx + 2 dx = 4,
D −1 −x−2 0 x−2 −1 0
Ï Z 0 Z −x Z 1Z x Z 0 Z 1
x dy dx = x dy dx + x dy dx = 2x dx + 2x dx = 0,
D −1 −x−2 0 x−2 −1 0
Ï Z 0 Z −x Z 1Z x 1
Z 0 1
Z 1
y dy dx = y dy dx + y dy dx = (−4x − 4) dx + (4x − 4) dx
D −1 −x−2 0 x−2 2 −1 2 0
Z 0 Z 1
= −2 (x + 1) dx + 2 (x − 1) dx = −2
−1 0

et on en déduit que Î Î
D x dy dx y dy dx 1
xG = Î = 0, yG = ÎD =− .
D 1 dy dx D 1 dy dx 2

En fait, aucune intégrale n’est nécessaire car

© G. Faccanoni 209
6 Intégrales multiples Dernière mise à jour : Jeudi 5 juin 2014

B la première intégrale correspond à l’aire de la figure qu’on peut calculer géométriquement,


B la deuxième intégrale est forcément nulle car le centre de gravité appartient à l’axe des y par symé-
trie donc xG = 0, y
1
B la troisième intégrale n’est pas nécessaire car pour calculer yG . Il suffit en effet de découper la plaque
comme dans la figure ci-contre : la plaque supérieure a centre de gravité en (0, 0), la plaque inférieure
−1 1x
a centre de gravité en (−1, 0) et les deux plaques ont la même aire donc la plaque totale a centre de −1
gravité en (0, −0,5) (ce raisonnement permet en effet de calculer directement xG et yG ) :
−2
Aire supérieure Aire inférieure 1 1 1
yG = yG supérieure + yG inférieure = 0 + (−1) = −
Aire totale Aire totale 2 2 2

Exercice 6.17
Calculer les coordonnées du centre de gravité G de la plaque supposée homogène dessinée ci-dessous (éviter le calcul
d’intégrales lorsqu’il est possible, mais justifier le raisonnement) :

−1 G 1

−1

C ORRECTION . Pour calculer les coordonnées (xG , yG ) du centre de gravité on doit calculer d’abord les trois intégrales doubles
suivantes :
Ï Z 0 Z −x Z 1Z x Z 0 Z 1
1 dy dx = dy dx + dy dx = (−x + 1) dx + (x + 1) dx = 3,
D −1 −1 0 −1 −1 0
Ï Z 0 Z −x Z 1Z x Z 0 Z 1
2
x dy dx = x dy dx + x dy dx = (−x + x) dx + (x 2 + x) dx = 0,
D −1 −1 0 −1 −1 0
¸1
0 x 1 x3
−x Z 1Z 0 1 1 ·
1 1 1 2
Ï Z Z Z Z Z
y dy dx = y dy dx + y dy dx = (x 2 − 1) dx + 2
(x − 1) dx = 2
(x − 1) dx = −x =−
D −1 −1 0 −1 2 −1 2 0 2 −1 2 3 −1 3

et on en déduit que Î Î
D x dy dx y dy dx 2
xG = Î = 0, yG = ÎD =− .
D 1 dy dx D 1 dy dx 9
En fait, aucune des trois intégrales est nécessaire car
B la première intégrale correspond à l’aire de la figure qu’on peut calculer géométriquement,
B la deuxième intégrale est forcément nulle car le centre de gravité appartient à l’axe des y par symétrie donc xG = 0,
B la troisième intégrale n’est pas nécessaire car pour calculer yG . Il suffit en effet de découper la plaque en deux (la plaque
supérieur correspond aux points d’ordonnée positive, la plaque inférieure aux points d’ordonnée négative). La plaque
supérieure a aire 1 et centre de gravité en (0, 1/3), la plaque inférieure a aire 2 centre de gravité en (0, −1/2), donc la
plaque totale a aire 3 et centre de gravité en (0, −2/9) car

Aire supérieure Aire inférieure 1 1 1 2 1


yG = yG supérieure + yG inférieure = − =−
Aire totale Aire totale 33 23 2

Exercice 6.18 Aire d’une châtaigne


Calculer l’aire de
D= (x, y) ∈ R2 x 2 < y 4 (y + 4)
© ¯ ª
¯ y < 0, .
y

0 x

−4

210 © G. Faccanoni
Dernière mise à jour : Jeudi 5 juin 2014 6 Intégrales multiples

C ORRECTION .
p p
Ï Z 0 Z y2 y+4 Z 0 Z 0
y2 y+4
2y 2
p
Aire = 1 dx dy = p 1 dx dy = [x] 2 p dy = y + 4 dy
D −4 −y 2 y+4 −4 −y y+4 −4
¸2
2 2 t7 t5 t3 212
Z Z ·
=4 t 2 (t 2 − 4)2 dt = 4 t 6 − 8t 4 + 16t 2 dt = 4 − 8 + 16 = .
0 0 7 5 3 0 105

Exercice 6.19 Aire d’un cœur


On se propose de calculer l’aire de l’ensemble D = (x, y) ∈ R2 ¯ 2x 2 − 2|x|(y + 1) + (y + 1)2 − 1 ≤ 0 .
© ¯ ª

y
1. Déterminer tout d’abord les deux fonctions y = ϕ(x) et y = ψ(x) telles y = ψ(x)
que
D = (x, y) ∈ R2 ¯ −1 ≤ x ≤ 1 et ϕ(x) ≤ y ≤ ψ(x) .
© ¯ ª

2. Par le changement
p de variable x = sin(t ), montrer qu’une primitive de la −1 1 x
fonction 1 − x 2 sur [0; 1] est
p
x 1 − x 2 + arcsin(x)
Z p
1 − x 2 dx = .
2
y = ϕ(x)
3. En utilisant la symétrie de la figure, calculer l’aire de l’ensemble D.
−2

C ORRECTION .
1. Pour trouver les équations des deux fonctions y = ϕ(x) et y = ψ(x) on remarque que
p p
(y + 1)2 − 2|x|(y + 1) + (2x 2 − 1) = 0 ⇐⇒ (y + 1) = |x| ± |x|2 − (2x 2 − 1) ⇐⇒ y = −1 + |x| ± 1 − x 2
p p
donc ϕ(x) = −1 + |x| − 1 − x 2 et ψ(x) = −1 + |x| + 1 − x 2 .
p
2. On calcule une primitive de la fonction 1 − x 2 sur [0; 1] par le changement de variable x = sin(t ) :
Z p Z q Z Z
1 − x 2 dx = 1 − sin2 (t ) cos(t ) dt = cos2 (t ) dt = sin(t ) cos(t ) + sin2 (t ) dt
Z Z Z p Z p
= sin(t ) cos(t ) + 1 dt − cos2 (t ) dt = sin(t ) cos(t ) + t − cos2 (t ) dt = x 1 − x 2 + arcsin(x) − 1 − x 2 dx

donc p
x 1 − x 2 + arcsin(x)
Z p
2
1 − x dy = .
2
3. En utilisant la symétrie de la figure par rapport à la droite d’équation x = 0, on trouve
Ï Z 1Z ψ(x) Z 1
Aire(D) = 1 dx dy = 2 1 dx dy = 2 ψ(x) − ϕ(x) dy
D 0 ϕ(x) 0
Z 1p " p #1
x 1 − x 2 + arcsin(x)
=4 1 − x2 dy = 4 = 2 arcsin(1) = π.
0 2
0

© G. Faccanoni 211
6 Intégrales multiples Dernière mise à jour : Jeudi 5 juin 2014

Intégrales doubles, coordonnées polaires

Exercice 6.20

Calculer l’intégrale double y


y=x
D

Ï
(x 2 + y) dx dy. x
D

x 2 + y2 = 1
y = −x
x 2 + y2 = 4

C ORRECTION . La région D s’écrit en coordonnées cartésiennes

¯ p
½ ¯ ¾
1 p
D = (x, y) ∈ R2 ¯¯ − 2 ≤ x ≤ − p et − x ≤ y ≤ 4 − x 2
2
½ ¯ ¾
2¯ 1 1 p p
∪ (x, y) ∈ R ¯ − p ≤ x ≤ p et 1 − x ≤ y ≤ 4 − x
¯
2 2
2 2
p
½ ¯ ¾
2¯ 1
p
∪ (x, y) ∈ R ¯ p ≤ x ≤ 2 et x ≤ y ≤ 4 − x 2
¯
2

donc p p p Z p
Ï Z − p1 Z 4−x 2 Z p1 Z 4−x 2 Z 2 4−x 2
2 2
(x 2 + y) dx dy = p (x 2 + y) dy dx + p (x 2 + y) dy dx + (x 2 + y) dy dx.
D − 2 −x − p1 1−x 2 p1 x
2 2

C’est un peu compliqué. . . Si on décrit la région D en coordonnées polaires centrées en (0; 0) on obtient

π
½ ¯ ¾

D = (r, ϑ) ¯¯ 1 ≤ r ≤ 2, ≤ ϑ ≤
¯
4 4

donc
3π 3π
Ï Z
4
Z 2 Z
4
Z 2
(x 2 + y) dx dy = (r 2 cos2 (ϑ) + r sin(ϑ))r dr dϑ = (r 3 cos2 (ϑ) + r 2 sin(ϑ)) dr dϑ
π π
D 4 1 4 1
3π 3π 3π 3π
ÃZ ! µZ ¶ ÃZ ! µZ
Z
4
Z 2 Z
4
Z 2 4 2 4 2 ¶
3 2 2 2 3 2
= r cos (ϑ) dr dϑ + r sin(ϑ) dr dϑ = cos (ϑ) dϑ × r dr + sin(ϑ) dϑ × r dr
π π π π
4 1 4 1 4 1 4 1
3π ¶ ÃZ 3π
ÃZ ! µZ ! µZ
1 + cos(2ϑ)
4 2 4 2 ¶
3 2
= dϑ × r dr + sin(ϑ) dϑ × r dr
π 2 π
4 1 4 1
¸ 3π · 4 ¸2 · 3 ¸2 µ
r r π 1 15 p 15 7 p
· ¶
2ϑ + sin(2ϑ) 4 ¤ 3π 7 15
π−
£
= × + − cos(ϑ) π4 × = − × + 2× = + 2.
4 π
4
4 1 4 3 1 4 2 4 3 16 8 3

Exercice 6.21
1
Ï
Soit D = (x, y) ∈ R2 ¯ x 2 − 4x + y 2 + 2y + 3 ≤ 0 . Calculer
© ¯ ª
dx dy.
D 1 + (x − 2)2 + (y + 1)2

p
C ORRECTION . Remarquons que l’équation x 2 − 4x + y 2 + 2y + 3 = 0 définit un cercle de centre (2, −1) et rayon 2. On passe
alors aux coordonnées polaires :

x = 2 + r cos(ϑ),

y = −1 + r sin(ϑ),

dx dy = r dr dϑ,

212 © G. Faccanoni
Dernière mise à jour : Jeudi 5 juin 2014 6 Intégrales multiples

et on obtient
p p
2π Z 2 2 p
1 r 2r
Ï Z Z
¤ 2
dr dϑ = π dr = π ln(1 + r 2 ) 0 = π ln(3).
£
dx dy =
D 1 + (x − 2)2 + (y + 1)2 0 0 1+r2 0 1+r 2

Exercice 6.22
Trouver les erreurs :
1. soit D = (x, y) ∈ R2 ¯ x 2 + y 2 ≤ 4 , alors
© ¯ ª

Ï Z 2π Z 2 µZ 2π ¶ µZ 2 ¶
16
(x 2 + y 2 ) dx dy = r 2 dr dϑ = 1 dϑ × r 2 dr = π;
D 0 0 0 0 3

2. soit D la région délimitée par la droite d’équation x = 1, la droite d’équation y = 0 et la droite d’équation y = x ;
alors, si on passe aux coordonnées polaires, on a
Ï Z π/4 Z 1
x dx dy = r 2 cos(ϑ) dr dϑ.
D 0 0

C ORRECTION .
1. Quand on passe aux coordonnées polaires on a x = r cos(ϑ), y = r sin(ϑ) et dx dy = r dr dϑ donc
Ï Z 2π Z 2 µZ 2π ¶ µZ 2 ¶
(x 2 + y 2 ) dx dy = r 3 dr dϑ = 1 dϑ × r 3 dr = 8π.
D 0 0 0 0

D = (x, y) ∈ R¯2 ¯ 0 ≤ x ≤ 1, 0 ≤ y ≤ ªx est un triangle (hachuré dans la figure ci-dessous) tandis que le do-
© ¯ ª
2. Le domaine
maine (r, ϑ) ∈ R∗ × [0, 2π[ ¯ 0 ≤ r ≤ 1, 0 ≤ ϑ ≤ π (colorié dans la figure ci-dessous) est un huitième du disque unitaire :
©
+ 4
y

0 1 x
Comme les domaines d’intégration sont différents, les deux intégrales sont différentes aussi. Si on utilise les coordon-
nées polaires pour décrire l’ensemble D on a

π π
o ½ ¯ ¾
n

¯ p
∗ 1
(r, ϑ) ∈ R+ × [0, 2π[ ¯ 0 ≤ ϑ ≤ , 0 ≤ r ≤ 1 + tan (ϑ) = (r, ϑ) ∈ R+ × [0, 2π[ ¯ 0 ≤ ϑ ≤ , 0 ≤ r ≤
¯
¯ 2 ¯ .
4 4 cos(ϑ)

y
O A = OD = R
B
OC = R cos(α)
A B D = R tan(α)
p
OB = R 1 + tan2 (α)

O C D x
En effet,
Ï Z 1Z x Z 1£ ¤ y=x
Z 1 1
x dx dy = x dy dx = xy y=0 dy dx = x 2 dx = ;
D 0 0 0 0 3
π 1 π 1
¸r = cos(ϑ) Z π
Z
4
Z
cos(ϑ)
Z
4
·
r3 1 4 1 1h iπ 1
2 4
r cos(ϑ) dr dϑ = cos(ϑ) dϑ = dϑ = tan(ϑ) = .
0 0 0 3 r =0 3 0 cos2 (ϑ) 3 0 3

© G. Faccanoni 213
6 Intégrales multiples Dernière mise à jour : Jeudi 5 juin 2014

y
7
Exercice 6.23 x 2 + (y − 6)2 = 1
(x + 1)2 + (y − 5)2 = 1 6
Calculer Ï 5
y dx dy
D 4 (x − 1)2 + (y − 5)2 = 1
où l’ensemble d’intégration est celui colorié 3
dans la figure ci-contre.
2
(Le trait discontinue suggère un découpage du
domaine d’intégration) 1

−2 −1 0 1 2x

C ORRECTION .

Méthode I On décompose le domaine d’intégration en cinq parties sur lesquelles sera facile de calculer l’intégrale donnée :
y
7
Ï Ï Ï Ï Ï Ï E
y dx dy = y dx dy+ y dx dy+ y dx dy+ y dx dy+ y dx dy 6
D A B C E F
F 5 A C
2
A = (x, y) ∈ R ¯ −1 ≤ x ≤ 1, 4 ≤ y ≤ 6 ,
© ¯ ª
4
n ¯ y yo
B = (x, y) ∈ R2 ¯ 0 ≤ y ≤ 4, − ≤ x ≤ ,
¯
¯ 4 4 3 B
n π πo
C = (r, t ) ∈ R∗+ × [−π; π[ ¯ 0 ≤ r ≤ 1, − ≤ t ≤ , avec x = 1 + r cos(t ), y = 5 + r sin(t ),
¯
2 2 2

E = (r, t ) ∈ R+ × [0; 2π[ ¯ 0 ≤ r ≤ 1, 0 ≤ t ≤ π , avec x = r cos(t ), y = 6 + r sin(t ),
© ¯ ª
½ ¯
π 3π
¾ 1

F = (r, t ) ∈ R+ × [0; 2π[ ¯ 0 ≤ r ≤ 1, ≤ t ≤
¯
¯ , avec x = −1 + r cos(t ), y = 5 + r sin(t ),
2 2
−2 −1 0 1 2 x

1 6 1 · 2 ¸ y=6 6
y
Ï Z Z µZ ¶ ¶ µZ
y dx dy = y dx dy = y dy = 2 ×
1 dx × = 20
A −1 4 −1 4 2 y=4
Z 4 Z y/4 Z 4 · 3 ¸ y=4
1 4 2 y 32
Ï Z
£ ¤x=y/4
y dx dy = y dx dy = y x x=−y/4 dy = y dx = =
B −0 −y/4 0 2 0 6 y=0 3
Ï Z 1 Z π/2 µZ 1 ¶ µZ π/2 ¶ µZ 1 ¶ µZ π/2 ¶
2
y dx dy = r (5 + r sin(t )) dt dr = 5r dr × 1 dt + r dr × sin(t ) dt
C 0 −π/2 0 −π/2 0 −π/2
2 ¸r =1 · 3 ¸r =1 h
r r
· h it =π/2 it =π/2 5π
= 5 × t + × − cos(t ) =
2 r =0 t =−π/2 3 r =0 t =−π/2 2
Ï Z 1Z π µZ 1 ¶ µZ π ¶ µZ 1 ¶ µZ π ¶
2
y dx dy = r (6 + r sin(t )) dt dr = 6r dr × 1 dt + r dr × sin(t ) dt
E 0 0 0 0 0 0
· 2 ¸r =1 h i · 3 ¸r =1 h
r t =π r it =π 2
= 6 × t + × − cos(t ) = 3π +
2 r =0 t =0 3 r =0 t =0 3
Ï Ï
y dx dy = y dx dy.
F C

Conclusion :
34
Ï
y dx dy = 20 + + 8π.
D 3
Méthode II La fonction et le domaine d’intégration sont symétriques par rapport au plan d’équation x = 0, par conséquent
il suffit de calculer l’intégrale sur une moitié du domaine d’intégration :

214 © G. Faccanoni
Dernière mise à jour : Jeudi 5 juin 2014 6 Intégrales multiples

y
7
E0
µÏ ¶ 6
Ï Ï Ï Ï
y dx dy = 2 y dx dy + y dx dy + y dx dy + y dx dy
D A0 B0 C E0 5 A0 C

4
A 0 = (x, y) ∈ R2 ¯ 0 ≤ x ≤ 1, 4 ≤ y ≤ 6 ,
© ¯ ª

B 0 = (x, y) ∈ R2 ¯ 0 ≤ x ≤ 1, 4x ≤ y ≤ 4 , 3 B0
© ¯ ª
n ¯ π πo
C = (r, t ) ∈ R∗+ × [−π; π[ ¯ 0 ≤ r ≤ 1, − ≤ t ≤ , avec x = 1 + r cos(t ), y = 5 + r sin(t ), 2
¯
2 2
n ¯ π o
E 0 = (r, t ) ∈ R∗+ × [0; 2π[ ¯ 0 ≤ r ≤ 1, 0 ≤ t ≤ , avec x = r cos(t ), y = 6 + r sin(t ), 1
¯
2

0 1 2 x

Z 1Z 6 1 · 2 ¸ y=6
6
y
Ï µZ ¶ ¶ µZ
y dx dy = y dx dy = y dy = 1 ×
1 dx × = 10
A0 0 4 0 4 2 y=4
Z 1 · 2 ¸ y=4 ¸x=1
x3
Z 1Z 4 Z 1
y
·
16
Ï
y dx dy = y dy dx = dx = 8 (1 − x 2 ) dx = 8 x − =
B 0 0 4x 0 2 y=4x 0 3 x=0 3
Ï Z 1 Z π/2 µZ 1 ¶ µZ π/2 ¶ µZ 1 ¶ µZ π/2 ¶
y dx dy = r (5 + r sin(t )) dt dr = 5r dr × 1 dt + r 2 dr × sin(t ) dt
C 0 −π/2 0 −π/2 0 −π/2
· 2 ¸r =1 h i · 3 ¸r =1 h
r t =π/2 r it =π/2 5π
= 5 × t + × − cos(t ) =
2 r =0 t =−π/2 3 r =0 t =−π/2 2
Ï Z 1 Z π/2 µZ 1 ¶ µZ π/2 ¶ µZ 1 ¶ µZ π/2 ¶
2
y dx dy = r (6 + r sin(t )) dt dr = 6r dr × 1 dt + r dr × sin(t ) dt
E0 0 0 0 0 0 0
· 2 ¸r =1 h i · 3 ¸r =1 h
r t =π/2 r it =π/2 3 1
= 6 × t + × − cos(t ) = π+
2 r =0 t =0 3 r =0 t =0 2 3

Conclusion : µ ¶
17
Ï
y dx dy = 2 10 + + 4π .
D 3

Exercice 6.24
Après avoir représenté graphiquement l’ensemble A , calculer l’intégrale

x
Ï
A = (x, y) ∈ R2 ¯ 1 ≤ x 2 + y 2 ≤ 4, |y| ≥ |x| .
© ¯ ª
dx dy avec
A y

C ORRECTION . L’ensemble A est la partie coloriée dans la figure ci-dessous :

y
2
1

−2 −1 0 1 2x
−1
−2

En passant en coordonnées polaires

x = r cos(ϑ), y = r sin(ϑ), dx dy = r dr dϑ

© G. Faccanoni 215
6 Intégrales multiples Dernière mise à jour : Jeudi 5 juin 2014

on obtient
π
½ ¯ ¾ ½ ¯ ¾
3π 5π 7π
A = (r, ϑ) ∈ R+ × [0; 2π[ ¯¯ 1 ≤ r ≤ 2 et ≤ ϑ ≤ ∪ (r, ϑ) ∈ R+ × [0; 2π[ ¯¯ 1 ≤ r ≤ 2 et ≤ϑ≤
¯ ¯
4 4 4 4

et donc
3π 7π 3π 7π
ÃZ ! µZ
Ï
x
Z
4
Z 2 cos(ϑ)
Z
4
Z 2 cos(ϑ) 4 cos(ϑ)
Z
4 cos(ϑ) 2 ¶
dx dy = r dr dϑ + r dr dϑ = dϑ + dϑ r dr = 0.
y π sin(ϑ) 5π sin(ϑ) π sin(ϑ) 5π sin(ϑ)
A 4 1 4 1 4 4 1

car
cos(t )
Z
dt = ln(| sin(t )|).
sin(t )

Exercice 6.25
Après avoir représenté graphiquement l’ensemble A , calculer l’intégrale

y
Ï
A = (x, y) ∈ R2 ¯ 1 ≤ x 2 + y 2 ≤ 4, |y| ≤ |x| .
© ¯ ª
dx dy avec
A x

C ORRECTION . L’ensemble A est la partie coloriée dans la figure ci-dessous :

y
2
1

−2 −1 0 1 2x
−1
−2

En passant en coordonnées polaires

x = r cos(ϑ), y = r sin(ϑ), dx dy = r dr dϑ

on obtient
π Z 5π Z 2
Ï
y
Z
sin(ϑ)
4
Z 2 4 sin(ϑ)
dx dy = r dr dϑ + r dr dϑ
x π
A − 4 1 cos(ϑ) 3π
4 1 cos(ϑ)
ÃZ π Z 5π ! µZ
4 sin(ϑ) 4 sin(ϑ) 2 ¶
= dϑ + dϑ r dr = 0
− π4 cos(ϑ) 3π cos(ϑ)
4 1

car
sin(t )
Z
dt = − ln(| cos(t )|).
cos(t )

Exercice 6.26
y

Soit D la partie coloriée en figure.


3 B Décrire D en coordonnées cartésiennes.
B Trouver un changement de variables adéquat pour décrire D en coordon-
5 nées polaires.
+
x x−y
Ï
= B Calculer dx dy.
y p
D (x + 2)2 + (y − 3)2
−2 − 12 x

216 © G. Faccanoni
Dernière mise à jour : Jeudi 5 juin 2014 6 Intégrales multiples

C ORRECTION . Le domaine d’intégration est le demi cercle de centre (−2, 3) et rayon r = 32 qui se trouve sous la droite d’équa-
tion y = x + 5, autrement dit ½ ¯ ¾
9
D = (x, y) ∈ R2 ¯¯ (x + 2)2 + (y − 3)2 < , y < x + 5 .
¯
4
Avec le changement de variables (
x = −2 + r cos(φ),
y = 3 + r sin(φ),
on obtient
x−y −2 + r cos(φ) − 3 − r sin(φ)
Ï Ï
p dx dy = r dr dφ
D (x + 2)2 + (y − 3)2 D r
où ½ ¯ ¾
3 5 9
D = (r, φ) ∈ R × [π/2, 5π/2] ¯¯ 0 < r < , π < φ < π .
¯
2 4 4
Donc
3 9

ÃZ !
−2 + r cos(φ) − 3 − r sin(φ)
Ï Z
2
r dr dφ = −5 + r (cos(φ) − sin(φ)) dφdr
D r 0 5

3 9 3 9
4π 4π
à Z Z ! ÃZ Z !
2 2
= −5 1 dφdr + r (cos(φ) − sin(φ)) dφdr
5 5
0 4π 0 4π
3 9 3 9
4π 4π
ÃZ ! ÃZ ! ÃZ ! ÃZ !
2 2
= −5 1 dr 1 dφ + r dr cos(φ) − sin(φ) dφ
5 5
0 4π 0 4π
15 9p
=− π+ 2.
2 4

Exercice 6.27

y
Soit D la partie coloriée en figure.
3 B Décrire D en coordonnées cartésiennes.
y= B Trouver un changement de variables adéquat pour décrire D en coordon-
− nées polaires.
x+
x−y
Ï
5 B Calculer dx dy.
D (x − 2)2 + (y − 3)2
1 2 x
2

C ORRECTION . Le domaine d’intégration est le demi cercle de centre (2, 3) et rayon r = 23 qui se trouve sous la droite d’équa-
tion y = −x + 5, autrement dit ½ ¯ ¾
9
D = (x, y) ∈ R2 ¯¯ (x − 2)2 + (y − 3)2 < , y < −x + 5 .
¯
4
Avec le changement de variables (
x = 2 + r cos(φ),
y = 3 + r sin(φ),
on obtient
x−y 2 + r cos(φ) − 3 − r sin(φ)
Ï Ï
dx dy = r dr dφ
D (x − 2)2 + (y − 3)2 D r2
où ½ ¯ ¾
3 3 7
D = (r, φ) ∈ R × [0, 2π] ¯¯ 0 < r < , π < φ < π .
¯
2 4 4
Donc
3 7
2 + r cos(φ) − 3 − r sin(φ) 1 1 3 4π
Ï Ï Ï Z Z
2
r dr dφ = − dr dφ+ cos(φ)−sin(φ) dr dφ = −π dr + cos(φ)−sin(φ) dφ = ∞.
D r2 D r D 0 r 2 3

© G. Faccanoni 217
6 Intégrales multiples Dernière mise à jour : Jeudi 5 juin 2014

Calcul d’aires et centres de gravité, coordonnées polaires

Exercice 6.28
Un disque de rayon 5 cm a densité 10 g cm−2 dans son centre, densité 0 g cm−2 sur le bord et la densité est une fonction
linéaire de la distance du centre. Calculer la masse du disque.

C ORRECTION . Décrivons le disque en coordonnées polaires centrées dans le centre du disque. Soit % la densité : elle ne
dépend que de la distance r du centre et l’on a %(0) = 10, %(5) = 0 et elle est linéaire donc %(r ) = 2(5 − r ). La masse du disque
est donc
· 2 ¸r =5
r3 53
Z 5 Z 2π Z 5 Z 2π µZ 5 ¶ µZ 2π
r

ρ(r )r dϑ dr = 2 r (5 − r ) dϑ dr = 2 r (5 − r ) dr 1 dϑ = 4π 5 − = 2π.
0 0 0 0 0 0 2 3 r =0 3

Exercice 6.29
Considérons un disque de métal de rayon 3 cm et prenons un repère centré dans son centre. À la distance r de l’origine,
la densité du métal par unité de surface est r 21+1 . Calculer la masse du disque.

C ORRECTION . Décrivons le disque en coordonnées polaires centrées dans le centre du disque. La masse du disque est
Z 3 Z 2π µZ 3 ¶ µZ 2π µZ 3
r r
¶ ¶
2r ¤r =3
π = π ln(r 2 + 1) r =0 = π ln(10).
£
2
dϑ dr = 2
dr × 1 dϑ = 2
dr
0 0 r +1 0 r +1 0 0 r +1

Exercice 6.30 Une ville entoure une baie comme dans la figure ci-contre. La
densité de population de la ville (en milliers de personnes par
y [km]
4
kilomètre carré) est δ(r, ϑ), où r et ϑ sont les coordonnées po-
laires et les distances sont mesurées en kilomètres. Plus loin
on vit de la baie et plus la densité de population diminue. De
même, plus loin on vit de l’océan et plus la densité de po-
pulation diminue. Laquelle des fonctions suivantes décrit au
Ville 1
mieux cette situation ?
Ocean

Bay
x [km]

−1
1. δ(r, ϑ) = (4 − r )(2 + cos(ϑ))
2. δ(r, ϑ) = (4 − r )(2 + sin(ϑ))
3. δ(r, ϑ) = (4 + r )(2 + cos(ϑ))

En déduire une estimation de la population totale de la


−4
ville.

C ORRECTION .
B La ville (domaine d’intégration) s’écrit
π
½ ¯ ¾

(r, ϑ) ∈ R∗+ × [0; 2π[ ¯¯ 1 ≤ r ≤ 4, ≤ ϑ ≤
¯
.
2 2
B La fonctions qui décrit au mieux la densité de population est δ(r, ϑ) = (4 − r )(2 + cos(ϑ)) ; en effet, pour tout ϑ0 fixé dans
[π/2; 3π/2], la fonction f ϑ0 (r ) ≡ δ(r, ϑ0 ) est monotone décroissante et pour tout r 0 fixé dans [1; 4], la fonction g r 0 (ϑ) ≡
δ(r 0 , ϑ) est monotone décroissante sur [π/2; π] et monotone croissante sur [π; 3π/2].
B La population totale qui vit dans la ville est
Z Z 3π
4 Z Z 3π 4 µZ ¶ ÃZ 3π 4
!
2 2 2
δ(r, ϑ)r dr dϑ = r (4 − r )(2 + cos(ϑ)) dr dϑ = r (4 − r ) dr × (2 + cos(ϑ)) dϑ
π π π
1 2 1 2 1 2
¸r =4 h
r3 i 3π
·
2
= 2r 2 − × 2ϑ + sin(ϑ) π = 18(π − 1).
3 r =1 2

218 © G. Faccanoni
Dernière mise à jour : Jeudi 5 juin 2014 6 Intégrales multiples

Exercice 6.31
1. Après avoir représenté graphiquement la région

D = {(x, y) ∈ R2 | y ≥ 0, 1 ≤ x 2 + y 2 ≤ 4},

calculer les coordonnées de son centre de gravité en supposant la région D homogène ; calculer ensuite
Ï
(3x + 4y 2 ) dx dy.
D

2. Après avoir représenté graphiquement la région

D = {(x, y) ∈ R2 | x ≥ 0, 1 ≤ x 2 + y 2 ≤ 4},

calculer les coordonnées de son centre de gravité en supposant la région D homogène ; calculer ensuite
Ï
(3x + 4y 2 ) dx dy.
D

3. Après avoir représenté graphiquement la région

D = {(x, y) ∈ R2 | y ≤ 0, 1 ≤ x 2 + y 2 ≤ 4},

calculer les coordonnées de son centre de gravité en supposant la région D homogène ; calculer ensuite
Ï
(3x + 4y 2 ) dx dy.
D

4. Après avoir représenté graphiquement la région

D = {(x, y) ∈ R2 | x ≤ 0, 1 ≤ x 2 + y 2 ≤ 4},

calculer les coordonnées de son centre de gravité en supposant la région D homogène ; calculer ensuite
Ï
(3x + 4y 2 ) dx dy.
D

5. Après avoir représenté graphiquement la région

D = {(x, y) ∈ R2 | y ≤ x, 1 ≤ x 2 + y 2 ≤ 9},

calculer les coordonnées de son centre de gravité en supposant la région D homogène ; calculer ensuite
Ï
(3x + 4y 2 ) dx dy.
D

6. Après avoir représenté graphiquement la région

D = {(x, y) ∈ R2 | y ≥ |x|, 1 ≤ x 2 + y 2 ≤ 4},

calculer les coordonnées de son centre de gravité en supposant la région D homogène ; calculer ensuite
Ï
(3x − 4y 2 ) dx dy.
D

C ORRECTION .
1. La région D est la zone coloriée suivante

© G. Faccanoni 219
6 Intégrales multiples Dernière mise à jour : Jeudi 5 juin 2014

Le centre de gravité a coordonnées (xG , yG ) données par


Î Î
x dx dy y dx dy
xG = ÎD , yG = ÎD .
D 1 dx dy D 1 dx dy

Il convient de passer aux coordonnées polaires et on obtient


Z πZ 2 µZ π ¶ µZ 2 ¶
3
Ï
1 dx dy = r dr dθ = dθ r dr = π,
D 0 1 0 1 2
Ï Z πZ 2 µZ π ¶ µZ 2 ¶
2 2
x dx dy = r cos(θ) dr dθ = cos(θ) dθ r dr = 0,
D 0 1 0 1
Z πZ 2 µZ π ¶ µZ 2 ¶
14
Ï
y dx dy = r 2 sin(θ) dr dθ = sin(θ) dθ r 2 dr = .
D 0 1 0 1 3
Donc
14 2 28
xG = 0, yG = = < 1.
3 3π 9π
On calcul maintenant l’intégrale donnée
Ï Z πZ 2
(3x + 4y 2 ) dx dy = (3r cos(θ) + 4r 2 sin2 θ) r dr dθ
D 0 1
Z π
£ 3 ¤2
= r cos(θ) + r 4 sin2 θ 1 dθ
Z0 π
= 7 cos(θ) + 15 sin2 θ dθ
0
Z π
1 − cos(2θ) 15
= 7 cos(θ) + 15 dθ = π.
0 2 2

2. La région D est la zone coloriée suivante

Le centre de gravité a coordonnées (xG , yG ) données par


Î Î
x dx dy y dx dy
xG = ÎD , yG = ÎD .
D 1 dx dy D 1 dx dy

Il convient de passer aux coordonnées polaires et on obtient


Z π/2 Z 2 µZ π/2 ¶ µZ 2 ¶
3
Ï
1 dx dy = r dr dθ = dθ r dr = π,
D −π/2 1 −π/2 1 2
Z π/2 Z 2 µZ π/2 ¶ µZ 2 ¶
14
Ï
x dx dy = r 2 cos(θ) dr dθ = cos(θ) dθ r 2 dr = ,
D −π/2 1 −π/2 1 3

220 © G. Faccanoni
Dernière mise à jour : Jeudi 5 juin 2014 6 Intégrales multiples

Ï Z π/2 Z 2 µZ π/2 ¶ µZ 2 ¶
y dx dy = r 2 sin(θ) dr dθ = sin(θ) dθ r 2 dr = 0.
D −π/2 1 −π/2 1

Donc
14 2 28
xG = = < 1, yG = 0.
3 3π 9π
On calcul maintenant l’intégrale donnée
Ï Z π/2 Z 2
2
(3x + 4y ) dx dy = (3r cos(θ) + 4r 2 sin2 θ) r dr dθ
D −π/2 1
Z π/2
¤2
r 3 cos(θ) + r 4 sin2 θ
£
= 1 dθ
−π/2
Z π/2
= 7 cos(θ) + 15 sin2 θ dθ
−π/2
Z π/2
1 − cos(2θ) 15
= 7 cos(θ) + 15 dθ = 14 + π.
−π/2 2 2

3. La région D est la zone coloriée suivante

Le centre de gravité a coordonnées (xG , yG ) données par


Î Î
x dx dy y dx dy
xG = ÎD , yG = ÎD .
D 1 dx dy D 1 dx dy

Il convient de passer aux coordonnées polaires et on obtient


Ï Z 0 Z 2 ¶ µZ 2 ¶
3
µZ 0
1 dx dy = r dr dθ =
dθ r dr = π,
D −π 1 −π 1 2
Ï Z 0Z 2 µZ 0 ¶ µZ 2 ¶
x dx dy = r 2 cos(θ) dr dθ = cos(θ) dθ r 2 dr = 0,
D −π 1 −π 1
Z 0Z 2 µZ 0 ¶ µZ 2 ¶
14
Ï
y dx dy = r 2 sin(θ) dr dθ = sin(θ) dθ r 2 dr = − .
D −π 1 −π 1 3

Donc
14 2 28
xG = 0, yG = − =− > −1.
3 3π 9π
On calcul maintenant l’intégrale donnée
Ï Z 0 Z 2
(3x + 4y 2 ) dx dy = (3r cos(θ) + 4r 2 sin2 θ) r dr dθ
D −π 1
Z 0
¤2
r 3 cos(θ) + r 4 sin2 θ
£
= 1 dθ
−π
Z 0
= 7 cos(θ) + 15 sin2 θ dθ
−π
Z 0
1 − cos(2θ) 15
= 7 cos(θ) + 15 dθ = π.
−π 2 2

4. La région D est la zone coloriée suivante

© G. Faccanoni 221
6 Intégrales multiples Dernière mise à jour : Jeudi 5 juin 2014

Le centre de gravité a coordonnées (xG , yG ) données par


Î Î
x dx dy y dx dy
xG = ÎD , yG = ÎD .
D 1 dx dy D 1 dx dy

Il convient de passer aux coordonnées polaires et on obtient


Ï Z 3π/2 Z 2 µZ 3π/2

3
¶ µZ 2
1 dx dy = r dr dθ = r dr = π, dθ
D π/2 1 π/2 1 2
Z 3π/2 Z 2 µZ 3π/2 ¶ µZ 2 ¶
14
Ï
x dx dy = r 2 cos(θ) dr dθ = cos(θ) dθ r 2 dr = − ,
D π/2 1 π/2 1 3
Ï Z 3π/2 Z 2 µZ 3π/2 ¶ µZ 2 ¶
y dx dy = r 2 sin(θ) dr dθ = sin(θ) dθ r 2 dr = 0.
D π/2 1 π/2 1

Donc
14 2 28
xG = − =− > −1, yG = 0.
3 3π 9π
On calcul maintenant l’intégrale donnée
Ï Z 3π/2 Z 2
(3x + 4y 2 ) dx dy = (3r cos(θ) + 4r 2 sin2 θ) r dr dθ
D π/2 1
Z 3π/2
¤2
r 3 cos(θ) + r 4 sin2 θ
£
= 1 dθ
π/2
Z 3π/2
= 7 cos(θ) + 15 sin2 θ dθ
π/2
Z 3π/2 1 − cos(2θ) 15
= 7 cos(θ) + 15 dθ = −14 + π.
π/2 2 2

5. La région D est la zone coloriée suivante


y

0 x
−2 −1 1 2 3
−1

−2

−3

222 © G. Faccanoni
Dernière mise à jour : Jeudi 5 juin 2014 6 Intégrales multiples

Le centre de gravité a coordonnées (xG , yG ) données par


Î Î
x dx dy y dx dy
xG = ÎD , yG = ÎD .
D 1 dx dy D 1 dx dy

Il convient de passer aux coordonnées polaires et on obtient


Ï Z π/4 Z 3 µZ π/4 ¶ µZ 3 ¶
1 dx dy = r dr dθ = dθ r dr = 4π,
D −3π/4 1 −3π/4 1
Ï Z π/4 Z 3 µZ 3π/2 ¶ µZ ¶ 3
26 p
x dx dy = r 2 cos(θ) dr dθ = cos(θ) dθ r 2 dr = 2,
D −3π/4 1 π/2 1 3
Ï Z π/4 Z 3 µZ π/4 ¶ µZ 3 ¶
y dx dy = r 2 sin(θ) dr dθ = sin(θ) dθ r 2 dr = −xG .
D −3π/4 1 −3π/4 1

Donc
26 p 1 13 p
xG = 2 = 2 < 1, yG = −xG .
3 4π 6π
On calcul maintenant l’intégrale donnée
Ï Z π/4 Z 3
(3x + 4y 2 ) dx dy = (3r cos(θ) + 4r 2 sin2 θ) r dr dθ
D −3π/4 1
Z π/4
¤3
r 3 cos(θ) + r 4 sin2 θ
£
= 1 dθ
−3π/4
Z π/4
= 26 cos(θ) + 80 sin2 θ dθ
−3π/4
Z π/4 p
1 − cos(2θ)
= 26 cos(θ) + 80 dθ = 26 2 + 40π.
−3π/4 2

6. La région D est la zone coloriée suivante

Le centre de gravité a coordonnées (xG , yG ) données par


Î Î
x dx dy y dx dy
xG = ÎD , yG = ÎD .
D 1 dx dy D 1 dx dy

Il convient de passer aux coordonnées polaires et on obtient


Z 3π Z ÃZ 3π ! µZ
2 ¶ 2
3
Ï
4 4
1 dx dy = r dr = π,
r dr dθ = dθ
π π 4
D 4 1 4 1
Ï Z 3π Z 2 ÃZ 3π ! µZ
2 ¶
4 4
2
x dx dy = r cos(θ) dr dθ = cos(θ) dθ r 2 dr = 0,
π π
D 4 1 4 1

ÃZ 3π
! µZ p
Ï Z
4
Z 2 4 2 ¶
7 2
y dx dy = r 2 sin(θ) dr dθ = sin(θ) dθ r 2 dr = .
π π 3
D 4 1 4 1

Donc p p
7 2 4 28 2
xG = 0, yG = = > 1.
3 3π 9π
On calcul maintenant l’intégrale donnée

Ï Z
4
Z 2
2
(3x − 4y ) dx dy = (3r cos(θ) − 4r 2 sin2 (θ)) r dr dθ
π
D 4 1
Z 3π
4 £ 3 ¤2
= r cos(θ) − r 4 sin2 (θ) 1 dθ
π
4

© G. Faccanoni 223
6 Intégrales multiples Dernière mise à jour : Jeudi 5 juin 2014

Z 3π
4
= 7 cos(θ) − 15 sin2 (θ) dθ
π
4

1 − cos(2θ) 15
Z
4
= 7 cos(θ) − 15 dθ = − (2 + π).
π 2 4
4

Exercice 6.32
Soit R > 1. Après avoir représenté graphiquement la région

D = (x, y) ∈ R2 ¯ y ≥ 0, 1 ≤ x 2 + y 2 ≤ R ∪ (x, y) ∈ R2 ¯ x y ≥ 0, 1 ≤ x 2 + y 2 ≤ R ,
© ¯ ª © ¯ ª

calculer les coordonnées de son centre de gravité en supposant la région D homogène. À partir de quelle valeur de R le
centre de gravité appartient à D ?

C ORRECTION . On trace d’abord les courbes d’équation x 2 + y 2 = 1 et x 2 + y 2 = R avec R > 1 et on colore l’ensemble D.

y On décrit ensuite D en coordonnées polaires centrées en


x2 + y 2 = R x2 + y 2 = 1 (0, 0) : on a une bijection de R2 \{ (0, 0) } dans lui même définie
R par x = r cos(ϑ), y = r sin(ϑ) pour (r, ϑ) ∈ R∗+ × [0, 2π[, ainsi
½ ¯ ¾

D = (r, ϑ) ∈ R∗+ × [0, 2π[ ¯¯ 0 < r ≤ R, 0 ≤ ϑ ≤
¯
.
1 2
yG Le centre de gravité a coordonnées (xG , yG ) données par
−R −1 xG 0 1 R x Î Î
D x dx dy y dx dy
xG = Î , yG = ÎD .
−1 D 1 dx dy D dx dy
1

−R

On calcule alors
R Z 3π/2 R 3π(R 2 − 1)
3π/2
Ï Z µZ ¶ µZ ¶
1 dx dy = r dr dϑ = r dr × ; 1 dϑ =
D 1 0 1 0 4
(R 3 − 1)
Ï Z R Z 3π/2 µZ R ¶ µZ 3π/2 ¶
x dx dy = r 2 cos(ϑ) dr dϑ = r 2 dr × cos(ϑ) dϑ = − ;
D 1 0 1 0 3
(R 3 − 1)
Ï Z R Z 3π/2 µZ R ¶ µZ 3π/2 ¶
y dx dy = r 2 sin(ϑ) dr dϑ = r 2 dr × sin(ϑ) dϑ = .
D 1 0 1 0 3

On obtient finalement
3 (R 3 −1)
− (R 3−1) 4(R 2 + R + 1) 3 4(R 2 + R + 1)
xG = =− , yG = = .
3π(R 2 −1) 9π(R + 1) 3π(R 2 −1) 9π(R + 1)
4 4

Le centre de gravité appartient à D si et seulement si 1 ≤ xG2 + yG2 ≤ R :

¶2 µ ¶2 ( R 2 +R+1 9π p
4(R 2 + R + 1) 4(R 2 + R + 1) R+1 ≥
p , c + c 2 + 4c
µ
1≤ − + ≤ 4 ⇐⇒ 4 p2 ⇐⇒ R > ≈ 4.826617132
2
R +R+1 9π 2
9π(R + 1) 9π(R + 1) 2
R+1 ≤ 4 ,


avec c ≡ p − 1.
4 2

224 © G. Faccanoni
Dernière mise à jour : Jeudi 5 juin 2014 6 Intégrales multiples

y
Exercice 6.33
Soit D la région représentée en figure ci-contre. Calculer x2 + y 2 = 4 x2 + y 2 = 1
les coordonnées de son centre de gravité (xG , yG ) en sup- 2
posant la région D homogène.
1
yG

−2 −1 xG 0 1 2 x

−1

−2

C ORRECTION . Le centre de gravité a coordonnées (xG , yG ) données par


Î Î
D x dx dy y dx dy
xG = Î , yG = ÎD .
D 1 dx dy D 1 dx dy

Il convient de passer aux coordonnées polaires et on obtient


Ï Z 3π/2 Z 2 µZ 3π/2 ¶
9
¶ µZ 2
1 dx dy = r dr dθ = r dr = π,

D 0 1 0 1 4
Z 3π/2 Z 2 µZ 3π/2 ¶ µZ 2 ¶
7
Ï
2 2
x dx dy = r cos(θ) dr dθ = cos(θ) dθ r dr = − ,
D 0 1 0 1 3
Z 3π/2 Z 2 µZ 3π/2 ¶ µZ 2 ¶
7
Ï
2 2
y dx dy = r sin(θ) dr dθ = sin(θ) dθ r dr = .
D 0 1 0 1 3

Donc
28 14 2 28
xG = − , yG = = .
27π 3 3π 27π

Remarquons
Î que seul le calcul explicite de la dernière intégrale est nécessaire car
B D 1 dx dy représente l’aire de la plaque que l’on peut calculer géométriquement comme la différence des 3/4 du cercle
de rayon 2 ôtés des 3/4 du cercle de rayon 1, i.e. (22 π − π)3/4 = 9π/4,
B xG = −yG car la droite d’équation y = −x est un axe de symétrie de la plaque.

Exercice 6.34
On se propose de calculer (xG , yG ) les coordonnées du centre de gravité de la plaque homogène Ω esquissée dans la figure
ci-dessous :
y
2
1
Ω3
−4 −3 −2 −1 0 1 2 x
−1
Ω2
Ω1 −2
−3
−4

1. On décompose la plaque comme union de trois plaques Ωi comme en figure. Décrire Ω1 et Ω2 en variables carté-
siennes et Ω3 en variables polaires (écrire explicitement le changement en variables polaires).
2. Pour i = 1, 2, 3, calculer l’aire de Ωi . En déduire l’aire de Ω.
3. Pour i = 1, 2, 3, calculer (xG i , yG i ) les coordonnées du centre de gravité de la plaque Ωi . En déduire (xG , yG ).
NB Lorsqu’il est possible, éviter le calcul d’intégrales !

© G. Faccanoni 225
6 Intégrales multiples Dernière mise à jour : Jeudi 5 juin 2014

C ORRECTION .
1. Le changement de variables polaires pour Ω3 est
(
x = −1 + r cos(ϑ),
avec (r, ϑ) ∈ R∗+ × [0; 2π[
y = −1 + r sin(ϑ)

et l’on a

Ω1 = (x, y) ∈ R2 ¯ x ∈ [−4; −1], y ∈ [−x − 5; x + 3] ,


© ¯ ª

Ω2 = (x, y) ∈ R2 ¯ x ∈ [−1; 2], y ∈ [−4; −1] ,


© ¯ ª

Ω3 = (x, y) ∈ R2 ¯ x ∈ [−1; 2], y ∈ [−1; 2], (x + 1)2 + (y + 1)2 ≤ 32


© ¯ ª
n ¯ h πio
= (r, ϑ) ∈ R∗+ × [0; 2π[ ¯ r ∈]0; 3], ϑ ∈ 0; .
¯
2

2. Aire(Ω)=Aire(Ω1 )+Aire(Ω2 )+Aire(Ω3 )=9 + 9 + 49 π = 9 2 + π4 .


¡ ¢

3. Pour calculer (xG , yG ) on calcule (xG i , yG i ) les coordonnées du centre de gravité de chaque plaque Ωi :
Î
Ω1 x dx dy
B xG 1 = Aire(Ω1 ) = −2 car

−1 Z x+3 −1 −1 −1 · ¸x=−1
2 3
Ï Z Z Z Z
£ ¤ y=x+3 2
x dx dy = x dy dx = x y y=−x−5 dx = x(2x + 8) dx = 2x + 8x dx = x + 4x 2 = −18,
Ω1 −4 −x−5 −4 −4 −4 3 x=−4

B yG 1 = −1 par symétrie de la plaque Ω1 par rapport à la droite d’équation y = −1 ;


B xG 2 = 12 par symétrie de la plaque Ω2 par rapport à la droite d’équation x = 12 ,
5
B yG 2 = −Î2
par symétrie de la plaque Ω2 par rapport à la droite d’équation y = − 52 ;
Ω3 x dx dy 4
B xG 3 = Aire(Ω3 ) = π − 1 car

Ï Z 3Z π Z 3Z π Z 3Z π
2 2 2
x dx dy = (−1 + r cos(ϑ))r dϑ dr = −r dϑ dr + r 2 cos(ϑ) dϑ dr
Ω1 0 0 0 0 0 0
µZ 3 ¶ ÃZ π ! µZ
3 ¶ ÃZ π !
2 2
2
=− r dr × 1 dϑ + r dr × cos(ϑ) dϑ
0 0 0 0
2 ¸3 3 ¸3 π
r π r π´
· · h i 9 2
³
=− × + × sin(ϑ) = − π + 9 = 9 1 − ,
2 0 2 3 0 0 4 4

B yG 3 = xG 3 par symétrie de la plaque Ω3 par rapport à la droite d’équation y = x.


On conclut que
π 2−π
µ ¶
(xG , yG ) = − , .
2+π 2+π

Exercice 6.35
La valeur moyenne d’une fonction f : R2 → R sur un domaine R ⊂ R2 est définie par

1
Ï
f moy ≡ f (x, y) dx dy.
Aire(R) R
³ 2
´
1. Une lentille circulaire de rayon 2 cm a épaisseur 1 − r4 cm en tout point qui se trouve à r cm du centre de la
lentille. Calculer l’épaisseur moyenne de la lentille.
2. La température (en Celsius) en un point (x, y) d’une plaque plane est donnée par la fonction T (x, y) = 5x y + x 2 où
x et y sont mesurés en mètres. Calculer la température moyenne de la portion de la plaque en forme de diamant
qui est définie par
P = (x, y) ∈ R2 tel que ¯2x + y ¯ ≤ 4 et ¯2x − y ¯ ≤ 4 .
© ¯ ¯ ¯ ¯ ª

226 © G. Faccanoni
Dernière mise à jour : Jeudi 5 juin 2014 6 Intégrales multiples

C ORRECTION .
1. L’épaisseur moyenne de la lentille est donné par
R 2 R 2π ³ r2
´ R2³ r3
´
¸2
1 − 4 r dr dϑ 2π r − 4 dr 1 r2 r4
·
0 0 0 1
= = − = .
4π 4π 2 2 16 0 2

2. Le domaine est symétrique par rapport à l’axe des abscisses et T (x, −y) = −T (x, y) donc
Ï Ï
T (x, y) dx dy = 2 T (x, y) dx dy
P P+

P + = P ∩ (x, y) ∈ R2 ¯ y ≥ 0
© ¯ ª

= (x, y) ∈ R2 ¯ −4 ≤ x ≤ 0 et 0 ≤ y ≤ 2x + 4 ∪ (x, y) ∈ R2 ¯ 0 ≤ x ≤ 4 et 0 ≤ y ≤ −2x + 4


© ¯ ª © ¯ ª

et
Aire(P ) = 2Aire(P + ) = 32 cm.
Donc la température moyenne de la plaque est donné par
Î
2 P + T (x, y) dx dy
Tmoy =
32
µZ 0 Z 2x+4 Z 4 Z −2x+4 ¶
1
= (5x y + x 2 ) dy dx + (5x y + x 2 ) dy dx
16 −4 0 0 0
ÃZ ·
2 ¸2x+4 ¸−2x+4 !
0 y2
Z 4·
1 y 2 2
= 5x +x y dx + 5x +x y dx
16 −4 2 0 0 2 0

(2x + 4)2 (−2x + 4)2


µZ 0 Z 4 ¶
1
= 5x + x 2 (2x + 4) dx + 5x + x 2 (−2x + 4) dx
16 −4 2 0 2
µZ 0 Z 4 ¶
1 3 2 3 2
= 3x + 7x + 2x dx + 2x − 9x + 10x dx
4 −4 0
µ·µ ¶ ¸0 ·µ ¶ ¸4 ¶
1 3 2 7 2 1 2
= x + x +1 x + x − 3x + 5 x 2
4 4 3 −4 2 0
16
=− .
3

y
Exercice 6.36 Centre de gravité d’un
poisson 3
Calculer les coordonnées du centre de
gravité G de la plaque D en figure sup- 2
Ellipse d’équation
posée homogène (éviter le calcul d’inté-
1 (x − 4)2 (y − 2)2
grales lorsqu’il est possible, mais justi- + = 12
fier le raisonnement) : 4 1
0 1 2 3 4 5 6 x

C ORRECTION . Tout d’abord remarquons que le corps du poisson est une ellipse d’équation

¯ (x − 4)2 (y − 2)2
½ ¯ ¾
2 2
(x, y) ∈ R ¯
¯ + =1 .
22 12

Pour calculer les coordonnées (xG , yG ) du centre de gravité on doit calculer d’abord les trois intégrales doubles suivantes :
Ï Z 2Z 4−x Z 2π Z 1 Z 2 Z 2π
1 dy dx = 1 dy dx + 2r dr dϑ = (4 − 2x) dx + 1 dϑ = 1 + 2π,
D 1 x 0 0 1 0
Z 2Z 4−x 2π Z 1 2¡ 2π µ ¶
4
Ï Z Z Z
2
¢
x dy dx = x dy dx + (4 + 2r cos(ϑ))2r dr dϑ = 4x − 2x dx + 4 + cos(ϑ) dϑ
D 1 x 0 0 1 0 3
Z 2µ ¶ · ¸2π
2 2 3 4 4
= 2x − x dx + 4ϑ + sin(ϑ) = + 8π,
1 3 3 0 3

© G. Faccanoni 227
6 Intégrales multiples Dernière mise à jour : Jeudi 5 juin 2014

y y

3 ¡4 ¢ 3
GT = 3,2
G E = (4, 2)

2 2

Aire(T ) = 1 Aire(E ) = 2π
1 1

0 x 0 x
1 2 3 4 5 6 1 2 3 4 5 6

F IGURE 6.1: Exercice 6.36

(4 − x)2 x 2
Z 2Z 4−x 2π Z 1 Z 2µ ¶ Z 2π µ ¶
2
Ï Z
y dy dx = y dy dx + (2 + r sin(ϑ))2r dr dϑ = − dx + 2 + sin(ϑ) dϑ
D 1 x 0 0 1 2 2 0 3
Z 2 · ¸2π
2
= (8 − 4x) dx + 2ϑ − cos(ϑ) = 2 + 4π
1 3 0

et on en déduit que Î 4 Î
D x dy dx + 8π y dy dx 2 + 4π
xG = Î = 3
, yG = ÎD = = 2.
D 1 dy dx 1 + 2π D 1 dy dx 1 + 2π
En Î
fait, aucune des trois intégrales est nécessaire car on peut les calculer géométriquement comme suit (cf. figure 6.1)
B D 1 dy dx = Aire(D) = Aire(T ) + Aire(E ) = 1 + 2π,
Aire(T ) Aire(E ) 4/3 + 8π
B xG = xG + xG =
Aire(D) T Aire(D) E 1 + 2π
B yG = 2 car la droite d’équation y = 2 est un axe de symétrie pour la plaque.

Exercice 6.37 Aire de la cardioïde


L’équation de la cardioïde est paramétrée par ϕ comme y
suit : (x = r (ϕ) cos(ϕ), y = r (ϕ) sin(ϕ)) où r (ϕ) = 1 − sin(ϕ)
et ϕ ∈ [−π; π]. Considérons le domaine délimité par cette
−1 1 x
courbe, i.e. l’ensemble décrit en coordonnées polaires par
©
(r, ϕ) ¯ −π ≤ ϕ ≤ π,
¯
0 ≤ r ≤ 1 − sin(ϕ)
ª −1

Calculer l’aire de cette ensemble. Calculer ensuite son −2


centre de gravité en la considérant une plaque homogène
de densité surfacique de masse 1.

C ORRECTION . Rappelons d’abord que

cos2 (t ) = 1+cos(2t )
cos2 (t ) dt = 1+cos(2t )
dt = 2t + sin(2t )
R R
2 =⇒ 2 4 +C ,
2 1−cos(2t ) 1−cos(2t ) t sin(2t )
sin2 (t ) dt =
R R
sin (t ) = 2 =⇒ 2 dt = 2 − 4 +C ,

et que

sinn+1 (ϕ) cosn+1 (ϕ)


Z Z
cos(ϕ) sinn (ϕ) dϕ = , sin(ϕ) cosn (ϕ) dϕ = − , n ≥ 0.
n +1 n +1

On peut alors calculer la surface de la cardioïde :

π ¸1−sin(ϕ) Z π π
1−sin(ϕ) r2 (1 − sin(ϕ))2
Z Z Z ·
Aire = r dr dϕ = dϕ = dϕ
−π 0 −π 2 0 −π 2
Z π Z π Z π
1 sin2 (ϕ)
= dϕ + −2 sin(ϕ) dϕ + dϕ
−π 2 −π −π 2

228 © G. Faccanoni
Dernière mise à jour : Jeudi 5 juin 2014 6 Intégrales multiples

Z π 1 − cos(2ϕ) π 3
= π+0+ dϕ = π + + 0 = π.
−π 4 2 2

Pour calculer le centre de gravité on doit calculer les intégrales


Z π Z 1−sin(ϕ) Z π Z 1−sin(ϕ)
r cos(ϕ)r dr dϕ, r sin(ϕ)r dr dϕ.
−π 0 −π 0

On a

π π π ¸1−sin(ϕ)
1−sin(ϕ) 1−sin(ϕ) r3
Z Z Z µZ ¶ Z ·
r cos(ϕ)r dr dϕ = cos(ϕ) r 2 dr dϕ = cos(ϕ) dϕ
−π 0 −π 0 −π 3 0
Z π π
(1 − sin(ϕ))3 1
Z
= cos(ϕ) dϕ = cos(ϕ)(1 − sin(ϕ) + sin2 (ϕ) − sin3 (ϕ)) dϕ = 0
−π 3 3 −π

et
π π π ¸1−sin(ϕ)
1−sin(ϕ) 1−sin(ϕ) r3
Z Z Z µZ ¶ Z ·
r sin(ϕ)r dr dϕ = sin(ϕ) r 2 dr dϕ = sin(ϕ) dϕ
−π 0 −π 0 −π 3 0
Z π π
(1 − sin(ϕ))3 1
Z
= sin(ϕ) dϕ = sin(ϕ) − 3 sin2 (ϕ) + 3 sin3 (ϕ) − sin4 ϕ dϕ =
−π 3 3 −π
µ ¶
1 3 5
= 0 − 3π + 0 − π = − π
3 4 4
car
Z π
£ ¤π
sin(ϕ) dϕ = cos(ϕ) −π = 0,
−π
ϕ sin(2ϕ) π
Z π Z π · ¸
1 − cos(2ϕ)
sin2 (ϕ) dϕ = dϕ = − = π,
−π −π 2 2 4 −π
Z π Z π Z π ¸π
π
cos3 ϕ
Z ·
sin3 (ϕ) dϕ = sin(ϕ)(1 − cos2 (ϕ)) dϕ = sin(ϕ) dϕ −sin(ϕ) cos2 (ϕ) dϕ = − cos(ϕ) + = 0,
−π −π −π −π 3 −π
sin(2ϕ) π
Z π Z πµ Z π
1 − cos(2ϕ) 2 1 π 1 π π
¶ · ¸
1 1 3
Z Z
sin4 ϕ dϕ = dϕ = dϕ − cos(2ϕ) dϕ + cos2 (2ϕ) dϕ = + 0 + ϕ+ = π.
−π −π 2 −π 4 2 −π 4 −π 2 4 2 −π 4

On obtient les coordonnées du centre de gravité


R π R 1−sin(ϕ)
−π 0 r cos(ϕ)r dr dϕ 0
xG = = 3
= 0,

Aire
R π R 1−sin(ϕ)
−π 0 r cos(ϕ)r dr dϕ − 54 π 5
yG = = 3
=− .

Aire 6

Exercice 6.38 Aire des trèfles de H ABENICHT


Calculer l’aire de la région définie en coordonnées polaires par

D = {(r, φ) ∈ R+ × [0, 2π] : r ≤ 1 + cos(kφ) + sin2 (kφ), k ∈ N∗ }.


Suggestion : utiliser les formules de duplication cos(2θ) = 2 cos2 (θ)−1 = 1−2 sin2 (θ) et calculer au préalable les intégrales
suivantes
Z 2kπ Z 2kπ Z 2kπ Z 2kπ Z 2kπ Z 2kπ
2 2 2
1 dα, cos(α) dα, cos (α) dα, cos(α) sin (α) dα, sin (α) dα, sin4 (α) dα.
0 0 0 0 0 0

© G. Faccanoni 229
6 Intégrales multiples Dernière mise à jour : Jeudi 5 juin 2014

C ORRECTION . On a
Z 2kπ h i2kπ Z 2kπ h i2kπ
1 dα = α = 2kπ, cos(α) dα = sin(α) = 0,
0 0 0 0
Z 2kπ 1h i2kπ Z 2kπ h sin3 (α) i2kπ
cos2 (α) dα = sin(2α) + 2α = kπ, cos(α) sin2 (α) dα = = 0,
0 4 0 0 3 0
Z 8π 2kπ
1 1h i2kπ 3
h i2kπ Z
sin2 (α) dα = 2α − sin(2α) = kπ, sin4 (α) dα = 24α − 8 sin(2α) + sin(4α) = kπ.
0 4 0 0 32 0 4

L’aire est donnée, en coordonnées polaires, par


Ï Z 2π Z 1+cos(kφ)+sin2 (kφ)
r dr dθ = r dr dφ
D 0 0
1
Z 2π h r 2 i1+cos(kφ)+sin2 (kφ)
= dφ
2 0 2 0
Z 2π
1
= (1 + cos(kφ) + sin2 (kφ))2 dφ
2 0
1 2kπ
Z
= (1 + cos(α) + sin2 (α))2 dα
2k 0
1 2kπ
Z
= 1 + cos2 (α) + sin4 (α) + 2 cos(α) + 2 sin2 (α) + 2 cos(α) sin2 (α) dα
2k 0
·Z 2kπ Z 2kπ Z 2kπ Z 2kπ Z 2kπ Z 2kπ ¸
1 2 4 2 2
= 1 dα + cos (α) dα + sin (α) dα + 2 cos(α) dα + 2 sin (α) dα + 2 cos(α) sin (α) dα
2k 0 0 0 0 0 0
· ¸
1 3 23
= 2kπ + kπ + kπ + 0 + 2kπ + 0 = π.
2k 4 8

y y y

k=1 k=2 k=3

x x x

y y y

k=4 k=5 k=6

x x x

230 © G. Faccanoni
Dernière mise à jour : Jeudi 5 juin 2014 6 Intégrales multiples

Intégrales triples

Exercice 6.39
Calculer
p
Z 1 Z 1−z 2 Z 2−x 2 −z 2
(x 2 + z 2 ) /2 dy dx dz;
3
a) p
−1 − 1−z 2 x 2 +z 2
1
Ñ
b) p dx dy dz où D = {(x, y, z) : 1 < x 2 + y 2 < 5 − z};
D (4 − z)2 x2 + y 2
1
Ñ
c) p dx dy dz où D = {(x, y, z) : 5 − y < x 2 + z 2 < 1}.
(4 − y)4 x 2 + z 2
D
Z 1 Z p1−z 2 Z 2−y 2 −z 2
(y 2 + z 2 ) /2 dx dy dz.
3
d) p
−1 − 1−z 2 y 2 +z 2

C ORRECTION .
a) En coordonnées cartésiennes on doit intégrer sur le solide défini par les inégalités
p p
− 1 − z2 ≤ x ≤ 1 − z2, x2 + z2 ≤ y ≤ 2 − x2 − z2 − 1 ≤ z ≤ 1.

Il est naturel de passer en coordonnées cylindriques



x = r cos(θ),

y = t,

z = r sin(θ),

et on obtient l’intégrale triple


p
Z 1 Z 1−z 2 Z 2−x 2 −z 2 Z 1Z 2π Z 2−r 2
(x 2 + z 2 ) /2 dy dx dz = r 4 dt dθ dr
3

p
−1 − 1−z 2 x 2 +z 2 0 0 r2
¸1
2π 1 1 r7 r5
µZ ¶Z ·
8
Z
2
= 1 dθ r 4 [t ]2−r
r2
dr = 4π r 4 (r 2 − 1) dr = 4π − = π.
0 0 0 7 5 0 35

b) On doit calculer l’intégrale triple


1
Ñ
p dx dy dz
D (4 − z)2 x2 + y 2
où le domaine D est décrit, en coordonnées cartésiennes, par

D = {(x, y, z) : 1 < x 2 + y 2 < 5 − z}.

On passe en coordonnées cylindriques :



x = r cos(θ),

y = r sin(θ),

z = t,

et le jacobien du changement de variables est r . Alors

D = (r, θ, t ) ∈ R+ × [0, 2π] × R ¯ 1 < r 2 < 5 − t


© ¯ ª

i.e.
D = (r, θ, t ) ∈ R+ × [0, 2π] × R ¯ r > 1 et t < 5 − r 2
© ¯ ª

et on a
Ñ
1
Z 2π Z +∞ Z 5−r 2 1
dx dy dz = r dt dr dθ
(4 − t )2 r
p
D (4 − z)2 x2 + y 2 0 1 −∞
2
1 t =5−r
Z +∞ · ¸ Z +∞
1
= 2π − dr = −2π dr = ∞.
1 4 − t t →−∞ 1 r2 −1

© G. Faccanoni 231
6 Intégrales multiples Dernière mise à jour : Jeudi 5 juin 2014

c) On doit calculer l’intégrale triple


1
Ñ
p dx dy dz
D (4 − y)4 x2 + z2
où le domaine D est décrit, en coordonnées cartésiennes, par

D = (x, y, z) ¯ 5 − y < x 2 + z 2 < 1 .


© ¯ ª

On passe en coordonnées cylindriques :



x = r cos(θ),

y = t,

z = r sin(θ),

et le jacobien du changement de variables est r . Alors

D = (r, θ, t ) ∈ R+ × [0, 2π] × R ¯ 5 − t < r 2 < 1


© ¯ ª

i.e.
D = (r, θ, t ) ∈ R+ × [0, 2π] × R ¯ 0 < r < 1 et t > 5 − r 2
© ¯ ª

et on a
Ñ
1
Z 2π Z 1 Z +∞ 1
Z 1 Z +∞
1
p dx dy dz = 4
r dt dr dθ = 2π 4
dt dr
D (4 − y)4 x 2 + z 2 0 0 5−r 2 (4 − t ) r 0 5−r 2 (4 − t )
Z 1· ¸t →+∞ Z 1
1 1
= 2π − 3
dr = 2π − 2 − 1)3
dr = ∞.
0 3(4 − t ) t =5−r 2 0 3(r

d) En coordonnées cartésiennes on doit intégrer sur le solide défini par les inégalités
p p
y 2 + z2 ≤ x ≤ 2 − y 2 − z2, − 1 − z2 ≤ y ≤ 1 − z2, −1 ≤ z ≤ 1.

Il est naturel de passer en coordonnées cylindriques



x = t ,

y = r cos(θ),

z = r sin(θ),

et on obtient l’intégrale triple


p
Z 1 Z 1−z 2 Z 2−y 2 −z 2 Z 1Z 2π Z 2−r 2
(y 2 + z 2 ) /2 dx dy dz = r 4 dt dθ dr
3

p
−1 − 1−z 2 y 2 +z 2 0 0 r2
¸1
2π 1 1 r7 r5
µZ ¶Z ·
8
Z
4 2
= 1 dθ r [t ]2−r
r2
dr = 4π 4 2
r (r − 1) dr = 4π − = π.
0 0 0 7 5 0 35

232 © G. Faccanoni
7 Fonctions vectorielles d’une variable réelle :
courbes paramétrées
Définition Courbe
Une courbe C de Rn est l’image d’une application continue γ : [a; b] ⊂ R → Rn . On dit que γ est un paramétrage de C .

Attention
Une même courbe C admet une infinité de paramétrages.

Exemple
1. Une droite de R2 est une courbe. Un paramétrage possible de la droite passant par le point (x 0 , y 0 ) et dirigée par le vecteur (u, v)
est l’application γ : R → R2 définie par γ(t ) = (x 0 + t u, y 0 + t v). Un autre paramétrage possible est l’application γ : R → R2 définie
par γ(t ) = (t , uv t + y 0 − uv x 0 ).
2. Un cercle de R2 est une courbe. Un paramétrage possible du cercle de centre (x 0 , y 0 ) et de rayon r est l’application γ : [0; 2π] → R2
définie par γ(t ) = (x 0 + r cos(t ), y 0 + r sin(t )).
3. Si f : [a; b] ⊂ R → R est une application continue alors son graphe γ : R → R2 définie par γ(t ) = (t , f (t )) est une courbe (appelée
courbe représentative).

Un problème se pose. Étant donné un ensemble C de points du plan, comment déterminer si C est une courbe ? Par exemple,
une ellipse est-elle une courbe ? Le carré [0, 1] × [0, 1] est-il une courbe ? Il va falloir manipuler la notion de courbe avec
beaucoup de soin. D’abord, avec la définition de courbe que nous venons de donner, un ensemble réduit à un point M est
automatiquement une courbe car toute application constante γ : R → { M } en est un paramétrage. Ensuite, en 1890, Giuseppe
P EANO a montré qu’il existe une application continue surjective γ : [0, 1] → [0, 1] × [0, 1] (c’est-à-dire que la courbe passe par
chaque point du carré : elle «remplit l’espace»). Cela montre, contrairement à ce que notre intuition pourrait nous laisser
croire, que le carré [0, 1] × [0, 1] est une courbe !

Définition Courbe paramétrée et support


Une courbe paramétrée de Rn est une application continue γ : [a; b] ⊂ R → Rn . La courbe C = γ([a; b]) est appelé le support
de γ.

Lorsque nous étudierons une courbe paramétrée γ : [a; b] ⊂ R → Rn , nous aurons généralement une vision cinématique du
problème. Nous considérerons par exemple que le point γ(t ) est un point du plan (n = 2) ou de l’espace (n = 3) qui évolue
avec le temps t ∈ [a; b]. C’est pourquoi le paramètre sera habituellement noté t (pour temps).
Rappelons qu’une application γ : [a; b] ⊂ R → Rn qui s’écrit en coordonnées γ(t ) = (x 1 (t ), x 2 (t ), . . . , x n (t )) est continue si, et
seulement si, les n fonctions coordonnées x i : [a; b] → R sont des fonctions continues.

Exemple y
Considérons les fonctions x : [−2; 2] → R et y : [−2; 2] → R défi-
nies par x(t ) = t + cos(3t ) et y(t ) = sin(2t ). Ces deux fonctions
sont continues sur [−2; 2] et induisent donc une courbe para-
métrée γ : [a; b] → R2 définie par γ(t ) = (t + cos(3t ), sin(2t )).
x

γ(t ) = (t + cos(3t ), sin(2t ))

233
7 Fonctions vectorielles d’une variable réelle : courbes paramétrées Dernière mise à jour : Jeudi 5 juin 2014

Définition
Soit γ : [a; b] ⊂ R → Rn une courbe paramétrée de Rn et k un entier positif. La courbe paramétrée est dérivable (resp. de
classe C k ou C ∞ ) si l’application γ est dérivable (resp. de classe C k ou C ∞ ).

Définition
Soit γ : [a; b] ⊂ R → Rn une courbe paramétrée de Rn de classe C 2 admettant une représentation paramétrique γ(t ) =
(x 1 (t ), x 2 (t ), . . . , x n (t )). La longueur de γ, indépendante du paramétrage choisi, est
s
Z b n
`=
X
(x i0 (t ))2 dt .
a i =1

Exemple Deltoïde
Considérons la courbe paramétrée γ : [0; 2π] → R2 définie par γ(t ) = 2 cos(t ) + cos(2t ), 2 sin(t ) − sin(2t ) .
¡ ¢

−1 0 1 2 3 x

−1

−2

La longueur de la courbe est


Z 2π r³
¢0 ´2 ³¡ ¢0 ´2
`=
¡
2 cos(t ) + cos(2t ) + 2 sin(t ) − sin(2t ) dt
0
Z 2π r³ ´2 ³ ´2
= − 2 sin(t ) − 2 sin(2t ) + 2 cos(t ) − 2 cos(2t ) dt
0
Z 2π r ³ ´
= 4 sin2 (t ) + sin2 (2t ) + 2 sin(t ) sin(2t ) + cos2 (t ) + cos2 (2t ) + 2 cos(t ) cos(2t ) dt
0

Z r ³ ´
= 8 1 + sin(t ) sin(2t ) + cos(t ) cos(2t ) dt
0
Z 2π r ³ ´
= 8 1 + 2 sin2 (t ) cos(t ) + cos(t )(1 − 2 sin2 (t )) dt
0
Z 2π r ³ ´
= 8 1 + cos(t ) dt = 16.
0

Exemple
Considérons la courbe paramétrée γ : [a; b] → R2 définie par γ(t ) = (t , f (t )). Alors la longueur de la courbe est
Z bq
`= 1 + ( f 0 (t ))2 dt .
a

234 © G. Faccanoni
Dernière mise à jour : Jeudi 5 juin 2014 7 Fonctions vectorielles d’une variable réelle : courbes paramétrées

7.1 Courbes planes en coordonnées polaires

Définition
Une courbe paramétrée γ : [a; b] ⊂ R → R2 est définie en coordonnées polaires par la fonction % : [a; b] ⊂ R si % est une
fonction continue et
γ(ϑ) = (%(ϑ) cos(ϑ), %(ϑ) sin(ϑ)).
La fonction % peut prendre des valeurs positives ou négatives. Le réel |%(ϑ)| est la distance de l’origine à γ(ϑ).
Si γ est de classe C 2 , sa longueur est
Z bq
`= (ρ(ϑ))2 + (ρ 0 (ϑ))2 dϑ.
a

Exemple y
Soit % : [0; 2π] → R définie par %(ϑ) = r > 0 avec r constante. La
courbe définie en coordonnées polaires par ρ est un cercle de
centre (0, 0) et rayon r . On a bien x
Z 2π q Z 2π p Z 2π
`= (ρ(ϑ))2 + (ρ 0 (ϑ))2 dϑ = r 2 + 02 dϑ = r dϑ = 2πr.
0 0 0

Exemple y

Soit % : [0; 2π] → R définie par %(ϑ) = sin2 (ϑ/2). La courbe défi-
nie en coordonnées polaires par ρ est une cardioïde et l’on a
Z 2π q Z 2π q x
`= (ρ(ϑ))2 + (ρ 0 (ϑ))2 dϑ = sin4 (ϑ/2) + (sin(ϑ/2) cos(ϑ/2))2 dϑ
0 0
Z 2π Z π
= | sin(ϑ/2)| dϑ = 2 sin(ϑ/2) dϑ = 2 [−2 cos(ϑ/2)]π
0 = 8.
0 0

© G. Faccanoni 235
8 Champs de vecteurs, formes différentielles
Définition Ouvert étoilé
Un ouvert U de Rn est étoilé s’il existe a ∈ U tel que, pour
tout x ∈ U , le segment d’extrémités a et x soit inclus dans
U , autrement dit, si tout segment reliant un point de l’en-
semble à son centre est inclus dans l’ensemble.

Définition Ouvert simplement connexe


Un ouvert U de Rn est simplement connexe si
1. U est connexe par arc, i.e. tout couple de points (x 0 , y 0 ) et (x 1 , y 1 ) de U peut être joint par une courbe continue :

0
γ ∈ C ([a, b]),


∃γ : [a, b] → U t.q. γ(a) = (x 0 , y 0 ),
© ª
∀ (x 0 , y 0 ), (x 1 , y 1 ) ⊂ U

γ(b) = (x 1 , y 1 );

2. tout courbe fermée contenue dans U peut être ramenée, par déformation continue, à un point.

Exemple
Considérons une pomme et imaginons un élastique autour de cette pomme. En faisant glisser l’élastique tout doucement, il est pos-
sible de le comprimer en un point de la pomme, sans couper l’élastique ni lui faire quitter la surface de la pomme. Prenons maintenant
un anneau et imaginons un élastique enfilé autour de l’anneau. Cette fois, il est impossible, sans couper l’élastique ou l’anneau, de
réduire juste par glissement et compression l’élastique en un point. La pomme est donc une surface simplement connexe dans R3 ,
alors que l’anneau ne l’est pas.
B R2 est simplement connexe
B un disque de R2 est simplement connexe
B une couronne circulaire de R2 n’est ni étoilée ni simplement connexe
B tout ©ensemble connexe de Rª2 ayant un trou¯ n’est ni étoilée ni simplement connexe
B U = ©(x, y) ∈ R2 ¯¯x 2 + y 2 ≤ 1 ∪ (x, y) ∈ª R2©¯ (x − 1)2 +¯(y − 1)2 ≤ 1 est simplement
¯ © ª
connexe
B U = (x, y) ∈ R2 ¯ (x + 1)2 + (y + 1)2 ≤ 1 ∪ (x, y) ∈ R2 ¯ (x − 1)2 + (y − 1)2 ≤ 1 n’est pas connexe donc n’est ni étoilé ni simple-
ª

ment© connexe ¯
B U = © (x, y) ∈ R2 ¯¯ (x + 1)2 + y 2 ≤ 1 ª ∪© (x, y) ∈ R2 ¯¯ (x − 1)2 + y 2 ≤ 1ª est simplement connexe
ª © ¯ ª
2 2 2 2 2 2
B U = (x, y) ∈ R ¯ (x + 1) + y < 1 ∪ (x, y) ∈ R ¯ (x − 1) + y < 1 n’est pas connexe donc n’est ni étoilé ni simplement connexe
B R2 privé d’un carré n’est ni étoilé ni simplement connexe
B R2 privé d’un point n’est ni étoilé ni simplement connexe
B R3 est simplement connexe
B une sphère de R3 est simplement connexe
B un tore (=doughnut) de R3 n’est ni étoilé ni simplement connexe
B R3 privé d’une sphère est simplement connexe
B R3 privé d’un point est simplement connexe
B R3 privé d’une droite n’est ni étoilé ni simplement connexe

237
8 Champs de vecteurs, formes différentielles Dernière mise à jour : Jeudi 5 juin 2014

8.1 Formes différentielles 8.2 Champs de vecteurs

Définition Définition
Une forme différentielle de degré 1 définie sur un ouvert U On appelle champ de vecteurs défini sur un ouvert U ⊂ R2
de R2 est une application toute application de U dans R2 telle que

ω(x, y) = ω1 (x, y) dx + ω2 (x, y) dy ~ (x, y) = (V1 (x, y),V2 (x, y))


V

où ω1 et ω2 sont fonctions de U dans R. où V1 et V2 sont fonctions de U dans R.


De la même manière, une forme différentielle de degré 1 De la même manière, on appelle champ de vecteurs dé-
définie sur un ouvert U de R3 est une application fini sur un ouvert U ⊂ R3 toute application de U dans R3
telle que
ω(x, y, z) = ω1 (x, y, z) dx + ω2 (x, y, z) dy + ω3 (x, y, z) dz
~ (x, y, z) = (V1 (x, y, z),V2 (x, y, z),V3 (x, y, z))
V
où ω1 , ω2 et ω3 sont fonctions de U dans R.
Si tous les ωi sont de classe C k sur U , on dit que ω est de où V1 , V2 et V3 sont fonctions de U dans R.
classe C k sur U . ~ est de
Si tous les Vi sont de classe C k sur U , on dit que V
k
classe C sur U .

Définition Forme exacte et primitive Définition Champ conservatif et potentiel


Une forme différentielle ω est exacte (ou totale) s’il existe Un champ de vecteurs V ~ est conservatif (i.e. il est un
une fonction f : U → R, de classe C 1 , telle que df = ω. On champ de gradients) s’il existe une fonction f : U → R, de
dit alors que f est une primitive de ω sur U . ~ . On dit alors que f est un po-
classe C 1 , telle que ∇ f = V
tentiel de V~ sur U .

Exemple Exemple
Soit ω = 2x dx + 2y dy une forme différentielle dans R2 . Elle est ~ = (2x, 2y) un champ de vecteurs dans R2 . Il est conser-
Soit V
exacte car f : R2 → R définie par f (x, y) = x 2 + y 2 vérifie df = ω. ~.
vatif car f : R2 → R définie par f (x, y) = x 2 + y 2 vérifie ∇ f = V

238 © G. Faccanoni
Dernière mise à jour : Jeudi 5 juin 2014 8 Champs de vecteurs, formes différentielles

Définition Forme différentielle fermée Définition Champ à rotationnel nul


Une forme différentielle ω définie sur un ouvert U de R 2 ~ défini sur un ouvert U de R2 est
Un champ de vecteurs V
est fermée si à rotationnel nul si
∂ y (ω1 ) = ∂x (ω2 ).
∂ y (V1 ) = ∂x (V2 ).
Une forme différentielle ω définie sur un ouvert U de R3
est fermée si Un champ de vecteurs V ~ défini sur un ouvert U de R3 est
∂ y (ω3 ) = ∂z (ω2 ),

 à rotationnel nul si
∂z (ω1 ) = ∂x (ω3 ),
∂ y (V3 ) = ∂z (V2 ),
 
∂x (ω2 ) = ∂ y (ω1 ).
 
∂ (V ) = ∂x (V3 ),
 z 1
∂x (V2 ) = ∂ y (V1 ).

Théorème de S CHWARZ : condition nécessaire pour ω Théorème de S CHWARZ : condition nécessaire pour V
exacte conservatif
Une forme différentielle exacte de classe C 1 est toujours ~ de classe C 1 conservatif est à
Un champ de vecteurs V
fermée. rotationnel nul.

Théorème de P OINCARÉ : condition suffisante pour ω Théorème de P OINCARÉ : condition suffisante pour V
exacte conservatif
Si U est un ouvert étoilé ou si U est simplement connexe Si U est un ouvert étoilé ou si U est simplement connexe
et ω est une forme différentielle de classe C 1 , alors ~ est un champ de vecteurs de classe C 1 , alors
et V

ω est exacte sur U ⇐⇒ ω est fermée sur U . ~ est conservatif sur U ⇐⇒ V


V ~ est à rotationnel nul sur U .

Exemple Exemple
Soit ω = (2x +y) dx +(2y +x) dy une forme différentielle dans R2 . ~ = (2x + y, 2y + x) un champ de vecteur dans R2 .
Soit V
CN pour ω exacte : comme ∂ y (ω1 ) = ∂ y (2x + y) = 1 et CN pour V ~ conservatif : comme ∂ y (V1 ) = ∂ y (2x + y) = 1 et
∂x (ω2 ) = ∂x (2y +x) = 1, la forme différentielle est fermée ; ∂x (V2 ) = ∂x (2y + x) = 1, le champ est à rotationnel nul ;
Domaine de définition : R2 est simplement connexe donc la Domaine de définition : R2 est simplement connexe donc la
CN est aussi une CS ; CN est aussi une CS ;
grâce au théorème de P OINCARÉ on conclut que ω est exacte, grâce au théorème de P OINCARÉ on conclut que V ~ est conser-
c’est-à-dire qu’il existe f : R2 → R telle que d f = ω. Par exemple 2 ~ . Par
vatif, c’est-à-dire qu’il existe f : R → R telle que ∇ f = V
f (x, y) = x 2 + x y + y 2 . 2
exemple f (x, y) = x + x y + y .2

Opérateurs classiques
~ un champ vectoriel différentiable définis comme suit :
Soit f une fonction différentiable et V

f : U ⊂ R3 → R ~ : U ⊂ R3 → R3
V
µV
1 (x,y,z)

(x, y, z) 7→ f (x, y, z) ~ (x, y, z) ≡
(x, y, z) 7→ V V2 (x,y,z)
V3 (x,y,z)

Gradient Le gradient de f , noté ∇ f ou encore grad f , est le champ vectoriel

∇ f : U → R3
µ∂
x f (x,y,z)

(x, y, z) 7→ ∇ f (x, y, z) ≡ ∂ y f (x,y,z)
∂z f (x,y,z)

Si f et g sont deux deux fonctions différentiables de U dans R, on a


B ∇( f + g ) = ∇ f + ∇g

© G. Faccanoni 239
8 Champs de vecteurs, formes différentielles Dernière mise à jour : Jeudi 5 juin 2014

B ∇(λ f ) = λ∇ f pour tout λ ∈ R


B ∇( f g ) = f ∇g + g ∇ f
~ , notée ∇ · V
Divergence La divergence de V ~ ou encore divV
~ , est la fonction

~ : U →R
∇·V
~ (x, y, z) ≡ ∂x V1 (x, y, z) + ∂ y V2 (x, y, z) + ∂z V3 (x, y, z)
(x, y, z) 7→ ∇ · V

~ et W
Si V ~ sont deux champs vectoriels différentiables de U dans R3 , on a
~
B ∇ · (V + W ~ ) = ∇·V~ +∇·W ~
~ ~
B ∇ · (λV ) = λ∇ · V pour tout λ ∈ R
B ∇·(f V ~ ) = f ∇·V~ +V~ · ∇ f (V
~ · ∇ f désignant le produit scalaire de V
~ par ∇ f )
Laplacien Si f est de classe C 2 (U ), le laplacien de f , noté ∆ f , est la fonction

∆f : U → R
(x, y, z) 7→ ∆ f ≡ ∇ · (∇ f (x, y, z)) = ∂xx f (x, y, z) + ∂ y y f (x, y, z) + ∂zz f (x, y, z)

Si f et g sont deux deux fonctions différentiables de U dans R, on a


B ∆( f + g ) = ∆ f + ∆g
B ∆(λ f ) = λ∆ f pour tout λ ∈ R
B ∆( f g ) = f ∆g + g ∆ f + 2(∇ f ) · (∇g ) ((∇ f ) · (∇g ) désignant le produit scalaire de ∇ f par ∇g )
Rotationnel Le rotationnel de V ~ , noté rotV ~ , est le champ vectoriel

~ : U → R3
rotV
µ ∂ y V3 (x,y,z)−∂z V2 (x,y,z) ¶
~ (x, y, z) ≡
(x, y, z) 7→ rotV ∂z V1 (x,y,z)−∂x V3 (x,y,z)
∂x V2 (x,y,z)−∂ y V1 (x,y,z)

Si f et g sont deux fonctions différentiables de U dans R et V ~ et W~ deux champs vectoriels différentiables de U


3
dans R , on a
B rot(V ~ +W ~ ) = rotV
~ + rotW~
~ ~
B rot(λV ) = λrotV pour tout λ ∈ R
~ ) = f rotV
B rot( f V ~ +∇f ∧V ~ (le symbole ∧ désignant le produit vectoriel)
~ ~ ~ ~ ~ ) · (rotW
B ∇ · (V ∧ W ) = (rotV ) · (W ) − (V ~)
~
B ∇ · (rotV ) = 0
B rot(∇ f ) = 0

240 © G. Faccanoni
Dernière mise à jour : Jeudi 5 juin 2014 8 Champs de vecteurs, formes différentielles

Définition Intégrale curviligne d’une forme différen-


tielle
Soit ω(x, y) = ω1 (x, y) dx + ω2 (x, y) dy une forme différen- Définition Circulation d’un champ de vecteurs
tielle définie sur U ⊂ R2 et soit γ : [a, b] → U un arc orienté ~ (x, y) = (V1 (x, y),V2 (x, y)) un champ de vecteurs dé-
Soit V
défini par γ(t ) = (x(t ), y(t )). ω1 et ω2 étant des fonctions fini sur U ⊂ R2 et soit γ : [a, b] → U un arc orienté défini
continues, on appelle intégrale curviligne de la forme dif- par γ(t ) = (x(t ), y(t )). V1 et V2 étant des fonctions conti-
férentielle ω le long de l’arc γ le nombre noté nues, on appelle circulation de V ~ le long de l’arc orienté
Z γ le nombre noté I
ω ~
V
γ γ

et défini par et défini par


Z b£
Z b£
V1 (x(t ), y(t ))x 0 (t ) + V2 (x(t ), y(t ))y 0 (t ) dt .
¤
0 0
ω1 (x(t ), y(t ))x (t ) + ω2 (x(t ), y(t ))y (t ) dt .
¤
a a

Soit ω(x, y, z) = ω1 (x, y, z) dx +ω2 (x, y, z) dy +ω3 (x, y, z) dz Soit V~ (x, y, z) = (V1 (x, y, z),V2 (x, y, z),V3 (x, y, z)) un
une forme différentielle définie sur U ⊂ R3 et champ de vecteurs défini sur U ⊂ R3 et soit γ : [a, b] → U
soit γ : [a, b] → U un arc orienté défini par un arc orienté défini par γ(t ) = (x(t ), y(t ), z(t )). V1 , V2 et
γ(t ) = (x(t ), y(t ), z(t )). ω1 , ω2 et ω3 étant des fonc- V3 étant des fonctions continues, on appelle circulation
tions continues, on appelle intégrale curviligne de la ~ le long de l’arc orienté γ le nombre noté
de V
forme différentielle ω le long de l’arc γ le nombre noté I
Z ~
V
ω γ
γ
et défini par
et défini par
Z bh
Z bh V1 (x(t ), y(t ), z(t ))x 0 (t ) + V2 (x(t ), y(t ), z(t ))y 0 (t )
ω1 (x(t ), y(t ), z(t ))x 0 (t ) + ω2 (x(t ), y(t ), z(t ))y 0 (t ) a
i
a
i + V3 (x(t ), y(t ), z(t ))z 0 (t ) dt .
+ ω3 (x(t ), y(t ), z(t ))z 0 (t ) dt .

Exemple
L’intégrale de la forme différentielle ω(x, y) = y dy définie sur R2 le long du graphe d’une fonction f : [a; b] → R de classe C 1 est égale
à ab f (t ) dt .
R

Exemple
~ = (−y, x) le long de la courbe γ : [0; 2π] → R2 définie par γ(t ) = (cos(t ), sin(t )) est égale à
La circulation du champ de vecteurs V
I Z 2π Z 2π Z 2π
~=
V −y(t )x 0 (t ) + x(t )y 0 (t ) dt = (− sin(t ))(− sin(t )) + (cos(t ))(cos(t )) dt = 1 dt = 2π.
γ 0 0 0

Proposition Proposition
Si ω est une forme différentielle exacte définie sur U ⊂ Rn ~ est un champ de vecteurs conservatif défini sur U ⊂
Si V
et de primitive f et si la courbe γ : [a; b] → U a pour ori- n
R et de potentiel f et si la courbe γ : [a; b] → U a pour
gine et extrémités respectivement les points γ(a) et γ(b) origine et extrémités respectivement les points γ(a) et
alors Z γ(b) alors I
ω = f (γ(b)) − f (γ(a)). ~ = f (γ(b)) − f (γ(a)).
V
γ
γ
En particulier, si la courbe γ est fermée, c’est-à-dire si ses En particulier, si la courbe γ est fermée, c’est-à-dire si ses
deux extrémités sont égales, alors la circulation sur γ de deux extrémités sont égales, alors la circulation sur γ de
toute forme différentielle exacte est nulle. tout champ de vecteurs conservatif est nulle.

© G. Faccanoni 241
8 Champs de vecteurs, formes différentielles Dernière mise à jour : Jeudi 5 juin 2014

Remarque
En mécanique, la circulation d’une force est égale au travail de cette force. Donc si la force dérive d’un potentiel, son
travail ne dépend que du point de départ et du point d’arrivé.

Exemple
Soit le champ de vecteurs
~ (x, y, z) = (2x y + z 3 , x 2 , 3xz 2 ).
V
On veut montrer qu’il dérive d’un potentiel scalaire et déterminer tous les potentiels scalaires dont il dérive.
D’une part, comme le champ est défini sur R3 qui est un ouvert étoilé, pour prouver qu’il dérive d’un potentiel il suffit de montrer qu’il
est à rotationnel nul :
2 2
∂ y (V3 ) = ∂z (V2 ), ∂ y (3xz ) = ∂z (x ),
  
  0 = 0,

∂z (V1 ) = ∂x (V3 ), ⇐⇒ ∂z (2x y + z 3 ) = ∂x (3xz 2 ), ⇐⇒ 3z 2 = 3z 2 ,
  
∂x (V2 ) = ∂ y (V1 ), ∂x (x 2 ) = ∂ y (2x y + z 3 ),
  
2x = 2x.

D’autre part, il est possible de calculer les potentiels : on cherche en effet f de R3 dans R tel que

3
∂x f (x, y, z) = 2x y + z ,


2
∂ y f (x, y, z) = x ,

∂z f (x, y, z) = 3xz 2 .

On résout ce système d’équations aux dérivées partielles : la deuxième donne par exemple f (x, y, z) = x 2 y + h(x, z). En la dérivant par
rapport à z on trouve ∂z f (x, y, z) = ∂z h(x, z) ; en utilisant la troisième équation on trouve ∂z h(x, z) = 3xz 2 d’où h(x, z) = xz 3 + g (x)
et donc f (x, y, z) = x 2 y + xz 3 + g (x). En la dérivant par rapport à x on trouve ∂x f (x, y, z) = 2x y + z 3 + g 0 (x) ; en utilisant la première
équation on trouve g 0 (x) = 0 d’où g (x) = C et finalement

f (x, y, z) = x 2 y + xz 3 +C , C ∈ R.

Exemple
On veut montrer que la forme différentielle
y x
ω(x, y) = − dx + dy,
(x − y)2 (x − y)2
définie sur U = (x, y) ∈ R2 ¯ x > y , est exacte et en déterminer toutes les primitives dont elle dérive.
© ¯ ª

D’une part, comme la forme est définie sur U qui est un ouvert étoilé, pour prouver qu’elle est exacte il suffit de montrer qu’elle est
fermée :
x y −2x y −2x y
µ ¶ µ ¶
∂ y (ω1 ) = ∂x (ω2 ) ⇐⇒ ∂ y = ∂ x − ⇐⇒ = .
(x − y)2 (x − y)2 (x − y)4 (x − y)4
D’autre part, il est possible de calculer les primitives : on cherche en effet f de R2 dans R tel que
−y x
ω1 (x, y) = , ω2 (x, y) = .
(x − y)2 (x − y)2
y
On résout ce système d’équations aux dérivées partielles : en intégrant la deuxième on obtient par exemple f (x, y) = x−y + h(x). En la
−y 0 0
dérivant par rapport à x on trouve ∂x f (x, y) = 2 + h (x) ; en utilisant la première équation on trouve h (x) = 0 et finalement
(x−y)

y
f (x, y) = +C , C ∈ R.
x−y

Proposition Formule de G AUSS -R IEMANN


Soit U un ouvert de R2 ayant un bord de classe C 1 par morceaux orienté de sorte que U soit à sa gauche. Soit ω une forme
différentielle de classe C 1 sur un ouvert contenant U . Alors
Z Z
dω = ω,
U ∂U

U (∂ y (ω1 ) − ∂x (ω2 )) dx
R R
où on a défini U dω = dy.

242 © G. Faccanoni
Dernière mise à jour : Jeudi 5 juin 2014 8 Champs de vecteurs, formes différentielles

Proposition Calcul de l’aire


Soit γ : [a; b] → R2 une courbe fermée simple de classe C 1 définie par γ(t ) = (x(t ), y(t )) et soit U la région délimitée par
γ. On suppose que lorsque t croît, le point γ(t ) parcourt le bord de U en laissant U à gauche. Alors l’aire de U est donnée
par
Rb
x(t )y 0 (t ) − y(t )x 0 (t ) dt
Z b Z b
Aire(U ) = x(t )y 0 (t ) dt = − y(t )x 0 (t ) dt = a .
a a 2

Exemple y

Soit % : [0; 2π] → R définie par %(ϑ) = sin2 (ϑ/2). La courbe γ(ϑ) =
(x(ϑ) = %(ϑ) cos(ϑ), y(ϑ) = %(ϑ) sin(ϑ)) est une cardioïde et l’on a
x
1 2π ¡ 1 2π 3
Z Z
x(ϑ)y 0 (ϑ) − y(ϑ)x 0 (ϑ) dϑ = sin4 (ϑ/2) dϑ = π.
¢
Aire(U ) =
2 0 2 0 8

© G. Faccanoni 243
8 Champs de vecteurs, formes différentielles Dernière mise à jour : Jeudi 5 juin 2014

UUUUUUUUUUUUUUUU Astuces UUUUUUUUUUUUUUU


Pn
ω(x) = i =1 ωi (x) dx i

Oui Non
∃ f tq df (x) = ω(x)

sais pas
Je ne
Non
ω est fermée

Oui

Oui
Dω est S.C.

Non

γ : [a; b] ⊂ R → Dω
t 7→ x(t ) Oui pour ce γ
B régulière particulier
B γ(a) = γ(b)

γω=0
R
ω exacte ω non exacte
Oui pour
tout γ Non

244 © G. Faccanoni
Dernière mise à jour : Jeudi 5 juin 2014 8 Champs de vecteurs, formes différentielles

............... Exercices ..............


Exercice 8.1 Exercice 8.2
Soit Soit
B V~ (x, y) = V1 (x, y)~
i +V2 (x, y)~
j un champ de vecteur dé- B ω(x, y) = ω1 (x, y) dx + ω2 (x, y) dy une forme différen-
2
fini sur R \ { (2, 2) }, tielle définie sur R2 \ { (2, 2) },
B γ1 la courbe d’équation γ1 (t ) = (2 + cos(t ), 2 + sin(t )), B γ1 la courbe d’équation γ1 (t ) = (2 + cos(t ), 2 + sin(t )),
t ∈ [0, 2π], t ∈ [0, 2π],
B γ2 la courbe d’équation γ2 (t ) = (cos(t ), sin(t )), t ∈ B γ2 la courbe d’équation γ2 (t ) = (cos(t ), sin(t )), t ∈
[0, 2π]. [0, 2π].
Dire si les affirmations suivantes sont V raies ou Dire si les affirmations suivantes sont V raies ou
F ausses. F ausses.
~ est conservatif alors V
1. Si V ~ est à rotationnel nul. 1. Si ω est exacte alors ω est fermée.
~ = 0 alors V
2. Si rotV ~ est conservatif. 2. Si ω est fermée alors ω est exacte.
~ est conservatif alors ~ 3. Si ω est exacte alors γ1 ω = 0.
H R
3. Si V γ1 V = 0.

~ est conservatif alors ~ 4. Si ω est exacte alors γ2 ω = 0.


H R
4. Si V γ2 V = 0.

~ est à rotationnel nul alors ~ 5. Si ω est fermée alors γ1 ω = 0.


H R
5. Si V γ1 V = 0.

~ est à rotationnel nul alors ~ 6. Si ω est fermée alors γ2 ω = 0.


H R
6. Si V γ2 V = 0.

C ORRECTION .
C ORRECTION .
1. V Si ω est exacte alors ω est fermée.
~ est conservatif alors V
1. V Si V ~ est à rotationnel nul. C’est le théorème de S CHWARZ.
C’est le théorème de S CHWARZ. 2. F Si ω est fermée alors ω est exacte.
~ = 0 alors V
2. F Si rotV ~ est conservatif. Pour qu’elle soit vraie pour toute forme différentielle fer-
Pour qu’elle soit vraie pour tout champ à rotationnel mée il faudrait que le domaine de définition soit simple-
nul il faudrait que le domaine de définition soit simple- ment connexe ou étoilé, ce qui n’est pas le cas ici.
ment connexe ou étoilé, ce qui n’est pas le cas ici. 3. V Si ω est exacte alors γ1 ω = 0.
R

~ est conservatif alors ~


H
3. V Si V γ1 V = 0.
La courbe est fermée, c’est-à-dire que ses deux extrémités
La courbe est fermée, c’est-à-dire que ses deux extrémi- sont égales γ1 (0) = γ1 (2π) = (3, 2), alors l’intégrale curvi-
tés sont égales γ1 (0) = γ1 (2π) = (3, 2), alors la circulation ligne de toute forme différentielle exacte le long de l’arc
sur γ1 de tout champ de vecteurs conservatif est nulle. γ1 est nulle.
4. V Si ω est exacte alors γ2 ω = 0.
R
~ est conservatif alors ~
H
4. V Si V γ2 V = 0.
La courbe est fermée, c’est-à-dire que ses deux extrémités
La courbe est fermée, c’est-à-dire que ses deux extrémi-
sont égales γ2 (0) = γ2 (2π) = (1, 0), alors l’intégrale curvi-
tés sont égales γ2 (0) = γ2 (2π) = (1, 0), alors la circulation
ligne de toute forme différentielle exacte le long de l’arc
sur γ2 de tout champ de vecteurs conservatif est nulle.
γ2 est nulle.
~ est à rotationnel nul alors ~
H
5. F Si V γ1 V = 0. 5. F Si ω est fermée alors γ1 ω = 0.
R

Pour qu’elle soit vraie il faudrait que V~ soit conservatif.


Pour qu’elle soit vraie il faudrait que ω soit exacte.
Comme le domaine de définition n’est pas simplement Comme le domaine de définition n’est pas simplement
connexe ni étoilé, on ne peut pas garantir la conservati- connexe ni étoilé, on ne peut pas garantir que ω est
~ donc la circulation sur γ1 de V
vité de V ~ peut être non
exacte donc l’intégrale curviligne de ω le long de l’arc γ1
nulle. peut être non nulle.
~ est à rotationnel nul alors ~
H
6. V Si V γ2 V = 0. 6. V Si ω est fermée alors γ2 ω = 0.
R

La courbe est contenue dans le carré [−1; 1]2 qui est sim- La courbe est contenue dans le carré [−1; 1]2 qui est sim-
plement connexe. Comme le champ est à rotationnel plement connexe. Comme la forme différentielle est fer-
nul, il est conservatif sur ce carré. De plus, la courbe est mée, elle est exacte sur ce carré. De plus, la courbe est
fermée donc la circulation sur γ2 est nulle. fermée donc l’intégrale curviligne sur γ2 est nulle.

© G. Faccanoni 245
8 Champs de vecteurs, formes différentielles Dernière mise à jour : Jeudi 5 juin 2014

Exercice 8.3 Champ de vecteurs


Considérons le champ de vecteurs
~ (x, y, z) = x ~
V j + 3z ~
i + 2y ~ k.
~ est conservatif et en calculer un potentiel.
Montrer que V

C ORRECTION . On a V1 (x, y, z) = x, V2 (x, y, z) = 2y et V3 (x, y, z) = 3z.


~ =~0 :
1. On vérifie que rotV 

∂ y (V3 ) − ∂z (V2 ) = 0 − 0 = 0,


∂z (V1 ) − ∂x (V3 ) = 0 − 0 = 0,


∂x (V2 ) − ∂ y (V1 ) = 0 − 0 = 0.

~ est défini sur R3 (simplement connexe) et est à rotationnel nul donc il est conservatif.
V
2. On cherche f : R3 → R telle que ~ ~.
∇f = V

x2
∂x f = x donc f (x, y, z) = 2 + g (y, z) donc ∂ y f = ∂ y g = 2y
2
donc 2
g (y, z) = y + h(z) donc f (x, y, z) = x2 + y 2 + h(z)
donc ∂z f = h 0 = 3z donc h(z) = 32 z 2
x2
donc f (x, y, z) = 2 + y 2 + 32 z 2 + c.

Exercice 8.4 Forme différentielle


On considère la forme différentielle
y x
ω(x, y) = − dx + dy,
x2 + y 2 x2 + y 2
définie sur le demi-plan U = (x, y) ∈ R2 ¯ x > 0 . Montrer que ω est exacte et déterminer ses primitives sur U .
© ¯ ª

C ORRECTION . D’une part, comme la forme est définie sur U qui est un ouvert étoilé (par exemple par rapport au point
(1, 0)), pour prouver qu’elle est exacte il suffit de montrer qu’elle est fermée :

y x y 2 − x2 y 2 − x2
µ ¶ µ ¶
∂ y (ω1 ) = ∂x (ω2 ) ⇐⇒ ∂y − = ∂ x ⇐⇒ = .
x2 + y 2 x2 + y 2 (x 2 + y 2 )2 (x 2 + y 2 )2

D’autre part, il est possible de calculer les primitives : on cherche en effet une fonction f de R2 dans R tel que
y
ω1 (x, y) = − x 2 +y 2 ,
(

x
ω2 (x, y) = x 2 +y 2
.

On résout ce système d’équations


³ ´ aux dérivées partielles : en intégrant la deuxième par rapport à y on obtient f (x, y) =
¡y¢ x y
arctan x + h(y) = − arctan y + h(y). En la dérivant par rapport à x on trouve ∂x f (x, y) = − x 2 +y 2 + h 0 (x) ; en utilisant la
première équation on trouve h 0 (x) = 0 et finalement
³y´
f (x, y) = arctan +C , C ∈ R.
x

Exercice 8.5 Forme différentielle


Montrer que la forme différentielle
2x y dx + (1 − x 2 ) dy
ω(x, y) =
(1 − x 2 )2 + y 2
est fermée. Dans un ouvert à préciser déterminer une fonction f telle que df = ω.

246 © G. Faccanoni
Dernière mise à jour : Jeudi 5 juin 2014 8 Champs de vecteurs, formes différentielles

2x y (1−x 2 )
C ORRECTION . La forme différentielle ω est définie sur Dω = R2 \ {(−1, 0); (1, 0)}. À partir de ω1 = (1−x 2 )2 +y 2
et ω2 = (1−x 2 )2 +y 2
,
on observe que
2x(1 − x 2 )2 − 2x y 2
∂x (ω2 ) = ¡ ¢2 ,
(1 − x 2 )2 + y 2 ]
2x(1 − x 2 )2 − 2x y 2
∂ y (ω1 ) = ¡ ¢2 .
(1 − x 2 )2 + y 2
Elle est donc fermée.
Elle est exacte sur tout ouvert simplement connexe (ce qui n’est pas le cas de son ensemble de définition).
Cherchons un potentiel f de ω sur un ouvert à préciser :

∂x f = ω1
∂ y f = ω2

Il est plus simple de partir de la seconde égalité et de se placer dans un ouvert où x 2 6= 1, par exemple A = {(x, y) : |x| < 1},
qui est simplement connexe. On peut alors écrire
1
1−x 2
∂y f = ³ ´2
y
1+ 1−x 2

donc
y ´ ³
f (x, y) = arctan + g (x).
1 − x2
En calculant ∂x f et en imposant qu’elle soit égale à ω1 on obtient
2x y
(1−x 2 )2 2x y
´2 + g 0 (x) =
(1 − x 2 )2 + y 2
³
y
1+ 1−x 2

d’où g 0 (x) = 0, soit ³ y ´


f (x, y) = arctan +K.
1 − x2

Exercice 8.6 Forme différentielle


Montrer que la forme différentielle

ω(x, y, z) = 2y z dx + 2z(x + 3y) dy + (y(2x + 3y) + 2z) dz

est exacte et en calculer un potentiel.

C ORRECTION . Comme la forme est définie sur R3 qui est un ouvert étoilé, pour prouver qu’elle est exacte il suffit de montrer
qu’elle est fermée :
∂ y (ω3 ) − ∂z (ω2 ) = (2x + 6y) − (2x + 6y) = 0,


∂z (ω1 ) − ∂x (ω3 ) = 2y − 2y = 0,

∂x (ω2 ) − ∂ y (ω1 ) = 2z − 2z = 0.

Néanmoins, il est possible de calculer directement les primitives : on cherche une application f de R3 dans R tel que

∂x f (x, y, z) = ω1 (x, y, z) = 2y z,




∂ y f (x, y, z) = ω2 (x, y, z) = 2xz + 6y z,

∂z f (x, y, z) = ω3 (x, y, z) = 2x y + 3y 2 + 2z.

On résout ce système d’équations aux dérivées partielles : en intégrant la première par rapport à x on obtient f (x, y, z) =
2x y z + h(y, z). En la dérivant par rapport à y on trouve ∂ y f (x, y, z) = 2xz + h y (y, z) ; comme ∂ y f (x, y, z) = ω2 (x, y) = 2xz + 6y z
alors h y (y, z) = 6y z, d’où h(y, z) = 3y 2 z + k(z) ce qui implique f (x, y, z) = 2x y z + 3y 2 z + k(z). En la dérivant par rapport à z
on trouve ∂z f (x, y, z) = 2x y + 3y 2 + k 0 (z) ; comme ∂z f (x, y, z) = ω3 (x, y, z) = 2x y + 3y 2 + 2z alors k 0 (z) = 2z, d’où k(z) = z 2 + c
et finalement
f (x, y, z) = 2x y z + 3y 2 z + z 2 +C , C ∈ R.

© G. Faccanoni 247
8 Champs de vecteurs, formes différentielles Dernière mise à jour : Jeudi 5 juin 2014

Exercice 8.7 Forme différentielle


Montrer que la forme différentielle
ω(x, y) = y dx + (2x − ye y ) dy
n’est pas exacte dans R2 . Trouver une fonction y 7→ g (y) telle que la forme différentielle ψ(x, y) ≡ g (y)ω(x, y) soit exacte
dans R2 . Déterminer ensuite les potentiels de ψ(x, y) dans R2 .

C ORRECTION . On a ω1 = y et ω2 = 2x − ye y . Comme ∂ y (ω1 ) = 1 et ∂x (ω2 ) = 2, la forme ω n’est pas fermée. Elle ne peut donc
pas être exacte.
On a ψ1 = g (y)ω1 et ψ2 = g (y)ω2 . Comme R2 est simplement connexe, d’après le théorème de Poincaré on a

ψ est exacte ⇐⇒ ψ est fermée.

Comme ∂ y (ψ1 ) = g 0 (y)y + g (y) et ∂x (ψ2 ) = 2g (y), on cherche g telle que y g 0 (y) = g (y) : ψ est fermée (et donc exacte) si et
seulement si g (y) = Ay avec A ∈ R.
Si ψ(x, y) = Ay 2 dx + A(2x y − y 2 e y ) dy, on sait qu’il existe un potentiel f de ψ, c’est-à-dire une fonction f telle que ∂x f = ψ1
et ∂ y f = ψ2 , soit
∂x f = Ay 2 ∂ y f = A(2x y − y 2 e y ).
Comme ∂x f = Ay 2 alors f (x, y) = Ax y 2 +h(y). En reportant dans la seconde équation on obtient 2Ax y+h 0 (y) = A(2x y−y 2 e y ),
c’est-à-dire h 0 (y) = −Ay 2 e y . Soit après deux intégrations par parties h(y) = −A(y 2 − 2y + 2)e y + C avec C ∈ R. Les potentiels
cherchés sont donc définis par
f (x, y) = A(x y 2 − (y 2 − 2y + 2)e y ) +C .

Exercice 8.8 Forme différentielle


Considérons la forme différentielle
y 2x y
µ ¶ µ ¶
2
ω(x, y) = + ln(1 + y ) dx + g (x) + dy
cos2 (x) 1 + y2

définie sur D = (x, y) ∈ − π2 ; π2 × R . Déterminer la fonction g de classe C ∞ (] − π/2; π/2[) telle que g (0) = 0 et ω soit
© ¤ £ ª

exacte. Déterminer ensuite pour la forme différentielle ainsi trouvée une primitive.

C ORRECTION . La forme différentielle ω est définie sur D (elle est même de classe C ∞ (D)). Comme le domaine de définition
est simplement connexe, la forme différentielle est exacte si et seulement si elle est fermée, c’est-à-dire

1 2y 2y 1
∂ y (ω1 ) = ∂x (ω2 ) ⇐⇒ + = g 0 (x) + ⇐⇒ = g 0 (x) ⇐⇒ g (x) = tan(x) +C , C ∈ R.
cos2 (x) 1 + y 2 1 + y2 cos2 (x)

Pour avoir g (0) = 0 il faut C = 0, ainsi g (x) = tan(x) et la forme différentielle s’écrit

y 2x y
µ ¶ µ ¶
2
ω(x, y) = + ln(1 + y ) dx + tan(x) + dy.
cos2 (x) 1 + y2

Pour trouver une primitive, c’est à dire une fonction f : D → R telle que df = ω, on intègre ω1 par rapport à x et on obtient
f (x, y) = y tan(x) + x ln(1 + y 2 ) + h(y). Pour déterminer h on dérive l’expression ainsi obtenue par rapport à y : ∂ y f = tan(x) +
2y
1+y 2
+ h 0 (y) ; cette fonction doit être égale à ω2 donc h 0 (y) = 0 et finalement

f (x, y) = y tan(x) + x ln(1 + y 2 ) +C , C ∈ R.

Exercice 8.9 Champ de vecteurs


Considérons le champ de vecteurs

y
µ ¶
~ (x, y) = + e i + g (x) + xe y ~
y ~
¡ ¢
V j.
cos2 (x)

248 © G. Faccanoni
Dernière mise à jour : Jeudi 5 juin 2014 8 Champs de vecteurs, formes différentielles

~ soit
défini sur D = (x, y) ∈ − π2 ; π2 × R . Déterminer la fonction g de classe C ∞ (] − π/2; π/2[) telle que g (0) = 0 et V
© ¤ £ ª

conservatif. Déterminer ensuite pour le champ ainsi trouvé un potentiel.

C ORRECTION . Le champ V ~ est défini sur D (il est même de classe C ∞ (D)). Comme le domaine de définition est simplement
connexe, le champ est conservatif si et seulement si il est à rotationnel nul, c’est-à-dire

1 1
∂ y (V1 ) = ∂x (V2 ) ⇐⇒ + e y = g 0 (x) + e y ⇐⇒ = g 0 (x) ⇐⇒ g (x) = tan(x) +C , C ∈ R.
cos2 (x) cos2 (x)

Comme on veut que g (0) = 0, alors g (x) = tan(x) et le champ de vecteurs s’écrit

y
µ ¶
~ + e i + tan(x) + xe y ~
y ~
¡ ¢
V (x, y) = j.
cos2 (x)

Pour trouver un potentiel, c’est à dire une fonction f : D → R telle que ~ ~ , on intègre V1 par rapport à x et on obtient
∇f = V
f (x, y) = y tan(x) + xe + h(y). Pour déterminer h on dérive l’expression ainsi obtenue par rapport à y : ∂ y f = tan(x) + xe y +
y

h 0 (y) ; cette fonction doit être égale à V2 donc h 0 (y) = 0 et finalement

f (x, y) = y tan(x) + xe y +C , C ∈ R.

Exercice 8.10 Champ de vecteurs


Considérons le champ de vecteurs

~ (x, y) = (y sin(x) + x y cos(x) + e y )~


V i + (g (x) + xe y ) ~
j.

~ soit conservatif. Déterminer ensuite pour le champ ainsi


Déterminer la fonction g de classe C ∞ (R) telle que g (0) = 0 et V
trouvé un potentiel.

C ORRECTION . Le champ V~ est défini sur R2 (il est même de classe C ∞ (R2 )). Comme le domaine de définition est simple-
ment connexe, le champ est conservatif si et seulement si il est à rotationnel nul, c’est-à-dire

∂ y (V1 ) = ∂x (V2 ) ⇐⇒ sin(x) + x cos(x) + e y = g 0 (x) + e y ⇐⇒ sin(x) + x cos(x) = g 0 (x) ⇐⇒ g (x) = x sin(x) +C , C ∈ R.

Comme on veut que g (0) = 0, alors g (x) = x sin(x) et le champ de vecteurs s’écrit

~ (x, y) = (y sin(x) + x y cos(x) + e y )~


V i + (x sin(x) + xe y ) ~
j.

Pour trouver un potentiel, c’est à dire une fonction f : R2 → R telle que ~ ~ , on intègre V1 par rapport à x et on obtient
∇f = V
f (x, y) = x y sin(x) + xe + h(y). Pour déterminer h on dérive l’expression ainsi obtenue : ∂ y f = x sin(x) + xe y + h 0 (y) ; cette
y

fonction doit être égale à V2 donc h 0 (y) = 0 et finalement

f (x, y) = x y sin(x) + xe y +C , C ∈ R.

Exercice 8.11 Champ de vecteurs


Considérons le champ de vecteurs

g (y)
µ ¶
~ (x, y) =
V + cos(y) ~i + (2y ln x − x sin(y)) ~
j.
x

~ soit conservatif. Déterminer ensuite pour le


Déterminer la fonction g de classe C ∞ (R) telle que g (1) = 1 de sorte que V
champ ainsi trouvé le potentiel φ(x, y) tel que φ(1, π/2) = 0.

© G. Faccanoni 249
8 Champs de vecteurs, formes différentielles Dernière mise à jour : Jeudi 5 juin 2014

C ORRECTION . Le champ V ~ est défini sur R∗+ × R. Comme le domaine de définition est simplement connexe, le champ est
conservatif si et seulement si il est à rotationnel nul, c’est-à-dire

g 0 (y) y
∂ y (V1 ) = ∂x (V2 ) ⇐⇒ − sin(y) = 2 − sin(y) ⇐⇒ g 0 (y) = 2y ⇐⇒ g (x) = y 2 +C , C ∈ R.
x x

Comme on veut que g (1) = 1, alors g (y) = y 2 et le champ de vecteurs s’écrit

~ (x, y) = (y 2 /x + cos(y))~
V i + (2y ln x − x sin(y)) ~
j.

Pour trouver un potentiel, c’est à dire une fonction φ : R2 → R telle que ~ ~ , on intègre V1 par rapport à x et on obtient
∇φ = V
2
φ(x, y) = y ln x +x cos(y)+h(y). Pour déterminer h on dérive l’expression ainsi obtenue : ∂ y φ = 2y ln x −x sin(y)+h 0 (y) ; cette
fonction doit être égale à V2 donc h 0 (y) = 0 et finalement

φ(x, y) = y 2 ln x + x cos(y) +C , C ∈ R.

Parmi ces potentiels, celui qui vérifie φ(1, π/2) = 0 est

φ(x, y) = y 2 ln x + x cos(y).

Exercice 8.12 Champ de vecteurs


Considérons le champ de vecteurs

~ (x, y, z) = (2x y sin z)~


V j + (x 2 y cos z)~
i + (x 2 sin z) ~ k.

1. Montrer que V~ est conservatif.


~.
2. Calculer un potentiel de V

C ORRECTION . On a V1 (x, y, z) = 2x y sin z, V2 (x, y, z) = x 2 sin z et V3 (x, y, z) = x 2 y cos z.


~ =~0 :
1. On vérifie que rotV 
2 2
∂ y (V3 ) − ∂z (V2 ) = x cos z − x cos z = 0,



∂z (V1 ) − ∂x (V3 ) = 2x y cos z − 2x y cos z = 0,


∂x (V2 ) − ∂ y (V1 ) = 2x sin z − 2x sin z = 0.

~ est défini sur R3 (simplement connexe) et est à rotationnel nul donc il est conservatif.
V
2. On cherche f : R3 → R telle que ~ ~.
∇f = V

∂x f = 2x y sin z donc f (x, y, z) = x 2 y sin z + g (y, z) donc ∂ y f = x 2 sin z + ∂ y g = x 2 sin z


donc g (y, z) = h(z) donc f (x, y, z) = x 2 y sin z + h(z)
donc ∂z f = x 2 y cos z + h 0 = x 2 y cos z donc h(z) = c
donc f (x, y, z) = x 2 y sin z + c.

Exercice 8.13 Champ de vecteurs


Considérons le champ de vecteurs

~ (x, y, z) = (y 2 sin z)~


V j + (x y 2 cos z)~
i + (2x y sin z) ~ k.

1. Montrer que V~ est conservatif


~.
2. Calculer un potentiel de V

C ORRECTION . On a V1 (x, y, z) = y 2 sin z, V2 (x, y, z) = 2x y sin z et V3 (x, y, z) = x y 2 cos z.


~ =~0 :
1. On vérifie que rotV 

∂ y (V3 ) − ∂z (V2 ) = 2x y cos z − 2x y cos z = 0,


∂z (V1 ) − ∂x (V3 ) = y 2 cos z − y 2 cos z = 0,


∂x (V2 ) − ∂ y (V1 ) = 2y sin z − 2y sin z = 0.

~ est défini sur R3 (simplement connexe) et est à rotationnel nul donc il est conservatif.
V

250 © G. Faccanoni
Dernière mise à jour : Jeudi 5 juin 2014 8 Champs de vecteurs, formes différentielles

2. On cherche f : R3 → R telle que ~ ~.


∇f = V

∂x f = y 2 sin z donc f (x, y, z) = x y 2 sin z + g (y, z) donc ∂ y f = 2x y sin z + ∂ y g = 2x y sin z


donc g (y, z) = h(z) donc f (x, y, z) = x y 2 sin z + h(z)
donc ∂z f = x y 2 cos z + h 0 = x y 2 cos z donc h(z) = c
donc f (x, y, z) = x y 2 sin z + c.

Exercice 8.14 Champ de vecteurs


Considérons le champ de vecteurs

~ (x, y, z) = (2xz sin(y))~


V j + (x 2 sin(y))~
i + (x 2 z cos(y)) ~ k.

1. Montrer que V~ est conservatif.


~.
2. Calculer un potentiel de V

C ORRECTION . On a V1 (x, y, z) = 2xz sin(y), V2 (x, y, z) = x 2 z cos(y) et V3 (x, y, z) = x 2 sin(y).


~ =~0 :
1. On vérifie que rotV 

 ∂ y (V3 ) − ∂z (V2 ) = x 2 cos(y) − x 2 cos(y) = 0,


∂z (V1 ) − ∂x (V3 ) = 2x sin(y) − 2x sin(y) = 0,


∂x (V2 ) − ∂ y (V1 ) = 2xz cos(y) − 2xz cos(y) = 0.

~ est défini sur R3 (simplement connexe) et est à rotationnel nul donc il est conservatif.
V
2. On cherche f : R3 → R telle que ~ ~.
∇f = V

∂x f = 2xz sin(y) donc f (x, y, z) = x 2 z sin(y) + g (y, z) donc ∂ y f = x 2 z cos(y) + ∂ y g = x 2 z cos(y)


donc g (y, z) = h(z) donc f (x, y, z) = x 2 z sin(y) + h(z)
donc ∂z f = x 2 sin(y) + h 0 = x 2 sin(y) donc h(z) = c
donc f (x, y, z) = x 2 z sin(y) + c.

Exercice 8.15 Champ de vecteurs


Considérons le champ de vecteurs

~ (x, y, z) = (z 2 sin(y))~
V j + (2xz sin(y))~
i + (xz 2 cos(y)) ~ k.

1. Montrer que V~ est conservatif.


~.
2. Calculer un potentiel de V

C ORRECTION . On a V1 (x, y, z) = z 2 sin(y), V2 (x, y, z) = xz 2 cos(y) et V3 (x, y, z) = 2xz sin(y).


~ =~0 :
1. On vérifie que rotV 

 ∂ y (V3 ) − ∂z (V2 ) = 2xz sin(y) − 2xz sin(y) = 0,


∂z (V1 ) − ∂x (V3 ) = 2z sin(y) − 2z sin(y) = 0,


∂x (V2 ) − ∂ y (V1 ) = z 2 cos(y) − z 2 cos(y) = 0.

~ est défini sur R3 (simplement connexe) et est à rotationnel nul donc il est conservatif.
V
2. On cherche f : R3 → R telle que ~ ~.
∇f = V

∂x f = z 2 sin(y) donc f (x, y, z) = xz 2 sin(y) + g (y, z) donc ∂ y f = xz 2 cos(y) + ∂ y g = xz 2 cos(y)


donc g (y, z) = h(z) donc f (x, y, z) = xz 2 sin(y) + h(z)
donc ∂z f = 2xz sin(y) + h 0 = 2xz sin(y) donc h(z) = c
donc f (x, y, z) = xz 2 sin(y) + c.

© G. Faccanoni 251
8 Champs de vecteurs, formes différentielles Dernière mise à jour : Jeudi 5 juin 2014

Exercice 8.16 Champ de vecteurs


Considérons le champ de vecteurs

~ (x, y, z) = (8z 4 cos(2x) cos(y))~


V j + (16z 3 sin(2x) cos(y))~
i + (−4z 4 sin(2x) sin(y)) ~ k.

1. Montrer que V~ est conservatif.


~.
2. Calculer un potentiel de V

C ORRECTION . On a V1 (x, y, z) = 8z 4 cos(2x) cos(y), V2 (x, y, z) = −4z 4 sin(2x) sin(y) et V3 (x, y, z) = 16z 3 sin(2x) cos(y).
~ =~0 :
1. On vérifie que rotV


 ∂ y (V3 ) − ∂z (V2 ) = −16z 3 sin(2x) sin(y) − 16z 3 sin(2x) sin(y) = 0,


∂z (V1 ) − ∂x (V3 ) = 32z 3 cos(2x) cos(y) − 32z 3 cos(2x) cos(y) = 0,


∂x (V2 ) − ∂ y (V1 ) = −8z 4 cos(2x) sin(y) + 8z 4 cos(2x) sin(y) = 0.

~ est défini sur R3 (simplement connexe) et est à rotationnel nul donc il est conservatif.
V
2. On cherche f : R3 → R telle que ~ ~.
∇f = V

∂x f = 8z 4 cos(2x) cos(y) donc f (x, y, z) = 4z 4 sin(2x) cos(y) + g (y, z)


donc ∂ y f = −4z 4 sin(2x) sin(y) + ∂ y g = −4z 4 sin(2x) sin(y)
donc g (y, z) = h(z)
donc f (x, y, z) = 4z 4 sin(2x) cos(y) + h(z)
donc ∂z f = 16z 3 sin(2x) cos(y) + h 0 = 16z 3 sin(2x) cos(y)
donc h(z) = c
donc f (x, y, z) = 4z 4 sin(2x) cos(y) + c.

Exercice 8.17 Forme différentielle


Trouver toutes les fonctions f : R → R de classe C 1 qui satisfont f (0) = 2 et telles que la forme différentielle

ω(x, y) = (y 2 + 1) sin(x) dx − f (x)y dy

soit exacte. Pour la forme différentielle ainsi trouvée, calculer toutes ses primitives.

C ORRECTION . Le domaine de définition de ω est R2 qui est simplement connexe : pour que la forme différentielle proposée
soit exacte il faut et il suffit que pour tout x ∈ R

∂ y ((y 2 + 1) sin(x)) = ∂x (− f (x)y),

ce qui implique f 0 (x) = −2 sin(x) ; toutes les fonctions f : R → R de classe C 1 qui rendent la forme différentielle ω exacte sont
de la forme f (x) = 2 cos(x) + c, c ∈ R. On obtient f (0) = 2 pour c = 0 et la forme différentielle devient

ω(x, y) = (y 2 + 1) sin(x) dx − 2 cos(x)y dy.

Pour trouver une primitive, c’est à dire une fonction ϕ : R2 → R telle que dϕ = ω, on intègre ω1 par rapport à x et on obtient
ϕ(x, y) = −(y 2 + 1) cos(x) + h(y). Pour déterminer h on dérive l’expression ainsi obtenue : ∂ y ϕ = −2y cos(x) + h 0 (y) ; cette
fonction doit être égale à ω2 donc h 0 (y) = 0 et finalement

ϕ(x, y) = −(y 2 + 1) cos(x) +C , C ∈ R.

Exercice 8.18 Champs de vecteurs


~ le champ de vecteurs défini comme suit :
Soit V

z ~ z ~
µ ¶ µ ¶ µ ¶
1 ~
~ (x, y, z) = x +
V i + y + j + z − k.
x2 y x y2 xy

252 © G. Faccanoni
Dernière mise à jour : Jeudi 5 juin 2014 8 Champs de vecteurs, formes différentielles

~ est conservatif.
1. Montrer que V
2. En déterminer un potentiel.
~ le long de la courbe γ de paramétrisation t 7→ (t , t 2 , t 3 ), t ∈ [1; 3].
3. Calculer la circulation de V

C ORRECTION .
~ est défini sur D = (x, y, z) ∈ R3 ¯ x y 6= 0 . Comme le domaine de définition est la réunion de
© ¯ ª
1. Le champ de vecteurs V
quatre parties simplement connexes, le champ est conservatif si, et seulement si, il est à rotationnel nul, ce qui est le
cas car  1
= 1 ,
∂ y (V3 ) = ∂z (V2 ),

 x y2 x y2
 

∂z (V1 ) = ∂x (V3 ), ⇐⇒ x 12 y = x 12 y , ⇐⇒ (x, y, z) ∈ D.
 
∂ (V ) = ∂ (V ), z z
 
x 2 y

1− =− ,
x2 y 2 x2 y 2

~.
2. On cherche un potentiel f : D → R tel que ∇ f = V
x2
(i) En intégrant V1 par rapport à x, on obtient f (x, y, z) = 2 − xzy + h(y, z).
z
(ii) Pour déterminer h, on dérive l’expression ainsi obtenue par rapport à y : ∂ y f = x y2
+∂ y h(y, z) ; cette fonction doit
y2 x2 y2
être égale à V2 . On a donc ∂ y h(y, z) = y, ce qui implique h(y, z) = 2 + g (z). Finalement f (x, y, z) = 2 − xzy + 2 +
g (z).
(iii) Pour déterminer g , on dérive l’expression ainsi obtenue par rapport à z : ∂z f = − x1y + g 0 (z) ; cette fonction devant
z2
être égale à V3 , on déduit g 0 (z) = z ce qui donne g (z) = 2 +C .
~ est de la forme
On conclut alors que tout potentiel de V

x2 z y 2 z2
f (x, y, z) = − + + +C , C ∈ R.
2 xy 2 2

3. On remarque que la courbe γ a pour origine le point A = (1, 1, 1) et pour extrémité le point B = (3, 9, 27). Par conséquent

32 92 272
¶ µ 2
12 12
µ ¶
27 1 1
I
~ = f (B ) − f (A) =
V − + + − − + + = 408.
γ 2 3×9 2 2 2 1×1 2 2

Exercice 8.19
Soit ω(x, y) = 3(x 2 − y) dx + 3(y 2 − x) dy une forme différentielle et notons Dω son domaine de définition. Montrer qu’elle
fermée. Expliquer pourquoi on peut en déduire qu’elle est exacte et en Rcalculer une primitive. Soit γ : [0, 2π] → Dω la
courbe définie par γ(t ) = (2 cos(t ), 3 sin(t )). Calculer l’intégrale curviligne γ ω.

C ORRECTION . ∂ y 3(x 2 − y) − ∂x 3(y 2 − x) = 0 : la forme différentielle est fermée.


¡ ¢ ¡ ¢

Dω ≡ R2 est simplement connexe et ω est fermée : le théorème de P OINCARÉ permet de conclure que ω est exacte.
On cherche f : R2 → R telle que df (x, y) = ω(x, y) pour tout (x, y) ∈ R2 . Comme ∂x f (x, y) = 3(x 2 − y), alors f (x, y) = x 3 −
3x y + g (y) où g : R → R est une fonction à déterminer ; alors ∂ y f (x, y) = −3x + g 0 (y) et on sait que ∂ y f (x, y) = 3(y 2 − x) donc
g 0 (y) = 3y 2 , par conséquent g (y) = y 3 + c et on conclut que f (x, y) = x 3 + y 3 − 3x y + c pour tout c ∈ R.
Comme ω est exacte et γ(0) = γ(2π), alors γ ω = 0.
R

Exercice 8.20
Soit ω(x, y) = (ye x y − 1) dx + xe x y dy une forme différentielle et notons Dω son domaine de définition. Montrer qu’elle
est fermée. Expliquer pourquoi on peut en déduire qu’elle est exacte et en calculer une primitive. R Soit γ : [0, 8π] → Dω la
courbe définie par γ(t ) = (t + arcsin(sin(t )), (2/π) arcsin(sin(t ))). Calculer l’intégrale curviligne γ ω.

© G. Faccanoni 253
8 Champs de vecteurs, formes différentielles Dernière mise à jour : Jeudi 5 juin 2014

C ORRECTION .
∂ y (ω1 (x, y)) − ∂x (ω2 (x, y)) = ∂ y ye x y − 1 − ∂x (xe x y ) = 0 : la forme différentielle est fermée.
¡ ¢

Dω ≡ R2 est simplement connexe et ω est fermée : le théorème de P OINCARÉ permet de conclure que ω est exacte.
On cherche f : R2 → R telle que df (x, y) = ω(x, y) pour tout (x, y) ∈ R2 . Comme ∂ y f (x, y) = xe x y , alors f (x, y) = e x y + g (x) où
g : R → R est une fonction à déterminer ; alors ∂x f (x, y) = ye x y + g 0 (x) et on sait que ∂x f (x, y) = (xe x y − 1) donc g 0 (x) = −1,
par conséquent g (x) = −x + c et on conclut que f (x, y) = e x y − x + c, c ∈ R.
Comme ω est exacte et γ(0) = (0, 0), γ(8π) = (8π, 0), alors γ ω = f (8π, 0) − f (0, 0) = 8π.
R

Exercice 8.21 Champ de vecteurs


1. Établir si le champ de vecteurs suivant est conservatif sur R2 \ {(0, 0)} :
y
x 3 + x y 2 + y ~ 3x 2 y + 3y 3 − x ~ 1
~=
V i+ j.
x2 + y 2 x2 + y 2 1
0<r < 2

2. Soit γr la famille de courbes représentée ci-contre et définie par −1 0 1 2 3x


1 −1 r> 1
2
γr (t ) = (1 + 2r cos(t ), r sin(t )), t ∈ [0, 2π], r > 0, r 6= .
2
I
Calculer ~ (t ) dt en fonction de r .
V
γr

C ORRECTION . Remarquons que le champ de vecteurs peut être mis sous la forme

y x
· ¸ · ¸
~
V = x+ 2 ~i + 3y − 2 ~
j.
x + y2 x + y2

1. Commençons d’abord par vérifier si les deux conditions pour la conservativité sont vérifiées.
B CN1 : rotationnel su champ de vecteurs

x2 − y 2
∂ y (V1 ) = = ∂x (V2 ).
(x 2 + y 2 )2

Le champ de vecteurs V ~ est bien à rotationnel nul.


~
B CN2 : circulation de V sur une courbe fermée quelconque. Considérons la courbe fermée `(t ) = (cos(t ), sin(t )) pour
t ∈ [0, 2π], alors
I Z 2π Z 2π ¤2π
~ (t )d t = 2 cos(t ) sin(t )−1 dt = sin2 (t ) − t 0 = −2π 6= 0.
£
V (cos(t ) + sin(t )) (− sin(t ))−(3 sin(t ) − cos(t )) (cos(t )) =
` 0 0

La circulation sur au moins une courbe fermée qui tourne autour du point (0, 0) est non nulle.
~ est à rotationnel nul (et donc conservatif sur tous sous ensemble simplement connexe de R2 \{(0, 0)})
On conclut que V
mais non conservatif sur R2 \ {(0, 0)}.
2. Comme γr (π) = (1 − 2r, 0), on en déduit que
I
B si r > 1/2 alors ~ (t )d t = −2π (car la circulation le long de courbes homotopes est la même et γr est homotope
V
γr
à la courbe γ du point
I précédent si r > 1/2),
B si 0 < r < 1/2 alors ~ (t )d t = 0 car γr est contenue dans un sous-ensemble simplement connexe de R2 \ {(0, 0)}
V
γr
~ est à rotationnel nul donc conservatif dans ce sous-ensemble.
et V

Exercice 8.22 Forme différentielle


Considérons la forme différentielle suivante
ω(x, y) = (2x y + y 2 − 1) dx + (2x y + x 2 ) dy.
1. Exprimer l’intégrale curviligne de ω le long du segment de droite reliant les points A = (1, 0) et B = (0, 1) (orienté de
A vers B ), puis calculer cette intégrale.

254 © G. Faccanoni
Dernière mise à jour : Jeudi 5 juin 2014 8 Champs de vecteurs, formes différentielles

2. Déterminer si ω est exacte. Si c’est le cas, déterminer une fonction f telle que df = ω.
3. Calculer l’intégrale curviligne de ω le long de la courbe γ de paramétrisation t 7→ (cos5 (t ), sin4 (t )), t ∈ [0; π/2].

C ORRECTION .
1. Un paramétrage possible du segment est l’application γ2 : [0; 1] → R2 définie par γ2 (t ) = (1 − t , t ). Donc
Z Z 1 Z 1
¤1
ω= (2(1 − t )t + t 2 − 1)(1 − t )0 + (2(1 − t )t + (1 − t )2 )t 0 dt = (2 − 2t ) dt = 2t − t 2 0 = 1.
¡ ¢ £
γ2 0 0

2. La forme différentielle ω est définie sur R2 . Comme le domaine de définition est simplement connexe, la forme diffé-
rentielle est exacte si et seulement si elle est fermée, c’est-à-dire

∂ y (ω1 ) = ∂x (ω2 ) ⇐⇒ ∂ y (2x y + y 2 − 1) = ∂x (2x y + x 2 ) ⇐⇒ 2x + 2y = 2y + 2x ⇐⇒ (x, y) ∈ R2 .

On cherche une primitive f : D → R telle que df = ω. On intègre ω1 par rapport à x et on obtient f (x, y) = x 2 y + x y 2 −
x +h(y). Pour déterminer h on dérive l’expression ainsi obtenue par rapport à y : ∂ y f = x 2 +2x y +h 0 (y) ; cette fonction
doit être égale à ω2 donc h 0 (y) = 0 et finalement

f (x, y) = x 2 y + x y 2 − x +C , C ∈ R.

3. On remarque que la courbe γ a pour origine et extrémités respectivement les points A et B . Par conséquent
Z Z
ω= ω = f (B ) − f (A) = 1.
γ γ2

Exercice 8.23 Forme différentielle


Considérons la forme différentielle
x x2
ω(x, y) = 2 dx − 2 dy
y y
définie sur U = (x, y) ∈ R2 ¯ y > 0 .
© ¯ ª

1. Montrer que ω est fermée sur U .


2. Montrer de deux façons différentes que ω est exacte. I
3. Soit γ une courbe de classe C 1 (U ) par morceaux d’origine A = (1, 2) et d’extrémité B = (3, 8). Calculer ω.
γ

C ORRECTION .
2
1. En posant ω1 = 2 xy et ω2 = − xy 2 , on a

x x
∂ y (ω1 ) = ∂x (ω2 ) ⇐⇒ −2 = −2 2 .
y2 y

2. La forme différentielle est fermée sur U étoilé, d’après le théorème de Poincaré elle est donc exacte. On peut aussi
prouver qu’elle est exacte en calculant ses primitives, c’est-à-dire en recherchant f telle que ω = df . On doit alors
résoudre le système d’équations aux dérivées partielles

∂x f (x, y) = 2 xy ,
(
2
∂ y f (x, y) = − xy 2 .

x2 2
La première équation donne f (x, y) = y + h(y) ; en la dérivant par rapport à y on a f (x, y) = − xy 2 + h 0 (y) ; en la intro-
duisant dans la deuxième équation on trouve h 0 (y) = 0 et finalement on obtient une primitive de ω :

x2
f (x, y) = .
y

3. Comme la forme différentielle est exacte, le calcul ne dépend pas du chemin choisi, mais uniquement des extrémités.
On trouve ainsi
9 1 5
I
ω = f (3, 8) − f (1, 2) = − = .
γ 8 2 8

© G. Faccanoni 255
8 Champs de vecteurs, formes différentielles Dernière mise à jour : Jeudi 5 juin 2014

Exercice 8.24 Champ de vecteurs


Montrer que le champ de vecteurs
y ~ x ~
~ =−
V i+ 2 j
x2 + y 2 x + y2
est non conservatif bien qu’il soit à rotationnel nul.

C ORRECTION . Le champ de vecteurs est à rotationnel nul car :


y 2 − x2
∂x (V2 ) = = ∂ y (V1 ).
(x 2 + y 2 )2
Comme il est définie sur R2 \ { (0, 0) } qui n’est
H pas simplement connexe, il peut être non conservatif. Pour le prouver on va
exhiber une courbe fermée γ pour laquelle γ V ~ 6= 0. Comme le champ est à rotationnel nul, si on prend une courbe fermée
γ contenue dans un ensemble simplement connexe, on aura V ~ = 0. On doit alors prendre γ qui encercle le point (0, 0), par
H
γ
exemple la courbe γ(t ) = (cos(t ), sin(t )) pour t ∈ [0, 2π]. Dans ce cas on a
Z 2π µ ¶ Z 2π
sin(t ) cos(t )
I
~=
V − (cos(t )) 0
+ (sin(t )) 0
dt = 1 dt = 2π 6= 0.
γ 0 cos2 (t ) + sin2 (t ) cos2 (t ) + sin2 (t ) 0

Exercice 8.25 Forme différentielle


Soit ω la forme différentielle
ω = (y 3 − 6x y 2 ) dx + (3x y 2 − 6x 2 y) dy.
1. Montrer que ω est une forme exacte sur R2 .
2. En déduire l’intégrale curviligne le long du demi-cercle supérieur de diamètre [AB ] de A = (1, 2) vers B = (3, 4).

C ORRECTION .
1. En posant ω1 = y 3 − 6x y 2 et ω2 = 3x y 2 − 6x 2 y, on vérifie qu’elle est fermée car
∂x (ω2 ) − ∂ y (ω1 ) = 3y 2 − 12x y − 3y 2 + 16x y = 0.
Comme R2 est étoilé, elle est exacte.
2. Pour calculer l’intégrale curviligne on cherche d’abord une primitive f de ω : on cherche f telle que ∂x f = ω1 et ∂ y f =
ω2 , c’est-à-dire (
∂x f (x, y) = y 3 − 6x y 2 ,
∂ y f (x, y) = 3x y 2 − 6x 2 y.
En intégrant la première par rapport à x on trouve f (x, y) = x y 3 −3(x y)2 +g (y) ; en la dérivant par rapport à y on obtient
∂ y f (x, y) = 3x y 2 − 6x 2 y + g 0 (y) qui est égale à 3x y 2 − 6x 2 y d’où g 0 (y) = 0. Une primitive de ω est donc

f (x, y) = x y 3 − 3(x y)2 .


L’intégrale curviligne le long du demi-cercle supérieur de diamètre [AB ] de A = (1, 2) vers B = (3, 4) est donc
Z
ω = f (3, 4) − f (1, 2) = −236.
[AB ]

y
3
Exercice 8.26 Forme différentielle
Considérons la forme différentielle 2
y −x
ω(x, y) = 2 dx + 2 dy.
x + y2 x + y2 1
0<r <1
1. Montrer que ω est fermée.
0
2. Montrer que ω n’est pas exacte. 1 2 3x
3. Soit γr la famille de courbes
−1 r >1

γr (t ) = (1 + r cos(φ), 2r sin(φ)), φ ∈ [0, 2π], r > 0, r 6= 1.


I −2
Calculer ~ (t )d t en fonction de r .
V
γr −3

256 © G. Faccanoni
Dernière mise à jour : Jeudi 5 juin 2014 8 Champs de vecteurs, formes différentielles

y x
C ORRECTION . Posons ω1 = x 2 +y 2
et ω2 = − x 2 +y 2.

1. La forme différentielle est fermée car

x2 − y 2
∂ y (ω1 ) = = ∂x (ω2 ).
(x 2 + y 2 )2

2. Puisque ω est définie sur R2 \{(0, 0)}, la forme peut ne pas être exacte. On a deux possibilité pour démontrer qu’elle n’est
pas exacte : soit on prouve qu’elle n’admet pas de potentiel (difficile), soit qu’il existe une courbe fermée le longue de
laquelle la circulation de ω est non nulle. Puisque ω est fermée, l’unique possibilité pour trouver une courbe fermée le
longue de laquelle la circulation de ω est non nulle est de considérer une courbe qui tourne autour du point (0, 0) (car
si elle ne tourne pas autour de ce point, alors il existe toujours un sous-ensemble simplement connexe du domaine de
définition de ω qui contient la courbe et dans ce sous-ensemble la forme est exacte donc la circulation sera nulle). Soit
γ le cercle de centre l’origine et rayon unitaire

γ(θ) = (cos θ, sin θ), γ0 (θ) = (− sin θ, cos θ), θ ∈ [0, 2π]

et calculons la circulation de ω sur γ :


I Z 2π sin(θ) − cos(θ)
Z 2π
ω= (− sin(θ)) + (cos(θ)) dθ = −1 dθ = −2π 6= 0.
γ 0 cos2 (θ) + sin2 (θ) cos2 (θ) + sin2 (θ) 0

3. On en déduit que I
B si r > 1 alors ~ (t )d t = −2π (car la circulation le longue de courbes homotopes est la même et γr est homotope
V
γr
à la courbe γ du Ipoint précédent si r > 1),
B si 0 < r < 1 alors ~ (t )d t = 0 car γr est contenue dans un sous-ensemble simplement connexe de R2 \ {(0, 0)} et
V
γr
ω est fermée donc exacte dans ce sous-ensemble.

Exercice 8.27 Intégrale curviligne


Calculer le travail du champ de vecteurs V ~ = (y 2 , 2x y) pour déplacer un point matériel de (0, 0) à (1, 1) le long de γ1
d’équation y = x et le long de γ2 d’équation y = x 2 .

C ORRECTION . On peut bien évidemment calculer directement le travail le long des deux chemins. Néanmoins il est évident
que la fonction f : R2 → R définie par f (x, y) = x y 2 est un potentiel de V ~ , autrement dit V
~ est conservatif, donc le travail ne
dépend que du point de départ et du point d’arrivé :
I I
~=
V ~ = f (1, 1) − f (0, 0) = 1.
V
γ1 γ2

Exercice 8.28 Champ de vecteurs


1. Établir si le champ de vecteurs suivant est conservatif sur R2 :

~ (x, y) = (x − y)~
V i − x~
j.

2. Soit γ1 la courbe d’équation


γ1 (t ) = (t 2 − t + 1, t 2 + 2t ), t ∈ [0, 1]
et γ2 le segment
I de droite allantIdu point A ≡ (1, 0) au point B ≡ (1, 3).
Calculer ~ (t ) dt . En déduire
V ~ (t ) dt .
V
γ2 γ1

© G. Faccanoni 257
8 Champs de vecteurs, formes différentielles Dernière mise à jour : Jeudi 5 juin 2014

C ORRECTION .
1. Deux méthodes :
~ est conservatif car les fonctions f : R2 → R définies par f (x, y) = x − x y + c en
2
Méthode I : Le champ de vecteurs V 2
~.
sont des potentiels, i.e. elles sont telles que ∇ f = V
Méthode II : Le champ de vecteurs est à rotationnel nul et V ~ est défini sur un domaine simplement connexe donc,
par le théorème de P OINCARÉ, le champ est conservatif.
2. Deux méthodes :
Méthode I : Comme le champ est conservatif, alors
µ ¶ µ ¶
1 1
I
~ (t ) dt = f (B ) − f (A) =
V −3+c − + c = −3.
γ2 2 2

La courbe γ1 va elle aussi du point A au point B car γ1 (0) = (1, 0) et γ1 (1) = (1, 3). Comme le champ est
conservatif alors I
~ (t ) dt = −3.
V
γ1
I
Méthode II : Sans calculer le potentiel, on peut calculer ~ (t ) dt directement :
V
γ1

I Z 1¡
~ (t ) dt = ((t 2 − t + 1) − (t 2 + 2t ))(t 2 − t + 1)0 + (−(t 2 − t + 1))(t 2 + 2t )0 dt
¢
V
γ1 0
Z 1¡
(−3t + 1)(2t − 1) + (−(t 2 − t + 1)(2t + 2) dt
¢
=
0
Z 1¡
−2t 3 − 6t 2 + 5t − 3 dt
¢
=
0
¸1
t4 t3 t2
·
= −2 − 6 + 5 − 3t = −3.
4 3 2 0

De la même manière, un paramétrage possible du segment est l’application γ2 : [0; 3] → R2 définie par
γ2 (t ) = (1, t ) donc
I Z 3¡ Z 3
~ (t ) dt = (1 − t )(1)0 + (−1)(t )0 dt = −1 dt = [−t ]30 = −3.
¢
V
γ2 0 0

Exercice 8.29
~ (x, y) = e y~
Soit V i +xe y ~
j un champ de vecteurs. Montrer qu’il est conservatif et calculer le travail nécessaire pour déplacer
une particule du point (−1, 0) au point (1, 0) le long du demi-cercle de rayon 1 et centre (0, 0) dans le plan y ≥ 0.

C ORRECTION . Le champ est défini sur R2 qui est simplement connexe et il est à rotationnel nul car ∂ y (e y )−∂x (xe y ) = 0 donc,
~ (x, y) = ∇ f (x, y) avec f (x, y) = xe y + c, c ∈ R. Par conséquent,
pour le théorème de P OINCARÉ, il est conservatif. En effet, V
le travail nécessaire pour déplacer une particule du point (−1, 0) au point (1, 0) ne dépend pas du chemin choisi et vaut
f (1, 0) − f (−1, 0) = 1 − (−1) = 2.

Exercice 8.30 Forme différentielle


Déterminer la valeur du paramètre µ de sorte que la forme différentielle

ω = (x 2 − 5µy + 8y z) dx + (−5x + 8µxz + 2) dy + ((7 + µ)x y − 6z) dz

soit exacte.
Pour cette valeur de µ, déterminer la primitive ϕ de ω telle que ϕ(3, 1, −2) = 0.
Calculer la circulation de ω pour aller du point (1, 0, 0) au point (0, 0, 1) le longue de la courbe paramétrée γ(t ) =
(cos(t ), cos(t ) sin(t ), sin(t )).

258 © G. Faccanoni
Dernière mise à jour : Jeudi 5 juin 2014 8 Champs de vecteurs, formes différentielles

C ORRECTION . ω définie sur R3 simplement connexe : exacte ssi fermée.

∂ y ω3 − ∂z ω2 = 0 (7 + µ)x − (8µx) = 0


 
 
∂z ω1 − ∂x ω3 = 0 ⇐⇒ (8y) − (7 + µ)y = 0 ⇐⇒ µ = 1.
 
∂x ω2 − ∂ y ω1 = 0 (−5 + 8µz) − (5µ + 8z) = 0
 

On obtient la forme différentielle

ω = (x 2 − 5y + 8y z) dx + (−5x + 8xz + 2) dy + (8x y − 6z) dz.


3
Calcul des primitives ϕ telles que dϕ = ω : comme ∂x ϕ(x, y, z) = ω1 (x, y, z) alors ϕ(x, y, z) = ω1 (x, y, z) dx = x3 − 5x y +
R

8x y z + f (y, z), ce qui implique ∂ y ϕ(x, y, z) = −5x + 8xz + ∂ y f (y, z) ; comme ∂ y ϕ(x, y, z) = ω2 (x, y, z) alors ∂ y f (y, z) = 2, ainsi
3
f (y, z) = 2y + g (z) et par conséquent ϕ(x, y, z) = x3 −5x y +8x y z +2y + g (z) ; comme ∂z ϕ(x, y, z) = ω3 (x, y, z) alors g 0 (z) = −6z
d’où g (z) = −3z 2 + c. En conclusion toutes les primitives de ω sont de la forme

x3
ϕ(x, y, z) = − 5x y + 8x y z + 2y − 3z 2 + c, c ∈ R.
3
L’unique primitive qui vérifie la condition ϕ(3, 1, −2) = 0 correspond à c = 64.
Comme la forme est exacte, la circulation ne dépend pas du chemin parcouru mais seulement du point de départ et d’arrivé,
ainsi Z
ω = ϕ(0, 1, 0) − ϕ(1, 0, 0) = 5/3.
γ

Exercice 8.31 Champ de vecteurs


Déterminer la valeur du paramètre µ de sorte que le champ de vecteurs

~ = (x 2 + 5µy + 3y z)~
V j + ((2 + µ)x y − 4z)~
i + (5x + 3µxz − 2)~ k

soit conservatif. Pour cette valeur de µ, déterminer le potentiel ϕ de V ~ tel que ϕ(3, 1, −2) = 0. Calculer le travail nécessaire
pour déplacer une particule du point (1, 0, 0) au point (0, 1, 0) le longue de la courbe γ(t ) = (cos(t ), sin(t ), cos(t ) sin(t )).

~ défini sur R3 simplement connexe : conservatif ssi à rotationnel nul.


C ORRECTION . V

∂ y V3 − ∂ z V2 (2 + µ)x − (3µx)
     
0
~ = ∂z V1 − ∂x V3  =  (3y) − (2 + µ)y  = 0 ⇐⇒ µ = 1.
rotV
∂ x V2 − ∂ y V1 (5 + 3µz) − (5µ + 3z) 0

On obtient le champ de vecteurs

~ = (x 2 + 5y + 3y z)~
V j + (3x y − 4z)~
i + (5x + 3xz − 2)~ k.

~ : comme ∂x ϕ(x, y, z) = V1 (x, y, z) alors ϕ(x, y, z) = V1 (x, y, z) dx = x + 5x y + 3x y z +


3
Calcul des potentiels ϕ tels que ∇ϕ = V
R
3
f (y, z), ce qui implique ∂ y ϕ(x, y, z) = 5x + 3xz + ∂ y f (y, z) ; comme ∂ y ϕ(x, y, z) = V2 (x, y, z) alors ∂ y f (y, z) = −2, ainsi f (y, z) =
3
−2y + g (z) et par conséquent ϕ(x, y, z) = x3 + 5x y + 3x y z − 2y + g (z) ; comme ∂z ϕ(x, y, z) = V3 (x, y, z) alors g 0 (z) = −4z d’où
~ sont de la forme
g (z) = −2z 2 + c. En conclusion touts les potentiels de V

x3
ϕ(x, y, z) = + 5x y + 3x y z − 2y − 2z 2 + c, c ∈ R.
3
L’unique potentiel qui vérifie la condition ϕ(3, 1, −2) = 0 correspond à c = 4.
Comme le champ est conservatif, le travail ne dépend pas du chemin parcouru mais seulement du point de départ et d’arrivé,
ainsi I
~ = ϕ(0, 1, 0) − ϕ(1, 0, 0) = −7/3.
V
γ

© G. Faccanoni 259
8 Champs de vecteurs, formes différentielles Dernière mise à jour : Jeudi 5 juin 2014

Exercice 8.32 Forme différentielle


Considérons la forme différentielle
1 1
ω(x, y) = − dx + dy.
(x − y)2 (x − y)2
1. Trouver le domaine de définition de ω (qu’on notera Dω ).
2. Établir si ω est exacte sur son domaine de définition.
3. Soit γr la famille de courbes

γr (φ) = (2 + r cos(φ), −2 + r sin(φ)), φ ∈ [0, 2π], r > 0.


Z
Pour quels valeurs de r on a γr ⊂ Dω ? Pour ces valeurs calculer ω(φ) dφ.
γr

C ORRECTION .
1. Dω = (x, y) ∈ R2 ¯ y 6= x .
© ¯ ª

2. On remarque que Dω n’est pas simplement connexe. On commence alors par vérifier si la forme différentielle est fer-
mée :
1
∂ y (ω1 ) = 2 = ∂x (ω2 ),
(x − y)3
donc la forme différentielle ω est fermée. On va alors en chercher un potentiel, i.e. une fonction f telle que ∇ f = ω :
1 1 1 1
∂x f (x, y) = − (x−y)2 donc f (x, y) = x−y + g (y) donc (x−y)2 = ∂ y f (x, y) = (x−y)2
+ g 0 (y) donc
0 1
g (y) = 0 donc f (x, y) = x−y + c.

1
La forme différentielle est exacte sur Dω et un potentiel est f (x, y) = x−y .
3. Les ³courbes γr sont
´ les cercles de centre (2, −2) et rayon r . Pour que γr soit contenue dans Dω il faut alors que r <
p
dist (2, −2); y = x = 2 2. Puisque toute γr est contenue dans un sous-ensemble simplement connexe de Dω et puisque
Z
ω est fermée, elle est exacte dans ce sous-ensemble. On en déduit que ω(t )d t = 0.
γr
y
x
=
y

2
x
r
<
2 p
2

−2 γr

260 © G. Faccanoni

Vous aimerez peut-être aussi